Download as pdf or txt
Download as pdf or txt
You are on page 1of 182

Unit 11 PROBABILITY

206
A. Probability (Simple Cases) 206
B. Binomial Distribution 213
Unit 12 STATICS
221 SOLUTIONS TO MODEL QUESTIONS
238
Unit 13 STATICS (CONTINUED)
253
2068 (SET I)
Unit 14 DYNAMICS Full Marks: 100
A. Motion in a Straight Line 253 Time: 3 hrs Pass Marks: 35
Candidates are required to give their answer in their own words as far as practicable. The figures in the margin indicate full
B. Mouon under Gravity 256 marks.
C. Motton Down an Inclined Plane 262 Attempt ALL questions of group A and group Bor C.
Unit 15 DYNAMICS (CONTINUED) 265 Group A

A. Newton's Law of Motion, Impulse 265 1. a. It is required to seat 5 boys and 4 girls in a row so that the girls occupy the even places. How
many such arrangements are possible? 2
B. Projectiles 270
Solution
C. Work, Energy and Power 277 Since the girls occupy only the even places, they can take 2nd, 4th, 6th and 8th seats. Hence, 4 girls in 4
places can be arranged in P(4,4) ways = 4! ways
Unit 16 LINEAR PROGRAMMING 285
A.Graphical Method 285 B
And, 5 boys in remaining 5 seats can be arranged in P(5,5) ways = 5! ways.
B. Simplex Method 309 T o t a l number of arrangements = 4! * 5!
Unit 17 COMPUTATIONAL METHOD 321 4x 3 2x 1 x5x4 x3 x2x 1
x
-2880
A. Number System 321
B. Bisection Method 324
b. Prove
that257 2(1-In2). 21
C. Newton Raphson's Method 330 Solution
Unit 18 cOMPUTATIONAL METHOD (CONTINUED) 337 LH.S 5 .
A. Gauss Elimination Method 337
B. Gauss-Seidel Method 342
C. Matrix Inversion Method
348 4.4441.1. 2 [1- In (1 + 1)] =2 (1 - In2)
Unit 19 NUMERICAL INTEGRATION
353
A. Trapezoidal Rules
353 C. Leta* b= 3a + 2b for a, b eZ. Verify that is a commutative binary operation on Z. 121
B. Simpson's Rule Solution
361 Let a, b e Z. Then 3a, 2b e Z.
Since sum of any two integers is again an integer, a"b - 3a + 2b e Z.
Hence, for all a, b e Z, a*b=3a+ 2b e Z uniquely. So, *is a binary operation.
Again, take a = 2, b =3 in Z. Then,
a * b = 2 * 3 = 3 x 2 + 2 x 3 = 12

b a 3*2 3 x 3+2x 213

a*b b*a
Hence, * is not conmmutative.

2. a. Find the equation of a hyperbola in standard position such that the length of transverse axis
is 6 and it passes through (4, 2) (21
Solution
Let the equation of hyperbola be

By question, length of transverse axis


.e. 2 a 6
=
6

a = 3

Putting the value of a in (i), we get

If the hyperbola passes through the point (4, 2), then


Mathematics-l SOLUTIONS To MoDEL QUESTIONS
2 Asmita's NEB Solution of Basic

dx
b. Find the integral1+2sin
or, -1 Solution

dr dr dr sec?2
Putting the value of a and b in (i), we get Let1+ 2sina |cnsin2 2sin~cos co sin2.2sinacos sec
sec dr
or, 1
4 x - 7 y 2 = 36 tan+4 tan+1
which is the required equation of hyperbola

b. Find the locus of points which are equidistant from


the points (1, 2, 3) and (3, 2-1). 12 Put tan y

Solucion
z) be any point on the locus.
Let P(x, y,
secddy
By given, distance of (1,23) from P(x, y, z)
=
distance of (3, 2,-1) from P (xy,z)
seca dr= 2dy
(r 1)? (y-2}*+(z-3)P (t-3)2 (y-2)+ (z 1)2
+ + =
+
or, -

+z2+ 2z t+1
-21r+1+y2-4y +4+z2-6z+9=?-6x+9+ y2-4y +4
or,
X- 22=0
c. Find the cosines of the angle between the vectors:
= (1-2,-2), b (21-2). tan2-5 C
Solution
Given, a - (1,-2,-2), b (21,-2) n26
Then, a b =(1,-2 -2) (21,-2) 2 - 2 + 4 - 4
121
a a l =y12+ (-2}+ (-2=3 Find the
integra 7 2
b B =V2 13 (-2=3 Solution
If e be the angle between the vectors a and b, then
Put 2 - x = y2

or, -dx =2ydy


cose= . dx= -2ydy
Now,
Cosine of the angle between the vectors is

3. a. Find the derivative of (ln x)sinh 121 Ja7 J9Js-2 -c-je -c


Solution
Let y = (Inx)vnkr
4. a. Solve the differential equation: ey + 3a er.

Taking 'in' on both sides, we get,


Iny = sinhr In (lnx)
Solucion
Differentiating both sides with respect to '* Given equation is:

d
drny) d a (sinhx ln (lnx)) dyr e y + 312 ey
d

iny)dsinhx dln (n) in (imeinh) or, d e (ex + 3x)

1a=sinhr. d {In (nu) dnx) in


or y dx cosh
d (Inx) dx
,
(Inx).
or, ey dy - (e* + 3x*) dx

On integration, we get
or yay
dr = sinhx a t coshx. In (lnr)
-e-y=e+
or, ddy y sinhr
a tcoshx. In (In) e t e-y + x+C= 0

(Inx)sinhrSnr
xlnr
coshr. In (In)
Mathematics-ll SOLUTIONSTO MoDEL QUESTIONS 5
4 Asmita's NEB Solution of Baslc
number of
calculate the expected value of Y
when X =
25, arrangement there are
in a circle, we n arrangements in a
straight line. Thus, if P is the'
b.
From the following data, X arrangements in a circle, have
5.6 12.5 nP-n
Average
Standard deviation
3.2 2.4

21
nnnn)- (n -1)!
n n

and correlation coefficient r= 0.95.


b. What is a group? Ifa binary operation is defined on a set S (a, b, c) by the following Caley's
Solution
Here, X =5.6, Y = 12.5, o 3.2, o -2.4, T= 0.95 table.

bx =0.95 0.7125

of Y on Xis
The regression equation
Y-Y = bx (X-X)
Show that (S, ") is a group. 141
or, Y-12.5 = 0.7125 (X-5.6)
or, Y- 125 = 0.7125X -3.99 Solution *
defined G.
set and is an operation
or, Y = 0.7125X + 12.5-3.99 Group: Let G be a non empty on

Y=0.7125X + 8.51 Then,


(G, *) is said to be a group if satisfies the following axioms.
*

When X = 25, Y = 0.7125 x 25 8.51 = 26.3225


1. Closure axiom: G is closed under the operation.
Required expected value of Y is 26.3225. i.e. a"b e G for all a, b eG
The average percentage of failures in a certain examination is 40. What is the probability that out of 2. Associative axiom
5 candidates, at least 3 will be passed in the examination? (b*c) (a*b)*c for all a, b, c, e G
3.
a =

Identity axiom: For all a e G there exists an element ee G such that


Solution Here, p = P(failure) = 40% = 04 ae= e'a =a
9 P(not failure) = 1 - p =1 -04 = 0.6 The element e is called the identity element.
n=5 Inverse axiom: Each element of Gpossesses inverse i.e. for all a e G, there existsbe G such that a'b
Therefore, the probability that at least 3 will be passed in the examination ie. at most 2 will be failed =b'a = e.
The e l e m e n t b is called the inverse of a. We write b = a-1.
is given by
Prs 2) P(0)+ P(1)+ P2) = *Co ptqs-+ C p'q'+ Gp4* Second Part
1 x
(0.6) + 5x (0.4) x (0.6) +10 x (0.4)2 (0.6)
x Here, S la, b, c)
0.07776 +0.2592 0.3456 = 0.683 i. Closure property: Since all the entries in the Cayley's table are element of S, S is closed under*.
i. Associative property:
5. a. Show that the number of combinations of n different objects taken r at a time is given by C a* (b*c) = a'a = a

(ab)*c = b'c = a
(n, r) rit Also, prove that C (n, n -r) = C(n, r).
a (bc)= (a"b)*¢
Solution This result is true for all elements of S.
Consider any one of the Cn.r) combinations. This combination contains r objects. These r objects Associativity is satisfied.
among thenmselves can be arranged in ! different ways. So, for each combination, there are r! ii. From table,
for
permutations. Hence, there are Ctnr)
the C(nr) combinations, different
these are al possible permutations ofn objects taken r at a time. So, we have
r permutations. Since a*a =

b'a = a*b =b

Cnr) = P(n.r) & c'a a ' c =c


n a is theidentity element.
or, C(nr) (n-r
V. aaa
b'c= c'b a
Cnr) n-7 a , b, care inverse element of a, c and b respectively.
Second Part From (i) - (iv) we conclude (S, ") is a group.

Cn, n-
n! OR
(a-(n-r)(n-r) n- ri! r Let a, b, c and x be elements of a
group G. Solve for x if axb= c and *b= xac.
OR
Solution
State the multiplication principle of counting. Prove that the number of circular permutations of Let, a, b, c, x e G.
n different objects taken all at a time is (n -1)! Here, xbxa'e
4
Solution or, xl (xxb) = x-1 (xa-lc)
Multiplication Principle of Counting or, (xx) (xb) = (x*l-x) arle (by associativity)
If one be done independently in n1 different
thing can ways and if a second thing can be done in n2 or, e xb =e alc
different ways and if a third thing can be done in ni ways and so on for or, xb= a-lc
then the total number of ways in which all the any finite number of things
thingscan be done in the given order is ni nz ns. or, (xb) b:1= (a-c) b
Next Part Or, x(b-b")=a-cb-
Let Xi, X2, Xn be the objects arranged in a circle, then the
..., or, xe = a-'cb-1

xiXix5.. Xax1X2 are not different. But if


arrangements x1x2*3 Xny X2X3X4 .XnX\ ...

...

they are arranged in a row (straight line), to each


Asmita's NEB Solution of Basic Mathematics-
SoLUTTONS TO MoDEL QUESTIONS 1
Xa-cb-1
Putting the value ofx in axb=c we get Next Part:
a(a- cb")b =c dy
or, (a a-l) c(b-' b) =c
Here, (x+ 1)d 2xy 3x
or, e ce s c
Cc(true)
x=a-'c b' Comparing (i) with dx+ Py =Q we get
which is the required solution.

P i
6.Find
theintegral d 14
I.F =e= = elog (o~e) =xx+1
Solution Multiplying both sides of (1) by (x+1), we get
2X
(x2+1)day+ (+ 1) +1'y= (* +1) +1
or, d{(x2 +1) y} = 3x? dx

Bx +C Integrating both sides, we get


Letc1)-x+1)*1*-x*1 yx+1) =3xdx + C

*)+Bx+ Cx+1)D
O(1)(-r+T (x+1)(3-x*1) y+ 1)-3+C
or, x A(x- x+ 1)+ (Bx+C) (x +1)
y(x2+ 1)=x+
Putx -1, then A = -
7.
Statefhe first mean value theorem of differential ealculus and interpret it geometrically.
a.

Again, putx = 0, thenC = Using it to f{x) sinx [0, x1 prove that sin sx for x2 0.
=
on x 4
Solution
Statement of mean value theorem
And putx =1, then B= If a function f(r) is
Now, from() (a) continuous in the closed interval [a, b]
(b) differerntiable in the open interval (a, b)
1/3 1/3xX+1/3
then there exists at least one c e (a, b) such that f(¢) = fb)-ffa)
b-a
Second part
Geometrically, Lagrange's mean value theorem says that in a continuous curve, in which tangent
can be drawn at every point, there is at least one point where the tangent is parallel to the secant
joining the end points as shown in the figure.

X X

Next Part

Let x 2 0. For all x 2


0, fo)
=
sin is defined in
x [0. x]. So, fo)
=
sin x is continuous in
[0, x. Again, f(x) = cos x which exists for all x 20. So, f(x) is differentiable in (0, x). Hence, all the

of mean value theorem are satisfied. There existsce (0, x) such that
conditions
fx)=t _= P(c)= cos c
X - 0
b. What is a
linear differential equation? Solve: Or, Sin = cOS C
Linear Differential Equation (+1)2ry 34. 41 k- 0

Solution or, = cos c S1


A differential equation of the
formd Py-Q where P and or, s1
not of y is called a linear differential equation. Q are functions of r
x or constants but
sin x Sx for all x 2 0.
SoLUTIONS TO MODEL QUESTIONS

Mathematics-Il
8 Asmita's NEB Solution of Basic
vectors. Show that the following vectors are linearly dependent.
It three balls are dra Define linearly independent
six red and two green
marbles.
drawn b.
41
four white, eight black, 2 red and 1 green marbles, T +4T-3K
D. An urm contains

random, find the probability


of getting (i) all
white marbles (1) 2T T K,3T -

2 +K,
at Solution
Linearly Independent Vectors
SolutionNumber of white marbles = 4
said to be linearly independent if in the relationn
Number of black marbles = 8 A set of vectors a, b, c , . , vis
red marbles 6 xa+ yb + zc +... + tv= 0
Number of
Number of green marbles = 2
all the scalars x, y, Z,.. tare zero.
Total number of marbles = 4 + 8+6+2=20
out of 20 Next part:
Total number of possible cases (n) =Number of selection of3 marbles Let a 21 +j -k = (2 1,-1)
20!
C173 20xx2x1
19* l8_ 1140
1140
6-37 -2+k -(3,-2,1)
i) All white marbles
Total number of (m) =
Number of selection of 3 white
marbles out of 4 white marbles -T+4 -3k =(1,4,-3)
possible cases
Let x, y, z be the scalars such that

-C 4 xa +yb+zc =0
x(2,1,-1) + y(3, -2, 1) + z(1, 4,-3)= 0
Pall whitemartio- or,
or, (2x + 3y + z, X -2y +4z, -x +y -3z) =0
the vectors, have
(i) 2 red and 1 green By equality of we

Total number of possible cases (m) = Number of selection of 2 red out of 6 red marbles and 1 green 2x +3y +z=0
x - 2y + 4z =0
out of2 green marbles = G x G * 4 2 1 1 3 0 ,2 -X+y 3 z = 0

P(2 red and 1 green marbles) =

a What is a conic section? Find the equation of the tangent to the parabola y2 = 8x which is
parallel to the straight line2x-3y +7=0.Also find its point of contact.
AX= 0
4
Dolution Where, A
Conic Section
The locus of a point which moves in a
plane in sucha way that the ratio of its distance from a fixed
point to its distance from a fixed straight line is constant is called conic section. A
Second Part =4-3-1 -0
-2(6-4)-3(-3+4) +1 (1-2)
The given equation of parabola is
Thus, X 0
y2 8x
Comparing () with y2= 4ax, we get
4a-8
which shows that all the scalars are not zero. Hence, the vectors are linearly dependent
a2
The equation of tangent to the parabola (i) is OR

+
Prove that if 0 is the angle between the vectors a and b, then a. b=ab cos . 4
y mx
m
Solution
or,
ymx m ...i) LetOA a,OB =
b and LAOB =
0. Then

If this tangent is parallel to 2x -3y +7=0, their slopes must be OA a, |OB| =b.
equal.
m AB -OB-OA
- Ib al
Substituting the value of m in (i) Applying cosines law to AOAB,

AB OA+OB-2(0A)(OB) Cos
b - a |2= a+ b-2ab cos 0
or, y-x+3 or, (b a) (b - a) = a +b - 2ab cos ala-a)
2x-3y +9 0 or, b b -b a - ab+a a a*b-2ab cos
or, b-2a-b +a a+ b2 -2abcos ('a b b a)
Point of contact
ab ab cos 0
Basic Mathematics-Il
10 Asmita's NEB Solution of SOLUTIONS TO MoDEL QUESTIONS 11

that:
9. For any positive integer n, prove
r*.+C (n, n) rn.
V2+ m+ n2
C(n, 1)a-lx+C(, 2) a
a+xC(n,0)
an+
1
of 1-5. m3.nwhich gives the d.cs of the first line.
Find the ternm containingr, ifany, in the expansion
Again, from () and (iv), we have
2+ 2m-n =0
Solution al+ C(1, 1) x
1, (a x)' a + x = C(1,0)
=
21+ m+ 0 n = 0
+
When n =

theorem holds forn =1. By the rule of cross multiplication, we have


This shows that true for n k. =

Assume that the theorem is

Then (a x) C{k, 0) ak C(k, 1) ak-l x


+ =
*... +C(k, k) x*
+
...0 o12-0-2-4
k*1 =

We wish to prove that the theorem is


true for n
we get
or
or,
Rmn
- 2 - 2 1 2 ( - 2 + (-2)
Multiplying both sides of (i) by (a+ x),
+C(KR) *
(a+ x) (a + x) {C(KO) ak +C(k1)a-lx+.. 1))ak1 x + . .
+ {C(k, 2)+0
C(k, 0) ak-1
+
{C(k1)+C(k 0)) alx
+{C(kk)+C([k, k -1)} axk + C(kk)
x*"
..
1-3,mn-
+C(k+ 1, k) ax +C(k + 1, k+1) xk which gives the d.c's of the second line.
Ck+1, 0)ak-1 +C{k +1,1) atx Ck*1,2) ak-lx2 +..
+

C(k. 0) =1 = C{k +1, 0)


Using OR
CKk) 1=C{k*1,k+1) Prove that a plane through three points (x, yu, Zi), (*2 y2, z») and (*u y, z») is given by
Ckr)+C(kr-1) =Ck+ 1,r) k.
k +1 whenever it is true for n
=
x-1 y-y1 z-Z1
which shows that theorem is true for n
=

theorem is true for all X yy Zz-z 0


Hence, by priniple of mathematical induction, the
ne N. X-X1 y3-y1 Z3-Z1

Next Part Also, find the angle between planes 2x-y+z=6and x +y +2z -3.
6
Solution
Let t be the general term in theexpansion of3 First Part
The equation of plane through the point (ri, y1, zi)is

Then, t- C(6.)
) a(r- xi) + b(y - y1) + c(z - zi) = 0 ()

Since () passes through the points(x y2 z:) and (r, ys zi),


a ( t - m) + b{y2- y1) + c(z2- z1)=0 . i ) D
so

Clen )*-can)( & a(ri-n ) + b{y1-y) + c(z1 - zi) =0 (ii)

For the coefficient of the term containing x, we have Eliminating a, b and c from (i), (i) and (ii), we have
X-X1 y- yi Z-Z1
6-2r
or, 2r =4 2-1X - X1
y2-yi
ys - y1
Z2-Zi
Z - Z1
0is the required equation of plane.
I 2
Second Part:
t 3 d term contains?.
Given equations of plane are:

Coefhioient of -
C(62) { a
2x-y+z6 and x+ y+2z 3
2 , bi = -1, ca =1 and a = 1, ba =1, ca =2

Let 0 be the angle between the planes. Then


10. Find the direction cosines of two lines which are connected by the relations 21 + 2m - n
mn+nl + Im =0.
0, a1a2 + bibz +Cicz_ 2x1+(-1)x1+1x2
V2(IP+1y+2-i-cos
Solution var+b+cr Vaz bz»c
Given relations are:
21 2m-n =0
mn +nl+ Im =0 11. Lives of tywo models of refrigerators turmed in for new models in a recent
Eliminating n betweern () and (i)
(ü) survey are
No. of years No of retrigeratoS
m(21+ 2m)+ (2+ 2m)i + Im=0 ModelA Model B
or, 212+ 5lm + 2m2=0 0-2
or, 1+ 2m) (2 +m) =0
Either1+ 2m =0 4-6
(ii) - 8
or 21+ m= 0 (iv) - 10
From (i) and (ii)
21+ 2m- n=0 10-12
I+ 2m+0n =0 What is the average life of each model of these refrigerators? Which model has more uniformity?16]
By the rule of cross nmultiplication, we have
Mathematics-l
2 Asmita's NEB Solution of Basic SoLUTIONS ro MoDEL QUESTIONS 13

Solution Model B or, Px 2 =Qx 1


Model A Q-2P
No. of year Mid value Therefore, the force on the prop at C is double of than that of the other.
0-2 63 A pump having a power of 392 W pumps water at the rate of 100 litres per minute. Find the
60
2-4 65 325 300 height to which the water is raised. (g= 9.8m/s, 1 litre of water = 1kg) [21
4-6
343 133 31 Solution
81
6-8
8-10 5
405
11 729 Mass (m) = 100

Time (t) = =1 minute = 60 secs


14 484
10-12 Efx2=1706I N=50 Ex=308 2d Power (P) = 392 W

N 50 2 =256 g 9.8 m/s


For ModelA Height (h) =?
We know that
A) -5.12 P-mgh

o(A)- --1/-(5.12 -34.12-26.214 =V79056 =281 X98 xh


or, 392=
2.81 50
CV(A) oA 100 15.12 100 54.88%
(A) Or, h 392
100 xx9.8
60 24m
For Model B
308 Required height (h)=24m
x (B) 50 6.16
13. A body of weight w is suspended
a.
by strings of length 3 m and 4 m attached to two points in
the same horizontal line whose distance apart is 5m. Find the tensions
oB) =N- V50 (6.16 P-223 the along strings. 4
Solution
Let CA and CB be the strings so that CA 3m, CB= =
4m and AB =
5m.
Cv) *1006100=36.20% Since 32+ 42= 52, so LACB = 90°.

Let the line of action of weight


Average life of model A = 5.12 years, Average life of model B = 6.16 years Wbe produced to meet AB at E. LetZACE= 0 so that CBE= 0. Let
Ti and T2 be the tensions along the strings CA and CB
Since CV(B)<CV{A), model Bhas more uniformity. W acting at C are in
respectively. Since the three forces T, Tz and
equilibrium,
so Lami's theorem,
by
Group B
12 a. Three forces P, Q and R acting on a
particle are in equilibrium, the l2 W
the Pand Q is 60° and that between Q and R is 150. Find the ratios of the forces.
angle between sin BCD sin ACD sin ACB 5m
14 T T2
Solution
Given, angle between P and Q is 60° and angle between Q and Ris
Or,
sin (90° +0)
sin
W
(180 0) sin90
3m 4m

Angle between R and P = 360°- 60° - 150° =150°


150 = W
or,
Since P, Q and R are in equilibrium, cos sin 0
by Lami's theorem, we have
P T= W cos 6 and T2= W sin 6 180- 0 90
sin 150sin 150°sin 60° BC
Butcos BA andsin0-AB
Or, 37B/2
150
Hence, T-W N
150 -W N
P:0:R-1:1:3 b. A body of mass 49 kg is falling freely under gravity at the of 20
rate m/s. What is the uniform
b. A uniform beam, 4m
long is supported in a horizontal force that will stop it (i) in 2 sec (i) in 50 em? (g 9.8 ms/)=

apart, so that the beam projects one meter beyond position by


one of the
two props which a f Solution
the props is double of that on the other. props. Show that the force on o Here, m= 49 kg
2
Solution u 20 m/s
Let O be the centre of a uniform beam AB v =0 m/s
=
4 cm
by two props at A and C. Then, supported i. t=2 sec
B Let a be the retardation. Then
OA-2 m, OC = OB- BC 2-1 =1 m
A(2 cm cm Cn
If W be the weight
of the
beam, P and Qbe the forces on the
a-1 0
props at A and C respectively. W Retardation (a)
Then,
=
10 m/s
AO W
AC
Resistance force () =
m(g + a) =
49(9.8+ 10) 49x 19.8 =
970.2 N =970.2
9R kg-wt 99 kg-wt
or, Px AO=Qx CO
SoLUTIONS TO MoDEL QUESTIONS 15
of Basic Mathematics-
14 Asmita's NEB Solutlon

ii. s= 50 cm = 0.5 m

We have,
v2 = u2-2as
0.5
or C
or, - 202-2 x ax

or, -400 =-a


S x OC = Q* CB .(ii)
as400
20,080.2 N 20,080kt w
=8kt wt
From (i) and (i), we have
Px AC=Q* CB
the resistance
force F=ma g) =49(400+9.8)=
+
- 2,049 kg wt
or C4C
Therefore,

i.e. C divides AB internally in the inverse ratio of the forces.


OR
after penetrating
acm. How much further win
half its velocity OR
A bullet fired into a target losses
penetrate?
Define the moment of a force. Forces 1, 2, 4, 5 kg-wts. act along the sides of a square taken in
Then, order. Prove thattheir resultant is parallel to a diagonal and find where it cuts the side along
Solution before the penetration
and a be its retardation.
which the first force acts.
Let u be the velocity of the bullet just
uucm/s, v=cm/s, s =3cm Solution
Moment of a force
The moment of a force about a given point is the product of the
We have,
v2= u2-2as magnitude of the force and the perpendicular distance of the
point from the line of action of the force.
r, -2a3
Thus if F be a force and p be the perpendicular distance of the
u2-8a () .

point O from AB then the moment ofF about O is Fx OM =FxP A M


Let (3 + x) cms be the thickness penetrated just before coming to rest. Then,
Second Part
0 u2-2a (3+x)
or, u2=2a(3 + x)
Let the forces 1, 2, 4, 5 kg wts act along the sides AB, BC, CD and
DA respectively of a square ABCD.
or, 8a = 2a(x +3)
(using 6)
x=1
Since 1 and 4 kg wts are horizontal parallel forces, so
Required further penetration =1cm.
X= 1
-4=-3 kg wts
14.
The resultant of two like P and Q acting on rigid body is a force of magnitude
parallel forces P Again, since 2 and 5 kg wts are vertical parallel forces, so
Q in the same direction as P and Q are. If A and B are any points on the lines of action of P and0 Y2-5=-3 kg wts
respectively, prove that the resultant divides line segment AB internally in the inverse ratio o Let R be the resultant. Then
the forces.
Solution
R- Y-y-33+(-=V18 2kg
IfR makes an angle of 0 with AB, then
wt

Let P andQ be two like parallel forces


acting at respectively
the points A and B of a rigid
body represented by the lines S tan- -1= tan 225 ( X<0, Y <0)
AD and BE. Join AB. Let us introduce two
equal and opposite 225°
forces each equal to S along BA and AB
two forces being equal and
These
opposite have no effect upon the
respectively. aPps Hence, the resultant is parallel to the diagonal CA
Suppose that the resultant cuts BA produced at E and AE = x. Then the moment of the resultant
system. Let these forces be represented by AH and BG. about E is zero. Hence
Complete the parallelogram ADIH and BEFG. Also let the 2(EA+ AB)+ 4x AD -5 x EA = 0
diagonals IA and FB be produced to meet at O. Through O, or, 2(x+ a) + 4a- 5x =0, where a is the length of the square ABCD.
draw OC parallel to the given forces P and
Q meeting AB at or, 2x+ 2a + 4a-5x = 0
point C.
or, - 5 x - 8 a
Now, the resultant of the
forces P and Q is
application of Ri may be transferred from Aequivalent
to the resultant of
Ri and R2. But the po t
P which are
to O and it can
be resolved into two X5a
parallel to their ongnal directions.
into two forces Q ands. Similarly, Ra may be taken to act componenred
at O ana
je 15. A man travels from A to B in 45 minutes. At C, somewhere between A and B, it attains its
The two equal and
have finally Jeft with
opposite forces each
equal to S acting at the
point nd W
, maximum velocity of 45 m per hr. If he travels with uniform acceleration from A to C andd
the force Pand Q acting along OC whose resultant is O balance each other a uniform retardation from C to B, find the distance between A and B, it being supposed that the
Now, the triangles OAC and AID P+ Q.
are similar
man starts from rest at A and comes to rest at B.
Solucion
Let a, s and t denote the acceleration, distance and time taken tromA to C respectively. Also, let f, x
or,
and T be the retardation, the distance and time taken from C to B
Then, t+ T = 45 min = 45 x 60= 2700s
respectively
S x OC =Px AC ..i)
Similarly, the triangles OBC and BEF are simnilar,
Mathematics-ll
6 Asmita's NEB Solution of Basic
TO MoDEL QUESTIONS 17
From A to C
SOLUTIONS
Group C
A
45
30 m/s 16. a. Deteprine graphicaly the solution set of the following system of inequalities:
u=0,v= 45 m/hr60 x60m/s y22,3x +2ys 4, x20, y20
We have, Solution
Vfhe corresponding equations of boundary
V=utat
lines are:
Or, 800+ at 2x +y =2
3x+ 2y 4 i)
x=0
. at 80
y0 (iv
Again, we have From ) 2x+ y=2

Sut+at
Taking testing point (0, 0) in x+ 2y 2 2 **
or, s-0+x get
we
y0
0+2x0>22 (false) 3x+2y 4
2
From(i) 3x +2y-4
...i) 3

02
From Bto C

1 m/s,v=0m/s
2 Taking testing point (0, 0) in 3x + 2y s 4,
u 80 we get
We have, 3x0+2x 0s4(true)
From (ii) x =0which is y-axis
V=u-fT From (iv) y = 0 which is X-axis

x20represents the closed right half plane


or, 0-fT and y 20 represents the closed upper half plane.
The shaded portion in the figure represents the required solution.
Again, we have b. Write a short note on accuracy of a numerical method.

Solution
Everywhere we have to do measurements. Sometimes it becomes very hard or impossible to get the
x=uT
exact value. In later case, we find some kind of result, that may not be exact but as close as possible
to the exact value. Such a result is called approximation
or, x80
Such type of approximation can be done by using numerical method. The absolute eror due to
approximation is given by exact value approximate value|. The error may arise due to
or, x160 i) truncation or rounding off the numbers.
Every method of numerical computation produces results with some errors. The results we obtain
From (i) and (ii) must have the degree of accuracy as required. In order to minimize the error, we should apply the
suitable method according to the problem.
S+X

c. Apply the Simpons's rule to approrimate the value of e lnr dx with n =3.
T6+T) 1

T66 2700 using() Solution


Here, f()=e nx,n=3
14 a1,n=4
vehave, h 4 - 1
Therefore, the total distance from A to Bis 16, m. The four points to be consicdered are xo = 1, X1 2, X = 3, X1= 4. The values of the function at these
ponts are as follows
3
yfx)= e Inx 5.1217 22.0662 75.6891
Mathematics-l SOLUTIONs TO MoDELQUESTIONSs
18 Asmita's NEB Solution of Basic

have OR
Applying Simpson's rule, we

3 22.0662)+75.6891] 58.9698 Writehree methods for measuring error. Approximate y11 by Newton-Raphson's method with
3h
f enxdx 3(y1+ y)+ys) -0+3(5.1217+ accaracy 0.00001. 4
1
Solution
Let us select an error tolerance t> 0, we generate xi,x2, Xy., until one of the following 3 conditions
6xr-9y subject to = s met
17. a.
singthe simpler method, maximize p i. Xn-Xn-1 E

2x-3y s6,x+ys20, x20,y20 14


ii.
n-Xn-1l -<6
Solutionand sbe the slack variables. Then given
LPP can be written as:
ii.
Xn
f(xn)| <e
Letr non-negative
2x-3y+ r=6 Next part
x+y+s=20 Letx=11
P 6x 9y Then, x2=11
or, x-11 =0
2x-3y +r+0s+ 0p =6
. a=11
x+y+ Or+s+0p= 20 By Newton's-Raphson method, we have
6x +9y +0r+0s+ p=0
Simplex tableau
R.H.S
1 0
20 3.333

0 0

Here, -6 is the most negative entry in the last row. So, first column is the pivot column. Since3 -i(s2167 3133162
20 and 3 <20, so 2 is the pivot element

Applying R~r Error 0.00001


RHS
-3/2 L/2 So, 3.31662 is approximate value of V11.
20
0
18Find
. the appronimate solution of the following system of equation by matrix inversion method
-y -2,x+y -2z= -9,x+ 2y+ z=9. [6
Applying RRa-Ri and Rs R + 6R Solution
R.H.S Writing the given equations in matrix form
-3/2 1/2
2
5/2 -1/2 0
1
In the last row, there is no negative numbers, so this table
maximum value of p =18 at (3,0).
gives the optimal solution. Hene which is in the form of AX=B
X= A-1B
Now, we agument A with identity matrix I
b. Use Bisection method to find solution accurate to
within 10 for 3-
7a 0 on
interval 1, 32] + 14x -
6 tn 2-1 1 1 0 01
4 [A| -2 0 10
Solution
Aet f(1)
fx) x -72+ 14x -6
=

0
0 10 107
=1 -7 x 12+ 14x1 -6=2
1 1 0 0RR
f3.2)- (3.2)-7(3.2+14 x3,2-6-0112
0 0 1
Since f(1) and f(3.2) have opposite signs, 50 there 1s a -2 0 1
root between 1 and 3.2. RRa-28
m f(a) fb) f(m) 0 3 0
-2-1 R>Rs-Ri
2.1
2.1 .2 2,65
2 -0.112 1.791
-2 1
2.65
2.925
.2
2
2.925
3.0625
1.791
0.5521
0.0858
-0.112
-0.112
0.5521
0.0858
0
0
1 -5/3 -1/3 2/3 0Ra
-1 ()
2.925 3.0625 2.9937 0.0858
-0.112 -0.0544 10 -1/3 1/3 1/3 RR-Ra
Here, f(m)| = }0.0063 | = 0.0063 < 10-2 -0.0544 0.0063 o1 -5/3 -1/3 2/3
1/3-5/3
RR-Ra
1K R-Ra
So, the required solution is 2.9937 00 14/3
SoLUTIONS TO MODEL QUESTIONS 21
Mathematics-l
Asmita's NEB Solution
of Basic
20

1 0 -1/3 1/3 1/3 - cro) + 2f(r) +..+ 2f(n-)+ ftr))


-1/3 2/3
1/14-5/14 3/14

1 0 0 : 15/42
9/42 1/14 7RRi+R Hence, t dr2co)+ 2f()* +
24(orn-i) ¥¢rmt*
0 1 0 : -9/42
3/42 5/14 a

001: 1/14 -5/14 3/14 JRR+R Next part


a0, b 8, n=8
15/42 9/42 1/14
A=-9/42 3/42 5/14 h - a 8-0,
n
1/14-5/14 3/14 have
From ()
Using Tapezoidal rule, we

15/42 9/42 1/14 8


X = A-B -9/42 3/42 5/14 idtio 24i + t iat is +i4t ist s+ i) +
is)
1/14 -5/14 3/14-2
30/42- 81/42 + 9/14
18/42 27/42+45/14-3 - (25.2+ 2(29+31.8+ 36.5+33.7+ 31.2+29.6+273)+28.61 246
-2/14 +45/14+ 27/14J
X=-2y=3 andz=5 OR
Ji dt,
of where i is the current
of a battery is a measure
19. Derive he trapezoidal rule. The capacity whose
current was
measured over
the
using the Trapezium rule, capacity of a battery
Estimáte,
results shown below.
Compute an approximate value of
J(1+ 1 dr by using the composite trapezoid rule with
an
eight hour period with the 0
Time/hours 1 2 34 67 three points. Then compare with the actual value of the integral. Nert, determine the
eror
28.6 formula and numerically verify an upper bound on it. 6
Current/Amps 25.2 29.0 31.8 | 36.5 33.7
31.2 29.6 27.3
Solution
Solucion
Statement: If a function fis continuous on the closed interval [a, b),
1

1+1) d
then fr) dr 2 fo) + 2f(m) + 2f(r) +.+ 2f(ta-) + f{an) ] where [a, b} has been partitioned 0
Here, a 0, b =1
For three points, n = 2

into n equal subintervals [ta z, , a, a , za, each oflength h = Then,h - a 1-0 =0.5
.
Then,h 2
Proof Let fbe defined on [a, bj. Consider the area bounded by the graph of f, the ordinates Now, 3 pointstobe considered arexoO, X1=0.5,xa=1.The value of thefunction at these pointsare
b
0.5
0.8 0.5
a , xband the x-axis. This area is thegeometrical representation of f t ) dx..
y )1
,

Using Trapezoidal rule, we have


1
fa+d 0 2yt y)2208+0.5]-0.25 31-0775
Actual value = ( +gdx = [tanx] ta (1)- tam (0)=-0-0.785

Error in Trapezoidal rule = 0.785 - 0.775 0.01

Here,
X -2x
b-
Let divide [a, b} into n
us
sub-intervals .Xo, ri}, [, xil,.. xi-, x ...
(xn-l, Xnl; each of lengtn
The ordinates corresponding to the points of subdivisions f'(x)-5X-1)

fx, ., =
ns
f(za).. f{x-i), {(x), n-1}, IGn) b. If we join the Xo, X1, X,... Ki1, X, In-l, Xn are
f(o)=
..,
he line
f')(1+ x
The maximum value of |f'(x)| occurs at x =1.It is 2.
segments, we get n consecutive points on the graph t th
rapezolab. 1ne sum o areas of these n trapezoids 'M 2
area under the curve. We know that, is an approximan
1, Error bounds = - 0.0417 (upper bound)
the area of a
trapezium=2(sum of paralel sides) (distance between them).
22 Asmita's NEB Solution of Basic Mathematic-
SoLUTIONS To MoDEL QUESTIONS 23

Or -22,3 -1
16 7
)
SOLUTIONS TO MODEL UESTIONS Comparing equation (i) with ,
2068 (SET I) Full Marks: 100
Weget
h 2, k= -3
a2 16, b2 = 4

Time: 3 hrs Pass Marks: 35 a=4, b 2


own words as Tar as practicable.The
andidates required to give their answer in their
are
Since a> b,
so major axis is along x -axis.
figures in the margin indicate full marks. Centre (h, k) (2,-3)
We have,
Attempt ALL questions of group A and group B or C
Group A Eccentricity (e)=
1 a. In an examination papercontaining 10 questions, a candidate has to answer 7 questions. If
wo questions are made compulsory, in how many ways can he choose 7 questions in all? 2 Foci
(h t ae, k) |2 =

-a:a
Solution 2 =5
b. Find the point where the line throgh the points (1,2, 3) and (4,-4, 9) meets the
If
2 questions are compulsory then the candidate has to select 7 -

questions from 10 -

2=8 zr-plane. 1
Solution
question5. Let zx-plane divide the line joining the points (1, 2,3) and (4,4, 9) in the
n=8,T=5 we have y = 0. ratio mi: m2
At zx-plane,
Total number of selections 8 8x 7x6x5! Now, by section formula, we have
=
C(8, 5) (8-51!5 5! x3x 2x1 miy2 + may1
mi t m2

b. Find the middle term in the expansion of | 121 Or,


0=* 4+m 2
mi+m2
Solution or, -4m1 +2m2=0
or, 4m 2m2
The number of terms in the
expansion of is 9+ 1=10, which is even. So, there are two or, 1
middle terms. They Or ma 2
are
t1 and ty1, ie. and ts
Again, by section formula, we have
We have, t i = C(n,r) an x
x=*
miX2 t m2X1 7=12*
Z
m2z1
mi tm2 m+ ma2
tst C(OA) (214. or, x
14+2.1
1+2 :-23
1+2
Again, or, x = 2, z = 5

ts1C(95) (2x)5 (5-5.2 Required pointis (2, 0,5)


c. Are the three points with
c. Let S=-1,1)
and " denote the usual operation of position vectors i+2+4k,21+5-k and 3i+87 -6K
Show that is a binary
"
on S. operation multiplication. Represent it by Cayley's tabie. collinear? Justify your answer.
121
Solution
Solution Let O be the Let
Cayley's table origin. A, B and C be the three points with position vectors
-1 7+27+4K, 2+57-Kand 3i 87-6k. Then
From above
So, * i5 a binary
Cayley's table we see that product of any two
operation on5.
elements of S is unique and belongs to
ok-27 5-K
oc -31+87-6K
2 a. Find the eccentricity and the foci of the ellipse:
2+4y2-4x + 24y + 24 = 0.
A -Ob-oA
Then, -27.57-F-0-27.sR-7.37-sR
Solution AC-oc-ôÀ -si87-6R-(T+27 4R)-2î+67-10
Given equation is
2+ 4y2-4x+24y +24 0
2(i+3-5k) 2
This shows that AB and AC are
or, - 4 x + 4y2 + 24y + 24 0 pårallel. But they start from the same point A. So, A, B and C are
or, x2-4x + 4 -4 + 4(y? + by +9-9) + 24 0
collinear.
or, (r-2)2+ 4(y + 3)2=16
SOLUTIONS To MoDEL QUESTIONS 25

of Basic Mathematics- Multiplying both sides of (i) by x, we get


24 Asmita's NEB Solution

lim e r t e -2co dyy 1


d
sin?x
3. a. Using L'Hospital's rule, evaluate y-0 or, d (y ) = dr

Integrating ay =X+C

Solution 10, 2X 60, EY 60,


lim et+ e -2c0s2
b. Ifn = = =
EX =
400, Y =
580 and 2XY =
415, find the correlation coefficient
between the two variables.
x0 sin X
Solution
lim er-e+ 2sin r Given, n = 10
EX 60
r>0 2sin r cos x EY 60 EX2 400

lim e-e+ 2sinr


EY2 580 EXY=415
Correlation coefficient (r) = ?
2sin 2r
We have, correlation coefficient () = nEXY-EX EY 10x 415-60 x60
lim e+e t 2c0s I4=2
2 cos 2
nEX-(EXP VnEY2 -(EY? V10x 400 60V10x 580- 60?
4150-3600 500 = 0.59

dx (>a).
400 V2200 202200o
b.Evaluate: J -ae- ) c. Two dice are rolled once. What is the probability of getting a total of 9 or 6?
(2
Solutions
Solution If two dice are rolled once, then the possible cases of uning up are
dr (1,1),(1,2),.. (6,6))
Let e-- (B>a) There are 36 possible cases.
There are 4 cases having a total of 9, whose set is
Putx-a= y (3,6), (4, 5), (5, 4), (6, 3)}
are 5 cases
dr 2y dy And, there having a total of 6. They are
(1,5,(2, 4), (3,3), (4, 2), (5, 1)}
(2y dyoa-2JVra-
JVF N-2JVA-0-
F+a-) P l a total of 9 or 6) =P(a total of 9) + P(a total of6) = :

2 iog (+Ny-(B-aP)+C =2log (Vz-a+yz-a-ß+a)+C 5. a. In how ways can the letters of the word "COMPUTER" be arranged so that

-2log a+vr-B)+C i. all the vowels are always together?


ii. the vowels may occupy only odd positions? 14
IE=67+37-sk and-7-47+2 show thata Bis perpendicular to a. Solution
Solution )
There are 8 letters in the word COMPUTER'
There are 3 vowels in the word 'COMPUTER'. Consider 3 vowels as one letter. Then 6 letters
Here, -67+37-57 namely C, M, P, T, R, (O, U, E) can be arranged in P(6, 6) ways
6! ways
B-7-472 Also, 3 vowels among themselves can be arranged in P(3, 3) ways
3! ways
Total number of arrangements 6! x 3!
6x5x 4x3 x 2x 1 x3x2x1=4320 ways

Now,
(6-20)7-(12+5)7+(-24-3)k- 14T -177-27 (i) If the three vowels 0, U, E occupy only in odd positions namely 14, 3, 5th and 7h ie. 4 positions,
then the total number of arrangements of 3 vowels in 4 places = P(4,3)

Again, the remaining 5 letters in remaining 5 places can be arranged in P(5,5) ways 5! ways
B)-a (147 -177-2k)- (6T +37-5k)- -84 -51+135-0
This shows that (a x b) is perpendicular to a. Total number ofarrangements P(4,3) x 5! -x54x3x2x1 x5x4x3x2x1 -2880ways

b. Given the algebraic structure (G, *) with G (1, o, a] where o represents an imaginary cube root
a. Solve:+y 1-0. of unity and * stands for the binary operation of multiplication, show that (G, *) is a group.

Solution Solution
Here, G (1, o, o
Here, d+y-1-0 i)Since the product of any two elements of G is also an element of G. So G is closed under
ii) 1 (o o ) =1* o'=1*1=1
dy and(1 o) * a2=a *a2=o 1
1 ( o ) (1 *o)*o2
Comparing () with dy Py =Q we get This result is true for all elements of G.
'Associativity is satisfied.

1.F= etd= elogs


Mathematics-ll SOLUTIONSTO MODEL QUESTIONS 27
Basic
26 Asmita's NEB Solution of
Second Part
ii) Here, 1*1 1 Let S be the focus and ZM be the directrix of the
1*o =o*1=o parabola. Let SZ be drawn perpendicular to ZM. Let Oo M-a.y
1*o2=o2*1=o2 be the middle point of SZ i.e. SO = 0Z. Then O is the
So, 1 is the multiplicative identity. vertex of the parabola and SZ is the axis of the parabola.
iv) Here, 1*1=1 Let OS a. Then the coordinates of S, O and Z be
1*o = o ' =1
(a, 0), (0, 0) and (-a, 0) respectively.
2oo*=1
inverse of 1, o and o respecthvely Let P(x, y) be any point on the parabola. Join PS and
1, and o? are multiplicative
o
draw PM perpendicular to ZM. Then by definition of
So,
Hence, from (i) - (iv), (G,") is a group. X Z(-a, 0)O S (a, 0)
>X

4ar at the point xu yi). Express it in the parabola,


the parabola y
=

6. a. Find the equation of the tangent


to PS PM
or, PS2 = PM2
slope form.
the parabola or, (r- a} +(y-0)2- (r+ a)?+ (y -y
Solution 4axr. Let us take another point Q2 y») on
or, x?- 2ax + a2+ y2 =x2+ 2ar +a2
P(a, yi) be the point on the parabola y?
=

Let
y2 4axr which is the required equation of parabola in standard form.
which is very cdose to P. Then,
...) equation of
y24ar
...(i)
b. Find the
plane through the point (21, 4) and perpendicular to each of the planes.
9x-7y +6z + 48 = 0 and x+ y +z=0.
&y2=4arn 41
Subtracting, we have atya.r3 Solution of
y?-yi= 4ar - 4ar2
Py) Equation any plane through the point (2, 1, 4) is
a(x-2)+ b(y - 1) + c(z-4) = 0 )
Or, y2-y) (y2+ yi) = 4a (*a - i )
Since (i) is perpendicular to the planes 9x - 7y+ 6z+ 480 a n d x + y +z=0, we have
9 a - 7 b + 6c = 0
2 - 1 y2+ y
.(ii) ..)
a+b+c=0 1)
The equation of PQ is Solving by the method of cross multiplication, we have

y-y a-a)
fusing (ii)] b C
or, y - y y a y - 1 ) 13 -i6-k (say)
ie. when x2>X1 and y2-> yi.
By definition, PQ becomes the tangent at P when Q> P a - - 1 3 k , b = -3k, c= 16k
Hence, the equation of tangent at P is
Substituting these values in (i), we get
a -13(x-2)-3(y -1)+16(z - 4) 0
y-y12,-i) or, 13x-26+ 3y -3-16z + 64 =0
or, yyi-yi= 2ar -2ar1 + 35 =0
Or, yy1- 4ar1 = 2 a x - 2 a using ()]
13x +3y-16z
or, yy1= 2a(r + i ) which is the equation of tangent to the parabola y2 = 4ax at (71, y1).

The above equation can be written as 7. a.


Evaluate:atbcosz (a>b>0),
yV Solution
2a
Writing=m, then y1 dx dr sec
a+b cos x
Sincey2 4ar, so 4ar1. si (-s Ja ba-b sie
sec
m
Then from ) . (a+b)+(a -b)tan*
a/m
y mr+ 24 2a/m
Put ya-b tan= y
y = mx which is the equation oftangentin slope form

OR
a-b sec dr=dy
Then the integral () becomes
What is a conic section? Find the equation of the parabola in the
standard form.
Solution
Conic Section Va-baby*yi tan"ya
Please see Model Set I, QNo. 8a
SoLUTONS TO MODEL QUESTIONS
29
28 Asmita's NEB Solution of
Basic
Mathematics-l
** t

yy1-=sin-lr+ c
8. a. Find Karl Pearson's coefficient of skewness from the following distribution.
4 Marks Above 20 Above 30 Above 40 Above50 Above60
b. Solve:x yy. No. ofstudents 50 46 30 8
Solution
Solucion Calculation of Karl Pearson's Coefficient of Skewness
Given equation is fd?
Marks Mid value d'al45)| fd

d+yy 20-30 25 -2 8 16
30-40 35 16 20 -16 16
Then, 40-50 45 6 26 0
This is a homogeneous equation. So, put y=Vx. 50-60 55 16 16

+ x dx 60-70 65 50 16 32
Hence (i) becomes
N=50 fd' 8 fd-80
y+
dvx-VX- yX Mean (x)=a+xh=45+50x 10-46.6

or, v =v-v? Metian tem tem-


Median lies in (40-50) classinterval.
Or, d x - v 2
1 40, f=6, cf-20, h 10
or, -v2dv = -C.f
On integration, we have We have, Median (Ma)= 1 + h-40+ 10-48.33
-logx S.D (o)=hx*\ 10x 16-0.0256
orlogx+c 10xV1.5744 =12.55
We know, Karl Pearson's coefficient of skewness
x y (logx+c)
OR Se (P)-=Ma) 3(46.6-48.33) -0.41
12.55

Solve:(1 9 y-1 4 b. The chance that A can solve a certain problem is Y and the chance that B can solve it is % Find
the chance that (i) the problem will be solved if they both try (i) A solves but B cannot.
Solution
Solution
Here,(1- 1+zy i The problem will be solved if A or B solve it
Probability of solving the problem by A, P(A)=
or, - y-1
Probability of solving the problem by B, P(B) =5
Probability of solving the problem, P(A orB) -
P{AUB) ?
Comparing equation () withd+Py Qwe get We have,
P(AUB) = P(A) + P(B) - P(AnB)

P P(A) + P(B) - P(A) P(B)

Multiplying both sides of equation (i) by1 - , we get

ii.
P(B)1-P(B) -1-
The probability of solving by A but not by B is

d(eT-)
Integrating we have
PAn)-P(A) P)-*-
Mathematics-l
30 Asmita's NEB Solution of Basic SOLUTIONS TO MoDEL QUESTIONS 31

OR are m a l e .
Suppose we select
t5 bi Similarly, a.b (magnitude of b) (projection of a on b)
births
the recorded
of all
in certain city 60%
ouPpose that probability
that
Next part
records from population. What is the
Let XOX' and YOY be twomutually perpendicular straight
i exactly three of them are male? lines representing X-axis and y-axis respectively. Let 2 XOP
ii. 4 or more are male? A and
and 20OX2.Then
=Bsothe
thatcoordinates
POQ--(A+B). Also,let OP
=
1 OQ= and of P Qare
Solution ( cos A, rIsin A) and (ra cos (T- B), T2 sin (7 B))
- =
(-n cos B, 71 sin X- B A
Given p =60% 100 B).
So OP ( i cos A, ri sin A)
1-p-1--j &0Q(-ncos B, ra sin B)
n=5
Now, OP.O0 = (n cos A, n sin A). (-rncos B, ra sin B)
) P3) = ?
We have, P() = «Cp q n - * -12 COS A cos B+ Ti T2 sin Asin B
-T (cos A cos B- sin A sin B)
Since -(A +B) is the angle between OP and 00 so
ii. P(4 or more are male) = P ( r 2 4

1053 cos T-(A+ B)] =


13125* 1 3125 OPI |0Q
-P(4)+ P(5) =SC

or, -cos (A+B) = cos AcosB-sin AsinB)


9. Show that: e -.
n 1 = e -1
cos
(AB= cos
Acos B- sin Asin B
n=1
State
11.
Rolle's theorem. Interpret it geometrically. Verify Rolle's theorem for the function.
Solution ) = xla 1) in [0, 1
-

The nth term of the series is


Also, find the point on the curve where the tangent is parallel to thex-ris.
61
n 41
t(n+1)!(n+ 1 n+ nn+1) nn"(n +1! Solution
First Part

n-1 n + 1 Statement of Rolle's theorem:


Ifa function f(r) is
(a) continuous in [a, b]
(b) differentiable in (a, b)
Now by addition, the sum of given series
( fa)- fb)
then there exists at least one point c e (a, b) such that f(c) =0
Second Part
- e-e+1+e-2-e-1
-*)Gdr ee-1)+ (e-2) the
Ifthealltangent
conditions of Rolle's theorem
is parallel to x-axis.
are satisfied then there is at least one point c efa, b) where

10. Define scalar product of two vectors. Find the geometrical interpretation of scalar product or
vectors. Prove vectorically that

cos(A+ B)= cos


Acos B sin Asin B
Solution ta) (b)
jt(a) (6 t(a)
Scalar produet of two vectors: Scalar product of two vectors a and b, denoted a b, is
by a.b al|b| cos 0 ab cos 6where=
by .

den
Geometrical Interpretation is the angle between the two vectors.
Last Part
Let OA a and OB b . Let ZAOB= 6. Draw BE Here, f(x)= x(x -1)2=
*-2x+x
OB. perpendicular to OA and AD perpena Since f(x) is a polynomial, it is continuous in [0, 1].
Now, Px) 3x-4x+1 which exists for all x e(0, 1)
a.b a |b|cos So, f(x) is differentiable in (0, 1)
a b cos 6 Also, f(0)=0 andf(1)= 0
= (OA) (OB) cos 0 f(0)= f(0)
All the conditions of Rolle's theorem are satisfied. So, there exists a number c e (0, 1) such that
=
(OA) (OB cos 0)
- (OA) (OE) Pc)=0
or, 3c2-4c+ 1 0
(magnitude of a) (projection of b on a) or, (c- 1) (ac -1) =0
Mathematics-ll
of Basic 33
32 Asmita's NEB Solution SOLUTIONS To MoDEL QUESTIONS

C1, Solution
Initial velocity (u) = 19.6 m/s

Weight (h) 294


(0.1) butc-1e(0.1)
=
m

Clearly, ce verified.
Time (t) =?
Hence, Rolle's theorem is , Taking upward direction as positive, we have,

-) to xaxis. or,
-h- ut-gt

-294 19.6t-5x 9.8 x t2


the tangentis parallel or, 4.9-19.6t-294 0
or, ?-4t60 = 0
OR or, (t+6) (-10) = 0
cosx.
derivative of In
from first principle the
Find Either ts-6 (notpossible)
Solution or
t=1
Let fr)= n cosr lim f(xh)-f)
t 10 sec

We have, f(a) h0 h c. A body of mass 50kg fallíng from a certain height is brought to rest after striking the ground
with a speed of 5 m/s. If the resistance force of the ground is 500N, find the duration of thhe
d (n cos) lim ncos (1+h)-Inaos ...) contract.
dr h+0 h
Put cos1 Fy and cos1 (x+ h)= y+ k Solucion Mass (m) = 50kg
and x+h= cos (y + k)
then r = Cosy
Initial velocity (u) = 5 m/s
h cos
(y+k)-cosy Final velocity (v) = 0
Also, as h>0,k>0 Resistance force (F) = -500 N
From () Duration of contact (t) =?
We have,
lim n (y+)-nyim Im(y-u
d(ncos h>0 h h0 F t

-500- 50(0-5)
0.5 sec
2sin 13. a. P and Q are two like parallel forces acting at A and B. Show that if they interchange

positions, the point of application of the resultant is displaced by a distanee p+AB. 4


siny.-1
siny-11 cos-x1-co y cos-lrV1 -
Solution

--9- Let P and Qbe two like parallel foroes acting at the points A and B, so their resultant P+Q acts at
the point C as shown in the figure.
Since the forces are like parallel, so
P+Q
Group B BC AC AB
12 a. Forces equal to 7P, 5P and BP acting on a particle are in betwee ACp+ AB .)
the latter pair of forces equilibrium. Find the ange P+Q
If forces P and Q be interchanged in their positions, let its resultant act at point D as showrm in the
Solution figure below. Then,
Let a be the angle between the forces 5P and 8P. Since the ilibrium
so the resultant of 5P
forces 5P, 8P and 7P are in equ
and bP 15 equal and opposite to the force 7P. So,
(7P)2 (5P2 + (8P)2 * 2:5P-8Pcos a
or, 49P2= 25P2 + 64P2 + 80P2 cos a
5P AB
or, 40P2 = 80P? cos a )
AD P+Q P +Q

or, cos a= -80P2


40 P2 =cos 120 From () and (i)
a120° 8P AD-AC P+Q P+Q
b. A body is projected vertically
upwards with a velocity of 19.6 m/s. How long wil Hence, the line of action of the resultant is displaced along AB through a distancep+ AB where P>Q
a point 294m below,the point of projection? (g98 19.6 m/s. How it take
m/s)
Mathematcs-l SoLUTIONS To MODEL QUESTIONS 35
4 ASmita's NEB Solution
of Basic

OR parallel
to the
sides of an equilateral triangle
Greatest height (H) =
sin
2
d and
and
direction

point in
Forces 1N, 2N and 3N act at a
taken in order. Find their
resultant. Horizontal range (R) =u sin 2a
CA
CA of an equilateral triangle of sidea
equilateral
BC and of an u2sin a
Solution and 3N act along
the sides AB,
have
to BC, we
Let the forces 1N,the2Nforces along and perpendicular
- 4
W Tesolving (a) L.H.S. sin 24
X =2 cos 0 +3 cos 120+1 cos 240
1N 3N 2u sin2 a
u2 x 2 sin a cos a
sin &sin--RHS
cosa u cos a
-2-13 ()2-5-*0 u2 sin2a
Y 2 sin 0° +3 sin 120° sin 240 C
--
3 2N
(b) L.H.S
2x03( (u2:2 sin a cos a
Let R be the resultant.Then, A u cosa
R=y+Y-y*+ (/3:=y3
Let e be the angle made by the resultant with BC.
Then,
(2usina 4u sin2a
tan tan 90°
4u sin?a 8Sina
90°

b. Prove that the sum of the kinetic and potential energies of a freely falling
body remains constant 31.H-RHS
throughout the motion. OR
A cat seeing a mouse at a distance of 15m before it, starts from rest with an acceleration of 2 m/s2
Solution at the point A which is at a height
h from the
Suppose that a body of mass m is initially of the body at any and pursues it. If the mouse be moving uniformly with a velocity of 14 m/s, find when and
falling from A and C be the positiorn
ground B. Let the body start
where the cat will catch the mouse. 6|
h-x
instantsuch that AC=. Then BC
=

Solution in t Then the


At the point A
Let t be the time taken by the cat to catch the mouse and let the mouse run x m sec.

E. =0 h-x distance covered by the cat to catch the mouse =(15+ x) m.


Since the m o u s e r u n s w i t h u n i f o r m velocity u = 1 4 m / s , using s = ut, we get

P.E.=mgh x=14t )
KE+ PE=0+mgh= mgh
At the pointC For cat, using s = u t + a t , we get
Let vi be the initial velocity of the body at the point C. Then. v? =2gx.

KE -mv-m 2g= mgr 15+x=0+5*2xe


P.E- mgh-) or, P=15+x ...i)
. KE. + P.E = mg+ mg{h - x) mgh From (i) and (i)
At the point B t215 + 14t
Let v be the velocity of the body when it reaches the ground. Then, or-14t - 15 0
or, (t+ 1) (t-15) =0
v2= 2gh
Eithert=-1 (impossible)
K.E-mv-im 2gh =mgh ort= 15sec
Now, from (4)
P.E 0 x= 14 x 15= 210 m
K.E + P.E = mgh + 0= mgh
in 15 sec after running a distance of 210 m.
So, the cat will catch the mouse
The sum of K.E and P.E of
the freely falling body at any instant is samee Prove that the
i.e. mgh) and
hence it is constant. 15. Define the moment of a point and interpret its geometrical meaning.
force about a

moments of two intersecting forces about any point


in their plane is equal
algebraic of the
sum
horizontal and the vertical [61
14. The components the initial of U and to the moment of their resultant about the same point.
respectively. If R be the nornzonal range and H, the greatest velocity of proj
are
a
height attained, prove that
i Soluion
i. Moment
of the
The moment of a force about a given point is the product
of the force and the perpendicular distance of the point
Solution magnitude
from the line of action of the force.
Let u be the initial velocity of projectile and a of the distance of the
Horizontal component (U) = u cos a angle of projection. Then, Thus if F be a force and p be the perpendicular
O is F x OM Fxp =

point O from AB then the moment of F about


Vertical component (V)=u sin a A

Also, we have
Mathematics-l
ofBasik SoLUTIONS TO MODEL QUESTIONS 37
36 Asmita's NEB Solution
Moment of a Foree
of the
ometric Meaning
represented in magnitude
a Total time '
force F be
Let a AB and O point. Join be a
aurection by OD perpendicular
to AD
1
and OB. e t us draw
or AB produced. or, 8x + 5y S 200.
the force F
about O is Hence, the mathematical model of given próblem is
m o m e n t of
Ihe AB x OD 2 (area of AOAB)
= maximize Z (xy) = x +y subject to
F * OD =
F
the force
the moment of 2x + 5y 100
numericaly. Hence, numerically by
is represented 8x + 5y S 200
about O whose base
is the fig (i)
of a triangle fig x, y20
times the area the force and vertex is the
ine representing
b. Convert the decimal number 2011 into octal form.
which moment is taken.
point about line of action of O a at
Last Part to P to meet th Solution
Let us draw OC parallel NOW, complete the narstt Remainder
Let O be a given point. P in magnutude.
AB represent 8 2011
in magnitude and
presentQ 8 251
ABDC. P and
the resultant R of O-
Then AD represents 31
8 3
Join OA and OB.
BAC in fig R
The point O may lie outside the 20111o 37338
The moment of
() and inside BAC in fig (i). in both c. Is the following equations diagonally dominant
P about O is 2 AOAB and is positive
figures. 12x+3y-5z =1 x+ 5y +3z = 28
3x+7y+13z =1? 21
The moment of Q about O is 2A0AC which is fig ) fig (i) Solution
positive in fig (i) and negative in fig (i) Here, |12 > 13|+ |-5] -8
In fig ) and|5|> |1| 13] =4 +

The sum of moments of Pand Q about O |13>13+ 17| =11


2AOAB+240AC Hence, the given system of equations is diagonally dominant.

2AADB+2AOAC (AOAB=AADB) 17. a. Using Gauss elimination method, solve the following system of equations:
2AOAD x+ 3y - z = - 2 3x+ 2y z=3 -6x-4y-2z 18 41
moment of R about O
Solution
In fig (i), X+3y -z=-2 .i)
The moment of P andQ about O
3x+ 2y-z=3 ii)
240AB-240AC -6x-4y 22= 18.ii)
Multiplying equation () by 3 and then subtracting from (i)
=
2AADB+ 240AC (: AOAB= AADB)
2AOAD = moment of R about O.
3x*4y
3x +9y 3z
==-6-6
Group C -7y +2z =9 (iv)
his
16. a. Ifa man
rides carat
25 km/hr, he has to spend Rs. 2
faster speed of 40 km/hr, the petrol cost increases to Rs. 5
per km on petrol. If he rides ita Again, multiplying equation () by 6 and adding with equation (ii)
-6x -4y -2z 18
per km. He has Rs. 100 to 5P
petrol and wishes to find the maximum distance he can travel within one hour. rormul 6x +18y -6z=-12
the above problem as a linear 14y-8z =6 (v)
programming problem. Multiplying equation (iv) by 2 and adding with equation (v)
Solution 14y - 8Z

Let x km and y km be the distances covered at 25 -14y t 4z 18


distance covered= (x+ y) km
km/hr and 40 km/hr respectively, - 42 24
When the speed is 25
(vi)
km/hr, expenditure
Hence, for x km, expenditure on petrol = Rs 2xpetrol= Rs 2 per km
on Now, we have the following three equations
+3y- z -2
When the speed is 40 -7y +2z =9 (iv)
km/hr, expenditure
Hence, fory km, expenditure on petrol Rs 5
on
petrol Rs 5 per km =
=

-42 = 24
(Vi)
y
Therefore, total expenditure Rs(2x+ 5y) = From equation (vi), we have z =-6.
But amount to spend on petrol Rs 100 =
Using z-6, (iv), we have
inequation
2 x + 5y s 100
-7y +2 x (-6) = 9
Again, time taken for x km is
or-7y = 21
y-3
Again, using y =
-3, and z =
-6 in equation (i), we have
Time taken for y km is X+3(-3) -(-6)
=
-2

1
Mathematics-l SoLUTIONS To MODEL QUESTIONS
38 NEB Solution of Basic
Smita's

or, *= 1
Solution
The required solution is: Given, y f ) 1 +
x*1,y=3, 2 6.
a 0, b 1, n =4
od:
OR Gauss-seidal
methoa:

Solve the following equations using We have, h- n


=025
21 =8; 3+7a-5 The five points to be considered are xo = 0, x1 = 0.25, x = 0.5, n = 0 . 7 5 , x4 - 1 .

Solution
Given equations are:

19 Thevalues ofthe function atthese points aretabulated below:


ndpoint X0 0 0.25 z 0.5 »=0.75 =1
x53) y-f)1+
0.9412 0.8 0.64 0.5

1 iteration:
Using Simpson's rule, we have
n-60)-4
6 3 x 4) -
-2429
(14d o+ 4y+2ya+ 4ys+yl3l+4x09412+2x0.8+4x0.64 0.5]
2 iteration

-8-242)-2786 -9.4254)-0.7854
3 x2786)- -1.908 18. Using Simplex method, maximize Z = 5x1 +7xasubjectto: 2r1+3 13, 3x +22S12, xu 2 0 . [6]

3nd iteration: Solution


Introducing the non-negative slack variables xs and xu, then given LPP can be written as
8-1.908) =3.046 2x1 +3x2 + xs= 13
3x1 + 2x2 + X4 = 12

x 63x 3.046)-2.020 Z= 5x1+ 7x2


2 x 1 + 3x2 t x3+0 x + 0 = 1 3
4h iteration:
3x1 +2x2+0 xat x4 +0 Z 1 2
8-2020) 2.990 -5x1-7x2+0 xst 0 xu+ Z-0
Initial tableau
=6+3 *
2990) -1.996 3 RHS
5h iteration 3 13

-8-1.996) =3.002
- 5 3 x 3.002) -2.001 Here,-7 is the most negative entry in the last row, so second column is the pivot column.
6th iteration:
Since4.33,-6 and 4.33 <6, so 3 is the pivot element
-8-2.001)= 3.000 Applying R j R
53x 3.000)--2 M
X3 R.HS
7h iteration: 2/3 1/3 0 13/3
12
--2)=3 5 0
Again, applying Ra > Ra - 2R1 and Rs> Ri+ 7R
X 53 3) =-2 XI A2 X1 R.H.S
From 6h and 7h iterations, the values of xi and xz are equal. 2/3 1 1/3 13/3
3 , x=-2 5/2 -2/3 10/3
b. Evaluate the following integral using Simpson's rule: -1/3 7/3 91/3

This is not the optimal solution as the last row contains negative entry.

S taking4 equal intervals (i.e. n- 4). Again, first column is the pivot column. Since 2 / 3 13/3 10/3
6.5, 5/3 2 and 2 < 6.5, so 5/3 is the pivot

element.
of Basic
Mathematics
a0 ASmita's NEB Solution

Applying Ra-R

2/3
1/3
A

3/5
0
0
R.H.S
13/3
2
UNIT 1
-1/3
0
7/3
-2/5 1 91/3
PERMUTATION AND
Applying R -Rr-R and Ri R+R

3/5
/5
R.HS
COMBINATION
-2/5 3/5 A. PERMUTATION
7/3 1/5
entry.
31
2 MARKS aUESTIONS
52063 Q.No.23
contains all non-negative How many numbers of three different
theoptimal solution as the
last row digits less
nis is 3. than 500 can be formed from the
integers 1, 2, 3,
value of Z 31 at xi 2 and xa
=

So, max.
= =

Using bisection method


.2058 Q.No.2 4,5,6? 2
19. Show that the equation
- posifiveroot.
f{x)= r 18 =0 has only oneinterval hod, find In a certain election, there are three candidates sOLUTION
in the (2, 3). for president, five for secretary and only two for
to 3 places of decimal
the positive root correct the treasurer. Find in how many ways the Siven integers are 1,2,3,4, 5, 6
Solution
Here, f(x) =x' - 18
election may turn out. 2
Since the numbers should be made from three
different digits less than 500, the hundred's
Since f{x) change
has one in sigm, so f{x) has only one pOsiive
root.
SOLUTION place has either 1 or 2or 3 or 4. So, there are 4
There 3 choices for president,
Given, a =2,b=3 are 5 for choices for hundred's place.
f(2)- 2- 18=-10 secretary and 2 for treasurer.
When the hundred's
f(3) = 3 - 1 8 9
Since they are all independent of one another, place has been filled up,
there will be 5 digits left. So, there are 5
Since f(2 and f8) have opposite signs,soa root lies between 2 and 3. the total number of ways in which the election choices for ten's place. In the same way, there
b a+b fla) f(b) f(m) may turn out =3 x5 x2=30 are 4 choices for unit's
2 place.
3 2.5 -10 -2.375 2.2059 Q.No.1E By basic principle of counting, total number of
275 375 9 2.79687 4 80
ways= 4 x5
3 x =
How many permutations are there of the letters
2.5 2.75 2.625 -2.375 2.79687 0.08789
-1.17358
of theword'mathematies' taken all together? 12] 2065 Q.No2a
25 2.625 2.5625 2.375 0.08789
2.5625 2.625 2.59375 -1.17358 0.08789 -0.55045 sOLUTION How many numbers are there between 100 and
259375 2.625 2.60938 -0.55045 0.08789 -0.23319 There are 11 letters in the word 1000 such that every digit is either 2 or 9?
260938 2.625 2.61719 -0.23319 0.08789 -0.07308 'MATHEMATICS' in which 'M comes twice, | sOLUTION
261719 2625 262110 -0.07308 0.08789 -0.00729 'A' comes twice, T' comes twice and the rest
2.61719 2.62110 2.61915
We know that the numbers between 100 and
0.07308 0.00729 -0.03287 are single. So
261915 2.621 2.62013 -0.03287 1000 are 3 digit numbers. Since each digit is
2.62013 262110 2.62062 0.01270
0.00729
0.00729
-0.01270
-0.00260
n=11, p 2,q=2,r=2 either 2 or 9, so there are 2 choices for unit's, 2
Total number of
2.62062 2.62110 2.62086 -0.00260 0.00729 0.00244
arrangementsS choices for ten's and 2 choices for hundred's
2.62062 2.62086 n!
-0.00260 0.00244 4989600
place
Since a and b have same value to three
places of decimal, the required root is 2.620. p! q! r! 2! 21 2 By basic prinaple of counting, total number of
OR 2060 O.No. 2 arrangements = 2 x 2 x 2 8

Find the numbers of permutation of the letters Note: Formed numbers are 222, 229, 292, 299,
Use Newton-Raphson method to find the
correct to three places of decimals.
positive root of z +3r- 5=0 lving between1 an ot the word 'MATHEMATICS'. [21 922, 929, 992, 999
Solution
SOLUTION 2066 CQNo.2
Let, f(x) = r'+3x - 5
Please refer to 2059 QNo. 1b
How many four digits odd numbers be can
f(x) =3x2+3 . 2061 Q.No. 2a formed using the digits 0, 1, 2, 3, 4, 5 no digit
Let us make an initial guess xo = 1 In how many ways can 6 different beads be being repeated? 21
By Newton-Raphson's method, we have strung on a necklace? [21 sOLUTION
Xn In P(xn) SOLUTION Given digits are 0, 1, 23, 4,5
Here, n= 6 For odd numbers there are 3 choices for unit's
The clockwise and place either 1 or 3 or 5. Also for 4 digit
1 X0 Fa2-514 anticlockwise arrangements
number starting digit cannot be 0. So, there
are same
0.1909 -PO14-R11811 in the necklace.
are 4 choices for thousand's place. And for
2-Fa)18)7.185 1.1545 L.1542
So, total number of
arrangements 2(n -1)! hundred's place there are 4 and tor ten's place
Comparing the values Is and
of
find the X-Fa1.1545 6.9986
digits first
X4 we a
cimal are sat
there are 3 choices.
So, by basic principle of counting, total
Hence, required root is 1.154. on
three places or 6-1y-6x4x3%2x1)- number of ways = 4 x 4x3 x3= 144
Mathematics-ll
PERMUTATION AND COMBINATION Unit 1 43
of Basic
42 NEB Solution 1
12.2070 (OldO.No. of three different digits |
ASmita's

many
numbers sOLUTION 20 2073 SetCONo.1
8. 20677OANo.2 of the
word
How
can be
rormed from thhe
integers 1,2 Here, n= 7
How many different numbers of five digits can
ways
letters
that all
the than 500 The required number of ways 7 students can
Tny
OUS c a n
be arranged
so
14
45,6,77

be seated in a circle be formed with the digits 0, 1, 2,3, 4 ?


vowels are always together? SOLUTION = (n -1)! = (7-1)! = 6! = 6 x 5 x 4 x 3 x 2 x 1 SOLUTION
Given integers
are
1, 2,3, 4, 5, 6,7, 720 ways Ihe number of 5 digits numbers formed from
SOLUTION formed numbers should
word is 'PRECARIOUS Since the given digits = P(55) = 5=5x4 * 3 x 2 x 1 - 120
Given always together
consider 500, so for hundred's place there may 17 2072Set D.No: 1
unce the
vowels are
Then
for But, some of these numbers have 0 in the first
O, U as a single letter.comes 34. So there are 4 choices In how many ways the letters of the word place, which are not 5-digit significant
the vowels E, A,6 I;letters or or
in which 'R' fill up the ten's place ELEMENT can be arranged so that vowels are
there wil be place. To there
choices and for unitS place there are 5 a always together?
numbers. So, we fix 0 in first place, then
twice. So the number of arrangements 2 remaining4 digits in 4 places can be arranged
in P(44)
6 6x5x4*3*E =360 Then by basic principle of counting, tal SOLUTION ways= 4! 4x3 x 2x1-
24 ways
2 2 number of three difterent digit The word 'ELEMENT consists of 3 vowels E, These are the numbers with 0 in first place.
Also, the 5 vowels can
be arranged among
that 500 that can be formed- 4 x 6x5= 1)
numbers E, E. Consider these 3 vowels as a single letter. Hence, the required number of 5-digit
themselves in 5! ways Then we have to arrange 5 different letters significant numbers =120 -24 96.
=5 x 4 x3 x2x1=120 ways
= 360 * 120 13. 2070Supp.QNo. 1a L,M, N, T, (E,E,E).
Required number of ways in which all the
21. 2073 Supp C.No. 1C
Required number of arrangements Six children are to be seated on a bench L
In how
= 43,200
vowels are always together many ways the letters of the word
many arrangements are possible
if the vams HEXAGON can be arranged so that vowels
left end of the bench. =5! =5x4 x3x2x1=120 are
2068 O.No. 2 child sits at the always together? [2
many license plates consisting
of
SOLUTION 18.2072 Set EQ.No. 1a SOLUTION
How be made out of given Fix youngest child at the left end of the ban Find the number of ways in which 4 men and 3
different digits can

women can be seated in a row having seven


Since the els are always together, so
integers 3,4,5,6,7? then remaining 5 chuldren can be arrangei consider (E, A,O) as a single letter. Then, there
seats so that the men and the women must will be 5 letters namely (E,A,O), H, X, G, N.
sOLUTION P5,5) ways alternate. 2 These letters can be arranged in 5! ways.
Given integers are 3, 4,5,6,7 = 5! ways
n= 5 SOLUTION Again, 3 vowels among themselves can be
& r-3
5x4x3x2x1=120 ways Here, number of men=4 arranged in 3! ways.
Total number of license plates that can be 14. 2071 Old Q.No. 2 a Number of women = 3 Total no. of arrangements = 5! x 3!
Seats = 5x4x3x2x1x3x2x2x1-720
5x4x32, 60
How many permutations are there of the lett
made P, 3) =(5-3 of the word "SAARC".
22. 2074 SetBO.No 1
M WM w M W M
10. 2069 (Set A) O.No.1a SOLUTION If men and sit
In how many ways the letters of the word
womenalternatively, then
should be in odd places (i.e. 1s, 3rd, 5th andmenCoMPLETE
In how many ways can four boys and three girls There are 5 letters in the word SAARC. A can be arranged so that the repeated
be seated in a row containing seven seats if they there are two A's and the rest are single 7m) | letters are always together?
and women should be in even place (i.e. 2nd,
may sit any where? 12 n=5, p=2 4th and 6th). sOLUTION
sOLUTION 4 m e n in 4 seats There are 8 letters in the word COMPLETE in
can be arranged in P(44)
Number of boys = 4 Total number of arrangements Pi 2 which E repeats 2 times. So. consider 2 Es as a
ways
Number of girls = 3 single letter, then there are 7 letters C,o,M, P,
If they may sit anywhere, then there is no
5x4x3E =
60
3 women in

ways
3 seats can be arranged in P(3,3)
L, T (E, E). The total no. of arrangement in
Total number of which 2 E's always come together
difference between the position of girls and arrangements
boys 2071 Supp. Q.No. 1a P(44) x P(3,3) .P(7,7)=7! =7x 6x 5x 4x3x2*1 =5040
n = 4 +3 7, r =7 n how many ways can eight different col0 4 ! x 3!
~P(nn) =nl) 23.2075 SotAQ.No. 1a
heads be made into a bracelet? =s 44xx3x2x1
3x2x x3x2x1 144
1
In how many ways can eight people be seated in
Total number ofarrangements P(7,7) ways| SOLUTION 19. 2072 Supp Q.No.1a a row of eight seats so that two particular
7! ways Here, number of beads = 8
Persons are always together? [21
Cire How many numbers between 3000 and 4000 can
7x6x5 x2x1=5040ways
x 4 x3 8 beads can be arranged on
a be formed with the 21
digits 2, 3, 4, 5, 6,77 SOLUTION

11. 2070 Set DQ.No.12 (8-1)! 7! ways SOLUTION Suppose two particular persons who are
n this case, clockwise and
ticdockw
Given digits are 2, 3, 4, 5, 6 and 7. Since the always together as one ànd their two seats as
In how many ways can the letters of the word one seat. Then there are 7 persons and 7 seats.
"ELEMENT" be arranged? arrangements are same. numbers lie between 3000 and 4000, there
(21 must be 3 at first place (thousand's place) and
Now, 7 persons in 7 seats can be arranged in
SOLUTION Total number of arrangements remaining 3 places be filled by
can the P(7,7) ways.
There are 7 letters in the word 'ELEMENT remaining 5 digits 2, 4, 5, 6 and 7. So, fix 3 in = 7! ways.

Also there are 3 E's and the rest are single.


n 7,p 3
7x6x5x4x3x2*
2
2520 the thousand's
place, then the number o
Also, 2 persons who are always together can
be arranged among themselves in 21 ways.
arrangements of 5 digits in 3 places can be
done in P(5, 3) ways
Total number of arrangements 16. 2072 Set GQ.No. 1a Hence,
Total No. of arrangements = 7! x 2!
In how many ways can 7 students be sea
7x6x5x4x3 840 circle? = 7x 6x 5 x4 x 3x2x1 x 2*1=10080
3!
Mathematics-ll

PERMUTATION AND COMBINATION Unit 1 45


of Basic
44 Asmita's NEB
Solution
6.2061 Q.No.7
can the letters contain
many ways the are n choices to First find the arrangements which
4 MARKS QUESTIONSs In how be arranged?
How NO up by n objects. Now, there we
of the has consider 2 C's as one. Now
"MONDAY"
begin with
M2Of fill up the first place. When first place 2 Cs together. For,
arrangements do
not l0w U's and
24. 2057 QNo.7 wora
and don't
end with ya been filled up, there will be left n
-1 objectsto there are 7 letters in which there are 2
the letters of the with M ill up the second place. So, there are (n -1) 2 L's.
n how many
m ways
can
that no. two R's come begin choices to fill up the second place. Similarly, The number of arrangements in which 2 Cs
ARRANGE be arranged so 14sOLUTION in the word
Total number of
leters
'MONDAY there are (n -2) choices to fill up
the third
together. 6!
arealwaystogether22-1260
=

Number of arrangements place and so on. Finally, to fill up the rh place,


SOLUTION 6x5x4 x3
x2x 1 720 = (r-1) n - r + 1 choices. which 2
Given word is
'ARRANGE =

arrangements t
there are n -
=

Required number of arrangements in


(n) = 7 To find the number of Then by basic principle of counting, total C's do not come together
Total number of letters with M, firstly we find
not begin number of ways = n (n -1). (n- 2)... (n - r+1) - 5040-1260 3780
that begin with h
Number of A's (p)=2 of arrangements For t
Number ofR's (g)=2
fix M at first place, then remaining 5 let P(nr)n (n-1)(n-2)..(n-r+ 1) 35 2070(Old)Q.No.7a
And the rest are single arranged in P (5, 5) ways
can be
30.2069 (Set A)Q.No..5a In how many ways can the letter of the word
= 5x4x3 x2x1 =120 ways the letters of the world be arranged? How many of these
Total number of arrangements
In how many ways can
Sunday
Required number
of arrangements that dn "ARRANGE" be arranged so that no two R is arrangements do not begin with S? How many
720 120 600 come together? begin with S and do not end with a? 41
begin with 'M'
= -

7x6 1260 first place and "Y' at last # Please refer to 2057 Q.No 7b SOLUTION
2x1 Again, fix 'M' at
in which be
remaining 4 letters
can There are 6 letters in the word 'Sunday'. Total
To find the number of arrangements
we first
find the =4 x x 3 x1
aranged1.
2
2069 (Set A Old Q.No. 75
24 number of arrangements
no two "R' come together, P(4, 4)ways the letters of the word n how many ways can
which two R's Required number of arrangements that bes P(6, 6) ways=6! ways
=

of arrangements in arranged? How many of these


number
consider two R's as a
'M'and do not end with 'Y' 120-24="MONDAY"
with =
arrangements do not begin with M? How many
be = 6x5x 4x3*2x1
Come together. For this,
single letter, then the number of letters will be 2062 QNo.7.b
with M and does net end with Y? begins 720 ways
To find the number of arrangements which do
Please refer to 2061 Q.No. 7b
6.
in which two In now many ways
can *
Art students and not begin with S, we first find the number of
Total number of arrangements Science students be arranged alternately al 32. 2069 (Set B)QNo.5a
arrangements that begin with S. For, fix 'S at
R's come together round table? In how many ways can the letters of the word first place, then remaining 5 letters can be
654*3*2
2 -360o SOLUTION
4 Art students at a round table an
"MONDAY" be arranged? How many of these
arrangements do not begin with M? How many
arranged in
P(5,5) ways -5! ways=5x4x3x2x1
arrangements in
number of which 3! ways - 3 x 2x1 begin with M and do not end with N? 14
arranged in (4-1)!
=

Required
Please refer to 2061 QNo 7b
120 ways
no two 'R' come together =6 ways The number of arrangements that do not
1260 360 900 33. 2069 Old (Set B) Q.No. 7b begin with 'S

26 2059C.No.7 In how many ways can ten people be seated in a 720-120= 600
round table if two people insists on sitting next Again, fix S at first place and 'a' at last, then
Prove that the total number of permutations of a
to each other? 41 remaining 4 letters can be arranged in
set of n objects taken r at a time is given by
P(44) ways=4! ways =4 x3x2x1 =24 ways
SOLUTION
14 Total number of arrangements that begin with
Lonsider two people who insist on sitting S and do not end with a = 120 2 4 9 6
Since Art and Science students must next to each other as one, then there will be 9
sOLUTIOL people. Then, 9 people in a round table can be 6 207K Sot CO.No.5
The number of permutations of a set of n
arranged alternatively, so 4 Science arrangedin (9-1)! ways = 8! ways In how many wàys can the letters of the word
objects taken r at a time is equivalent to the Students in 4 seats can be arranged in r
Also two people
number of ways in which r places can be filled ways= 4! ways = 4 x 3 x 2x1 =24ways sitting next to each other"TUESDAY" be arranged? How many of these
among themselves can be arranged in P(2, 2) arrangements do not begin with 17 Hlow many
up by n objects. Now, there are n choices to 6 x 241 =

Total number of arrangements begin with T and do not end with Y?


fill up the first place. When first place has r ways2 ways
been filled up, there will be left n-1 objects to 28. 2064 Q.No.7C Total number of
arrangements SOLUTION
There are 7 letters in the word TUESDAY'.
fill up the there are (n
second place. So, -1)| Show that
the number of ways ina
which
81 x21 8x7x6x5x 4x3x2x1*2x1 So total no. o f arrangements= P(7,7) = 7!
80640
choices to fill up the second place. Similarly, letters of the word "arrange" can bea
7x6 x5 x 4x3x2x1 =5040
there are (n -2) choices to fill up the third thattwo rs do not come together is 34. 2070 Set CQ.No. 5a To find the arrangements that do not begin
place and so on. Finally, to fill up the rh place, Please refer to 2057 Q.No. 7b In how many ways can the letters of the word, with T', we find the arrangements that begin
there are n - (r-1) = n - r + 1 choices.
CALCULUS' be arranged so that the two L's do |
Then by basic principle of 7 29. 2065 Q.No ofn o t come together? 14
with T'. For, tix "T' at first place.
counting, total Frove that the total no. o f p e r m u t atti o n s SOLUTION
Then, remaining 6 letters can be arranged in
number of ways (n -1).
=
n
(n-2)
. n-r+1)|of n objects taken r time is
by P(6, 6) ways= 6! =6 x 5x 4x3x2x1= 720
P(nr)n (n -1)(n -2)..(n -r+ 1) at a
gV There are 8 letters in the word CALCULUS. Number of arrangements that do not begin
nn-1) (n-2).. (n- r +1), n2. Also there are 2 C's, 2 U's and 2 L's and the
nn-1) (n-2)..n-r)n-r.. 321 with 'T' = 5040 - 720= 4320
(n-r)321 SOLUTION of a
set d rest are different.
The number of permutations ent Total number of arrangements 2121 2 5040
( n - n 2r objects taken r at a time is equiv bel
can
es
humber of ways in which r
Mathematics-ll

of Basic of 4 different digis PERMUTATION AND COMBINATIOON Unit 1


Smita's NEB
Solution
The
number
=
P(n, umbers h
last place. be formed
gain,
fix "T at first place
and Y' at
be arrangea
n
can

C(12,3) ways 9131


12123 x11 X10 220
*2x1 20 sOLUTION
hen remaining
5 letters can
x3x2x1=120 ways P$,4)- Selection of women
Here, n=10, r =7
=5x 4 numbers which are divisih
F,5) ways =5!arrangements begin
witn
For the
have 5 inunit's place. So,
be by 2 women out of 8 can be selected in
Required number of selections
The number of 600 must fix in u
end with Y= 720-120 remaining places C(10,7)=317 3x2*112
and do not then
by
digits can be filled up by P(4,3) waysmaininy
place, C(8,2) ways 61 2 22
37. 2071 Set DQ.No.5 of the word
Total number of committees 220 x 28 6160 49. 2069 (Sot B) O.No.1
can the letter
many ways all vowels From 10
nC Ohow
M P U T E R " be aranged
so that i) 45. 2062 Q.No.2 persons, in how many ways can
relative positions or
selection of 4 be made if two
together? i) the The number of 4 digit numbers which a From 10 persons, in
how many ways can a particular personsS
always
are vowels and consonants are notchanged?
24.
are
committee of 4 be made when one particular
are always excluded. (2
divisible by 5
=

the Please refer to 2066 QNo. 2a


Hence, the number of 4 different digits umbers always included?
person is 121
SOLUTION
There are 8 letters in the
word 'COMPUTER

which are not


divisible by 5 sOLUTION S0 2069 01d Sot B) Q.No. 1
the word 'COMPUTER Total number of players = 10 How
are 3vowels in
= t o t a l numbers - numbers which
There letters Te many different sums of money can be
letter. Then 6 Number of players to be selected = 4 made from 4 coins of different
Consider 3 vowels as one
can be divisible by5 denominations?
U, E)
namely C M, P, T, R, (O, = 120-24 96 When one particular player is always (21
arranged in P(6, 6) ways
6! ways =
included, then we have to select 4 -1=3 SOLUTION
Q.No.5a
Also, 3 vowels among themselves can De 42. 2074 Supp In how many ways can the letters of the w
players out of10-1 =9 Required number of different sums that can
=
3! ways Total number of selections be made
arranged in P(3, 3) ways
6! 3! ARRANGE" be arranged so that no two
Total number of arrangements
= x =

C(4,1)+C(4, 2)+ C(4, 3)+C(4, 4)


6x5 x4 x3 x2x1x3 x2x1=4320ways come together? -Ce.3)-3-3*22-84 4 L, 4
( ) There are 3 vowels and 5 consonants in ne Please refer to 2057 Q.N.7b 3! 1! 22*13*0 44+6+4+1 15
46. 2064 .No.2a
word 'COMPUTER". 51. 2070 Set CQ.No.1a
arranged in | 43. 2075 Set B Q.No. 5a
A person has got 12 acquaintances of whom 8 are
The3 vowels in 3 places can be How many words can be formed from theletrelatives. In how many ways can he invite 7 rom 10 persons, in how many ways can a
P(33) ways =3! that 5 of them selection of 4 be made when two
of the word ENGLISH'? How many of these 8uests so may be relatives? 21 particular
And 5 consonants in 5 places can be arranged | SoLUTION persons are always included?
not begin with E? How many of these be
ways= 5!
in P(5,5) with E and do not end with H? Total number of acquaintances -12 SOLUTION
T o t a l number of arrangements= 3! x 5!
Number of relatives = 8 When two particular persorns are always
=3x2x1x 5x4x3x2x1-720 words SOLUTION
There are 7 letters in the work ENGLISH
Number of non-relatives = 12-8 = 4 included, then we have to select 4 2 2
persons out of 10-2=8.
93 2017/2SuppONo.5 So, the letters can be arranged in 7! ways Relatives (8) Non-relatives (4)|Selection_ Required number of selections = C(8, 2)
Prove that P(n, r) = nn!
- r where the symbols 7x6x5 x4x 3*2x1 7-5 2 C(8, 5) x C(4, 2)

have their usual meanings.


5040
Total number of selections
= C(8,5) x C(4, 2) 2
Next, fix E at first place, then remainn
Please referto2059Q.No.7b letters can be arranged in 6! ways
2. 2070 Set DQ.No.5
From 6 gentlemen and 4 ladies, a committee of 5
99 2073 Set DONo.5 =6x5x4x 3x2x1 336 is to be formed. In how many ways can this be
In how many ways can the letters of the word 720 done so as to include at least two gentlemen? |2]
"COMPUTER" be aranged so that () all the
Hence, the no. of arrangements that
47.2066 Q.No. 2a
vowels are always together (ii) the vowels may From 10 persons in how SOLUTION
begin with E = 5040 - 720 4320 many ways can a The selection of the members in the
occupy only odd positions. last.
last selection
of 4 be made if two
Again, fix E at first place and H at particular persons
Please refer to Model Set LI, QNo. 5a are always excluded?
[21 committee can be made asfollow
remaining 5 letters can be arranged SoLUTION Gentlemen (6)| Ladies (4) Selection
40 2074 Sot AQ.No 5x 4x3x 2x1 120 Total number of C(6,2) x C(4, 3)|
Prove that the number of permutations of
n Hence, the no. of arrangementsu
that players 10
2 C(6,3) x C(4, 2)|
Number of players to
distinct objects taken r at a is time given by: with E and do not end with H 720- When two
be selected=4 4
C(6,4)x C(4,1)|
n! particular players are excluded, then
5
pn,)n-r (nar) 41
600 we have
to select 4
players out of 10-28
0 C6,5) xC4, 0
Required number of committees
Required number of selections
Please refer to 2059 QNo. 7b B. COMBINATION 8 8x7x65 =70
C(6,2)* C(4,3)+C(6,3) x C(4,2)+
=

C(8, 4)=4! 4 C(64) C(4, 1)+C(6, 5) * C(4, 0)


1. 2074 Sot AQNo.6a O 2MARKS QUESTIONSS 4x3x2x1
How many numbers of 4 48. 2069 (Set A) Old Q.No, 2a
formed from the digits 4, different digits |
can be
5,6,7,
8? How 4. 2057 Q.No. 2
many of
n
an examination paper containing 10
these numbers are divisible by 5? How A 12 6 4
these numbers are not divisible by 5?
many of committee is to be cho questions, a candidate has to answer 7 questions 1!51 4!0!
141 women and is to
consist of 3 men and y in how many ways can he choose the 60+120+ 60+6 246
SOLUTION Fiow many such committee can DE ome0questions?
Given digits are 4,5, 6,7,8
So,n - 5,r=4
SOLUTION
Selection of men
men out of 12 can be selected u"
Unit 1 9
PERMUTATION AND COMBINATION
of Basic
Mathematics-
Solution
48 ASmita's NEB Letr r2.

Required number of selections


Then, Gentlemen (6)| Ladies (4) Selection
53. 2071 Set C Q.No.1 out
2r 3 r - 1 C(6,4)x C(4,1)| C(6,1)x C(5,4) +C(6,2)* C(5,3)+
courses C(6,3) x C(4,2) C(6,3) x C(5, 2) + C(6, 4) * C(5,1)
in which 5 2
Find the number of ways
when 3
courses a or, 1 3r -2r C(6, 2)x C(4,3)|
or8can be selected 2 -1
6 5! 6 5 6
C6,1) * C4,4]
compulsory. Again, let ri+r2= n9
sOLUTION then 5
24
-35
Then,2+ (3r-1)
=
The required number of committees
= C(6, 4) x C(4, 1) + C(6, 3) * C(4, 2) + C{6, 2) *
=6x5+15 x10+20x 10
30+ 150+200 +75 455
15x5
wnen 3 ourses are compulsory, selected n
or, 5r =9+1 C(4,3)+C(6, 1) * C(4, 4)
5 be
courses out of 8 -
3 can
or,,I=Z 86. 2068 O.No. 7
5x4xS=10
C6. 2) ways 32 3 x2*1
*

r=1,2.
A person has got 12 acquaintances of whom 8 are
relatives. In how many ways can he invite
54 2071 Set DQ.No.1a 59. 2075 SetcQ.No.1a = 60+120+60 +6 246 guests so that 5 of them may be relatives? 141
to securm62. 2063 Q.No. 7 b
has 5 friends. În how many ways ca In an examination, anxaninee has Please refer to2064QNo. 2a
A man
[21 of the five subjects. In how
of them to a dinner? grade in each A candidate is required to answer 6 out of 10
invite one or more
ways can the
examinee fail to secure A-g a d questions which are divided into two groups 71. 2069(Set B) Q.No. 5 O
SOLUTON
Total number of friends = 5 each containing 5 questions and he is not From 6 gentlemen and 4 ladies, a committee of 5
can this be
to be formed. In how many ways
SOLUTION permitted to attempt more than 4 from any 1s
done so as to include at least 2 ladies?
Required number of ways
5) The candidate fails to secure A+ grade in egroup.
In how many different ways can he make 4
=C(5, 1)
+
C5, 2) + C5, 3) + C(5,4)+C5,4)+C5, choice? SOLUTION
secure up his
of the five subject if he can not The selection of questions can be made as The selection of the members in the
132'1'05 either in 1 or 2 or 3 or 4 or 5 subjects. f o l l o w s '

committee can be made as follows


5 10+10+5+1=31 Total no. of ways by which the candidate
to secure A+ grade 1t group (5) 2d group (5 | Selection Gents (6) Ladies (4) Selection
55. 2073 Set DQNo.1a C(5, 3) +
C(5, 4) +C5,5
| C(5, 4) x C(5, 2)| C(6,3) x C(4. 2)
C(5, 1) + C(5, 2)
a +
From 10 persons, in how many ways can 3 C(5,3) x C(5, 3)| 2 C(6,2) x C(43)
selection of 4 be made when two particular
2 C5,2) x C5,4]| C6.1) x C4 4]
persons are always excluded? Total number of selectionsS The required number of committees
Please refer to 2058 Q.No. 7b = 5+10+10+5+1 3 1 - C(6,3) x C(A,2) + C(6, 2) x C4, 3) +C(6, 1) x C(4, 4)
=
C(5, 4)x C(5, 2) C(5, 3) C(5, 3)+C(5, 2) C(5, 4)
+ x x

56.2074 Set AQ.No.1a 5 5 5 5 6


4 MARKS QUESTIONS 43122131 213 31 2 114
party and each two of
If there are 10 persons in a

them shakes hands with each other, how many80. 2058 Q.No.7 b
- 5 x 10 + 10 x 10+ 10x 5 200
20x 6+15 x 4+6 x1 =120+60 +6 186
hand shakes happen in the party? 2 From 10 football players in how many ways 83.2066 Q.No.7 88. 2070 SuPp. Q.No.5
A committee of 5 is to be formed out of 6 gents
SOLUTION aselection of a 4 be made (i) when one particu
A
person has got 12 acquaintances of whom 8 are
relatives. In how many ways can he invite seven and 4 ladies. In how many ways can this be done
when
player is always included (i)
Here, n = 10, r = 2
guests so that 5 of them may be relatives? when at least two ladies are to be included? 41
Total number of hand shakes C(nr) particular players are always excluded?
10! 10x 9x 8 = 45 SOLUTION
Please refer to 2064 Q.No. 2a Please refer to2069 (Set B)ONo.5a OR
C(10,2)=(10-2) 28*2x1 Total number of players = 10 84.2066 CQ.No. 7 9. 2071 OldO.No. &E
From 10 players in how many ways can a
57. 2074 Supp Q.No.1a Number of players to be selected=4 In how many ways a
selection of 4 be made, when one particular| selected from 8 gentlemen and 6 ladies, if the
committee of 8members be
In an examination, a candidate has to pass in When one particular player is always
player is always included, when two particular committee is to include not more than three
each of the four subjects. In how many ways can included, then we have to select 4-1 players are excluded?
the candidate fail?
41 ladies.
players out of 10 -1 = 9
Please refer to 2058 Q.No. 7b SOLUTION
SOLUTION Total number of selections The selection of the members in the
A candidate can fail in an examination if he C(9,3)= 9x8x7
66. 2067Q.No. 7b
committee can be made as folows
fails either in 1 or 2 or 3 or 4 subjects. -C9,3)6 33x2x 1 A committee of five is to be constituted from six
excludedboys and five girls. In how many ways can this Gentlemen (8)|Ladies (6) Selection
. Total no. of ways by which he can fail When two particular players are 10-4
ee be done so as to include at least one boy and
one
C(8,5) x C(6,3)|
C(4, 1)+C(4,2)+ C(4,3)+ C(4,4) we have to select 4 players out of girl?
C(8, 6) * C(6, 2)
Kequired number of selections C(8,7)x C(6, 1)|
C(8, 4) = 8x7x6 x570 SOLUTION
Cs, 8)xC6, 0)
4+6t4+1 = 15 =C(8,4)414 4x3 2*1 The selection of
committee can be made as follows
the members in the
The required number of committees

58. 2075 Set BO.No. 1 81. 2060 Q.No.7 commitee Boys(6) Girls (5) Selection
C(8, 5)x C(6, 3)+C(8, 6) C(6, 2) C(8, 7)
C(6, 1)+ C(8, 8) x C(6, 0)
x + x

Find the value ofr if C=C From 6 gentleme 4 ladies a


thi C(6, 1) * C(5, 4)
6!
6 8
2 is to be formed. and yO 6 8
can

SOLUTION In how many ways 2 C(6, 2) x C(5, 3) 315 313! 2161412 17 5!1 018610
=

aone so as to include at least one lauy C(6, 3) x C(5, 2) =1120+ 420 + 48+ 1 1589
Given
C Car- SOLUTION embers n
C64) xC5, 1)_
We know that if C, C = The selections of the men
mittee can be made as follow
thenr r or n+r= n
Mathematlcs-ll PERMUTATION AND COMBINATION Unit 1 51
of Basic
50 Solution
A.No.6
3. 2072 SetE
Asm
Smita's NEB

6 bo number of selections Total no. of selections


Required
70. 2071 Supp. Q.No.5a In
of
a
students,
group
10
4 students be
are
boys. In hoy 6 Csx C2+Gx C+CxC
consists of
12 questions many
ways
mathematical
can

competiti tition so selected t


as to
-C(6,4) x CA,2)-2141*212
examination paper
n parts A
and B. Part A contains inc
aivided into
two atmost wo girls?
contains remaining
questions 36 +60+ 20= 116
B
questions, part to attempt
8 quesnons
SoLOO
A candidate is required
least 3 from each part.
In how many
Here, total
number of students =
10 76. 2073 Supp Q.No. 6a 79. 2075 Set CQ.No. 5a
Selecting at candidates select the questionsí [*]| of boys 6
=
In how many ways a committee of three can be If C(n, r - 1) 36, C(n, r) = 84 and C(n, r+ 1)=
can the Number
ways 126, find the value of r and n.
Number of girls
=
10 -
6 4 formed out of 5 men and 2 women so that it 14
sOLUTION = 12 the selection of the memh. always consists at least one women? SOLUTION
Total number of questions Now,
A =
7 mathematical competition can be ma sOLUTION Given,
Number of questions in part
12-7=5
The selection of the members in the Cn, r-1)=36 ()
in part B follows C(n, r) = 84
=

Number of questions committees can be made as follows: ... (i)


The selection of 8 questions selecting
at least
Girls (4) Boys (6) Selection Men (5 Women (2) Selection Cin, I+1)= 126 ii)
rom each part can be made in the following8 C(4,2)x C(6,2 2 C(5,2) x C(2, 1) Dividing equation (i) by (i),
1 C(4,1)x C(6,3| C5, 1) x C(2, 2) _Cn, r_4
way's.
Selection C4,0) xC264 The required no. of committees C(n,r-1)36
Part A(7 Part B(5) selections
number of n
C7,3) x C(5,5) The required
= C(4,2) x C(6,2) + C(4,1) * C(6,3) + C(4,0) x C6a
=
C(5, 2) C(2,1)+C(5, 1)
x x
C(2,2) (n-! r!_
C7, 4) xC,4)| 2
4
C7, 5)xC5,3)]| 31211 1 O2 n
(n-r+1)! (r-1)!
Required number of ways 22313'T0 421 1205 n-r+1! (r-1!_
Or,
C73)x C6.5)+ C74) x C(5,4) + C75) x C(53) n-r)! r!
27. 2074 Set BQ.No,5 Or,
(n-r+1). (n-r!(r-12
90+80+15 185 From 3 men and 7 women a committee of 5 is to (n-r)! r. (r-1)!
be formed. In how many ways can this be done
74. 2072 SupPp Q.No.5a OR so as to include at least one man?
n-r*lZ
3
Prove that C(n, r) + C(n, r-1) = C(n+ 1 , ,wha sOLUTION
or, 3n-3r +3-7r
Cn, r) is the combination of n things taken ra or, 3n -10r = - 3
=35 175+210 420 The selection of'the members in the ..

(iv)
time. committee can be made as Again, dividing equation (ii) by (i)
12072 SetCONo. SOLUTION Men (3)|Women (7) Selection Cn,T+1) 126
A committee of five persons is to be selected C(n, r) 8 4
LHS-C(nr)+ C(n, r -1) C(3,1) x C(74)
from 5 men and 4 ladies. In how many ways can
n 3
n
this be done so that at least two ladies are C(3,2) x C(73)
( n - r)! r! (n -r+1)! (r - 1)!
2 (n--1)!(+1!
always included?
n The
C33) xC7,2)
SOLUTION required no. of committees (n-r)! r!
(n-r!r (r -

1)! (n -r+1) (n- =C3, 1) C(7, 4) +C3, 2) C7,3)+


The selection of
be made
the members
follows
in the
n
x
x
C(3,3) x
C7, 2) (n-! =
n-r-1)! (r+1): "2
committee can
Ladies (4) Men(5
as

Selection (n-7! (r-1) 213/4 112431 01 31312 n-.(n-r-1)!!-3


105+105+21 231 (n-r-1! (r + 1). r!2
C(4, 2)x C5,3) n!
[n-r+1t+I
78. 2076 Set AQ.No. 5a n-
2 C(4,3)x C(5, 2) (n-1(6-1 n-r1 Or,+12
C4,4)x C(5,1) An examination paper consisting of 10
T h e required number ofcommittees
-n+1) n! guestions, is divided into two groups A and B.
or, 2n-2r =3r+3
or, 2 n - 5r = 3

= C(4,2) C(5,3)+ C(43) C(5.2)+ C(4A) » C(5,1)


(n-r+1) (n - r)! r(r-1) Group A contains 6 questions. In how many (v
Multiplyingg (v) by 2 and subtracting from (iv)
n+ 1 R.H.S. ways can an examinee attempt 7 questions
+1, r) 3n - 10r= -3
=

n+1-rr!C(n
4 5 45 4 selecting at least two questions from each group. 4n- 10r= 6
75. 2073 Set CQ.No. 5a -n 9
here are ten electric bulbs in the stocSOLUTION n 9
shop out of which four are defectives.de Total no. of Putting the value of n in equation (v), we get
made No. of
questions =
10
60+ 40+5 105 nany ways can a selection of 6 bulbs
that 4 of them
questions in group A =
6 2 x 9-5r =3
No. of or, 1 8 - 3 = 5r
may be good bulbs questions in group B =
10 6 4
72.2072 Set DO.No.5
-

has
SOLUTION he selection of questions from two groups
or, r=3
A person got 12 acquaintances of whom8 are Here, number of bulbs = 10 Canbe made as follows: 3 , n=9
relatives. In how many ways can he invite 7 number of defective bulbs
-4 6
Group A(6)| Group B(4)Selection
guests so that 5 of them may be relatives?
14 number of good bulbs= 1ol0 Cx Ca
Please refer to 2064 QNo. 2a easfoll
ne selection of bulbs can be na Cx Cs
Selectio
CxC
Good bulbs (6) |Defective bulbs (4) J C 6 , 4 )* C A

6-4 2
BINOMIAL THEOREM Unit 2 3

UNIT 2064 Q.No.1b


Find the middle term in the expansion of
SOLUTION
Let t be the term free from r in the

BINOMIAL T H E EM () SOLUTION
2
Then,
Here, the numbers of terms in the expansion
coefficient of x, we have
)-cqas, r) -
For the

A. BINOMIAL THEOREM 5 7-2r is18+ 1 19


whichis odd. So - C(15, r) x03r

For the term free fromx, we have


there is only one middle term. The middle
r 1

2 MARKS QUESTIONS 30-3r 00


Coefficient of*=*2s: term is given by ti8. I= 10
2057 O.No.1E expansion or 1
tn is the term free fromz.
Write the middle terms in the 20611O.No.1E t+1 C(18,9) x( 9 And, t 15!
tio-1 = C(15, 1 0 ) 5 110! = 3005
n is odd of x in the
(a+ x when Find the term independent
sOLUTION
expansin 8 2065 Q.No 16
When n is odd, the number
of terms arter
Find the term free from x in the expansion of
13. 2069 (Set A Old QNo. 1Ein
Find the coefficient of the expansion of
So
of (a + x)P is n +1 which is even. |
expansion
there will be two middle terms namely SOLUTION
2]
be the term independent of x.
n+1 and +1 : . Let t
Then, sOLUTION Please refer to 2060 Q.No. 1b
Let tr+1 be the general term.
= C(12, 1) 14. 2070 (0ld) O.No. 2a
t1-C(12,1)( Then, Find the term independent of x in the expansion
For the term independent of x, we have
tr=C(9,) (
(5)().9| 24-3r 0
39-2r
12
. r-8
to is the term independent of .
=
C(9, T) 29- x18-3r Please referto2061 Q.No.1b
(). And, t ts +1- C(12, 8) For the term free from x, we have
18-3r =00
15. 2071 Set CO.No.1E
Find the term independent of x in the
12! 2x11 x10x9x8! expansion
. I=6
4!81 8! x4x3x2x1 t i s the term free from x. (21
52062Q.No. 1b and t
3-12 9
If Co, C, C2 ., Ca are the binomial coefficien ton=C(9, 6) 29-6 316 sOLUTION
in the expansion of (1 +x), show that: Let t be the term independent of x.
9 2066 CQ.No.1E
Co+ Ca+C+.. =20-1 Find the middle term in the expansion of Themt
2 2058 QNo.1 C(12, r) (r
Find the seventh term of (2x + y)2. 121 SOLUTION
sOLUTION
Seventh term =
t t=C(12 6) (2x)24ys
=
We have,
(1+ x = Co+ Cix + C2+ ...+ Ca ".
(
Please refer to 2064 Q.No. 1b
121 (-1 C(12, )**

12
Puttingx=-1 in (i), we get
10. 2067 Q.No. 16 (-1) a-4
61 6 1 2 * ys = 59136 1* y
1-1) =Co -C+C2 -Cs+Ca-Cst Find the middle term in the For the term independent of x, we have
or, 0-(Co+ Ca+Ct.) -(Ci+Ca+Cst expansion of
2060 O.No.1 24 4r 0, r =6

Find the coefficient of r*i


( 121
Co+ Ca+ G+

Co+C+Ca+Cs+ Ca+Cs+=(
Ci + Cs+ Cs+
Again putting x = 1 in (G), we get
( Please refer to 2064 QNo.1b
21 b is the term independent ofr
and = (-1)°
and= (-1)
0121
6 *243
308
2
SOLUTION Cot Ca+ C =Ci+ Ca+Cst 2 11 2068 Q.No. 1b
The general term in the expansion of Find the coefficient
of as in the expansion of
16. 2071 Sat DQNo.1
Find the coefficient of x in the expansion of
6. 2063 Q.No. 1
( Find the term independent of x in the expans

Please refer to 2060 Q.No. 1b


(21

C,) ( 12.2069 (Set A) Q.No. 1 SOLUTION


We have the general term of (a + x)" is

-co.n.- SOLUTION Which term is free from x in the expansion or


t -C(n, r) a"*
Please refer to 2061 Q.No. 1b
121 The general term of(x3+
Mathematics-lil
54 ASmita's NEB Solution
of Basic
soLUTION BINOMIAL THEOREM Unit 2 55
Here, the number of terms in the
tr C(5. r) (a is 121
expanslion taC(2,n)(1)2n-7m-n!ni
2n!
or, -2
( 13
which is -

there is only
one middle term.
The
2n (2-1)(2n -2.65
n ! n!
43.21 OF, n-r+1 -2r
or 3r n+1 iv)
in (r* 1) term is given by
i.e. t.
t mid
Suppose the coefficient of 12n(2n-2)...64 21 (2n-1)...53.11
a occurs
Again dividing (ii) by (i)
term. Then We have,
n'
n Cn 20
10-3r 1 = C(n, r) an-t x 2 n (n-1)3.2.11{(2n-1).5.3.1 n! n
C(n, r
3 n
coeficient of x
twill contain the

Coefficient of r =C(5, 3) a*= a-10


a* lor=C(12, 6) x12-
( n! (2n-1)...5.3.12

1:3-5n 2
n!n!
(n-r-1 (r+ 1_
n
12 n!
7. 2071 OldaNo.1 61 61
Find middle term or terms in the expansion
or
24. 2069 Q.No. 8C (n-r-1) (r+1)
121 21. 2074 Set BQ.No. 16of
If Co, C, C2 ., Ca are the binomial coefficients or, -
"
n-I-1)! 7
Find the coefficient x* in the in the expansion of (1 + x)" then prove that: (n-r-1)! (r +1) r!
expansior 2n! 141 n-I
or. r + 1 5
SOLUTION Cat.+CaConln
Here, the number of terms in the expansion of
()
sOLUTION
CCa+ Ci

sOLUTION
OT, 5n-5r = 7r+7
or, 5 n - 12r = 7
is 16+1 =
17 which is odd. So Here, or, 5n-4 (Br) = 7
Let t 1 be the general term in the
expansi(1
there is only one middle term
=Co+Cix + Cax t ...+Cn-11-l +
Cr"..-4)| or, 5n -4 (n+ 1) 7 using (iv)]
Also, n11
Middle term = tl6, t a (1+x)=Cnxn + Cn-ixn-1 +...+ Cix+Co (i)
Multiplying () and (i) 26. 2066 O.No. 8
Then, t1= C(n, r) an-.x (1+xn= (CCn+ Ci Cn-1t.. CaCo) m + . . . ( i ) f three consecutive coefficients in the expansion

Ccas.8)a)( tC(10,1)w0-(
Since (iii) is an
identity,
power of x of the LHS should be equal to the
the coefficient of any of (1+x} be 165, 330 and 462, find n.
Please refer to 2061 Q.No. 8b
[4]

Find the coefficient of the term containing 2 in


=
C(10, r) x*0-3, x
coefficient of the same power of x of the R.H.S.
The coefficient of xn in the L.HS of (ii)
27. 2069 Old Set B) O.No. a
C(10, r) x30-4r IE (1+r) = Ca+ Cur+ Car+. C Prove that
2n!
the expansion of 121 For the coefficient of x, we have
6 30-4r
C(2, n)-n!n
Again, the coefficiernt of xn in the R.H.S of (iii)
Ca+C+C+.. +C 2n)!
(nl
Please refer to Model Set I, Q.No.9 or, 4r 30-6 sOLUTION
or, 4r = 24
Cocn+Ci Cn-1t...Cn Co (1+x= Co+Car+Car+... +Ca
19.2072 Supp Q.No.1b Also,
I =6 Co Ca+ Ci Cat.+ C Con2n
Find the middle term in the
expansion of t contains the coefficient of x.
n! (r+1 Corn Cix+Car+..+Ca ()
Multiplying () and (i).
And, the coefficient of x* 25. 2061 Q.No. 8 b (1+x (C+Ci2+C2+.. C)a*.... (ii)
10! 10 x
9x8x7x6 the threeof consecutive coeficients in the Since (ii) is an identity, the coefficient of any
sOLUTION -C(10,6)= (10-6)! 61 4x3x2x1x6find
4 x 3 x 2x 1 x6expansion
n.
(1+x)n be 165, 330, 462, power of r of the LHS should be equal to the
Here, the number of terms in the coeficient of the same power of r of the RHS
expansion of = 210
SOLUTION The coeficient of
r
in the LHS of (ii)
is 18+1
19. So, there is only one 2. 2075 Set AO.No.18 Let C(n, r -1), C(n, r) and C(n, r *
1) be three C(2n, n)=n! n(n!F
Find the coefficient of
middle term. It is
x in the expans10n consecutive coefficients in the expansion of
given by tu ,ie. to (1+ a. By given, And, the coefficient of r in the RHS of ()
C(n, r - 1) = 165 C+CP+C+..C
We have, )
Cn, r)= 330 ..ii)
tC(nr) ax Please refer to 2071 Set DQ. No. 1b Cn, r+ 1) = 462
(1)
C+ C+C+...C=(n!
4 MARKS QUESTIONS Dividing (i) by (i)
tuo C(18,9)xu
)-() 23. 2068 O.No.8D
Cn)_330
C(n,r-1)165
28 2070 Supp. Q.No.5
If the coefficient of x in the expansion of
12,155
20. 2073 Set CO.No.16
128 ind the middle term in the expan
(1+xa, where n is a
positive integer.
(-r)! r! (SOLUTION
is 270, find k.

Find middle term in the


expansion of
SOLUTION (n r+ 1! (r - 1)! The general term in the expansion of
Here the number of
( 21 x) is 2n
terms in the
1 which is odd, so,there
Only one middle term. It is given by
exphere Or,

r,
n-r)! r!
-T+1)n-! (- 1Y (
(n- r)! r (r- 1)!
Mathematics-l
BINOMIAL THEOREM Unit 2
Solutlon
of Basic
56 ASmita's NEB

n!
213 T V T 3 3
131+n-n)! n! 36. 2072 Set DQ.No.9
toyC(696 (-q6K (n-r-1)! (r+ 1)! State Binomial theorem. In the expansion of

For the coefficient of ,


we must have
-T-1L(T +1)!
23n2 1,3n(n= 2 , +n the coefficients
a prove that the to theofsum
sum
of
coefficients
10-3r= 1
a
-r-2)! (r +2)! the odd terms is equal
and each to 2 .
( n - T - 1 :( n - T - 2 ! r + 1 ! of the even terms equals
-3
(r+1)* 23n 1 . - n - 2 .
Coefficient of r
C5,3) = k*
270
Or, (n--2)! (r+2) sOLUTION
coefficient of a
=

n-'C1+ n-iC2+... +n-1C-1) Bionomical Theorem


By question, 2n+ 3n(n-1Co+ For any positive integer n,
C(5,3) k' 270
= 2+3n (1+1)- 2) an2
5r+ 10 = 2+3n.2n-1 (at x= C(n0)an + C(n,1) a-x+ C(n,
4n-4r-4=
or, k'213 270 or,
4n - 9 r =
14 (v) = 2.2-1+3n.2n-1 . . Cn) an rt..+C(nn) n
or,
Multiplying (v) by
3 and subtracting 3n+2) 2n-l = RH.S. Next part: Please refer to 2062 QNo. 1b
or, k= 27 from 37. 2072 Set EQ.No. 9
k3 wege 14
6 MARKSQUESTIONS
4n- 9r = If (1+xp= C+ Cx +Ca.. +Ca prove
29 20741 01dO.No.7 term of the binomial
3n 9 r 6 33. 2069 (Set B) Q.No. that: Co Ca+C Ca1t+ C Ca-2+ .+Ca 2n
Define the general
the expansion or
If (1 x) = Co + Cix + Cx2 * + . = C a , prove
Co nln!
of (x + a)". In
expansion coefficients of n8 2n
( * xprove that
the s u m of the that: CoC+ CiCa-1+.+CaConini [6]
to the s u m of the coefficients
n =8 Pleaserefer to 2059 Q.No 8b
even terms is equal
207 Sot AC.No. 6
to 2 , Please refer to 2059 QNo. 8b
of odd terms and each is equal 1. 38. 2073 Set DO.No. 9
SOLUTION in the
how that the middle term in the expansion
Show that
the m i d d l e er
34.2070Set CQ.No.9 If(1+ x=Co+ Cx+ Cx... + Cx, prove that:
General term: (r + 1jth term term
The 13.52n-1)a Show that the middle term in the expansion of

expansion of (a 1)"is
called the general (1+x) is n! CaCa+GCat+ CaC i n [61
+

It is denoted by ti. it is given by


1.3.5 (2n-1 (2 6
n!
t c (n T) a
sOLUTION
Here, the number of terms in the expansion
Please refer to 2059 QNo.8b
Next part which is odd. So,thee
SOLUTION 39. 2073SuPpQ.No.
(1+ x) is 2n 1,
+
1b Here, the number of terms in the expansion of Provethat C-2Ca+3C-4C+ ..+n-1)Ca = 0,
Please refer to 2062 QNo. only one middle term, which is given by
where Co, Cu ., Ca the
30. 2071 Supp.Q.No. 55 is 2n1 which is odd. So there is coefficients.
are
binomial
[6
If the three successive coefficient in the expansion +1ie. tn+1.
of (1+x} are 28, 56 and 70, find n. only one middle term. The middle term is SOLUTION
Middle term =tn+1=C(2n,n)x L.HS C1-2Ca+3C- +(-1)"-, nCa
given by,
SOLUTION in 2n! n n n
If t , tr-2 and ts be three consecutive terms x .*
the expansion of (1 +1), then their respective (2n -n)! n! (n -1! 1! (n -2N!2*3 (n-3)!31-
coefficients are C(nr), C{nr*1), cn, r+2) 1:2-3.4(2n-2) (2n-1),2n. tn+1C(2n, n) (*)2n-n
respectively. By given, n n! -1nn-n)ln
C(nr)=28 ) _1-3.5 (2n-1)} {2.4 6 (2n-2.,2 -(-1) C(2, n) = (-1)" nl n
n-1-22n-2).21
C(nr+1) =56 n!!n!
C(nr+2)=70 135(2n-1)) 2{1:2:3 (n-1), 2n 2n (2n-1):(2n-2).6-5.4.3-2.1
-(-1)* n(n-1) -3)! +
(-1)*n
Dividing (i) by (i), we have n n n-3!.3.2
n! n --Ir{(2n-(2n-2).64.2 1(2n-1)..5-3.1)) nn-1) nn-1n-4-..+(-1)»-n
Cn- Cnr2 3-5: (2n-1)).2 n! x
n! n!
n! n

n 20 {n-(n-1)..3-2.1(2n-1)..5 3.1
n -

1! 2

n!n
Or, n--1 (1)-2
n!
1:3:5(2n-)2n
n!
x HP 2. nl {(2n-1)...5.3.1
n! n n{Cn-10)-Cn-11) Cn- 1)-.. *(-1*-iCin-Ln-1}
(n-r)! r!
32. 2075 Set BQ.NO. 56 3 52n-) (-1n, 20 n(l - 1)-

(n- 2
1

If Ca, Ci, Ca n
OT
(n-r-1)! (r +1)! ., C are
binomial co+4C C'nis replaced by n -1&x=-1 in (1 +x})]

n--(n-r-1)!. ne expansion of (1
7Ca+10C3 t..+(3n +1)C.= (3n+ 2)
+ x, prove that 1:3-52n- (-2 n!
nx0=0= R.H.S proved.
O(n-r-1)! 4r+ 1) r 40 2074 Supp Q.No.9
SOLUTION 35. 2070 Set DQ.No. 9 C»x?+.+Cax", provethat
If (1+ x) =Co+ Cx
+
o7 +12 L.H.S It (1+ x)* =
Co+ Cix + Cax+..+Car, prove
ve
C o t 4C + 7C2 + 10C+.. +(3n + 1)
2n n(n+l)
or, n-r= 2r +2
C3Ci)+ (Ca+ 6C2)+ (Cs*
that: Co CRn+Ci Ca- +.+CaCo(n!} 61 Co
or, n-3r =2 ..iv) SOLUTION
Again, dividing (ii) by (i), we get (3n Ca+ Ca) + (3C1+
Please refer to 2059 Q.No. 8b
L.H.S.
(Co+C+ Ca+ Cst ... +Cn)*
9Cst.+ 3nCn) Cn-1
2 +3(C1+ 2C2+ 3Cs+...+nn
: Co+ C + C 2 t
BINOMIAL THEOREM Unit 2
Asmita's NEB Solution of Basic
Mathematics-ll 59
58
Adding, we get &
n
n-1912(n-22 e-2* Subtracting (i) from (i)
n-1)! 1! (n-2)!2 -1 1 .. =e =RH.S e- e-
112
50.2072Set EQ.No. 1b
n-n)! n!
+n. or,i ( ) -
n
(n-n+ 1! (n - 1)! 95. 2070 Setca.No,1 Showthat: 1ti*ai.to co=e. 2
55.2075 Set BQ.No.16
n-1 (n-2 n.(n-1! Please refer to 2072 Set DQ.No. 1b
(n-1) (n -2)! (n -3): n! If
,n
nn-1(n-1(n-2!, (n-2)(n-3!*..n!
Showthat (-i)-1*2-i 12 51. 2073 Set DQ.No. 1E y
i 2
(n-1)! (n-2)! (n-3)
n+ (n-1)+(n-2)+...+1
sOLUTION Showthat . to o=e
12 show that x=y+++ [21
sum of 1sn natural numbers We havee=1+2 Please refer to 2072 Set DQNo.1b sOLUTION
Given,
nn - RHS. Putting x=1 and -1, we have 52. 2073 Supp Q.No.16
y=x- . show that: x =y* 2
e 1+
B. EXPONENTIAL AND LOGARITHMIC
SOLUTION -y
SERIES_
2 MARKS CUESTIONS
ei =1-23 y=x-*z Adding 1 to both sides,
Adding, we get or, yloge (1 +x)
1. 2059 ONo. 32
Prove that:
or, 1+x= ey
1-y1-
or,1-y=ex
log2 2
or,-x= loge (1 -y)
SOLUTION y2 y
We have
1-* y*23*
53. 2074 Set AQ.No. 16
Or,,-X

46. 2070 Set DQ.No. 1 Xy


Putting x 1 , we have
Showthatlog.2*6* Ify-x+2+3*show that: 56.2075 Set CQ.No.1b
log0-1)1-3i Please refer to 2059 Q.No.3a (2
Prove
that *^* *.= 21 -
In2). 12
Please refer to Model Set I Q.No. 1b
-)--)-6) 47 2070Supp.Q.No.1 sOLUTION
Here, y=x+
4 MARKS QUESTIONS
Please refer to Model Set I, Q.No. 1b
57 2057 O.No. &
42. 2066 ONo.1 48 2071 Supp.Q.No.13 or-y-x-- Show that 1
log,2* or, -y loge
=
Prove that (1 -

x)
12 Ifx y -2 + - show that or, 1- x=e-y *********

Please refer to 2059 Q.No. 3a SOLUTION


43.2069 (Set B) Q.No.1b y-x2
SOLUTION
2 Or,
1-xm1- We have
Prove that log22 or, -x
121 e-1
2 35
Please refer to 2059 Q.No. 3a Given,x=y- Puttingx 1 and -1, we get
=
4. 2069 Old (Set B) Q.No. 3 or, x =log (1+y) xy- e
or, 1+ y =ex 54. 2074 Supp Q.No.
1
Find the value
ofs (e + e). 21 or, 1+y=1+j*
Prove that:
ande-1 = 1

SOLUTIONN 2
Now,
We have e =1 +
y-x
SOLUTION
We have
e+e-l -2

Putting r =1 and -1, we have


49. 2072 Set DQ.No.1b
-1 Provethat* i . . Putting r = 1&x= -1, we get
SOLUTION
el -1-7*7- LHS
BINOMIAL THEOREM Unit 3 67
Asmita's NEB Solution
of Basic Mathematics-
62
verse.

T5. 2075 Set AQ.No. 9 ion.


Sum to infinity the series: 1+* à 1
"ove

SOLUTION NoW,
The nh term of given
series e2-1
LHS
2n-1 2n-2+12n-
(n-1 21 416
e-1
(n-1( n- 1 ) (n-1) (e+ 1) (e-1)
n-1) (n-2 n-1)
2
Cn-2 n-1 [Dividing numerator
=e+e=2e Putting, n=1,2,3...,we get, and denominator bye
72. 2072 Supp O.No s t-0O RH.S
Showthat + e-1. 61

Please refer to Model Set I1, Q.No.9


2073 SetcO.No. 2
Prove that 1 r6]
Adding all the terms, we get,

Please refer to 2057 Q.No. 8a


74.2074 Set BQ.No.9
Suntoinfinitytheserien 1
2e+e
SOLUTION 3e
Let ta be the nth term of given series.
t. n 76. 2075 Set CO.No.
Then, t n!n.(n-1 Write e in the series form. Using it show that

-1)!
(6
n-
(n-1) (n-2!" (n-1)

1 SOLUTION
(n-2(n-1)! For all values of x, the series form ofe* is
Putting n=1,2,3,4,... . , We get,
-1

t-00 Puttingx =1 and -1, we get

t0 1 e

and

el

Now,
1-13
e+e -2
Adding all the terms, we get,

****
1
e+e =2e Also,
e -e
ELEMENTARY GROUP THEORY Unit 3 65

UNIT 1 X7r =6
6
From table,
1x(-1)=1
2072 Sot C Q.No, 1c 1x1 1
ELEMENTAR9. Ifa b 3a + 2b for a, b e Z, the set of integers,
show that*is a binary operation on Z. (2
1 and

respectively
1 are the inverses of -1 and 1

SOLUTION 14.2073 Set DQ.No. 1


GROUP THEOR I f a and b are any wo integers then 3a and 2b
are also integers and their sum 3a *
again an integer.
2b is
Let G ={0,1, 21. Form a
under

element of2
addition modulo
composition table for G
3. Find the inverse

2
Cayley's table Thus, for all a, b e L, a'b =
3a * 2b ¬
15. 2073 Supp Q.No. 1c
10 1 uniquely. Showthatthe set G ={-1,1,-i, i, the fourth roots
Hence is a binary operation on Z.
2 MARKS QUESTIONS 0-1| of unity satisfies the binary operation of
10. 2072 Set DQ.No. 1c multiplication. (21
. 2069 (Set A) Q.No. 1 operation of In a Caley's table for a finite group, why does| SOLUTION
and denote the usual
*

Let S ={-1, 1) table.


exactly once in each row and Here, G -1,1,-i,i}
multiplication. Represent
it by Cayley's
From above Cayley's
From
Cayley's table, we see each elementinoccur
each column? 2 Cayle's table
is a binary operation two elements of S is tnexactly once
Show that the multiplication
product of any Let a1, a2,.,an be the distinct element ofa
on S. belong to S. So, * 15a binary operation onS
SOLUTION finite group (G,"). If possible,
Please refer to Model Set II, QNo.
1c 5Let2070 Supp. Q.No.1 c let a " a j a n d a * ax be the same in the Cayley's

mn = mn for m, n e Z. Verify tho table, then a*a= a * ak

2 2069 (Set B) Q.No. 1d


on Q the set of rational is not a binary operation on Z have
*
lt the binary operation operation By left cancellation law, we
From above Cayley's table, we see that the
a + bt ab for every which is contradictiorn as aj*
by a"b sOLUTION ak.
=
a
numbersis defined a a
product of any two elements of G is unique
Hence, in a Cayley's table for a finite group
satisfies associative property. We have, m'n =Vmn for m, n Z
"
e
a, beQ show that and belongs to G. So, x is a binary operation
We have to show * is not a b i n a r y ope each element occurs exactly once in each row

SOLUTION on Z. and exactly oncein each column. on G.


Here, ab=a+ b+ ab for every a,beQ For, take m =1,n=2. Clearly 1,2eZ 11. 2072 Set EQ.No. 1c 16. 2074 SetAQNo.
Let a»b
1G
a - b on Z. Show that "" is not an
Let a,b, ce Test the commutative property for the operation*
Now,
Now, 112=V2 e Z
defined by m *n=n, m, n E Z. [2 associative binary operation. 2
a (b°c) a* (b+c+ bc)+ a (b+c+ bc
(b+c+ bc)= a +
So, 1S not a binary operation on Z. SOLUTION sOLUTION
Here,
atbtc+bc +ab +ca+ abc 6. 2071 Set C Q.No.1c For any m, n ë Z
arb a -be Z for all a, b e Z.
And, Let G = {0, 1, 2]. Form a composition able m*n=n
So, * is a binary operabion on Z
(a*b)'c (a + b+ ab) + c+
b+ ab)*c (a
= + (a+b+ ab) c 3. Fi n* m= Im
a + b t a b + c + ac+ bc+ abc under the multiplication modulo Again, let a, b, c e Z.
of 2. So, m *n#n*m
a+b+c+ ab +bc + ca+ abc identityelement Then a(bc) a* (b - c)=a (b-c) = a -b+ c
= -

a*(bc)=(a*b)*c for all a, b, ce Q SOLUTION Hence, 1S not commutative And, (arb)*c = (a*b) -c= (a- b) - c s a - b - c
Composition table for G under m u 2 . 2072 Supp Q.No.1c
This shows that satisfies associative a(bc) * (a*b)*c for some a, b, c e Z.
modulo 3 Let G= {0, 1, 2). Form a composition table for G
property._ multiplication modulo 3. Find the inverse 1e. *IS not associative
3Let 2070 Set CQ.No.1c O12 under
element of 2. [21 Hence, * 1s not an associaäve binary operation
G = {0, 1, 21. Form a composition table for G
0 0 0 |0 SOLUTION 17. 2074 Set BQ.No.1
under addition modulo 3. Find the identity Here, G = {0, 1,2}
Prepare Cayley's table for the set S
=
{0,1, 2) under
element of 1.
sOLUTION 2 0 21
=1 x2=2
Composition table for Gunder X
0
theoperation multiplication modulo
sOLUTION
3.

Here, G= {0, 1,2 Here, 2 x31 Here, G= (0, 1, 2)


Composition table for G under addition 1 is the identity element of2 0
Composition table tor G under x
modulo 3 2071 Set DQ.No.1 is a binary 0 2
01 2 Dnow that the multiplication From above Cayley's table, 1 is the identity
001|2| on the set S={-1,0,1 element.
SOLUTION
Please refer to 2070 Set DQ.No.1
Here, 2 xi2 1
So, 2is the inverseelementot 2. 18. 2074 Supp Q.No.1c of is
From above table, 1 +s0= 0+1 =1 8 2071 Supp. Q.No.1d If the binary operation on 2, the set integers
f possible, solve 2x +1 6 in Zn 13 2073 Set CQ.No. 1 defined by m *n m+n+1 for every m, nez,
=

0 is theidentityelement of 1. nd the inverses of the elements of G


=
(-1,1} |
SOLUTION under multiplications, if exist. 2 show that satisfies associative property.
4 2070 SettheDQ.No. 1cS Given, 2x+1 =6 SOLUTION SOLUTIONN
Show that multiplication is a binary operation 2x+1 6 in Z Given, m *n m*n+1 =
for allm, n e Z
on the set S=(-1,0, 1} 12 2 X 7x + 71 =6
Cayley's table Let m, n, peZ
SOLUTION
Given, S-(-1,0,1} 2 x 7x + 71 + 76=6t 70 m'n "p) m*(n+p+1)
m+ (n+ p*1)+1|
2 x7x= 5 m+n+p*2
4 x7(2 x7 x) = 4 *5
Clearly, 1 is the identity elemen
4 x72) x7x = 6
Mathematics-ll

66 NEB Solution
of Baslc
1(-1*i) =i*(-1)
=-1
smita's
b) ELEMENTARY GROUP THEORY Unit 3 67
*in-1*i--i
(1 (-1))
*m n)'p
=
(m*n+1)P 1(-1*i)= (1true*(-1)) *i ii. Identity axiom
Here, (-1) x1-1 x (-1) =-1
So every element of Z possesses an inverse.
(m+n+1)+p*i
This result is ror every element of 1 x1 1 From ()-(iv), Zis agroup under addition.
m+n*p+2 Associativity is satisfied.
P¬Z 28. 2070 Set DQ.No. 5 bOn
*n)*pforall m,n 1is the identity element.
m(n p) (m c) Here If a and b are the elements of a
associabive propergy O iv. Inverse axiom: group (G, 0) prove
satisties the 1*1-1 that the equation a ox =b has a
and if (-1)*1 1*(-1)=-1 (-1) * (-1)1 unique solution
19.2075SetAQ.No.1
elements of a group
(G, ") and1 x 1 =1 in(G,0) 4
f a and b are the i*1 1 * i = i
So, -1 and 1 are the inverse elements of -1 and SOLUTION
a'be prove thatmSupp. i*1 1*(-i)=-i Since a
belongs to group G, so a-l G such
refer to 2070 Supp.QNo. 6a
Q.No. elemernt.
1 respectively. that anar =aa = ee
e

Please 1 is the identity From i) (iv), we conclude that T is a group


20. 2075 Set BQ.No 1 Now, ax =b
A 11=1 under multiplication. Operating both sides by a1 on the left
Solve 3x +6 5, in Z (-1)*(-1)=1 26. 2070 Set CQ.No.6b O ao (ax)a'ob
SOLUTIONN i(-i)--2=-(-1)=1 a, b, c are the elements of a group (G,o) (alo a)ox - aob (by associavity)
Here,
3x+6 5 in Z & (-i)*i= -j2 =1 i. if aob=aocprove that b =c. enr = aob
1,-1,-i and i are the inverse elem ifb oacoaprove that b = c. (as ar'oa= e)
3 *x +6-5 I=a'b (as eis the identity element)
i and -i respectively. SO SOLUTION This is the required solution.
3 *-x+-6t1=5*71 Sine a e G, has the inverse a-l such that
3 **x=6 Hence (G, is agroupP aoa-l =al oase
To show the
uniqueness of the solution,
5xB x-x)=5 x6 23. 2069 (Set A) Q.No. 5b on We have, aob=aoc let 1 and a be the solutions, then
ao b and aarz = b
( 5 x3) **x=2 Ifa and b are the elements of a group (G, o. Operating both sides of () by al on the left,
1 X7x =2
o b)1 bl o a1
that:(a
=
we have aoxi aoI2

X**2 sOLUTION a-l o (a o b) = a-lo (a oc) a"o (aari) a-lo (aar?) =

Let G be a group and a, be (G,0) or, (al o a) ob=(ao a) oc (ar oa) o = (aoa) or2 (by associativity)
21. 2075 Set CO.No. 1C (by
Prove that multiplication on the set Z-
of all Now, (aob)o (brloar-") associativity) e o=e o2 (as al oa=e)
negative integers is not a binary operation on Z. (aob)ob-")o ar1 (by associativity) or, eob=eoc as ao a=e (as e is the identity element)
[21 b=c (ase is the identity element) So, the solution is unique.
(ao (bob-1))» a-1 (by associativity) Again, we have b o a=coa
SOLUTION aoe )oa-1 (as bob-l= e) (i)
Let Z be the set of all negative integers. (as e is the identity element
Operating both sides of (ii) by a-l on the right, 29If 2070 Supp.theQNo.6
and b
aoa e have
a are elements of a group (G, ") such
Consider -1,-2 Z- e
se that
Then, (-1). (-2) =2 e Z- Similarly, (boa-1) o (aob) = e (boa) a-l=(co
o a) o a-l a*b=b, prove that a =e
So, multiplication on Z is not a binary
or, bo(aoa-l) co =

(ao a-)(by associativity) ii ab=e, prove that b= arl


(aub) o (b'oa-") = (b-loa-")o(aob) = ee or, boe=coe
operation on Z. (as a o a-l =
e) SOLUTION
4 MARKS QUESTIONS
Hence boa-l is the inverse of ab. bc (as eis the identity element) Since b is in group G, bri e G.

22 2069 (Set A) O.No. 5 aob b'oa 27. 2070 Set DQ.No. 5 b We have,
Show that the set of
G [1,-1 i, -i} where i is 24. 2069(Set B) Q.No.5bE integers Z forms a
group a'b= b
Define group. Letand an
the algebraic structure (G, der the operation of addition. 4 (a"b)* b l = b"b-1
imaginary unit stands for the
*
binaryGiven SOLUTION
G (1, a * (b*b))= b°b-{by associativity)
operation of multiplication. Show that (G, ) o, o} where o represents theimap We know,
Cube root of unity and * s t a n d s for the aee (as b'b-l= e)
forms agroup. Z={..., -3, -2, -1, 0, 1, 2, 3, .
sOLUTION operation of multiplication, show that ( > ase (ase is the identity element)
Group: Let G be a non empty set and * is an Osure axiom: Snce the so Z isorclosed
integers is again an integer,additon two . Since abelongs to group G, arl
operation defined on G. Then, (G, ) is said to group
sOLUTION under addition. We have,
e G

be a group if satisfies the


"
ii. Associative axiom abe
following axioms.
1. Closure axiom: G is dosed under the| 25. 2070 Set CQ.No. 5 b
Please refer to Model Set I1, Q.No.5b Let a, b, c eZ a" (a"b) =a-l"e
operation Show that the set T = (-1,1} forms a grou
Now, a + (b+c) = a+b+c
(a-l"a) *b=a-le(by associativity)
ie. ab e G for all a, b e G and (a + b)+ c=a+ b+c eb=arl e (as a"a = e)
2 Associative axiom multiplication operation. a+ (b+ c) (a+ b) + c for all a, b, c e b a (as eis the identity element)
=
Z
a (b'c)= (a"b)'c for alla, b, c, e G SOLUTION Hence, associativity is satisfied.
3.
Here, T{-1,1} 1l.
ldentity axiom: Let a e Z. 30. 2070 Supp. Q.No. 6 a OR
ldentity axiom: For all a e G there exists an Closure axiom: Also let e be the Let G 0,1,23,4, Construct cayley's table for G
element ee G such that identity element of a. Then, under the multiplication modulo 5. Find the
-1)* (-1) =1eT a+=a
a'e =e'a =a (-1) x 1= -1 eT e=0e Z inverse of each element of G. 4
The element e is called the
identity element. SOLUTION
4. Inverse axiom: Each element of G 1x(-1)= -1eT ..
0 is the
identity element. We have, G= {0,1, 2,3, 4)
possesses
inverse ie. for all a e G, there exists 1x1=1 eT Inverse axiom: Let a e Z. Also let b be the Cayley's table for Gunder xs
b inverse element of a.
such that a"b= b'a= e. e G Hence T is closed under multiplicau x 0 1|2 34
ii. Associative axiom: Then,
The element b is called the
inverse of atb=e 0 0 0000
write b a-
a. We Here, -1,1,1 eT Or, at b=0 O1|2 3
Next part: -1x(1*1) -1*1 = = -1
b -a
(e 0) 2 0 2 1
Here, G 1, -1, i, -i) and(-1*1)x1 = -1x1=-1 1e. -a is the inverse
303 1 4
a) Since the element of a. o4 3 2
product
of any two
elements of Gis
-1x
(1*1)=(-1*1) *
This result is true for all of
also an element of G, so G is closed elemen
under x.
Associativity is satisfied.
ics-l
68 Asmita's NEB Solution of Baslc Mathemaric Next Part
ELEMENTARY GROUP THEORY Unit 3 69

: Z) m e
that 1 is the
idenuy
Let G(2m V -(u1, uz ui) - u e V for all u e V. sOLUTION
rom above table, we see Closure property:
Let
2, 2n e G. Th. So, each elemernts of v possesses an inverse. We know that an element a in a group (G,o)
element. 2mn e G for all m, n e Z has an inverse a-l such that a oa-l = a l o a = e
The inverse of 0 does not exist. 2 2 G is closed under multiplic Hence (V, ) 1s a grouP
where e is the identity of element of G.
From table, Hence,
Let 2m, 2n, 2
37. 2072 8ot CQ.No.5b OR
1 x1 1 i. Associative property: If a, b e (G, 0) Where G is a group, prove If possible, let a' be another inverse of a.
Then, a o a' = a'ba = e
2 x3 1 Then,2 (2 2)= 2m -2n*p =2m*np () (aob)1=b"% a-l (i) (a-"-1 =a 4
2P= 2m*n+p sOLUTION Now,
3 xs2 1 2"=2m+n
and (2m 2) Let G be a group and a, be (G, 0) a'oe (as e is the identity element)
4xs 4 1
1, 3, 2 and 4 are inverse elements
of 1,2, 3 ana
2m (20 2")=(2m 20) 2 for all m,n
is satisfied,
i. (aobjo (b-loa-i)= (aob)ob-'}o a-1 by associativity) a'o(a oa-") (as e =aoa-')
So, associativity (ao (bob-))o a (by associativity)
(as bob-1 = e)
(a' o a) o a-1 (by associativity)
respectively of identity: Let 2m (aoe )oa-1 (as a' o a e )
ii. Existence
e G. Tho
hen ta a0a (as e is the identity element) e oa
31. 2071 Set CQ.No. 5 is 0 e Z such
that 20 e G. a- (as e is the identity element)
Given the algebraic structure (G, )with e
a a-i1
the cube root ot Now,2m 20 2m0= 2m Similarly, (boa-") o (aob) = e
G{1, o, o} whereo represents This shows that the inverse elements is
ot 20+m=2m
unity and for the binary operation
stands and20 2m =

.(aob) o (b"oa-")= (b"%a-')o(aob) =


e
unique.
element in G. Hence b-oa-l is the inverse of aob.
ordinary multiplication of complex numbers, 141 20is the identity
show that (G,) is a group. iv. Exristence of inverse: Let 2mEG. Since me (aob)-l =b-l%a-1 42. 2072 Supp O.No. 5b
SOLUTION -m e Z such that 2-m¬Z. i. We have, a oa =e Given
G
that the algebraic structures (G, ) with
1, whereo)
Please refer to Model SetI1, Q.No. 55 represents
Operating both sides on left by (a-), we get o, o an
imaginary
Now, 2m 2-m-
2m-m =20
(identity eleme cube root of and
(a-1)-1 o(a- o a) = (a-")" o0e unity stands for the binary
32 2071 Set CO.No. 5 bOR Hence, 2-m is the inverse of 2m.
(a-1)1 operation of multiplication, show that (G, ) is a
If a, b, the elements of
c are a group (G, "), prove From (i) (iv) we conclude that G is a or ((a-1)-1 o a-l) o a
=
(by associative law)
or, e o a = (a-l)-1 8roup.
that under multiplication.
a*b=atcab=cand b *a=c*'a»b=c
a (a Please refer to Model SetI, Q.No.5b
sOLUTION *al= a ase
36. 2072 Set CQ.No.5b
Show that the set of all vectors in space un 38.2072 Set DQ.No. 5a 3. 2073 Set CQ.No.56 Prove that
Sine a e G, has
the inverse a1 such that a
Let (G, ") be a group. If a, b e G, then prove that Define Abelian group. a group G is
Abelian if and only if (a o bi = al o bi for all a,
We have, a*b=a'C..) addition is a group. ( ) (a * b = b 1 * arl a n d (ii) ( a - 1 ) 1 = a
Operating both sides of (i) by a on the left | soLUTION 4
refer to 2072 'C' Q.No. 5b beG.
we have Let V be the set of all vectors in space. Also, letf Please sOLUTION
a-1 ( a b ) = a-1* (a *c) operation of addition of vectors be denoted by 39. 2072 Set DQ.No.5a OR Abelian Group: A group (G, 0) is said to be
or, (a-1 *a) b= (a-i*a) "c (by associativity) Define a group. Let a, b, c and x be elements of a an abelian group if aob = boa for all a, b e G.

OT, e'b =e'c (as a-i"a = e) () Closure property: Let u =


(u1, uz, us) and group G. Solve the following for x: x2 = a2 and Second part

b=c (as e is the identity element) V (V1, V2 Vs) be any two vectors in V. =e Suppose (G, 0) is abelian.
sOLUTION Then, aob = boa, for all a, b e G.
Again, we have b"a =c'a ..i) Now, u + v = (u1t Vi, u2t Va, us + V) e First Part: Please refer to 2069 Set A QNo. 5b
Operating both sides of (i) by a-l on the right, Now, (aob) = (boa)-" (- aub = baa)
Next Part
we have u+vEVfor all u, vev Given equations are: a- ub- (xoy) y-loxl for
Hence V is closed under addition.
(ba)a-i= (ca) *ai .. all x. y in a group G|
or, b* (aa-')=c (a'a") (by associativity) (ii) Associative property: Let u = (u, uz and = e ..(i) (aob)-l a'ob-l
or, b'e=ce (as aal =e) be any
Let be the binary operation on G. From (ü), Conversely, suppose that (aob)-1 =aobr
V
(V1, V2 Va) and w =
(wi, W2 ws) tnree =e or, b-barl = a-lubr!
bc(as eis theidentityelement) in V.
Or, X (r2 2 ) =e or, bo(b-oa-') .b = bo (a-"ob-"}ub
33. 2071Set DQ.No.6b Now,
Or (a4 a)=e using )] or, (bub-l)o (aab) = (buar )o(b-ob) (Associativity)
(Wi, W ws)
*
A binary operation defined
the set S= fa, on b, c) u+ (v+w) =
(un, u2, us)+ ((v1, V2, Va) +
or, x a = e or, ev(a'ob) = (bua-1)oe (Existence of inverse)
is presented in the following Cayley's table. ( u , uz, ua)+ (V1 +Wi, V2 + W2 Va t W ..ii)
Since a e G, a a a a =
at e G. Since Gis a or, a lob = bua-1 (Existence of identity)
*abc (ui+Vit Wi, uz t V2 + W2, u *vi*
8Toup, each element possesses an inverse, so or, ao (a-'ob)oa = ao(boa-"Joa
( ( u i + V1)+ wi, (uz+ v2) + W (3
1a bc ( a e G such that (at) (a)-l e . Operating| or, (aoa-")o(boa) = (a»b)o(akoa) (Associativity)
b |bl (uj+V1, U2 + V2, u3 + va) + (Wi, W2 o
(U1, u2, u3) + (V1, V2 va)) + (wi, W both sides of (ii) by (a)l on right. or, ebua) = (aubJue (Existence of inverse)
Lcca b (x a) (a'):1 =e (a') or, bua= aub for all a, b eG(Existence of identity)
Showthat: (S, ") forms a group.
sOLUTION 14 (u)w
u+(v w) = (u + V) +w for all u, v, W
or, X(a (a)-") =(a* (by associativity law) Gis abelian
or, x e = a4
Please refer to Model Set1, Q.No.5b Hence, associativity is satisfied. a+ (as e is the identity element)_ 44. 2073 Set GQ.No.5b OR
Show that the set of all positive rational numbers
34.2071 Set DQ.No.6 bOR (1) Existence of element: Let u =
(u, u,
u 40. 2072 Set EQ.No. 5b
Let a, b, c be the elements of a group (G, ab
defined by a«b=forms
") how that the set T (-1, 1) forms a group under under the composition
isa vector 0= (0, 0, 0) in V such thatu*
=
i. If a* b=b, prove that: a = e
ii. Ifa"b=e,prove that: b = a1 multiplication operation. 4 agroup.
SOLUTION 41 us)+(0,0,0) (u1, uz, us)= u
Please refer to2070Set C QNo. 5b_ sOLUTION
Please refer to 2070 Supp Q.No. 6a Similarly, 0+u u 41. 2072 Set EQ.No. 5b OR set of all rational Let O denote the positive
Thus, 0 e V is the identity element numbers. Here, the operation 'on Q* is
tOve that every element in a group (G, o) has
35. 2071 Supp.Q.No.6a (v) Eristence of inverse unique inverse. 14 defined by a *b=
Define group. Verify that :
Z is a group me
Let u (u, =
element inverse
ofu
with respect to multiplication. uz, u) e V. Let v be the
the
SOLUTION 14 u+V 0
First Part: Please reler to 2069 Set A Q.No. 5b or, (u, uz, us) + v- (0, 0, 0)
o (aob)=
Mathematics-"

'oa ') 0a ")


(b"adb ELEMENTARY GROUP THEORY Unit 3 71
70 Asmita's NEB Solution of
Basc
(aob) o(bthe inverse of
*b=
1 is

b e Q, oa sOLUTION
Let Q denote the set of all positive rational Closure property: Let ü - (u1, uz, us) and
property: Since a,
i . c . ( a o b * D ' 0 a

**
osure 6b
SuppQ.No. numbers. Here, the operation * on Q' is
V-(v, V2v) beany twovectorsin V.
5EQ'.Hence, Q is closed under 7.2073 clian 8(a"b)-l a1* group.
If G, ) is
an
i. Associativity: Suppose a, b, c eQ
Define

show
abelian

that
=
b' a,b eG.
a-1* abel ab
defined bya*b- Now, u v (u1+ V, uz + Vz, us + vi) e V
gTup,
2073 Set CQ.No.
5h u v e Vfor all u, v eV
Please refer to . Closure property: Since a, b e Q', so a*b
on
SuppQ.No.6b is an abelia
Hence V is closed under addition
Then, a (b*c) = a*5 . . 48. 2073that
Verify (2m : m Z where
e
abelian
Z
group wi
is th
eQ'Hence, O"is closed under. ii. Associative property: Let u (ur, uz us), =

to
multiplication,
i. Associativity: Supposea, b, ceQ'.
respect ( , V v) and w - (wi, wz w) be any three
vectors in V.
integers,
Now,
and (a"b)'c = SOLUTIONN

For a group.
Q' o,
Supp Q.No. 6a Then,a (b*c) =a* u+(v+w) = (u1, u u)+ ((Vi, Vz V) + (Wi, Wz w )

for all b, c to 2071


e
b)* a.
Please refer
*
c
a
*
(b'c) =
(a ( h , Uz th* (71+ Wi, V2+ Wz V+ n)
assoaativity is satistied. For abelian group,
be an idenaty
Let e EG. Then ((u1 + vn) + wi, (u+ V2) + wz (u + V) * wi)
ii. ldentity element: Let 2, 2
element Ife e Q, then 2m.2n2m*n and (a"b)'c-e abc
4 (u1, z ) + {7i, Vz Vi)) * (Wi, W2 W}
aesa 2n.2m 2n*m= 2m*n
and a (b c) = (a ' b) * c for all a, b, c e Q.
Hence, 2m, 2n
2.2m for all m, n eZ
So, associativity is satisfied.
- (u )+w
Or,
Hence Gis an abelian group under multiplicati ii. Identity element: Let e be an identity u(V+w)-(a 7) w for all u, ,weV
Hence, associativity is satisfied.
e=5
5 is the identity element
49.2074 Set AQ.No. 6a of
group (G,)..
element. Ife e Q', then
ii. Eristence of element: Let u =(u, 2 m) e V. There
elements tne
and a *e =a
element: Let a' be inverse elementt
a
and b are=e, prove that b =a-1 is a vector 0 (0, 0, 0) in V such that
W.
Inverse
an
i Ifa *b
b=b, prove that
=
* a e. u 0 (u1, uz u)+ (0,0, 0)= (ui, uz u)= u
of a. If a e
i. ifa or,

reter to 2070 SuppQNo. 6a


then a ' a' se=5

Please e = 22
Similarly, 0 u u
, 50.2074 Set BQ.No. 5b 2is the identity element. Thus, 0 e Vis the identity element
Show that {2 neZ} is an Abelian group wit Eristence of inverse elementE
iv. Inverse element: Let a' be an inverse element
t, a' respect to multiplication. of a. If a' e Q, Let u (u, uz us) e V. Let v be the inverse of u.
Please refer to 2071 Supp Q.No.6a then a a' =e=2 Then, u+ v =0
i s the inverse element of
d
a.
51.2074 Set BQ.No.5b OR or,
a a'
2 or, (u1, u2 ) + v= (0, 0, 0)
Hence, from (i) - (iv), (Q,") is a group.
Ifa, be (G, o) where G is a group. Prove that: V - u , uz u») - -u e V for all a e V.
Here, G= {0, 1,2 i (aobl= bloa1 or, a' 0 , each elements or v possesses an nverse.
Composition table for Gunder*s i (al}l =a. Commutative property:
Please refer to 2072 Set CQ.No. 5b OR i s the inverse element of a. Let u = (u1, u2 » ) and

52. 2074 Supp Q.No. 5b Hence, from (i) (iv), (Q", ) is a group. v (V1, v2 V3) be any two vectors in V.
Let G {1, -1, i, -i). Show that G forms a grou Also, for commutaive property, Now, u +v = (u1+ V1, u2+ V2 us+ V3)
under the operation of multiplication.
From abOve table, 0 is the identity element.
Here, 2 1 =1 +:2=0 Please referto 2069 Set A Q.N. 5b Letab vu (v1* u, Vz 2 V w)
53. 2074 Supp Q.No. 5b OR (u+ V, u2+Vz u+ vi)
1 isthe inverseelement of 2 Let G be a group. If a, be(G,+), and ba- u v v* u for all u, v eV
45. 2073 Set DQNo.5b prove
Given the aigebraic structure (G,) with (ab)= b1 *a1 a*b=b*a for all a, b e Q Hence, commutativity property is satisñed.
V is infinite set. Hence,
learly
Hence, (Q,) is an abelian group (V, +) is an ininite Abelian
G={1, o, o} where o represents the cube roots of Please refer to 2069 Set A Q.N. 5b OR
57. 2076 Set CQ.No. 5b group
unity and stand for the binary operation of 2076 Set AQ.No. 5 Detine a group. Prove that the set of all three 58. 2075 Set CQ.No. 5b OR
ordinary multiplication of complex numbers, a 8
Show that the set of integers Z forms If a and b are the elements of group (G, ) such
show that (G, ") is a group. dimensional vectors form an intinite Abelian
141 under the operation of addition. that
Please refer to Model Set l, Q.No. 5b Please refer to 2070 Set DQNo. 5b 8rOup under vector addition.
46. 2073 Set DQ.No.5b OR 56. 2075 Set AQ.No. 5b OR SOLUTION
First Part: Please refer to 2069 Set A Q.No. 5b
i
ii
a b=b prove that a = e.

If a, b e (G, o), prove that (a o bi = b'oa, a*be,prove that b=art


a n d bare the elements of a group (G, pro Next Part: Let V be the set of all vectors in space. Please refer to 2070 Supp. Q.No. 6a
sOLUTION that a "x =b Also, let the operation of addition of vectors be
(a o bho (boa-l) (aob) ob-") oa"(by associative
= has a unique solution in (G
law) lease refer to 2070 Set DQ.No. 5D denoted by +.
(ao(bob )) oa- (by associative law)
(aoe) oa "(as bob-l = e)
56. 2076 Set BQ.No. 6a
a0a (e s the
jdentity element)
=e Let be defined on Q* b y a b=Show
Also,
(b-oa-1) o (aob) = (bloa-) oa) ob
(b' o (a oa)) ob
(Q)is an Abelian group.
( b ' oe) ob = bl ob= e
CONIC TIONS
phadka n i t4
73

UNIT a-
Focus of the parabola (h + a, k)
m--2,
Putting m=-2 in (),

CONIC SECTION
)-)
Vertex of the parabola= (h, k) = {
y -2x 12+9
or, 2x+ y +3 =0 pha

2084 Q.No.6 Again, putingm-|in ()


2060 Q.No.2 Find the equation of the parabola in which the

Find the equation ofthe


normal to the ends f the latus rectum have the coordinates y-6n9
A. PARABOLA
yi= 5x
perpendiçalar
to theline x+2y= parabola (X5) and-1-11) and the vertex is (-5, -3).[2] or, 2y =x+24
2 MARKS QUESTIONS SOLUTION oT, x-2y +24 0
SOLUTION Required equations of tangents are 2r+y+3 =0
5r with y2 =
4ar Since the end points of latus rectum are (-1, 5)
2057 Q.No. 2 directrix of the paraboli
Conypáringy2= have and (-1, -11), so the equation of latus rectum and x-2y+ 24 =0.
Find the foçs and Aar5
is x-1. So it is parallel to x-axis. . 2068 Q.No. 2
- 4 y - 3 20 0
Here, vertex (h, k) = (-5,-3)

sOLITION We know that the equation of parabola is


Prove thatthe line k+ my +n
parabola y2 = 4ax if In = am?.
=
0 touches the

Given parabola is
2
+2y 7 is(m)= y-k24a(r -h) SOLUTION
Slopeofthe line
=
x
y2-4y 8x-20=0 or, (y +3)2 = 4a(x + 5)
Given parabola is y2 = 4ar
J=T1-
is
or, y2-4y
+4 20+4 8x + Since the normal perpendicular to the give If () passes through the point (-1, 5), then
+ 24
Equation of givenlineis lx + my +n =0 24.
or, (y-2)2 =8x (5+3)2-4a (-1+5)
or, y-2}? = 8{x + 3) line, so its slope (m) = =2 or, my= lx - n
64 16a
with
Comparing this equation Now, the equation of normal is .'. a=4 or, y m m
- k =4a(r - h), y
mx-2am -
am*
Putting the value of a in (i), we get We know that the line y = mr + c will be a
we have,h=-3, k=2, 4a=8 (y+3)2=4 x 4(r + 5)
as 2 ot, y-2x-2.2-.2 or, y2 + 6y +9 16x +80 tangent to the parabola y2 = 4ar ifc =
Focus (h +a, k)= (-3+2,2)= (-1,2
or, y = 2-5-10 y 2 + 6y - 1 6 x - 71 = 0
So, the given line will be a tangent to the
Equation of directrix is y 2-15
X=h-a 8. 2066 Q.No. 5 c given parabola if
or, X - 3 - 2 5 2062 Q.No.5 Find the eguatíons of the tangents from the
joining th point 9 ) to the parabola y2 = 24x m 1/m
X+5=0 Determine the equation of the chord4ax. 2
poinés ti and th on the parabola y2
=

2 2058 QNo.2 soLLTIONN or, ma


Comparing y2 = 24x ... i) with y2 = 4ar, we get
Find the equation of the tangent to the parabola | soLUTION
y2=16x at thepoint (4, 8). The equation of chord joining the point ti an 4a= 24, n am
SOLUTION ie. (at, 2ati) and (at:?, 2at:) of the paraboi a6 h0 2069 (Set A)Old Q.No.5
16x y4ar is n d the equation of the normal to the parabola
the equation of parabola y2
Comparing
=

y24ax at the point ( yi). 121


with y2= 4ax, wehave 2at- 2at
y-2athat?-at? ( -at9 SOLUTION
4a = 16

(-6,9)- The equation of tangent to the parabola y2 = 4ar


. a

The equation of tangent to the parabola at


Or
y-2ata(tu-t)(h+h)-at >X
at ( yi) is yy1 = 2a(r * n)

(x1, yi)= (4, 8) is or, (y-2at) (ta+ ti) =2(x- at)


yy2a(x+X1) Or, (ti+ ta) y - 2at1t2-2at2 2x- 2ati*
So, slope of the tangent=
slope of tangent -
y 8-2 4 (r + 4)
2x-(t+ t)y+2at1ta=0 The slope of the normal 2aa
y*x+4 62063 Q.No. 5 Hence the equation of the normal is
B 2059 2.No.2 Find the coordinates of the vertex and the to
The equation of line through the point (-6,9) is
Find e focus and directrix of the
parabolaofthe parabola whose equation is y -9 m (x + 6) y-y2-a)
2y. 121 6y-12x +45. Or, y = m t 6 m + 9
..i) 11. 2069 Old (Set B) Q.No. 2
SOLUTION
Comparing r2= 12y with x = 4ay, we have
sOLUTION he line i) will be tangent to the parabola
(i) if
Find the equation tangent to the
of the parabola
Given equation parabola
of is y2 16x at the point (4, 8).
4a = 12

y 6y-12x+45 óm
Please refer to 2058Q.No.2c
a =3 or, y - 6y +9= -12x+ 45+99
Focus= (0,a) = (0,3)
or, (y-3) -12x + 54
=
2Find2071 Supp-0.No.2a
the equation of the tangent to the parabola
or, 6m+ 9m= 6
The equation of directrix is
.0 or, 2m2+ 3m -2 0 y9xat (4,-6). 121
y= -
or, (y-3--12(- or, 2m2+ 4m
or, y=-3 h), wep m-2 0 SOLUTION
or, 2m (nm+ 2)-1 (m+2) = 0 Given equation of parabola is y2 = 9ax
4a (
' y +3 =0 Comparing (i) with(y k -
=

or, (m+ 2) (2m 1)=0


-

h-k3
4a- -12
74 smita's NEB Solution of Basic Mathematics- = 4ar
(mr +c)2 cONIC SECTIONS Unit 4 5
2a)x + c2
=
00
2(mc line will be a tangent (üi)
-

Comparing it with y2= 4ax, we have The straight


m2r2+ to
4a-9
or,
So, slope of the tangent 2a SOLUTION
the two values of x c y
equation if(ii) are equal. This ed6
be Comparing the given parabola y2 = 8x with
a-
parabola
trom The slope of the normalslope of tangent 2 a y2= 4ax, we have
inant of (ii) =
0 4a 8
Ihe equation of tangent to the given parabola discrimi m2.c2= 0 Hence the equation of the normal is a= 2
at (ru yi) = (4,-6) is ie. (2 (mc-2a)2-4
4m? c?
=
0 The equation of the tangent to the parabola is
(mc -2a)2 y-y-2 -i).6)
-

y. yi 2a (x+ ni) or, 4


mc?=0
mc-4amc
+
4ad - y mx+
or, If m be the slope of the normal, then m
y(-6)-2*á a+4) or,
4amc
= 4a2

9 m- or y =
mx
*
or, 6y-(+4)
yi-2am The tangent (i) passes through the point
7. 2060Q.No. 9
Since (1, y1) lies in y2 = 4ax,
(-2,3), so
oF, -2y-a+4 tangent t the
Deduce the of equation so, yi2=4ar1
or, 4y =3x+12
parabola y2= 4ar at (L Yi) on
the
parabola. Putting y1-2am in yif= 4ax1, we get 3=-2m +m
3x+ 4y +12 =0 Please refer to Model Set I1, Q.No. 6a (-2am)2= 4ax1 Or, m=-2m2+2
X1 a m 2 or, 2m2+ 3m -2 0
13 2075 Set BQ.No.2a 18 2061 Q.No.9
equation of the chord joining the | Prove that the latus rectum
Putting the values of X1 and y1 in (), we get or, 2m2 +4m m -2=0
of a parabola sect
Determine the 4ax. y +2am= m(r-am2) Dr, m (m+ 2) -1 (m+ 2) =0
points t and t on the parabola y?=
Please refer to 2062 Q.No. 5c
the angle between the tangent and the norm
y mx -2am - am' which is the required or, (m+ 2) (2m - 1) = 0
latus rectum.
either extremity of the equation of the normal to the parabola
4 MARKS QUESTIONSs
SOLUTION y2 4axin slopeformn.
.
m--2
4 2057 QNo.9b the
Let the equátion of parabola be y2 = 4ar. Ihen
20. 2063 Q.No.96 Product of the slopes (-2) *-1
Find the equation of the parabola in its end points of the latus rectum PQ ae
Prove that the line lx + my +n = 0 touches the
standard form y2= 4ax Pla, 2a) and Qla, -2a).
Please refer ó Model Set II, QNo. 6a o parabolay4ax if In =
am2 Hence the tangents are at right angle.
P(a, 2a)tangent SCLUTON 25.2068 QNo. 9b
5Show
2058thátQNo. 9
the normal to the parabola y2= 8x at
Given parabola is y2 = 4ar Find
the coordinates of the focus, the vertex, the
(2 4heets the parabola again in (18, -12). 14 Equation of given line is lx + my + n =0 eguation
latus
of the directrix and the
length of the
rectum of the
or, my = lx - n
parabola
y2 = 6y -12x + 45. [4]
scLUTION normal sOLUTION
The equation of tangent to the parabola or, y m *
X Siven equation of parabola is
O R
y2-8x .6) at (24)is We know that the line y2 = 6y - 12x + 45
y
=
mr + c will be a
y 4 4x+2) tangent to the parabola y2= 4ar if 6y +9= -12r+ 45+9
y =x+2 or, (y -3)?= -12r+54
C
Slope of tangent=1
Q(a,-2a) So, the given line will be or, (-3-12(-
Slope of normal slope of tangent a
tangent to the
The equation of normal at (2, 4) is Slopeof PQ=-2a
aa
-2 o given parabola if Comparing (i) with (y - k} = 4a (x- h), we get

to the axis ot t
y-4-1(x - 2)
or, y -4=-x +2
So, PQ is
parabola i.e. ZPSR = 2
perpendicular
PSA = 90°.
m /m hk3
4a = -12
X+y=6 (ü) Again, the equation of tangent at P(a, 2a)15 m2 a
Solving () and (i), we get y 2 a = 2a(x + a)
as 3
n=am Focus of the parabola (h+ a. k)
=

or,
y2-8(6-y
y2 + 8y 4 8 = 0
or,y x *a
21. 2064 Q.No. 9 b
is 1.
Slope of tangent at P(a, 2a)
or, (y+12)y-4)=0
ys4,-12
When y = 4, x =6 - y =6-4=2
i.e. tan 2PAR = 1 = tan 45°
PAR =45°
Deduce the
equation of the parabola in the
standard form y2 4ax. =
4
)-)
When y = -12, x = 6 - y =6-(-12) = 18
nen, from AAPS, ZAPS = 180° - 90°-4 Please refer to Model Set Il, Q.No. ba or Vertex of the parabola = (h, k)

The two points of intersection of the normal


F 45° 22 2066 Q.No 9
right ange
Equation of directix is x = h - a
Since tangent and normal are at ind the equation of the tangent to the
and parabola are (2, 4) and (18, -22). Hence, if
the normal is drawn at (2, 4), it again meets
S0 ZRPS=90° - 45° = 45°.
betwee
y 4ax at a point (v, yi) on the parabola.parabola
the patabola () at(18,-12).
Latus rectum PQ bisects the ange
P
Siml Please refer to Model Set ll, QNo. a 2x- 15 0
the
tangent AP and normal PR a 23 2066 CQ.No. 9
6 2056 O.No.9D that result holds at another end Q na the equation of the parabola in the
Length ot latus rectum =|4a| =14 * (-3)|
1 - 1 2 | = 12
Findshe condition the line y mx + c
may be 19 2062 Q.No. 9 b standard form y2 =
4ax. 14
a tahgent to the parabola y2= 4ax. 41 Find to the
p a r a b

Please refer to Model Set H,QNo. 6a or 26 2069 (Set A) Q.No. 6


SOLUTION the equation of the normal Find the equation of the normal to the parabola
Let y
y4ax in the slope form. 24 2067 Q.No. 9b y 4ax at the point (x, yi) and express this in
mr + c ...0) be a
straight line and SOLUTION
y2 = 4ax.(i) be a parabola. how that the pair f tangents from the
point
slope form.
41
From (1) and (i)
The equation of tangent to the parabola y?* 9 ) to thparabala yi=x a r e at right angle.[41| Please reter to 2062Q.No. 9b

at (1,
y1) is yy 2a(x+ Xn)
of Basic
Mathematics- CONIC SECTIONS Unit 4 77
76 Asmita's NEB Solution
From () and (i), 2 -
6 2071 Supp.ofQ.No.6b (m)
9
27 2069 (Set A) Old Q.No.parabola y=6y-12 Find the the
triangle formed by the lines
area
Given an equation of the diecti the parabola yt 12x to the =
Here, 9 45°
focus, vertex, equation
of the
141 or, oining the vertex of
ind the rectum.
rectum.
ends of its latus We have tand= mi-m2
length of the latus
and the t j mim2
9x SOLUTION
Please refer to 2068 Q.No. or,2( of
Comparing the equation parabola y? =
12r| or, tan 45°
28 2069(Set B) Q.No.6aunder which the line with y2= 4ax, we have 1+ m1/2
condition or.a 4r+4) 9
the 4ar.
Find 4a 12
1-2
=

the parabola yi
is tangent to 3
my*c in slope torm. =
8r a
2+ 4r+4
FindAhe equation of tangent 4 or,
+ 4 =
0
So the end points of latus rectum are (a, 2a) Taking +ve sign
fipd the point of contact. r - 4r and (a,-2a) i.e. (3, 6) and (3, -6). 2+m = 2m -1
or,

SOLUTION or, (r-2)2 =0 Vertex of the parabola is (0, 0) .. a =3

First Part
=2,2 Again, taking -ve sigrn
touches the parabola".
Please refer to 2059 Q.No.
9b Since the line (a,2a) 2+m=-m+1
coincident points, so it is a tangent
Last Part in the
to the parabola
parabola. Putting
x
=
2 in (i) .
m-
The equation of tangent
slope form is 3 When m=3, the equation of tangent is
y 2 - =-3

y m m (0,0 y-3x
the parabola Point ofcontactis(2,-3)
the equation of tangent to or, 3x-y +1 =0
Again,
is 32 2070 (Old) Q.No. 9b
y2= 4ax at the point (x1, yi) at point (at2 ,2at) on (a-2a) When m=, the equation of tangent is
yyi = 2a (r+ x1) u) If the normal any
parabola y2= 4ax meets the curve again Now, the area of triangle with vertices
Equation (i) and equation (ii) wll represen (x1, yi) = (0, 0), (*2, y2) =(3,-6) and y 1/3)
the same line if
Qlat, 2at), prove
that
t+ =0.
(3, 6) is
(x3,yo)
=

or, 3y=-x -27


OT, I * 3y + 27 0
SOLUTION A1
2
yi
y2 1
The equation of the tangent at P(ati2,2at)
From first two ratios
X3 y3 T
Hence the required equations of tangents are
3x - y+1=0 and x +3y + 270
y 2 a t 2 a (x + ati2) 0 We know that the point of contact of tangent
oT, yt=r+ ati2 - -6
Slope of the tangent 6
yi m
From last two ratios When m = 3,the point of contact is

Slope of the normal slope oftangent


LExpanding along Ril
a/m
22 2ax The equation of the
y - 2 a t 1 - t 1 (x - a t )
normal at P(atr, 2 36) 18 sq units. )G)
1 1m2 Since it passes through the point Q(at, 37 2072 Set CQ.No. 6aa When m t h e point of contact is
The point of contact is (i, y) = m SO
Show that the pair of tangents from the point
or, 2a(t - ti) = - at1 (t2 - ti) (ta+ ti) (-2, 3) to the parabola y2 = 8x are at right angle.
-173pP173 - 1 9 )
or, 2= -t (th + t)
29. 2069 Oid(Set B) Q.No.9b Please refer to 2067 Q.No. 9b Note: The point of contact also an be obtained b
Findthe equation of the parabola in standard or, = -(t, + t) solving the equation of tangent and equation of
form. 141
38 2072 Set DQ.No. 6a parabola.
if the tangent to the parabola y? = 12x makes an
Please refer to Model Set II, Q.No.6aor
30. 2070 Set CQ.No.6a . angle
find
45° with the straight line r 2y +3
its equation and the point of contact.
-

=0, | 39 2072 Set E Q.No.65


141 Find the condition under which the line
Find the equation of the tangent to the parabola mr * c is tangent to the parabola y* 4ar. Find =

. 2071 Set CQ.No.6 a


=
y
y2 = 4ax at the point (,y1) to the
parl SOLUTION the equation of the tangent in slope formn.
14 Find the equation of the normal Comparing the equation of parabola y* = 1x soLUTION

Please referto Model Set II,Q.No.6a y 4ax in the slope form. with y2 = 4ax, we have
Let y = m *c...() be a straight line and
1. 2070 Set DO.No.6a Please refer to 2062 Q.No. 9b 4a 12
Prove that the line 3x 4y + 6 0 15 tangent to|4. 2071 Set D
+
a =3
y 4ar. (ü) be a parabola.
the parabola 2y2 = 9x. Find its point of contact.14| Q.No.6 a r a b o l a

The equation of tangent to the parabola is


From (i) and (i)
Find the equation of the parave (mr*c=tar
SOLUTION standard form y2 = 4ax. y ma+ or, mr +2(mc 2a)x + c
-
=
0 (i)
Equation of line is Model Set l1, Q.No The straight line will be a tangent to the
3x + 4y + 6 - 0 Please refer to 3
or, y =
() parabola if the two values ot x obtained from
85. 2071 Old Q.No. 9 rabola A mx +
m equation (iüi) are equal. This will be so if
or. y - Deduce the equation of the pa Slope of tangent (mi) m discriminant of ( i i ) = 0

standard from y? = 4ax. ba or Slope oftheline a - 2y+30IS i.e. (2 (mc - 2a)]-4 . m2.d=0
And equation of parabola is 2y2 = 9x Please refer to Model Set 1, Q.No
(i)
Mathematice-"
cONIC SECTIONS Unit 4 79
of Baslc
78 Asmita's NEB Solution
, the equation of
hen m
tangent Since y? = 4axo, so
or, 4 (mc- 2a)?-4m? c=0 -1
Slopeof L'Z(m:) =
y iS
- a

or, me- 4amc+4a2 mic=0 Since, mi. ma =1 x (-1) = -1, LZ and L'Z are
-

4axo
or, 4amc = 4a2

or, 3y -*-27 perpendicular to each other.


or,
tangent
to or,
r+ 3y + 27 =0
equations of
Hence, 2LZL'=90°. Proved Xom
will be a
line y mr + c
Hence the required Set CQ.No.6 Then from (i)
49 2076 condition
=
the
nus

3r-y+1=0and x +3y + 27 =0 n of tangency of straight line a


the parabola y? = 4arifc= Find the a/m2
4ax. 14 mx +
2a 2a/m
2073 Set DQ.No.6a mx + cto a parabola y2
=
y
to the parabola 4 2 yPlease refer 2059QNo. 9b
So the equation of tangent
Find the equation of the tangent to the y mxm which is the equation of tangent
slope from is y = mr m
y24axat the point (XL yi). 6 MARKS QUESTIONs
Please referto Model SetII, QNo, a in slope form.
40. 2072 Supp Q.No, 6 or
2073 Supp Q.No.6a
60 2070Supp. Q.No.9 B. ELLIPSE
Define conic section. Find
the equation t4 Find the condition at a line y Prove that the tangent to parabola y2 4ax at
a =
a

parabola in its standard


form.
4ax.
mx + point (xo yo) on the+ parabola is given by
the| 2 MARKS QUESTIONs
parabola y2
=
the
Conic Section: tangent to equation yyo
=
2a (x Reduce the
Xo). equation in
No. 8a Please refer 2072 Set E Q.No. 6b
Please refer to Model Question
Set 1 Q. slope form. 16]1 2069(SetB) Q.No.2
Find the
42074 Set AQ.No. 6 eccentricity
and the foci of the ellipse
Next Part
Q.No. 6a or
SOLUTION
Let P(ro, yo) be the point on the parabola y2 = Y-S =
Please refer to Model Set I1,
Find the condition under which the
y = mx+c is tangent to the parabola y?s 16 9
4ax. Let us take another point Q,
yi) on
41 2073 Setca.No. of the
the| sOLUTION
y= 1x makes a Also find the equation tangent iint parabola which is very close to P. Then,
Ifa tangent to the parabolaline ..() Comparing , 1
angle 45 with the straight x
2y *3 | slope form.-
yo 4aXo
find the equation of the tangent. Please refer to 2072 Set E Q.No. 6b & y12= 4ar1 .. (i)
Subtracting, we have withb, (y-kz
b2 1, we have
SOLUTION
parabola y? =12r 5. 2074 Set BQ.No. 6a y 2 -y o = 4ar1 - 4axo

omparing the equation of Show that the pair of tangents from the or, (yi-yo) (yi + yo) 4 a ( i - xo)
h -2, k 5
with y24ax, we have (-2, 3) to the parabola y2 = 8x are at right a a2 16, b2=9
4a 12 y1-y
1 - Xoy1 + yo
4a
(ii) a 4 b 3
Since a> b, the major axis is along r-axis
. a-3
The equation of tangent to the parabola is
Pleaser refer to 2067Q.No. 9b We have,
2074 Supp Q.No. 6a
Find the equation of the tangent to the panh Eccentricity (e)=\
ymm y = 4ax in the slope form. Also find the poit
P(xa,yo)
or, y mr*m ..) contact.
First Part: Please refer to 2059 QNo. 9b -X Foci a-(2:45)-2-.
Slope of tangent (m1)= m
Slope of the line x - 2y +3 0 is
Second Part: Please refer to 2069 Set BQN 2 2070Set GO.No. 2a
Find the eccentricity and the foci of the ellipse:
2075 Set AQ.No.6a
Find the equation of the tangent to the parn
(m y2 = 4ax at the point ( , y1). The equation of PQ is
Here, =45
Please refer to 2060Q.No. 95 SOLUTIONN
We have tan 6 = +y=m2
y-yo -Xo)
48 2075 Set BQ.No.6b
Prove that the lines joining the ends o or, y-yo+V xo)
mparing -1
with -L we

using (ii)] have


or, tan 45*1+m1/2 of the parabola y2 4ax to the po a 9, b2 16
rectum
=

intersection of the directrix and the axs


By definition, PQ becomes the tangent at P
when QPie. when 1>Xo and yi > yo a=3, b4
or, 1
2m-1 right angles. Since b> a, so major axis is along y-axis.
2+ m Hence, the equation of tangent at P is
We have
Taking +ve sign SOLUTION a
La, 2a) and L'(a, -2a) be the ends
poin y-yo 2yo- Xo)
2+ m 2m -1 Let
the latus rectum of the parabola or, yyo- yok = 2ax-2axo
Eccentricity ()*
m= 3 Or, yyo- 4axo 2a - 2axo
Also, let Z (-a, 0) be the point ot using()
Again, taking -ve sign
of directrix and axis of the paraboia Or, yyo= 2a(x+ xo) which is the equation ot Fiai-(0 h)-(o. -0)
2+m=-2m+1
La2a) tangent to the parabola y 4ar
a3. 2070 Set Da.No. 2
m-
(Xo, yo). Find the eccentricity and the foci of the ellipse
The above equation can be written as
When m =3, the equation of tangent is (a.0 y 2ato
3ud+4y= 36. 12
yvo yo .() SOLUTION
y-3 Writing v2a m then yo 2a
m
Equation of ellipse is
3a2+ 4y2= 36
or, 3x-y+1 =0 L'a,-2a
Now, Slope of LZ(m»)a -a
0-21
80 Asmita's NEB Solution of Basic Mathematics-
cONIC SECTIONS Unit 4 31
S6. 2072 Set EQ.No.2a
o. Find the eccentricity and the foci of the ellina.
ipse
a 5 , b=4

b, the major axis is along x -

axis.
SOLUTION
Since a
Focus (0,-be) =
(0, -5)
Comparing ) with 1,we have We have, be
)
a2 12 b=9 sOLUTION e And e-
a=23,
Since
b=3
b, the major axis is along r-axis.
a > Comparing 21with 1, Putting the value of e in (i)
We have, have
b2 25 Foci (htae, k) b 15
a29,
Eccentricity (e) =1 a 3, b= 5
Since b > a, so the major axis 1s along the
-(1 +5..0) Again, we have

axis.
We have,
= (1 +3,0)
= (4, 0) and (-2, 0). 1-
Foa-(t ae, 0)= o-e. Eccentricity (e) V1-
a MARKS QUESTIONS

200
4Find2070 (Old) O.No.2 0.2057 Q.No.9 bOR The equation of ellipse is
the equation of the ellipse in the standard Find the eccentricity, length of the latus rectum
form whose focus is at (-20) and vertex at (5,0).
[2
and coordinates of the foci of the ellipse
1
SOLUTION
Vertex ( a 0) = (5, 0)
Fodi=0,tbe)(0,+5.3 (0.+4) 4
20 1
sOLUTION
a = 5 S7 2072 Supp Q.No.2a 1 with 32060 O.No. 9bOR
Focus (-ae, 0) = (-2 0) Find the eccentricity and the coordinates of the Compa ng 1, we
Find the eccentricity and the foci of the
ae2 ellipse:
5.e-2 using )]
foci oftheellipse 1 2 have
a2-16, b2=4 ,v-21
3 2-1 4
Please refer to 2070 Set C Q.No. 2a 4

b 2 sOLUTION
We have, 58 2074 Supp C.No.2 Since a > b, the major aris is parallel to z-axis.

Find the foci of the ellipse


We have, Comparing 1
b a (1-)=5 sOLUTION Eccentricity (e)= with 1,wehave
h=0, k2
Required equation ofellipseis 1 Comparing + 1 Length oflatus rectum= x2=2 a 8, b2=12
with x- h S=1, we have a 22, b =23
Focd-(tae,0)-(+40a6.0 Since b > a, so the major axis is parallel to y-

2071 Setca.No.2
Find the eccentricity and the foci of the ellipse
h 0,
a2 8,
k
b2 12
2 1. 2058 Q.No. 9b OR axis.
We have,
Find the eccentricity and the foci of the ellipse.
25x2+4y2= 100.
SOLUTION
2 a-
22, b=2/3
Since b> a, the
kh
16
2 y-5=1 Eccentricity (e) =\
Given equation of ellipse is
so
major axis is parallel toy- 9
2512+4y2 = 100
axis. sOLUTION
100
252 4y2
100 1
We have,
Comparing , 1 Foci (ktbe)-(0.2:5
Eccentricity (e) =\/ 0,2+2)= (0,0) and (0, 4)
or. 1 with- ,V-SE -1, we have
a 84 2061 Q.No.5
ellipse in the standard

Comparing ) with -1, we get


Foci- kt be)-(0.2:a5) hs-2, k =5
16,
Find the equation of the

(0, 2+2)= (0,0) and (0, 4) a 4,


b2=9
b =3
position with a
focus at (0,-5) and eccentricity
a-4, b 25 4
a= 2, b 5 59. 2075 Set CO.No. 2a nce a>b, the major axis is along X-axXS
to 2059QNo. 9b OR
P l e a s e refer
We have,
Since b> a, the
We have,
so
major axis is along y-axis Find the foci of the
ellipe 1 Eccentricity (e)=V 85. 2062 Q.No. 9 bOR coordinates
and the of the
Find the eccentricity
SOLUTION =1 4
Eccentricity (e)
Given equation of ellipse is 1 .. Foci-(ht k) (-24s)-(2:7.5)
ae, =
foci
oftheelipse: 9b OR
Please reler to 2060QNo.
2 2059
Comparing ()with , 1
Deduce Q.No. 9b
the
standawne OR
equation of the ellipse in the
64. 2064 Q.No. 9bORand the foci of the ellipse:
Find the eccentricity
(0be-(o.s.)-0 we get, h
a2
1, k = 0
25, b? = 16
*position if a focus is at (0,-5) an9r+5y-30y= 0

eccentricity is
Mathematics- CONIC SECTIONS Unit 4 83
Q.No.9
Basic
82 NEB Solution of
6 2066
Smta's

of the ellipse in
the equation
Find the Q.No, 9b OR 3. 2069 (Sot A) Old Q.No. 9b OR
SOLUTION position whose
latus
rectum is e
throu
qual to 1 2068 of the vertices, thee Show that 9x
equation of ellipse
Given
93+ 52 30y =0
is
major axis and
which passes
through the pal Find the coordinates
coordinates
eccentricity and the100.
of the foci of the
+
5y2 30y
equation of an ellipse. Find the eccentricity, the
=
0 represents the
ellipse 25x+ 4y?= 141 coordinates of the centre and the foci.
or, 9r2+ 5(y2 6y) =0 (V6.1).
or, 912+5(y2-6y +9-9)=0 SOLUTION
Let the equation of ellipse in standard fom
sOLUTION
Given equation of ellipse is
sOLUTION
Given equation of ellipse is
or, 93+ 5(y 3)-45 =0 -

25r2 + 4y2 = 100 9x2+ 5y2 - 30y =0


or, 92+ 5(y 3)=45 (1) 25x2 4y or, 9 + 5(y2 6y) = 0
or -1 or, 100 100
1
of, 9x+ 5(y2- 6y +9-9) = 0

By question,
latus rectum major axia or, 9x2+ 5(y -3)2 -45 0
1 o, 1 . (1)
Comparing() with or, 9x2+5(y -3 45
wehave
k=3
Comparing ( ) with a * = 1, we get
or, -1 .0)
h 0,
a2 5 , 9 2b2a a 4, b2 =25
Since equation () is in the form
3 From a2, b 5 of D
=1,
a-sb > a
1
VE
it
Sunce so major axis is along y-xIS. 26 -1 Since b> a, so the major axis is along y-axis represents an equation of ellipse.
have We have,
Since it passes through the point (/6,1.
Eccentricity (e) =
Eccentrity()y1- -3
Coordinates of centre h, k) =
(0,3)
Coordinates ofvertices = (h. k+b) -(0,3t3)
= (0,3-3) and (0, 3 *3)
(0, 0) and (, 6)
Foci -(h. k be) -0,3+3.5 (0.3+ 2)
be)-(0.5 (0.1) and (0, 5)
Foci k be)- (0,33. -(0,3
b24
Putting the value of b? in (i), we get
(0,
Foci
-0+v 74 2069 Old (Set B) Q.No. 9b OR
72 2069 (Set A) O.No.6a o Find the equation of the ellipse whose latus
(0.1) and (0.5 Find the eccentricity, the coordinates of the
rectum is 3 and
7. 2065 Q.No9 bOR vertices foci eccentricity is 4
Show that: 9+4y2 - 18x - 16y - 11 =0 2+2y2=8 andthe of ellipse
9x2 +5y2 30y=0.

represents the equation of an ellipse. Find


70. 2067 Q.No.9b OR SOLUTION
4SOLUTION
Latus rectum = 3
Show that 2 + 4y2- 4x + 24y + 24 = 0 represe
centre, vertex and focus. Given equation of ellipse is
SOLUTION the equation of an ellipse. Find centre, vet 912+ 5y2- 30y = 0
2b2 = 3
GIven equation is a

9x2+ 4y2- 18x 16y-11 =0 and focus. or,9x 5(y2- 6y) =0


SOLUTION or, 9x2+ 5(y2 - 6y +9-9) = 0
b
or, 9-2x+1)+4(y2-4y+ 4) 36 Given equation is or, 9x2+ 5(y -3)2 -45 = 0
or, , =1 ...(4) 2+4y2-4x + 24y +24 0 or, 9x+ 5(y - 3)2 = 45
Eccentricity (e)=
or, 2-4x+ 4y2+ 24y +24 0
Since equation () is of the form 1 or, 2-4x +4-4+ 4(y2+6y +9-9) + 24=0 or =1 We have
it
or, (T-2+ 4( +3)2=16
represents an equation of ellipse. Comparing ) with -1, we have
Here,
, k=2
or,2 +32
4 1 h0 , k =3

a=2,
a2= 4,
b=3
=9
Sunceequation () is of the form
a=y5 b
a5, b2 = 9

3
or1
Since b> a, so the major axis is it represents an equation of ellipse. Since b> a, so major axis is along y-axis. We Or, 1 - a
parallel toD Here,
y-axas have, 3
Centre (h, k) (1, 2)
= =
h 2, k - or, 2 a 2
Vertex = (h, k £b) 2 = 16, Eccentricity (e) =
a b2= 4 a3
=

(1,2 t3) =(1,2-3) and (1,2+3) 4, b 2 Coordinates of vertices (h, ktb) = Now from ()
(1,-1) and (1,5) nce a >b, so major axis is along *-axis
(0,3 t3)
Centre (h, k)= (2,-3)
Eccentricity (e)-1- We have, (0,3 -3) and (0, 3+3)
=

The equation of ellipse is


(0,0) and (0, 6)

Focus-(h,k be) -1.2+3-0.211) Eccentricity (e)=\V - - Faxi h, k #be)- (o,3:3 ) (0.3+2 -


a
1 ba 1

68. 2066 CQNo. 9 b OR -(0.1)and (0, 5) or,5/21


Find the eccentricity and the foci of
the ellipse:
9x2 + 5y230y =0
Vertex (hta, k) =
(2+4, -3) =
(6, )
3)and(-2 or, -1
Please refer to 2064 Q.No. 9b 4
OR +2y29
Foci-(h t ae, k) =

.-)-a:
Mathematics-

84 Asmita's NEB Solutio of Basic equation () with cONIC SECTIONS Unit 4 85


Comparing
-6, k
=
0
have, h
=

75 2071 Set DQ.No.6a OR se


We
b 2 36 Bccentricity (e) - 83. 2075 Sot AQ.No. 6a OR
the equation of the ellipse
whose a? 4,
ind
two foci is 8 and
the
semi-latus rectun
4 a 2,
b =6 Find the equation of the ellipse
form with vertex at
in the standard
a and (0, 8) passing through
Ween
is 6. Since b> a, so
the is is
major axis
alon8Y -axi Foci- (h, k t be) -.33)
sOLUTION
Distance between two foca
We have,

*
= (0,3+ 2)-(0,1)and(0,5 (H
sOLUTION
4
Ecentricity (e)
=

2ae==8 00. 2073 Supp Q.No. 6a o


Let the equation of ellipse in standard form be
Find the vertices, eccentricity and foci of the
aes 4
Semi-latus rectum =o
V- ellipse 5x2+9y2 45. 141
-1()
SOLUTION Given, vertex = (0, 8)
( Foci-(h ktbe) -6,0t6.
-

6, Given equation of ellipse is


5x2+9y2= 45
ie. (0,
b=8
b) =
(0,8)
or, b?= 6a ..i)
We have,
6a OR
78.2073 Set CQ.No.of Putting the value of b in (1), we get,
the ellipse whoe.
Find the equation
Eccentricity (e) =\ axis is twice its minor axis and passes hrou
ma
-1
or, ae= a?- b the point (0,1).
using ) and (i)] y o. 1.)
or, (4)2=a2-6a SOLUTION Comparing this equation with -1, we get
Let the equation of ellipse with a > b be If equation (2) passes through the
or, a 2 - 6 a - 16 = 0
points
or, a2-8a+ 2a-16=0 a2=9, b= 5

or, a (a - 8) +2(a-8)=0 a -3,b=y5


Or, a 8,a-2(rejected) Since a> b, major axis is along x- axis.
W h e n a=8, from (i) b=6 x 8=48 By given, major axis=2(minoraxis)
The equation of ellipse is ie. 2a 2(2b)
Vertices (ta, 0) (+3,0)
a2 64
a 2b (1i) Eccentricity (e)=\ (32)
Also, if equation (i) passes through the po oTa25 x641
(0,1), then
6448 =1 (t ae, 0) (t 3x.0)
Foci =

or,

76. 2071 Supp. Q.No. 6b OR 1 t2,0) or,


-1
Find the vertices, ecentricities, foci and length or, b2=1 81 2074 Set AQ.No.6b OR
Find the vertices, eccentricties, foci and length of
of major axes of the ellipse , -14 b=1
2 1 2 major axis of the ellipse + 1. 4
Then from (i), a x
=

SOLUTION Putting the values of a and b in (9, we geesoLUTION or,

Comparing -1 withh a25


a2
Given equation ofellipseis 1 . ( ) Substituting the value of a in (2), we get,
=1, we have +4y2 4, which is the required equaton
=1, 25 1 which is the required equation of
h -5, k 1 elipse. omparing equation () with
we geth 0, k= -2, a? =5, b2 = 3 epse.
a29 b2= 4
79 2073 Set DQ.No. 6a OR 4 2075 Set BQ.No.6b OR
a-3, b 2
ind the centre, ecentricity and foci av5,b= y3 Find the vertices, eccentricities, foci and length
Since a > b, so the major axis is along x -axis.
ellipse92+ 5y2 30y =0 Since a > b, the
major axis is along x-axis
Vertices (ht a, k) =
(-5+3,1) =

(-8,1) and SsOLUTION Vertices (hta, k) =(0*V5,-2) (#5,-2) ofmajoraxis of the ellipse +y =5.
2,1)
Given equation of ellipse is sOLUTION
Eccentricity (e)=V 9x2+5y2-30y =0 Eccentricity (e)=\
or, 9x2+ 5ly2 - 6y) = 0
Given, eyuation ofellipseis y=5
Foci-(h tae, k) = (0+y5
Foci (n ac, )=(-531-(525,1) or, 92+5(y? - 6y +9 -9) = 0
or, 9x2 + 5(y -3)2 -45 =0
Dividing both sides by 5, we get

Length ofmajoraxis =2a 2x3=6 -(t2, -2) 105=1 ...(1)


7 2072 SetDO.No. 6a OR
or, 9x+5(y -3)2 =45 Length of major axis = 2a 2* 5 2 N 5
82 2074 Set BQ.No. 6a OR Comparing (1) with
a* , = 1, we
Find the eccentricity and
coordinates of the foci O 1 n the eccentricity and the coordinates of the get
of the curveo,
1 comparing equation () with ,p 0ci of the ellipse: xi + 4y2 - 4x +24y +24 0
Fleaser refer to 2067Q. No. 9b OR
4 hs-2, k
a2 10, b2=5.
0

SOLUTION We get a-V10,b= y5


Since a>b, the major axis is along x - axis.
Given equation of ellipse is 1
) a 5, b2=9, h 0 and k=3 =

Coordinates ofcentre (h, k)= (0,) =


matics-ll

Mathemau
of Basic
86 Asmita's NEB Solution
7. 2067 Q.No.6e of hyperbola cONIC SECTIONS Unit 4 87
the equation
in the
verstanda
Vertices = (hta, k) Find at (0, 5) and
form with a focus
a
tey 1. 2071 Old Q.No. 2C SOLUTION
Given hyperbola is
-(-2tv10,0) Find the coordinates of vertices and eccentricity
0,-
SOLUTION 9k-16y2 =144
Eccentricity (e) =\ Vertex (0, -b)= (0, -3)
ofthehyperbola 6 1. 121
ori65-1
b - (0, 5) SOLUTION
Vi*V£° Focus (0, be)
Comparing this equation with 1,
Foci (ht ae, k)
be 5
3.e - 5 (.b 3)
paring T1wi
we have
1, we get
a2 16, b=9
2-0 e 5/3
We have, a4,
? = 16,

b
b2 4
2
a 4,b 3

= (-2+V5,0) We have, e-
2a =2xy10 =210.
Vertices (ta, 0)=(t4,0)
Length of major axis
=

C. HYPEREBOLA - Coordinate of foci ( t ae, 0)=


2 MARKS QUESTIONs -1- or, 25=9+a2
82 2072 Set CQ.No.2a
Find eccentricity and foci of the hyperbola
-

Coordinate of vertices= (ta, 0)= (t4,0)


(5,0)

S 2065 O.No5 a2=16 95. 2073 Set DONo. 2


Find the eccentricity and foci of the hyperbola The required equation of hyperbola is Find the equation ofa hyperbola with a focus at
32-4y2= 36. SOLUTION
sOLUTION (-7,0) and eccentricity / 2
is
Given equation of the hyperbola
32-4y2 36
Comparing A 1
with -1, we
Please refer to 2066 CO.No.5c
o -1 (i)
have
a2 36, b2= 64
9Determine
2073 Supp ONo.2
the equation of the hyperbola with a
88. 2069 (Set A) Q.No.2a a 6, b 4 focus at (-5,0) and a vertex at (3,0 2
Find the eccentricity and the foci of sOLUTIONN
Comparing with-=1, we have hyperbola 3a2-4y2 36.
We have,
Vertex = (3,0)

a-12b#9 Please refer to 2065 Q.No. 5c Eccentricity (e)= (a, 0)(3,0)


a= 2/3 b=3 a =3
-)
99. 2070 Supp. Q.No.2a Focus=(-5, 0)
Eccentricity(e)=V
Find the eccentricity of hyperbola x2 - 44-12 Foci
(tae, 0)-(+6. (#:10, 0) (-ae, 0) = (-5, 0)
Or, ae 5

SOLUTION 3 2072 Sat DQ.No. 2 or, 3 x e =5


[Using
Foci-(ae,0)-.Yo-. Given equation of hyperbola Find the equation of the hyperbola with vertex
-4y2-12 0 (8,0) and passing through the point (&2, 4). [2 e-
B62066 CO.No.6C
Find the equation to the hyperbola in the O *-4y=12 sOLUTION Again, we have,

standard form with a focus at (9, 0) and or, -=1 ) Let the equation of hyperbola be 1

eccentricity -1
Comparing() with 1, we get Vertex
SOLUTION (a, 0)
(8, 0)
a --12, b2 3 a=8
Focus (-ae, 0) = (-7, 0) or,
Then equation (i) becomes
ae = 7
y2 or, b2+ 9 25
Eccentricity ()
°

4-1
=

i) b2 = 16
Eccentricity (e) =4
80 2071 SetDO.No.2a of Also, equation (i) passes through the point Hence, the equation ofhyperbola is
Then from () aj -7 Find the eccentricity and the foci (&2,4)
a =4 hyperbola -1 or,4 -1
Again, we have sOLUTION
97. 2074 Set AQ.No.2a
e-1
Omparing -1
with - b2=16
Find the equation of a hyperbola in standard
position such that its transverse and conjugate
have Putting the value of b2 in ()
6-16 a2 9, b2 = 16 axes are respectively 4 and 5. 2
b2 33 a 3, b 4 4-T6 1 which is the reyuiredequationof SOLUTION
Given, length of transverse axis (2a) =4
The equation of hyperbola is We have, hyperbola. a 2
And, length of conjugate axis (2b)= 5
1 Eccentricity (e)= - 84 2073 Set GQ.No. 2a
Find the foci and vertices of the hyperbala b-
1 - (t5, 0) 9x-16y=144
Foci (tac, 0)=
Mathematics- CONIC SECTIONS Unit 4 89
Q.No.9 b OR
Basic
88 NEB Solution of
1 .2063 ordinates of th
ASmita's

position
is eccentricity and the coordinat or, 5(r-2)- 20y2 20

equation of hype
in
standard
Findthe
The
hyperbola j6--1 or, -1 )
foci ofthe x-h2 y-B2 -1,
-1,
If
equation () passes through the point
SOLUTION
Comparing ()witha (4,&2)then
or,
5/2)1 Comparing6i1with we have We have, h 2, k = 0 82 64
1
b24 a24, b2 1 or, a 1 6
X
O4(25/4) 1 a2 16,
b 2 a2, b 1 Putting the value ofa in equation ), we get
4
which is the required equation Vertices (h ta, k) (2:20)-(0,0) and (4,0)
4 251
We have,
16--1 which is the required equation of
hyperbola. Eccentricity (e)=* Eccentridty(e)1 1. hyperbola.
8 2074 Set BQ.No.2hyperbola with a focus at 109.2072 Supp Q.No. 6a OR
Findthe equation of

7,0) anda vertex (5, 0).


at Focd-(tae, 0)=(: t2/5) Fodi =(htae,)-22 -a5.0 Find the eccentricity, coordinates of the vertices
and the foci of
the hyperbola 5x- 20y2-20x=0.
SOLUTION 9b OR 105. 2070 Set DQ.No.6a O 141
Given, vertex = (5, 0)
102.2066Q.No, the hyperbola i Deduce the equation of a hyperbola with a focus
or, (a, o) = (5, 0) Determine the equation or
standard position with tocus at (-7, ) a at (6, 0) and a vertex at (4, 0). Please refer to2070 Set CQNo.6a OR
a-5 10. 207 Supp O.No. 6a OR
Also, focus = (7,0) sOLUTION Obtain the equation to the hyperbola in the
Vertex (a, 0) = (4, 0)
OT, (ae, o) = (7, 0) eccentricity standard form with a focus at -70) and
refer to 2066 CQ.No.5c
>a 4
ae Please
5.e=7
6a O
Focus (ae, 0) = (6, 0)
eccentricity
103. 2069 (Set B) Q.No.
of hyperbola with focusa
ae=6 Please refer to 2066 Q.N. 9b OR
Find the equation the
4.e=6 (.a=4)
-5,0) and
vertex at (2, 0). 1. 20175 Set CQ.No. Sa OR
sOLUTION Find the vertices, centre, eccentricity and foci of
We have e4* Vertex (a, 0) = (2,0)
the hyperbola 9(x 1}- 16(y
We have
-

+2)P 144.
a-2 sOLUTION
Focus-ae, 0) = (-5,0)
e 1+ Given hyperbola is
ae =5 9(x-1)-16(y+2) 144
b
or, 25+ b? = 35
2e=5 ('a=2)
b2
-1i6 20
Dividing both sides by 144 we get

9x-11O 4
b2 10
The equation of hyperbola is
The equation of hyperbola is- 1
We have, x-1_
16
=1 *
.
-1
101 e21a2 Comparing (1) with

8 2075 Set AQ.No. 2


x-hE
a- =1,1, we get,
get
we

Findthefoci ofthehyperbola - 1 2 106. 2070 (Old) Q.No. 9 bO h=1, k=-2


a2= l6, b? = 9
Find the equation to the hyperbola in standard form
Please refer to 2071 Set D Q.No. 2a whose focus is at(0, 5) and vertex at (0, -3). 14] a-4,b =3
4 MARKS QUESTIONS b2 21 Please refer to
2067Q.No. 5c Vertices (hta, k)

100.2061 No.9 bOR Hence, the required equation hyperbola


of h07.2071 Set CQ.No. 6aOR (14-2)
(5, -2) and (-3, -2)
Find the eccentricity and the foci of the ind
at
theandequation of the hyperbola with a focus
Centre (h. k)= (1,-2
3x2- 4y2= 36 (0,5) vertex
a
(0,-3). at
hyperbola: 4 Please refer to 2067 Q.No. 5c
sOLUTION
Given equation of the hyperbola is 08. 2072 Set EQ.No. 6b OR Ecventricity(e)-V1
312-4y2= 36 Find the equation of the hyperbola with vertex
2 21x-4y2=84 at (0, 8) and passing through the point (4,
or, 2 - 1 ) &2) Foci (htae, k)
04. 2070 Set CQ.No.6a o vertices
14
Find
Comparing () with 1, we have
the coordinates of
tnehe the focd o
SOLUTION -(4-
eccentricity and the coordinates of Vertex (0, b) = (0, 8)
a2 =12, b' = 9
hyperbola 5x2 20y2- 20x0. b 8 (l5,-2)
(1+5, -2) and (1 - 5, -2)
a= 23 b 3 SOLUTION The equation of hyperbola is (6, -2) and (-2)
Eccentricity (e) =*
ven equation of hyperbola is
5x-20y2-20x = 0
or, 5-20x 20y2 = 00
Foci -(tae,0)(25 (i.0) or, 5(x2-4x) - 20y2= 0
Or, 5(x2-4x +4 -4)-20y2 0
or, 5(x-2) -20 20y2 =0
co-ORDINATES IN SPACE Unit 5 91

JNIT SOLUTION
Given relations are
I+ m+ n =0
and2/m + 2/n - mn = 0
)
a-1, b 2.c=2
If1,m, n be the d.c's of the line, then

(i)
Co-ORDINATES IN SPA0 Eliminating n from (G) and (i), we have

AC 2/m+ 2 ( - l -m) -m ( - - m) = 0
or, 2lm-2-2/m+ Im+ m
or, 2/2- Im - m2=0
=0
m

Va+b ca122-22
=1+4+4 3 . or, ( - m) (2/ + m) =0
be the d.c's of AB, then
ACO-ORDINATE IN SPACE If 1, m, n Either, I- m =0 ii)
2-X2t1_1 21+ m 0 Hence, the required d.cc's
2 MARKS QUESTIONS IAB 3
Or,
Solving (i) and (ii) by cross multiplication
iv are3.3
2056 CNo. 5b AB -3 2064 O.ND.5
joining the points (1,
line
2 3) and T 0-1 -1 T1-10 "T)-17 Prove tht the points -4, 9, 6), (0, 7, 10) and
Sbow that the to the line joining
the points - 1 , 6) are the vertices of right angled a
4,5, 7) is parallel [2 n issceles triangle. 21
(4,3,-6) and (29, 2). SOLUTION
sOLUTION , Let
line joining the Required d.c's are A(-4,9,6), B(0, 7, 10) and C(- 1, 6,6) be the
The direction ratios of the given points. Now,
which are the d.c's of first line.
points (1, 2 3) and (4, 5, 7)
are

a 4 - 1 =3,
2059 Q.No. 5b Again, from () and (iv) we have
AB 04+(7-9* (10-6
Find the angle between the lines
b-5-2-3, dirgetion cosines are proportional to 1, 24
+m+n=0 =16+4+16 6
G7-3=4 2 + m+0 n =0 BC
V(-1-0P+ (6-7+ (6- 10¥ =18
The direction ratios of the line joining
the Z-9,5. By the rule of cross multiplication, we have AC
=(-1+4+ (6-9 (6- 6)}? =V18
points (43, 5) and (29,2) are Q-2+6=8 SOLUTION m Since BC AC, so AABCis an isosceles
=

a2 +4=6 b=-9-3 =6, Given, al, bi = 2,


C4 1.0-1 1 1:2-0.111. triangle.
Here anda=-2, b2=-9, c=5 Also, BC3 + AC3= 18+18 36
If e be the angle between the lines, then 2C=90
=6)-(AB
a
aia2+ b1bzt CiC2 Hence, AABC is a
right angled isosceles
cOS ar?+ b2+ c2ya2 + b2+ c triangle.
1:(-2)+2:(-9)+4:5 -2 10. 2065 Q. No. 5b
12+22+ 42(-2)+ (-9)2+52 ITm-ny
If e be the
angle between
Show that the
C-2,4, 2) are collinear.
points A (1, 2 3), B (4, 0, 4) and

-2-18+ 20 = 0 two lines, then 121


cos 90° Cos 6 hlh+ mim2 + nin2 sOLUTION
21 110 Please refer to 2062 Q.No. 5b
Hence, the given lines are parallel G=90°
11. 2066 Q.No. 5
2 2057 ONo.5b 5 2060 Q.No. 5b Find
thefatio in which the line
joining the
What are direction cosines of a line? Prove| Find he co-ordinates of the point which a
sin a+sin B+ sin?y=2 in the 0 120°
points2 4 5) and (3, 5,-4) is divided by
2 eline joining (2, 4, 3) and (5, 5, -6) aylane.
sOLuTION
a
2:1.
72062 QNó. 56 SOLUTION
line makes angle a. Bß and y with positive SOLUTION Show thdt
P (1, 2, 3), Q4, 0, 4) and R(-2, 4, 2) are
Here, (r, y, z1) = (24, 5)
I-axis, y-axis and z-axis respectively then cos Here, ( ,yl, zi) = (2, -4,3) coiéar. [21 y 2 z2)= (3.5, 4)
a, cos B and cos y are called the d.c's of the
ine.
2 y2, Z2) = (5, 5, -6) SOLUTION In xy-plane, we have z =0.
Now, rom section tormula, we have
m: m2 2:1 GIven points are A(1, 2, 3), B(4, 0, 4) and C(-2,4, 2).
Now, miZ2+ moz1
(x, y , z) = ? AB
sin a +sin2 ß+ sin?y From section formula, we have BC
V4 1)3 + (0-2)+(4-3) =v14
-
z
mi+m
=1 cos a+1 -cosp+1-cos?y V(-24)+ (4 -0)2+ (2-4) =V56 or, 0 9+om2.(5)
2 mex my2+ m2y1 mZ2
=

3-(cos a+ cos p+ cos ) (y, 2) = 214


m + m2 AC
3-1 ( cos a +cosp + cosy 1) =
n tm2
(-6)+
=V-2-1)+ (4-2}+ (2-3) -V14
Here AB+ AC =V14y14 214 BC
or, 4m 5m
2
(2x5+1x2 2x5+1* (4) 22 2 1 Thepoints A, B and C
3 2058 Q.No. 56 2+1 are
collinear. Required ratio is 5: 4.
Find direction cosines of a line
8. 2063 Q.No 8
joining the
whose 12.
2067 Q.No.5
points(-1, 2, 5) and (-2, 4, 3).
sOLUTION
21 3-6.2 nd
SOLLTTON
the rection
direçtiopfatios are 1, 2,2.
cosines of a line
12
The section of
Qtx, y, z) in the
two points P(2, -4, 3)
ratio 2:1 is (-2,2-3). Find Q/21
and

Given points are A(-1, 2, 5) and B(-2, 4, 3). 62061


the
Q.No. 56 Given direction ratios are SOLUTION
AB V-2+1}+ (4-2P (3-5 Section of two points e Please reter to 2061 Q.No. 5b
(, Y, z) in the ratio 2:1 is (-2, 2, -3), h
Mathematics-l

Basic co-ORDINATES IN SPACE Unit 5 93


Soluton
of sOLUTION
Asmita's
NEB
Given points are M(-2, 4,
3) and 4, 31
32 2.13.2)+4.1
S, 3) N cos
V22 +32+ 4212+ (-2) +12=
Now,
2, (-1,
13.2068 Q.No.6
points (3, 0,
1), (2 2 parallelo8ra
MN= V(-1+2) + (2-4)2+(6
how
and
that

(0, 1, 2are
the
the
vertices
of a
If l, m, n be the direcuon cosines ot
then
m-A
21. 2072 Supp Q.No. 2b
SOLUTON
0, 1), B(2, 2
2), C-1,
3, 3)
and D{0,
1, 2-1
MN Find the directioncosines of a line passing
through the points P(2, 3,4)
and Q1, 4, 6). 2
nA
Let A3, points.
2) be given
Mid point of
diagonal
AC m - MN SOLUTION
Here, x1 2 , yi =
3, z1 =
4
Required d.cs of AB are

25. 2074 St BQ.No. 2


- 15-() MN3-3 PQ V x -
x2 1, y2 4 , Z2 6
x1)2 + (y2 -yi)2 + (z2- z1)? Findthe direction cosines of the line PQ passing
point of diagonal
BD
Required direction cosines of MNar
Through P(2, 3, 4) and Q5,9, 13) 21
Mid
B2,2.2) V1-2+ (4-3)2+ (6-4) =v6 SOLUTION
A3,0,1) Direction cosines of PQ are

18. 2072 Set CQ.No. 26 X2-X1 Y2-Y1 Z2-Z1


Given points are
(x1, y1, z1) = (2,3,4)
the line
ratio
poinsthe
Find which
in
P(-2, 4, 7) and Q(3, -5, - iis div OPQPQ & (x2, yz, Z2)= (5,9, 13)
D0.1,2)
C-1,3,3) thoyz- plane. .e. PQVx)+(ya-y+ (z2-z}
sOLUTION V6-2+9-3+ (13-4
Given points are (x1, y1, z1)=(-24.7 e. - -9+36+81 =y126 -314
-(1) of the
and (x2, y2, 2)= (3,-5,-1) Tf the direction cosines of PQ be l, m, n then
coordinates of the midpoints The x-coordinate of any point on then 22.2073 SetcQ.No.25
Since the
and BD are the same, so is 0. Now, from section formula,weha If O is the origin, P{2, 3, 4) and Q1,-2,1) be any
diagonals AC each other.
and BD bisects two points, show that OP is perpendicular to
Y2-y 9-3
diagonals AC mi2+ m'1 6
ABCDisa parallelogram mi+mn2 0Q. 2 m
PQ 14 14 V14
2b mi.3+ m2.-2) SOLUITION
14. 2069 (Set B)Q.No. line which are or, 0 Direction ratios of OP are n=Z221
PO
Find
cosines
the direction
of a

12 or, 3m 2m2
mi*n2
a x2- X1 = 2 - 0 =2, bi = y2 - y1 = 3 - 0 = 3
N14 14 V14
inclined to the axes.
equally .

C Z2-Z1=4 -0=4 26. 2074 Supp Q.No. 2


SoLUTION and direction of ratios of OQ are Find the direction cosines of a line which is
Given that the line is equally inclined
to the or
m2 equally inclined to the axes.
axes. So a=p=y. Hence, the d.c's are 1 m2 =2:3 a2 Xa- X1 =1 -0=1, b2 = y2 - y1 =-2-0--2, (2
C2 Z2-Z1=1 -0=1 Please referto 2069 Set BQ.N. 2b
I= cos a, m= cos a and n COs a 19. 2072 Set DQ.No. 2b Now, 27. 2075 SetcQ.No.28
We have, If P and Q denote the coordinates (26 Find the direction cosines of the line passing
a1a2+ bibz+ CIc2= 2 * 1+3x (-2) +4 *1
+m2+n=1 (4,5, 0) respectively, find the directiono through the points A(-1,25) and B(-243 2
or, cosa + cosa
+ Cosa=1 2-6+ 4 0
the linePQ SOLUTION
or, 3 cos?a=1 Hence, OP is perpendicular to OQ
SOLETON =

2078 Set DQ.No.26


Given points A and B are
cos a Given points are P(2, 6, 2) and Q4 (N1, yr, z1) = (-1,25)

PQ =V4-22+ (5-6)2+ (0-23=


and y are the direction cosines of a line.
and (x y z2)=(-2 4,3)
then cos2at
nat cos2ß+cos2y+1=0 [21 AB xi(-y)+(z-z1)}
Required dc's are+ If1, m, n be the direction of PQ SOLUTION
2-X1-4-2-2
PO -2 L.HS. = cos2a + cos2pt cos2y t+1 =v-2+1+(4-2 3-5
15. 2070Set CQ.No.2b 2cosa - 1 +2 cosp -1+ 2cosy - 1+1
=v1+4+4
Show that the direction cosines of line
a
equally m = 3 2(cosa + cosB+ cos) -2 The direction cosines of AB are
inclined to the axes are:
+ 2 2 x1 2 (' cosa+ cos*p + cosy= 1 - -
SOLUTION n- =
0 =
R.HS.
Please refer to 2069 (Set B) QNo. 2b
'Required direction cosin
s of PQ are yA, 2073 Supp Q.No. 2b
Find the direction cosines of a line joining the
m
- AB

16. 2070 Supp.ONo.2E points (1,2,3) and (4,5,7). 2221


Find the locus of a point which is
20. 2072 Set EQ.No. 2 AB
from the points (1, 2, 3) and
(3,2,-1).
equidistant
Find the angle between the woSOLUTION
12 and 1 4 MARKS QUESTIONS
sOLUTION direction ratios are 2, 4 Let given points (1, 2, 3) and (4, 5, 7) be
denoted by A and B respectively. Then,
Please Model Set 1, Q.No. 2b 28. 2056 Q.No. 14a
see
SOLUTION AB
17.2071 Set DO.No. 2 Here a 2, bi 3 -V4-13 6-23+(7-3¥ Find the direction cosines of the line which is
b=-2,
=9+9+16 4 perpendicular to the lines with direction cosines
Find the direction cosines line of a
and 1, proportional to 3, -1, 1 and -3, 2, 4.
through the points M(-2,4,3) and N(-1, 2,passing 4
it , m, n be the d.c's of AB then
If0 be the angle betweecca
5). 12 bib; t Ci
aaz+ + b +

cos 0
a?+bi?+c#ya
Mathematics-l

CO-ORDINATES IN SPACE Unit 5 95


Basic
Solution
of
34 ASmita's
NEB
CCO,0a
line whch i 32. 2058 Q.No, 14a 37.2060 Q.No.14a OR
cosines of the line which is
sOLUTION whose Find the direction
d.c's ofthe d.cs Find the angle between the lines
perpendicular to the lines with direction cosiness
with
be the
Let, m, n
the
lines Ther D(0,a,a. P(a.4.a) direction cosines
are given by (h, mi, ni) and
P e r p e n d i c u l a r
to
-1, 1
and -3,
2, 4.
we
nave
( , ma na).
proportional to (1,-2,-2) and (0,2, 1). 4
to 3,
POporhonal

condition of
perpendicular,
Please refer to 2057 Q.No. 14a sOLUTION
the Let 1, m ,n be the d.c's of a line which is
ng
3 m+n=0
and-3/ + 2m+
4n =0 we
have
Aa0 3. 2058thatQ.No,
Show
14a OR
the angle between two diagonals of a perpendicular to the lines with d.c's
multiplication,
E(a,0,a) proportional to 1, -2, -2 and 0, 2, 1. Then, by
the rule of B(0.b,0)
cross

D 4] the condition of perpendicularity, we have,


cube is cos-
1+m. (-2)+n.(-2) =0
Here,
OP V a - 0 + (a - 0) + (a - 0 Please refer to 2057 Q.No. 14a or, 2m-2n0
or, 15* 34. 2059 QNo. 14a andl,0+ m.2+n.1=0
ay3
AD-V0-a)2 * (a - 0) +* (a -o-
Find the angle between the lines whose o2mn=omultiplication,
the rule of we
(i)
have
direction cosines are (h, mi, ni) and By cross

ay3 (, mz nz) 4 m
of OP are Please refer to 2057 Q.No. 14a OR -2).1-(-2).2 *2.0-1.1 *12-(-2.o
n Now, d.cs

Required dc's are


35. 2059 Q.No. 14a OR B(5,4,
or
-2+4-1
m
Given three collinear points A(3, 2, -4), -6)
and C(9,8, -10). Find the ratio in which B divides
29.2056 Q.No. 14 OR divides the ie. AC. 4 o
Find the ratioín which the yz-plane
line joinig(4, 6, 7)
and (-1, 2, 5).
the on the
point
Also find the
plane. 4
yz
And d.c's of AD are SOLUTION
Let (x, y, z) = (5, 4, -6) divide the line joining 1-,mn-
foordipátes of
Required dc's of the line are212
the points (1, y, z1) = (3, 2, 4) and
33 3
SO TION (x2, y2, Z2)= (9, 8,-10) in the ratio mi: m2.
Here, ( z , y , Z1) = (4,6, 7)
Then, by section formula, we have
38.2061 Q.No.14
(z y2 z2) =(-1,25) i
B(5,4-6) Prove that the lines whose direction cosines are
mi: m ? Ife be the angle between OP and AD the given by the relation al bm +cn = 0 and
In yz-plane, we have x = 0 A(3,2-4) C(9,8,-10)
is of the form (0, y, z). cos hl2 + mim2 + nin2
mi2+ m2r1
fmn + gnl + hlm = 0 are perpendicular if
1.e. any point in yz-plane
From section formula, we have 1 mi+m12
m9+ m2 3
X
m+m2
or, 5 mi * m2 SOLUTION
4 or, Smi+5m2 = 9mi + 3m2 Given relations are
F)+m
0 =

al+ bm+ a=0


or,
or, 4m= 2m2 ()
or, mm 4m2 31.2057Q.No. 14a OR or.
and fmn+ gnl+ hlm = 0
(i)
Findthe angles between the two lines Eliminating n between () and (i), we have
or.
difection cosines are (1, my n
m2

m m2 =4:1 h m, n2)
mi:m2=1:2 (4l+ bn him=0
Again from section formula, we have
36. 2060 Q.No.14a
SOLUTION The projeetíon of a line on the axes are 6, 2,3. or, ag (af+ bg - ch) Im+ vfm 0
Let 0 be the angle between two lines AB an
mya+may mz2+mzA Find the length of the line and its direction
(0,y.2( mtm'mitm whose d.c's are l1, mi, ni and lz, ma, na respa Tosines. 141 o a ( ) ich-af bg (m)b-0
- (a 2
Let the coordinates of A and B be (n, y, a SOLUTION
(2 ya z2) respectively. Then, Let , m, n be the d.c's of a line and r be its length. which is quadratic in m
Then,
32-1 B
Let the two roots and
Required point in yz-plane is AB be m m
X2-1= ABlh A(.Y1.2) ' I r = 6, mr = 2 and nr = 3 Then a t Product of the roots
30. 2057 Q.No. 14a Similarly, Squaring and adding
Show that the angle between two
diagonals of a Z2 Z =AB ni ya-y ABm r(2+m2+ n) = 36 + 4+9=49
r,
hlamime
b ag
cube is cos Or, r = 49 (:P+ m2 + n2= 1)
141 The projection of hla mim2
AB r=7 of, i/a (iii)
SOLUTION on CD is given So, the length of the line 7 g/b
Let us suppose
that one vertex of a cube of by (z2-21
Similarly, if we eliminate ! between (i) and ( i
'a' be at origin O(000). length| AB cos 0 (xa xi) la + (y2- Now, 1-.m- andn- we have
Consider two y+ABn
-

OP and AD where the coordinates of O, P, A diagonals| or, AB cos AB l+ AB M*


=
mimnhn iv)
D be (0, 0, 0), (a, a, and g/b h/c
a), (a, 0, 0) and
respectively as shown in the figure. (0, a, al
cos hl+ mimzt nmna Hence, d.c's of the line are j and~.
From (ii) and (iv),
Mathemarcs
Solution of Basic -l+ m+n)
96 ASmita's NEB
co-ORDINATES IN SPACE Unit 5 97
alzminm2 =k(suppse) cosy -m+n)
t/a 8/b h /
kh
3 81,2062 Q,No 14a OR
kt
Find theAatio in which the line joining the
(+ m-n) point 2, 4, 7) and (3, 5, -8) is divided
2 -cos 120
cost

lines will be
perpendicular
he two + cos-ß + cosy+ cos2t uyplane. 4
ifhl+ mmu+ nn:=0 Now,cosa sOLUTION 0 120°
m+n *3H+ m +
ng ++ -m
Here, (1, yy zi) = (-2, 4,7)

(X2,Y2, 72) (3,-5, -8)


43.2064 Q.No.14á OR
Find theAtio in which the line
We have, in xy-plane, z = 0 joining the
m-n Now, from section formula, we have
point- 3, 4, 8) and (5, - 6, 4) is divided by the
y plane. Find also the coordinates
of the point
39. 2061 O.No. 14b OR makes angle x, y, z,t (4R 4m+4n)- me+ n mi2 +m2z1
mi m2
of intersection of the line with the
place. [4]
Frove thata line which
with four diagonals of a cube is m -8)+ m:(7)
sOLUTION
or, 0 Here, ( , y1, zi) = (-3, 4, -8)
4 14
0. 2062 0.No. 14a
( 2 y2, Z2) = (5,-6, 4)
cos?t
cosx+cos y cos z
+ + or, 8mi = 7m2

Find the
direction cosines , m, n of Inry-plane, we havez=0
SOLUTION are
connected by the Le. any point in xy-plane is of the form (x, y,
or which nl=0. relatio
cube m8
of the vertex of the 0 and Im -
mn+
Suppose that one
O| 41+3m-2n= mi:m2 =7:8
O(0, 0)
0, and OA, OB and Now, from section formula, we have
length a is at origin Then the sOLUTION
are along the
coordinate axes.

F and P are (0,0, 0),


Given relations are
42.2064 Q.No.14 miz2+ mo21
coordinates of O, A, B, C, D, E,
a, 0)
4!+3m-2n =0 ) Find the direction cosines 1, m, n of two lines ni *m2
(a, a),
0, (a,
a, 0), (0,0, a), (0, a, a), (i) which are connected by the relations:
=m.4+ ma (-8)
+ ni= 0
(a, 0, andlm - mn
. . .

or, 0
and (a, a, a) respectively as
showrm in figure. Eliminating n from (i) and (i), we have 1 + m + n = 0 and mn - 2nl - 2m = 0
4 m+m2
sOLUTION or, 4mi 8m2
B/0.a.0) F(aa.0) Im- Given relations are
+mtn =0 ma
.. )
2/m- 4ml -3m2+ 4/2 + 3lm 0
=

D02 or, and2/m+2/n - mn = 0... i ) mi: m=2:1


or, 4/2 Im -3m2 0
+ =

Eliminating n from (G) and (i), we have The required point is


3/m-3m2= 0
or, 4/2+ 4lm 2/m+ 2(-1 - m) -m (-l-m) =0
-

0 (x, y,0)= I miy2* may


4 (+ m) -3m (+ m)
=

or, or, 2/m -22-2lm + Imt m2=0


m+m2
C(o,0a) E{a.0,.a) or, (+ m) (4/ -

3m) 0 =

or,22-Im -m2=0
Either I+ m=0 (ii) or, ( - m) (2+ m) = 0
-(51
The four diagonals of cube are OP, AD, BE and CF. and4-3m=0 ..(iv) Either, l - m=0
ii)
Now, From (i) and (ii) or, 21+ m= 00
The direction ratios of OP are a -0, a-0, a-0 iv 44.2065 Q N6. 14 a
41+3m-2n 0 Solving i) and (ii) by cross multiplication
ie. a, a, a.
Find the direction cosines of the line which is
I+ m+0. n =0
The direction ratios of AD are 0- a,a-0, a-0
By the rule of cross multiplication,
we hav 1:0-1 (-1) 1:1-1.0 1(-1)-1.1 peppéndicular to the lines with direction cosines
ie. -a, a, a proportional to 3,-1, 1 and -3, 2, 4.
The direction ratios of BE are a -0, 0-a, a -0
3-0-(-2) 1 (-2) 1 -0.44:1-3.1 sOLUTION
Please refer to 2056 QNo. 14a
i.e.a,-a, a
The direction ratios of CF are a -0, a -0,0-a
i.e. a, a, -a
or, 7 2+(2+1
m 45.2065 Q.No.14a OR
The projection of a line on the axis are 6, 2 3.
So, the d.c's of OP are which are the d.c's of first line.
Find the length of the line and its direction
n.n- which are the d.c's of fist Again, from (i) and (iv) we have
+mt n =0
cosines. 4
Va+a2 a a a+a ya-a Again, from (i) and (iv)
4+ 3m-2n =0
2+ mt0 n=0 sOLUTION
Please referto 2060 Q.No. 14a
By the rule of cross multiplication, we have -
i Similarly, the d.cs of AD, BE and CF are
41-3m+On =0
by the rule of cross multiplication,
e have

T-0-1.1 m
12-0.1"1-1-1:2
6. 2066 Q.No.14
Prove that the straight lines whose dc's are given
by ul+ vm+ wn =0 and fmn+ gnl + hlm = 0 are

3.0-(-2) -3) (2).4-0.4 . or, 2- perpendicular if =o 4


respectively.
SOLUTION
Given relations are
Let1, m,n be the d.cs of the lne
ul+ vm+ wn = 0 )
making angles a,
B.y andt with OP,AD, BE and CE, Then andfmun+ gnl+ hlm =0 (i)
which are the d.c's of second line
Eliminatingn between () and (ii), we have
It be the angle between two lines, then
cos a=1G m. m+n) which are the d.c's of second line Cos = hl + mmut niu
Similarly, Hence, the required d.e's are
Mathematics-
14hOR
49. 2067Q.No, which makes
Basic
Solution of
98 NEB anol
le, Co-ORDINATES IN SPACE
t a ' s

x, y, Unit 5 99
vfm=0 Prove thatline
Or, ug P+ (uf+ vg
-

wh) Im+
four diagonals
ofa
cube is
2,d 0 m+n+j(-* m* n"+ *-m+ npOLU
o, ug (wh -uf*v(m)v0 Cos+Cosy + Cos z+Coss- Let a, bi, Ci and a2,
bz, ca be the direction
So it has
two ro +m-n ratios of two lines whose
are l, m, and l,
corresponding d.cs
is quadratic in SOLUTION m mz, n. Then,
which

Suppose
that one of of
of the cube aP
the vertex Am2+ 4n)- (+ m+ n)-
length a is at oigin O(0, 0, 0) ancdO h
Let the two roots be m an m
the coordinate A Va by2 c'
OC are along axes. Then C:2+ m2 + n-1)
ar vf O, A, B, C, D, E, F mi
m'm2 ug
coordinates of D,E, Fandp 50, 2068 Q.No. 144a
0, 0, (a 0, 0), (0, a, 0), (0, 0 FHnd the angle between the two straight lines
or,
or, vf
mm
ug (0, a, a), (a, 0, a), (a, a, 0) and whose direction cosines are , m, nm and lz, ma
ar bc
respectively asshown in figure. na Also, find the condition for the two lines to
hlz mum: (i) be perpendicular to each other.
4
Or, f/ug/V
if we eliminate /
between ()
and (7),
B0,a,0) F(a.a,0) SOLUTION a+b +ca
Similarly, First part:
wehave m
... (iv) D(0,a,a)
Please refer to 2057 Q.No. 14a OR a b+
Second part:
8/v h/w P(ala, Two line will be perpendicular to each other if
From (in) and (iv), az+b2+c2
0 9 90 If e be the angle between two
hlemi- = k (suppose lines, then
f/ug/v h/w k (suppose)
o(0,0,00
Aa00 i.e. cos90° = l1l2t mim2 * n1na
cos hlt mima+ nmna
h 112 mim2*nn2= 0
C(0,0,a) E(a,0,a) aaz+ b1bz t CHC
lk mm 51. 2068 Q.No. 14 a O a?b+cyaz bz+c2
two ines will be perpendicular ir
The line
joining the points (1, 2, 3) and The two lies will be
The four diagonals of cube are OP, ADShow that theis perpendicuar if 0= 90P
hl:+mim+nine0 -1, 2, 3) parallel to the line
joining the a1a2+ bib2+ CiC
and CF ie. cos 90.
or, u Now, the direction ratios
of OP 0, a -0, a-0
are a -

P01nts (2, 3, 4) and (5, 9, 13). 4 ar by a yaz b+ c?


ie. a, a, a. SOLUTION a1a2 bba+ CcIG_
**u v w
The direction ratios of AD are 0-a,a-0a Let the given points be A(1, 2, 3),
B(-1, -2,-3), aO a+ br?+ c?ya?+ b?*
C(2 3, 4), D(5, 9, 13). We have to show that
i.e.-a,a,a a1a2+ bib2 + cic2 = 0.
37. 2066QNo.14aO The direction ratios of BE are a -0, 0-a AB is parallel to CD.
53. 2070 Set DQ.No. 6E
A(23,-1), B(5,2, 3), C43,-5),DE21,-3)are i.e. a,-a, a For AB:
Find the angle between two
four points in space. Find the projection of AB
The direction ratios of CF are a 0, a-00
If a1, bi, c1 are the direction ratios of AB then straight lines whose
on CD. a *2-X1 -1 -1=-2 direction cosiness are h, m, nm and k, me, 2
1.e. a, a,a

sOLUTION So, the d.c's of OP are


bi =y2-y1=-2-2= 4 SOLUTION
C1 22-z1=-3 -3 = -6 Please refer to 2057 Q.No. 14a ODR
(2 3, -1), B (5, 2, 3), C (4,3,-5)
Gien points are A For CD:
snd D(-2, 1,-3)
Now ya? a+a'Va+a+a'ya+a*a If az, bz c2 are the direction cosines of CD, 54.2071 Set CQ.Na. 6
Show that the line AB is perpendicular to CD if
then
CD=V-2-42+(1-3)2+(-3+5 A B, C, D are the points (2 3,
y36+4+4 = y44 = 2/1
a2 5-2 3 4),
(3,6, 2) and (1, 2 0) respectively.
b2 = 9 - 3 =6

If the d.c's of CD are l, m, n then Similarly, the d.c's of AD, BE and CF sOLUTION
C2 13 -4 9
I &2-4 -3 Here, For.AB:
If a, bi, a be the direction ratios of AB, then
a2 a 5-2-3, b 4-3 1, c-1-4=-5
y2-Y 1-3 respectively.
-CD 6
For CD:

Again, if a bz cabe the direction cosines of


CD, then
Let , m,n be the d.c's of the line making ang C1 a:1-3-2 b=2-6 4 C20-2=-2
C2
CF, Then Now,
Now, the projection of AB on CD is
B.Yand o with OP, AD, BE and a a2b b + a2=3(-2)+1(4)+ (-5) (-2) =0
a-i)I+ (y2 - yi) mt (z2-z») n to CD.
Hence ABis perpendicular
6- 0-90 cos m Hence, AB and CD are parallel lines. 55. 2075 Set AQ.No. 6b
Similarly, cos p- +m+ n) 52. 2069 (Sot A) Q.No. 6b Show that the angle between the two diagonals
Find the angle between the two lines whose
4
ection
Find the ratiosare
a, b. ca and az ba Also,ot acube is cos
ca

48 /2067theQ.No. 14 cOB mtn cordition under which the two lines are
perpengkcular 4
Please refer to 2057Q.No. 14a
Hnd angle befween the lines whose cos8 - n)
Adirection cosines are given by I + m +n
0 and
2 Im +2ln -
mn= 00
Please refer to 2064 Q.No. 14a 14
cosa+cos:p+ cosy+ cos*o
tics- that the quadratic

00 Asmita's NEB Solution


of Basic
Mathematics
We know
have equal rootsif its discriminat equation cO-ORDINATES IN SPACe Unit 5 101
(c?v + b
(2abw)-4 (C°u* a-w) w)- SOLUTION 63. 2074 Set AQ.No. 26
6 MARKS QUESTIONS
ie.
abaw2-cuv
-
bciuw -

aci
ac2 wy -ab
of plane through the point (3, -4,5)
or. * acvw =
0 The equation
a(r - 3)+ b(y + 4) + c(z - 5) - 0
is
Find the equation of the plane whose intercepts
or,
cuv
* b*uw 0 . () on the axesare 2, 3 and 4
respectively. 121
56. 2071 Supp.Q.No.9
=
direction| a*vw
I fequation () is parallel to 3x 4y + S 7, oLUTION
lines
whose
cuv* buvt
that the straight altbmtcn =0| or,
Dividing
both sides by uvw
OW equations get then - k(Suppose)
given by the Given, a = 2, b= 3, c = 4
a = 0 are
perpendicular
r
ul+Vm?+wn?
The equation of plane in intercept form is
0 and parallel
and
=
u v w a 3k, b - 4 k , c = 5k
+ c{utv)
a(v+w)+ b(u*w) Putting the values of a, b, c in (i), we have
a =0.
6 9
57. 2074 Set A Q.No. whose 3k(r -3) -4k(y + 4) + 5k(z -5) = 0 1
V w Prove that
the
lines direction.
the relations al + bm + or, 3x-9 -4y -16+5z -25 0
SOLUION given by =
0
3x -4y 5z = 50 is the required equation o f |
* him are
Given relations are

al+bm+cn =0
fmn +gnl
perpenditu plane.
& ulP+vm+wn?=0 ..(u) 60. 2070 Set DQ.No. 2
we have Find the pqúation of the plane which makes
Eliminating n
between () and (ü), 6x+ 4y 3z =
12 which is the required
sOLUTION equal intercepts on the axes and passes through
u+vmw 0 Please refer to 2066 Q.No. 14a theoint (2, 3, 4). equaton of plane.
(21
Put u a, v
=
b, w
=c SOLUTION 64. 2075 Set AQ.No. 2
The equation of plane in intercept form is Find the equation of the plane which makes
or, u+vm2+ v , 58.2075 Set BQ.No. equal intersepts on the axes and passes
through
ukc+ vmi+ a-w +2ablmw +
bm-w =0
Prove that the straight lines whose.
direy
cosines are given by the relations al +bm
1 the poin2, 3, 4).
or,
Since the plane makes equal intercepts on the Péase refer to 2070 Set DQ.No. 2b
or, (cu+ a-w) P+ (2abw) Im+ (cv b>w)
+
m2=0 and fmn + gnl * him = 0 a r e perpendin
axes, so a b c So the
=

equation of plane65. 2075 Set BQ.No. 2b


=

0 and parallel ifat Dgsa becomes Find the intercepts made by the
or, (u+ a~w) plane 2x+3y+
4z 24 on the coordinate axes.
(cv+bw)=0 ) aaa 4=1
or, x+y*z=a
SOLUTION
which is quadratic in Let the two roots be | sOLUTION 1 Given, equation of plane is
First Part: Please refer to 2061 Q.No. 14 Since equation (i) passes through the point
Second Part (2,3,4), so +3y+4z =24 )
and The lines are parallelif
2+3+4 a
a9
Dividing both sides of (i) by 24, we get
Now, product of the roots=
Substituting the value of a in equation (i),
or + bw Taking first two ratios, we have we have
or,
n/ m2 C u a w
m2
Itytzs9is the required equation of plane.
61. 2071 Set CQ.No. 2E Comparing (ü) with =1, we get
O y+ b~w cu+a*w Find the equation of the
plane through (1, 2, 3) x - intercept (a) = 12
(iv) mi m2 and parallel to the plane 3x -4y +5z=0.
y- intercept (b) = 8
Similarly, if we eliminate I between (i) and (i), We know that the quadratic equation SOLUTION
we have have two equal roots if the discriminatea The equation of plane through the point (1, 2, 3) is z-intercept (c)=6
a(r -1)+ b(y - 2)+ c (z -3) =0
mm ie. (af + bg- ch)? - 4.ag.bf= 0 [: BP-4AC- )
cu+ aw av +bu Since, plane (i) is parallel to the plane 3r- 4y+ 5z = 0, so 4 MARKS QUESTIONS
From (iv) and (v)
..(v) or,
(af+bg-ch}#=(+2/agbf
or, af + bg- ch- +2agbf
k (suppose) 86. 2069 (Set B) G.No. 6b
Fird the equation of the plane through the
mim2 a 3k, b= -4k, c =5k póints (1, 1, 0) (-22-1) and (1, 2,1).
C+bw cu+ away+buk (suppose) or, af+ bg t2ag.bf= ch Substituting the value of a, b and cin (i)
3k (x - 1) 4k (y -2) +5k (z-3) =0 sOLUTION
hlh=k(cv+ bw) or.
(yaitbe)-(Wah) The equation of plane through the point (1, 1, 0) is
mim k (cu+ aw) or, 3x-3-4y +8+ 5z -15 0
nn k{ay + bu)
The two lines will be
or,
Vaf +bg- +ych
or, yaf tbgt ych 0
3 x - 4y + 5z 10 is the =

required equation ot
a(r-1)+ b(y -1)+e (z -0) 0
If equation () passes
through the points
(i)

hla+ mim2+ nin2= 0


perpendicular if
lel i plane. (-2.2,-1) and (1, 2, 1), then
Hence the given lines are para
or, cay+ b>w+ c^u +a?w +a?v
62. 2071 Supp. Q.No. 20 a-2-1)+ b{2-1)+ c(-1 -0) =0
+
bu =0 af +bg Wch 0 k
ind so that the planes * 2y + or, 3a *b-c=0
+5y-z 0 are at right angles. kz=0 and 121anda(l - 1)+b{2-1)+c(1 -0) 0
-

ii)
a-(v+w)+b2 (u +w)+c(u+ v)=0
Again, the lines are parallel if B. PLANE SOLUTION
Two planes x - 2y + kz=0 and 2x+ 5y 2 0
or, 0 a t+b+c=0 ii)
Solving (iü) and (i) by the method of cross
m2 nz 2 MARKS QUESTIONS are at
right angles to each other if multiplication, we have
Taking first two ratios, we have
59. 2069 (SetA QNo. 2 1:2(-2) 5 k.(-1)-0 f aaa* brbz* cica= o
mi t or, 2-10 - k= 0
m2 Find the equation of the plane through t - 4 y +
k -8
(3,-4,5) and parallel to the plane 3*
1.e. mi m2 1-1-(-1) 1 F).0-(3 1 3) 1-0-1
2-(i
Basic Mathematics
102 Asmita NEB Solution of
CO-ORDINATES IN SPACE Unt 5 103
or, 12-12 0
. -k(wuppoe) . ie 0-0 (true)
a 2k b 3K.c= -34 in (1),
"e Hee, the planes( ) and (ii) are perpendicular
value of a. b and c
k (uppme)
Suhstituting the 71.2072 9et EQ.No.6
have -3k Find the equation of the plane passing through
1) 3k ( -1)+ (-3K) ( 0)0
2k, b= 2k,c=
2k(1 a

k Supoe)
-

-0
Substituting the value of a, b the points (1, 1,0), (-2, 2, -1)and (1, 2, 1). 14 o,
or, 21-2+ 3y -3-3z or
and
3v-32 =
5 is the required equahon equation (), we have
sOLUTION a - Zk, b -kc 5k
2k (r+1) 2k (y -1) - 3k (2 - 1) - 0
plane. Please referto 2069 (Set B) Q.No. 6b
3 =0 Putting the value of a, b, c in (i)
57.2070 Set CO.No.6b 2*2*2y -2-3z + 72.2072SuppQ.No,6
the or, 3z + 3 *
U1s the
required
Zk (x ) -3k (y -3) 5k (z - 4) - 0
of the plane through 21 2y Find
equation
and normal to the quation the equation of the plane through theor, 2x+4 -3y 24 Sn 20 0
póints (2 2 1) and (, 3, 6), 4
plane. points (2, 2, 1) and (9, 3, 6) and normal to the
plane 2x+ 6y +6z 9.
89. 2072 SetCQ.No. 6b plane 2x 6y 6z 9.
+ + =

[41
2x -8y
+3 -0
OLUTION
Find the equation ot the plane thro
sOLUTION 76. 2074 Set BQ.Mo. St
is intersection of the planes 2r 3y + 102 Find the equation of the
The equaton of plane through (2 2 1) Please refer to 2070 Set C QNo. 6b plane through the
and perpendicular to the intersection of the planesx + y +z =6
ax -2) b 2) cz -

1)=0 2r -3y+ 7z 2 pla 3. 2073 Set CQ.No.6t 2x+3y z


and
+ 5 = 0 and perpendicular to the
through the point 3-2y+4z =5.
f equation ) passes
Find the equation of theplane through the point plane 4x +5y 3z =8.
9.3, 6), then SOLUTION (2-3,1) and perpendicular to the line joining the sOLUTION
a9-2)b3-2) o -1)0 The equation of plane through the intersectiond two points (3, 4, -1) and (2, -1, 5). 4
or 7ab 5c-0 (i) the planes 2x + 3y + 10z = 8 and 2x -3y+72 The equation of the plane hrough the
Sance equaton (1) 15 normal to the plane 2x+3y+ 10z-8+ k (2x - 3y + 7z -2) =0
sOLUTION intersection of the planes x*y * z = 6 and
Direction ratios ot the line joining the ponts 2x
2x 6y 6z - 9, so 3y+ 4z 5 =0 is
2 a6 b+6.ce (11 or, (2+ 2kJx+(3-3k)y*(10+7k)z-8-2k =0. (3, 4,-1)and (2, -1,5)
are
x+y+z-6*k(Zx3y 4z 5) -0 6)
Since (i) is perpendicular to 3x -2y+42 a 2-3 -1, b= -1-4 -5, c=5-(-1) =6 =

Solving (ü) and (11), or, ( 1 2k)x (1 3k)y 1 4kjz 5k -

6-0
2+ 2k) 3+ (3-3k) (-2)+ (10 + 7k)-4-0 Now, the equation of plane through the point If this
or, 6+ 6k-6+ 6k + 40 + 28k =0
plane is
perpendicular to the plane
(2,-3, 1) is,
4x5y-3z=8 then
or, 40k+ 40 0 ax-2)+by+3)+ c(z -1) =0 ) 4. (1 2k) 5. (1
3k) 3)(14k) -0
267-12 k-1
Putting the value of k in equation (i), we ha
Since equation () is perpendicular to the
joining the points (3, 4, -1) and (2, -1, 5), so
line| or,
or,
4 8k 5 15 k
6+ 1lk = 0
-3- 12k 0
2r 3y+10z -8 -1 (2x-3y + 7z-2)-0
O6-30 10-2 2-2 or, 2x+3y + 10z -8 -2x +3y -7z +2-0 k(say) k-
a=-k, b = -5k, c = 6k
or, 6y +3z-6=0 Subsituting the value of k in (i)
Putting the values of a, b, c in (i), we get
2y+2 2
or. - kSuppowe) which isthe required equation of plane. -k(x-2)-5k(y +3)+ ók(z - 1) =0 xyz-6- 3yiz 5) -0
a 3k, b= 4k and c = -5k or, -(x-2) -5(y 3) +6( 1) =0
+ -

Or, 11x
11y 112 -

o6-12- 18y -24z -30 0


Substituting the values of a, b, c in (i),
70. 2072 SetDO.No.6b or, -X + 2-5y - 15 + 6z - 6 0 or, - x - 7y - 13z - 6 = 0
Show that the plane 2x+3y 4z 3 is paral X+5y -6z +19 = 0
we have
x *
7y + 13z 6 =0 which is required
the plane 10x 15y - 20z 12 and
3k(a-2) 4k (y -2)- 5k (z -1) =0 perpendicular to the 3z 5.
plane 3x + 2y + 74.2073 Set DQ.No. 6b equation of plane.
or, 3x -6+ 4y -8-52 * 5 =0
sOLUTION
Find the equation of the
plane passing through 2074 Supp QNo. 6
-52 the points (1,1,0), Show that the plane 2+3y z 3 is parallel to
3x4y =
9 1s the required
equaton of Given planes are (-2,2,-1) and (1, 2, 1).
plane 2x+3y 4z = 3 ) SOLUTION the plane 10x+ 15y -20z = 12 and perpendicular
to the plane 3+2y +3z = 5.
Please refer to 2069 (Set B) Q.No. 6b
68 2071 Set DQ.No.6D 10x+ 15y - 20z = 12
i) Please refer to 2072 Set DQN. 6b
Find the equation of the plane through the 3x+ 2y+ 32 = 5 111) 75. 2073 Supp Q.No. 6
Pgints (-1, 1, 1) and (4,-1,1) and perpendicular We know that two aix +
Find the equation of the plane through (-2, 3, 4) 78. 2075 Set CQ. Na. 6
planes biy Find the equation of the plane through (-2 34)
theplanea+2y+22 =5. 14 and azx t bey + c2z= d2 are parale and perpendicular to the pianes 2x + 3y + 4z =
and perpendicular to the planes x
SOLUTION and 3x + 2y +2z = 8.
s 0 and x + 3y + z - 6
2y
0.
+22 -8
The equation of plane through the
a+1)
point (-1, 1,1) is
b(y -1) * c(z -1) =0
SOLUTION Please reter to 2073 Supp Q.No. 6b
Equation of plane through the point (-2, 3, 4) 1s
Since ) 0.plane ()and (ü) are parallel i
equation () passes through the point alx - xu)+b(y - y1) * c(4 - z1) =0
MARKS QUESTIONS
(1,-1, 1), so a(l 1)* bl-1-1)+ o1
*
Or, ala+2) bty -3)*c(z -4) 0.. 0)
Or, 2a - Zb*0csU -1)-0 79.2070 Supp.Q.No. 19
(1) plane () Ls perpendicular tu the plane
Since equation Pruve that a plane through three points
(0) 15 perpendicular to the
plane1+ 2y+ 22 =5, so
a 1 +b 2+c 2=0
e-unue) x y + 42 6 and 3x + 2y + 22= 8, then
(L yu 21) (* y 2a) and (n yy z) is given by

(1) 1ence the planes (i) and (ii) are pard lel and 3a+ 2b 2
X - 1 y-yi 2- 2 1

Solving (i) and (i). we have Agan, we know tthat two - ya-y a-1a
àx +
plane g ihese equations by CrUss multiplcaluon
biy +
and azx
z di 0
*

ehod
Perpendicular if aiaz +bibz * cicular
planes (i) and (ii) are perpe
2-33 2+(4) 3
Mathematics-l

Asmita's NEB
Solution of Basic The equation ot plane throuo
( , y1, Z1) (22 0,-1), is(12, y2 the
104
passing
of the plane
Also, find

SOLUTION
the equation

through (22-1), (3,

The equation

(, y'1, Z1) is
4, 2) and (7,

of plane
0, 0).

through
the pOint
and (, y, Z3)= (7, 6)
X-2

3-24-2
7-2
y-2

2+1 =0
0-2 6+1
z+1

UNIT 6
b{y -y1) c(z-z1)=0
*

al n)+passes
Since ()
and
through the points
y, 2:),
so
(12 y2 Z3
VECTORS AND ITS
(r c(z- z1)
=0 )

a(r-X1)
+

b and
+

at2-n)+ b(y2 - y1) c{z1 - z 1 ) = 0 . (11)


b(v3 -y':) + and (11),
c from (i), (i)
- |--3| APPLICATIONSs
Eliminating a,
we have

X2-X1 y2-1 2-Z1 0 is the 1 -2) (14 * 6) - (y -2) (7-15) + (z + 1 ) .

16-12z -12=0 2-
A. ELEMENTS OF VECTORS AND ITS Putting the value of a in (i)
or, 20x-40+8y
-

APPLICATIONS
- 12z - 68 0
or, 20x+ 8y 2 MARKS QUESTIONSs (4.4)b 6.6
required euation plane.
ot
5x+2y 3z 17 is the required equa
plane. 2057 Q.No.4a or, b (5,6)-(4,4)- (1,2

ABCDEF is a regular hexagon. Express A and b=(1,2


AD in terms of AB and BC. 2060 Q.No. 4
If a (2 - 3) and b (4, - 2). Find the unit
sOLUTION
Let ABCDEF be a regular hexagon.
vector along 4 a 3b. 21
-

From AABC, AC = AB + B SOLUTION


Also, AD = 2BC
Here, a (2,-3)and b =(4,-2)
4a-3b = 4(2, -3) -3(4, -2)
(8,-12)-(12-6)
(8-12-12 +6) =(4,-6)
2058 .No.4 4a-3bYA6 -v16+36
-52-13
If a (3,-1,-4), b = (-2,4,-3) and
Unitvectoralong 4a -35 i a - 3 b
(-5,7,-1) find a -2b+c 2 4 a -3 b1
SOLUTION
Here, a (3, -1,-4), b =(-2,4, 3) and c - (-5, 7,-1)
.a
9-
-2b c
=(3, -1, -4) -2(-2,4, -3)+(-5, 7, -1)
3+4-5, -1 -8+ 7,-4+6-1)
(2,-2, 1) 2062 Q.No. 44
la-2b 1 - 2+7 - 3 uOP= 7+3j- 7k and o0= 5ï+27-4
2059 Q.No. 3 find PQ and determine its direction cosines. [2
fab (5,6) and -b- (,2,find a and b. 121 SOLUTION
SOLUTION Here, OP î +3 -7k and 00
Here, a b (5, 6)
+
(1) -5 -24k
a b.=(3, 2) PQ00-OP
Adding (i) and (i)
i-2+4) -(i +37 -7k)
2a (8,8) 4i -Sj 11k
a (4, 4) IPOI -VP+ (-5)3+113 - 2
Mathematics-ll

106
Asmita's NEB
Solution of Basic
Oc -3i+8 -6k
AB OB- OA i+3 VECTORS AND ITS APPLICATIONS Unit 6 107
direction ofPQ= pO
along the P
Unit vector
C OC-CA 2i+ 6j -1o -(OGGA) (OG +GB) +(OG+C)9 20705ot DQ.NoC 2of
- 2(i+3 -5k) =2A The vertices A, B,
(OG+GD) a
triangle are (2, -1, -3),
(4, 2, 3) and (6, 3, 4) respectively. Show that
-5-114i This shows that AB and AC are
N2 they start from
the s me pa
point A. So
40G+(CA +GC)+ (GD+GB) AB (2,3,6) and
AC 9.
20163 CLNo.JE vector
Care collinear.
-40CGA-GA)+ (GD-Gb) SOLUTION
find unit Let A (2, -1, -3), B(4, 2, 3) and
If a =
(2 3) and b =
(4. -2), 0. 2066 CO.No. 3 (: The diagonals of parallelogram
bisects each other)
C(6, 3, 4) be the
given vertices of AABC. Let 0 be the
2 Find a unit vector parallel too thhe origin.
along4a-3. Sum Then,
vectors 27+4-5k and î+27+ 40G
P l e a s e refer to
2060 QNo.4a OA- (2-1,-3)
SOLUTION 13. 2069(Sat A)Old Q.No.4a OB (4,2,3)
2064 .No Ifa = (3,4) and 3a +2b =(5, 6)find b.
Ifa (3,4) =
and3a +2b =(5,6),
find b. [21 Leta-2+4 -5k,b=+2
Now,
Please refer to2064 Q.No.4a
2
oc (6,3, 4)
=

Now,
SOLUTION a +b 3i +6j -4K 4 2069 (Sot B)C.No.2 AB OB-OA= (4,2,3) (2,-1,-3)
or-3-7k ando0 -s+ -274.
-

Here,
(4-2,2+1,3+3) = (2,3,6)
a (,4) abI y32 +62 +(4}P - Va find P and a unit vector along the direction of AC =
OC-CA= (6,3,4)- (2-1,-3)
3a +2b 5,6) ) A unit vector parallel to the
sum of (4,4,7)
From () and (i) Po 2
33,4)+2b 5,6)
Please refer to 2062 Q.No. 4a AC-1AC 7y1 -9
ot, 2b (5,6)- (9,12)
a b 15 2069 (Set A)Q.No.2
Show that the three points with position vectors
19 2071 Od O.No. 3E
If D is the middle
point of BC of the
triangle
or, 2b (5-9,6-12)
V61 61 61 T2 +4k, 2i +sj-X and si+87-6 re ABC showthat AB +AC =2D. 12
collinear.
or, b-4-6) sOLUTION
1 2067 Q.No. 4a Please refer to2066QNo.4a Let D be the middle
point of BC of the triangle
. b=(23 Determine the unit vector of 2a-3 16 2070(0ld)Q.No. 3 ABC.
Find the direction cosines f the vector
a-4T+3 j and B =-2T -3. From AABD, AB+ BD = AD
2065 O.No3E MN where
...)
If 3+-kand à î - 4j+4kare collinear | SOLUTION position vectors of Mis i +
6j +6k From AACD, AC + CD = AD
(ii)
vector. Find i 12 Here, a =4 i+3 j and b =-21-3 and Nis 4i+ 9j+ 6k. Adding i) and (i)
[21
SOLUTION 2a -36=2(4T +37 )-3(-27-37 SOLUTION
Let M andNbe two
AB AC+BD+ CD=24D
points with position or AB AC BD - BD=2AÐ
Since the vectors 37+ -k and .+ -4+ 4k 8 i +6+61+97=14
are collinear, so vectors-i +6j + 6k and 4i+ 9 +6k
2 -3B1=V142+15 -41 respectively. Let O be the origin. Then, C Dis the midpoint of BC)
A(3T+7-k) =i.+ -47+4k for some scalar A. Unit vector in the direction of 2 a-3 AB+ AC 2AD
OM= -i + 6j + 6k
or
3AT A Ak -47+4k -

2a-3
ON= 4i+ 9j + 6k
22 2071 Sotca.No.2
Equating the coefficient of i, j and k, we get
. 3A
12a-351 V421 If 3f k and
-

ai - 4j+ 4k are collinear


... (i) MN - ON-OM3 3 vectors. Find the value of
A =4 12. 2068 Q.No. 4a Please refer to 2065 Q.No. 3b
(i) ABCD is a parallelogram. G is the P
From i) and (i)
intersection of its diagonals and
IMN =V-33 +3 -32 21.2071 Set DQ.No. 2
3x (4) =-12 Show that the three points whose
2066 O.No. 4 point, show that: OÅ +OB +OC+0D Unit vector in the direction of MN
is MN position
Prove that the | MN| vectors are 7 +10k, - ï+67+6k and
points A, B, C are
collinear, if|sOLUTION
OA 727 4k,OB
-

4+9 6k form an isosceles triangle. 2


oc -37 +87 -6
2 5 -k and Given, ABCD is SOLUTION
Let A, B and C be the three points with
parallelogram. Also G position
SOLUTION is the point of
Required d.cs of the vector MN are
vectors 7+ 10k,-ï+ 67+6k and 4T +97+ 6k
Let A, B and C be three
points with position intersection of 7. 2070 Set CQ.No. 2 respectively. Let O be the origin. Then,
vectors i+2j 4k, 2i +5j - and + diagonals and O is any
point,
ABCD is a parallelogram. G is the point of
intersection of its diagonals
OA=7j +10k

3i+8-6k respectively. Let Obe the origin.


Then Now,
and if O is any pounOB7+6+6k
show that: OA OB +OC +OD 40G.
OA i+2 +4k OA+OB + OC + OD Please refer to 2068 QNo. 4a 2OC-4f+97+6k
OB 2 +5j -k
)
Mathematics- 10k
NEB Solution of Basic
and 7 j
+ respectively, Let o.
108 Asmita's
VECTORS AND ITS APPLICATIONS Unit 6 109
rigin. Then,
Now, SOLUTION
20 2072 Supp Q.No.2
+3k
OA i -2j
AB OB-OA =-i -j-4k Suppose OB and OCare two straight lines and D

BC OC-OB- -37+37
OB-2i+3j -4k 3 7 + - k and Ai - 4j + 4k are collinear is a point on BC such that BD:DC = m: n
vectors, find
the value of A. 12
OC-7 + 10k So, m
S0, DC n
AC-Oc-OA -4+2-4k Please refer to 2065 Q.No. 3b
AB 1ABV-1}+(-13* (4)=v18=32
Vow, AB
OB- OA
-iS-7 27 2073 Set CQ.No.20 Or,
BD I
n
BC BC=V-3}+3=v18 2 andAC OC-OA-i -57+7 Ifa 2i -3j +4k and b - i +2j -2k, finda

AC AC| =V4+2+(4)
=6
-(i+5 -7 unit vector along
the direction
of 2a +3b. 21 or, nBD =mDC
Here, AB = BC -AB sOLUTION
B and C
triangie.
vertices of a n isosceles
Since AC and AB are parallel and bo
or, n
(OD-OB) m(OC-OD)
So, A =
are
Here, a -2-3 4k or, nOD- nOB mOC- mOD
from the same point A; so A, B ana
2 2071 Supp. Q.No.2 points with position
value of a
Find the the if collinear. b-i+2 -2k or, nOD+ mOD = nOB+ moC

and Si- 10j+ k 2a+3b 2(2i -3j +4k )+3(-i +2j -2k)
vectors +2+k, 21-j+ 3k 24 2072 Set DQ.No.2c oD OBmoc
+n
are collinear.
Ifa (3, -1, 4, b (-2, 4, -3) find unit ve
=
-41-6+8k -31+6 -6k
SOLUTION =i +2k
32 2075 Set AQ.No. 2
posiion 2b.
along a
with
Let A, B and C be
three points -

Express r (4,7) as the linear combination of


5i - 10j+ k sOLUTION 12a +3b-T+2 =y5
vectors i+2j +k, 2i -j+3k and
Here, a = (3,-1,-4) Unit vector along the direction of 2 a + 3b is
a (5,-4) and b (-2,5). 2
respectivey. Let O be the origin. hen, sOLUTION
b (-2,4,-3) 2a +3b Let

2a +3b T=xa+ yb .(1) where x and y are


a-2b =

-2(-2,4,-3
(3,-1,-4) Scalars to be determined.
(3,-1,4) (-4,8,-6)= (023. 2073 Set DQ.No. 2c
-

or, (4,7) = x(5,-4)+ y(-2,5)


-8,-4+6) = (7,-9,2)
ABCD is parallelogram. G is the p01nt ot
a
or, (4,7) =
(5x 2y, - 4x + 5y)
-

intersection of its diagonals and if O is any point


a -2b1 =v7+(-9+2 Then,
OA-i+2-k -134 showthat: OA+OB +oC + Oð =40 5X-2y 4 .. (2)
Please refer to 2068 Q.No. 4a and,
OB-2- 3k 4x +5y =7 8)
2b is-
a - 2b
2 2073 Supp Q.No. 2 Solving (2) and (3), we get,
Unitvector along a -

OC-5i-10 +k -281 Showthat the vectorsî +27 +4k,2++57 -K x =2,y =3


AB OB-OA -9 and 3i +8j +6k are collinear.
Substituting the values of x andy in (1), we
121
=i-3j+2k ,9,2)134 13 Please refer 2066 Q.No. 4a get,
T-2a+35
AC OC-0A-4i-12j+ 0-1)% 25. 2072 Set E Q.No. 2 80. 2074 Set BQ.No. 2c
B =(-2, 4, -3) and T= (-5, 7 - . 2076Sat BQ.No.2
Since A, B and C are collinear, so AB is If OP = i + 3i 7k and 0Q= 5i -2t(,-1,-4),
-

find the unit vector along a - 2 +7. 121 Show that the points 21+-K,3i-27+kand
find PQ and its direction cosines. SOLUTION
parallel AC. Then, T+4-3 are collinear.
a(i -3j+2% ) = 4i - 12j +(.-1)k for some scalar a. SOLUTION a-2b+c -

(3,-1,4) -2(-2,4-3)+(-5,7, -1) sOLUTION


or ai -3a +2uk=4i -12j +(-1)k Here, OP-i +3-7k and 0Q-5i- (3,-1,4)-(4,8,-6)* (-5,7,-1)
(3*4-5, -1-8 +7,4*6-1)
Let O be the origin. Let A, B and C be there

Equating PQ-00-OP -(5i-2j+4k)-(i+3 = (2,-2,1) points with position vectors 2+ -K, 37


4
.(i) 27+ andï+47 -3k, respectively. Then,
&2a .-1 (i)
4 i -5j + 11k a-26 I -v2(2P+ -3
From () and (i)
2x4=-1
PQI =42+(-5) +112 =N2 Unit vector along a -2b + c OA-27 7-
OB-37-27
)
Ja-2b+ - )
=9 Unitvector in the direction of PO 15POl
32072 SptCO.No.2 oc-7-3
Now,
Prove that the vectors i -2 +3k,21+37-4 4i-5i 1. 2074 Supp Q.No. 2
AB 08-0OA
and 7 j + 10k are collinear. 2 and OC are two straight lines and D is a
12 POnt on BC suchthatBD: DC = m:n, show that
SOLUTION Required d.c's are -(7-27 )-ei+7-)
Let A, B and C be three points with position N o5.nOtmo
m+n
(21 -7-3 2
vectors i -2j 3k,2i +3 -4k
Mathematics -ll
Asmita's
NEB
Solution of Basic
Now,
AB OB -OA
110 Unit 6 111
VECTORS AND ITS APPLICATIONS
andA OC-OA -5
And,
BC OC-OB - - 3 + 3 SOLUTION
AC -O -0OA
74-3R-ei+7-
-(i3-
-AB CA -OA-OC 4T-27+4
Suppose OB and OC are two straight lines and D
19 a point on BC such that BD:DC = m: n

Since AC and AB are parallel ana


-i+37-2 from the same point A; so A.1 AB ABI -1+ 1 (-4P So,
B
- (i-37+2) colinear. and
BC BCI V-3+3-V18-2
- AB 36.2059 Q.No.10 CA CA. = V4+ (-23 +4 =6

AC-ABwhich
that AC and
shows
from the same point
A.
AB are

| Provethat the
vectors -a+4b+3:
-a+4...
or, nBD = mDC

But they start and2a+7b -3c are coplanars, where or, n (OD-OB) = m(OC-oD)
parallel. collinear.
So, A, B &C are are any vectors. or, nOD nOB = mOC- mOD

MARKS QUESTIONs SOLUTION oT, nOD + mOD = nOB+ mOC

42057 QNo. 10 Let, n -a +4b +3c OD nOB+ mOC


vector of M and Nare L m+n
If the position
n =2a -3b -5c
Here, AB= BC 40 2063 ONo. 10
3 3K and 4i - 2j+k respectively, -2 +7b-3c Also, ABCD is a parallelogram G is the point of
its direction cosines. 4 intersection of the diagonals and if Q is any
find MN and determine If three vectors are AB+BC2 (2+ (2)¥ =18+18 36 (CAp
coplanar than ones B = 90°
can be written as the
SOLUTION
Let M and N be two points with position multiple of other two say
sum of the
Hence, A, B and C are vertices of isosceles point, show that: OA+ OB+OC oD-406j4|
right angled triangle. SOLUTION
vectors 3 i +j -
3k and 4i 2 j +k xn +yn 2061 .No. 10
Given, ABCD is a
respectively. Let O be the origin. Then, parallelogram. Also G
2a +7b -3c =x(- a +4b +3c)+y2a- Prove that the following vectors are coplanar;
is the point of
OM-3i - 3 or, 2a +7b -3c =(R+2y) a +(4r-3y) b+ a-36+5, a -2b+3c,-2a +3b-4 14] intersection of its
diagonals and O is any
ON41-2- Equating we get,
SOLUTION point
-x+2y=2 (i) Now,
MN -oN-OM-7-s7-4 4x-3y-7 ...ii) Let, a -3b +5c

IMN| =V12 (-3+ - 26


x - 5
3
Solving (i) and (iii), x -4, y 3
iv
-2+3 OAOB OC-0D
Putting the values ofx and y in(iv), we Ts-2 a +3b -4c
Unit vector in the direction of MN is
If three vectors
=
(OG +GA) +
(OG +GB) (OC +GC)+
3 x4-5 x3 =-3 (true) are
coplanar then one
vector
MAN From (i), rs = 4ri +3r2
can be written as the sum of the scalars OGGD)
multiples of other wo say
| MN| Hence the given vectors arecoplanat r xn + yr
=

40G (GA +GC) (GDGB)


37 2060 Q.No. 10 where x and y are scalars. -4OG GA-GA)+ (CD-Cb)
Show that the three points whose p C: The diagonals of parallelogram
Or2a *3b -4c
=x(a -3b
*5c)+ y(a -2b +3c) bisects each other)
Required d.C's are 26 vectors are 7+10k,-7+6j+6k or-2 a +3b -4c = (x+ y) a *(3r-2y) b +(5x + 3y)C
Equating we have 40G
3. 2058 ONo. 10 9j +6k form an isosceles right ange
Show that the points A, B and C with
*y-2 (11) 81 2064 Q.No. 10a
position -3x-2y=3 ii) Show that the three points whose position
vectors i-2+3k, 27+3j-4k,-77+10 sOLUTION 5x+3y4 (iv)
Let A, B and C be three points Solving (iü) and (iii), x = 1, y = -3 vectors are 7 j +10k, - i +6j +6k and - 4i
respectively are collinear. W Putting the values of x and y in (iv), we get
SOLUTION 14 vectors 7 10 K, -i+6j+6 9j 6 k fornm isosceles
Let A,
5 x1+3 x (-3)=4 (true) an
right angled
Band C be three points with position -4i triangle. 14
vectors i-2j +3k,2i+ +9+6k respectively. Le tO bene So,Hence from (i) rs =n -3r2 Please refer to 2060 Q.No. 10a
3j -4k Then, the given vectors
are coplanar._ 2. 2065 Q.No 10 a
and 7 j+ 10k
respectively. Let O be the 89 2062 O.No. 10 Show that the following vectors are
linearly
origin. Then, OA 7+10K Oand OC are two straight lines and D is aaepenaen
Point on BC such that
OA i -2 +3k OB - i +6+6K
BD:DC m:n, show that 5i+6j +7k,
7î -8+9k and 3i *20+5k 141
=

OB 2i+3j - Ob.nOB+mOC SOLUTION


4k oc -4 i
+9j+6k =
m+n 4
oC -7 + 10k Given vectors 5i+ 6j+ 7k, 7i 8j+ 9K are -

AB OB-OA - i - i - 4 and3 20j +5k


+(-2+
Mathematics-l

of
Basic
AB AB|
-67
Solution
NEB
Asmita's

112 Unit 6 113


BC-BC » VECTORS AND ITS APPLICATIONS

Now
CA ICA| =V(-1)2 +32
B. PRODUCT OF VECTORS
82 2060Q.No.3
Find the cosine of the angle between the vectors
Here, 2 MARKS QUESTIONS
24)
BC2+CA
C 90
(N6 35)- 41 -

40 2067 Q.No.3
21 7+K and47+37+5k. [2
(35
27)+ 7 (140+ nd C sOLUTION
-6 B are the +3j -8k and
-

Hence A, vectors 2 f
5(40-180)
-1100- 48+1148 0
linearly
dependent.
angledtriangle. vertices n Show that the
Let -277-
are
given
vectors
27+4+2k are orthogonal. 2
the
32066 Q.No. 10 6 2068 Q.No.10 SOLUTION -47+37+5K
66+7ë,7 -86+9
Prove that the three vectors Now,
Prove that
the vectors5a+ 41 28 Let a-2+37-8k
and 3a+20b +5c are coplanar. -2a+3b-4 Cand - b+2c are
coplane b 2 i +4 +2k
l a - 2 12 -V6
bI P 3 5 = 52
sOLUTION
SOLUTION
Now,
5a6b 7 Let n a-2b +3 a.b (21 k).41 35k)
Let n
a.b -21+3 -8k).2í+4+2k = 8+3 +5=16
7a -85 9 - 2 a +3 b 4 c 4+ 12-16 =0
If e be the angle between a and b, then
n 3a 20b 5c then one
vector Thisshows that a and b are orthogonal.
three vectors are aoplanar s - b +2c C O s0 a b
16
f
written as the sum of the scalar 60. 2058 C.No.3D
can be three vectors are coplanar thenone Show that the area of the parallelogram
muliplies of other two. can be wTritten as the sum of the
Let TiXT+ yT2 .() multiples of other two vectors. So,let determined by i
+
j -

3k and i -2 -3k is 53. 2061 ONo. 3 6


+6b 7c)+y7a -8b +9c ) r x r 1 + yr2 118 sq. units Find a unit vector perpendicularto 2 i+3 -K
or 3a-20b
Sc
=(5a sOLUTION
+(6a -8y) b (7a +9y) c
where x and y are scalars
or, 3a 2 0 b 5 c
Equating, we have
(Sz 7y) a

Let a 7+-3k and +-2k. 21


or, -b+2cx(a - 2b +3c) +y(-2a+36. SOLUTION
51+7y=3 (4) b--21 -3k
6x 8 y 2 0 (i) or, -b +2c= (x -2y)a +(-2x +3y)b + (3r- Let a-27-37-R
Now,
(iv) Equating we have
7x+9y=5
Solving (i) and (ii), we get I 2,y
Putting the values of x & y in (iv)
=-1 =
x-2y 0 (i) -77-2
-2x+3y -1 ii) Now,
7x2-9x (-1)=5 (true) 3x-4y 2 (iv)
Solving (i) and (ii), we get x = 2,y=1
-
From (i) rs = 2r1 - r2

Hence given vectorsare coplanar. Substituting the value of x and y in (iv) 437-43|7:||
3x2-4x 1 =2 (true) -9i+6 -k
42066 CC.No.10 From
Show that the points A, B and C with position Area of the parallelogram determined by a and b
vectors ï - 27 3K. 27 37 - 4R, r3 2r1 +12
Hence the given vectors are coplanar.
is a b x
--53-K
-7 +10k respectively arecollinear. 4 =-9)262+ (-1)2 =y198 sq. units.
Please refer to 2058 QNo. 10a 47. 2069 (Set A) OldQ.No. 10a 51. 2059 Q.No. 4 a
ab-V-5-3-(-1)-V
45 2067 QNo. 10 OB and OC are two straight linesand Find theangle between two vectors: Unit vector perpendicular to a and b is
point on BC such that BD:DC = m:n, sho
Show that the three points whose position axb
are
2T-7 K,T- 37 5K and 3-
vectors
- 2 k andb-27-7-R. [21
47- 4K nOB+ mOC SOLUTION
Ts7-
-

form the sides of


right a
angled triangle. m+1n Here, a = i+ j -2k
SOLUTION Please refer to 2062 Q.No. 10a 54 2061 QNo. 4a
Let A, B and C be three
points with position
vectors 2T -)+K,i -3j -5k and 3í -4j 8 2070(Old) Q.No.10a 6-27-7-K If a and b are two vectors of unit length and
respectively. Let O be the origin. Then, -4k lal -12 12 (-2 =yo is the angle between them. Show that
Prove that the vectors a -2D
Tb
OA 27-T K
and - b +2c are coplanar.
V22+ (-1)3 +(-1) =y6 -B1-sin 121
OB 7-37 -5K Please refer to 2068 Q.No. 10a N
oC 3T 47 -4K 2-1+2 =3 ã- 5 (a b:
be
AB OB-OA -7 the angle between a andb, then a - 2 a b +b
-2j -6K Cos a b_ - 1 -2a b +1(" | al =|bl =1)
BC OC-
OB 27 - -7+K allb Vory
Cos 60
al
CA -OA-oC -T 375K 60°
2-2 |bl cos
('ab al |blcos 0)
VECTORS AND ITS APPLICATIONS Unit 6 115
-o+0+41 41
Mathematics-l"

NEB Solution of Basic la b x


A s m i t a ' s

®) =2 (2 sin?0/2) triangle determined by a.and


2 2068 Q.No.3 B6. 2069 Old (Sot B) O.No. 4a
=2(1 -cos Area of triangle determined by the Find the area of the triangle determine by the
2-2 cos Findthe of the
area

or, la-5|1 (2sin 0/2 b-ab-j*205sq,uni vectors 31 +4 and -


5T+7T. 121 vectors: 3 +4j +k and-5 +7}. 12
3b
-2 sin Please refer to2064 Q.No. sOLUTIOR
or, a-b| 4
S62065Q.No the vectors 3T-7.. 63. 2069 (Set A) Old Q.No.3 Let a 3 i +4j +k
lã-B-sin Find the
value of r if - l b - | a - b l, prove that a is perpendicular
are orthogonal. -5i +7
5 2062 Q.No.3D
determined|2-2j+rk

121
the of the
area
parallelogram
2 sOLUTION tob.
Find
by the vectors Let a 37-7-2K SOLUTION
Here, -5 7

T+2+3 and-3T -27+ -27-27+


a+bi-a -b
SOLUTION
The vectors
a and
bwill be orthogonal if bs +
- b|12
b |2= | a
|7|3|7:|%;|?
Let a i+2j
+3k ie. i - 7 - 2 k ) . ( 2 1 - 2 + r k ) =0
or, a - -7i+5 41k
b -3i -2+k or, (a b = (a - b}»
or, 6+2-2r =0
or, a2+2a.
b +b2 =a2 -2a b+b2 axb VT+5+41 =V175
Area of triangle determined by a and b is
59 2066 CO.No.4 b =0
or, 4a
Ifa=? +27 + 3k and b - 27+ 37+4
or, a b =0
Eaxbi =y1755 20.95sq units.
17-13i|7:|33|E|theprojection of a on b.
a 1s perpendicular to b. 7Find2069 (Set A)QNo.3d
the area of the parallelogram determined
sOLUTION
+3k 4 2069 (Set B)Q.No.3c by the vectors
=8i - 10j 4k
and b
Here, a i +2 a-i+7-2k and =2ï -7 - areany i+2 +3kand-3i -2+.
determined by a
Area of the parallelogram b-2+37+4k two vectors, find the cosine of the angle between
Please referto 2062 Q.No. 3b
65 sq. units
is a x b =ys+ (-10)2 +42 a.b i +3k).(2í +3+4k) thetwo
+2 vectors. (21
2070 Set CO.No.3
S6 20653 O.No. 4E = 2+6+12 20 SOLUTION
mutually perpendicular
unit
bI =22+32+4 29
=
Here, 7F-|7-F|.poethat a
If i, j, k are three
a-7+-2K
i - 2j+k,b=27-37-k b 2D is perpendicular to b. 21
vectors
and
find the cosine of the
a= angle between the wo Projection of a on
=

b27 -j -k Please refer to 2069 Set A(Old) Q.No.3b


2 b
vectors. al 13+(-2 =V6 aFind2070 Set DONo.3G
SOLUTION 80 2066 Q.No. 3E vectors í- 2+ b-2+(-1}+(-13 -v6 the sine of the angle between the two
the
Forwhat value of m
are vectors 2
Here, ai -27 27+77+mk orthogonal?
a.b -(i+ j -2k). (2i - j - k)
2i j +k and 3i 4 j - k
= 2-1+2-3
b-27-37-K SOLUTION
Ife be the angle between a and b, then sOLUTION
lal -v12 (-2P+1 -y Let a =7- 2 +4k
b 2+7 + mk cos
Let a-27-j

a.b -(-27+).27-37-K) The vectors a and b will be orthogonali


a.
6-37 47-T
+mk)=0 65. 2069 Old (Set B) Q.No. 3b
2+6-1=7 or, ( -2 +4K). (27 +7 For what value of x is the pair of vectors
If e be the angle between a and b then or, 2-14+ 4m 0
m=3 xi-2+ 4k and 27 +77 +k orthogonal?
a b [21 a
cos =
81.2067a Q.No.
Given (3, 3b1, 2) and B (2 - 2, 4, SOLUTION
Let a -xi -2 +4k
57. 2064 Q.No.3
Find the area of the triangle determined by the projection
SOLUTION of on b.
b 2 i +7j k 1|-134||3#|7
vectors 3 4 and -57+7. Here, a - (3,1,2)
Vectors and b will be orthogonal if a .b =0 -3i +5+11k
12 a

axbl V-3P +58+1t -v155


sOLUTION b (2,-2,4) e-274k)-
c.2* or, 2x-
(ei +7 +K)-o
Let a 3i +4 j b--5i+77 4)=6- 14+ 4 0 Ife be the angle between a and b, then
Now, a . b (3, 1,2)(2,-2, X=
5
axb
b-22 (-2}:+4 =y24 ne-larbl - 5
allb Vóv26V156
5
Projection of a on b
0i +0j +41k
Methematic8-i
Unit 6 117
Baslc VECTORS AND ITS APPLICATIONS
Solution of
NEB
116 Asmita's
m.-
Here,
or, 1a| -|b or, ab+c =0 . (i)
o 2070 (O1d)a.No.4 to the
vectos

From ()
unitthe
vector
perpendicular
Jal-b
ind

2072 Set EQ.No.3d


+ b+ axc -0
4i 2
+3k and 5T+-4k 77.
Find the area of the triangle determined by the or, 0+ a x b- - a x
(ax a =0)
-8 -8-8k
sOLUTION
31 +4j and-5 i +7 j. 2
axbl=+(-8)2+ (-8}¥ =& vectors or, a x b = c xa ..(i)
Let a-4-2+3k Please refer to 2064 Q.No. 3b Again, from (i)
2072 Supp Q.No.3
vector perpendicular to
b-5 -4k Unit
axb bx b+cxb =0
6 a-b| prove that a s 0T, ab+0-cxb
la x bl perpendicular to
b .

or, ax b = bxc .(ii)


||i|i|'|s -7 Please
79.2073Sot CQ.No.3c
refer to 2069 (Set A) Old QNo. 3b From (i) and (i),

20710ldQ.No.3 product x b is perpendicular to


axb bxc =c*a_
24 Show that vector
a

5i-31 14k Find the vector perpendicular to each o


both vectors a and b 12
81 20173 SuppQ.No.3
=V1182 (2, 1,-1).
=V5: (-31J3 14
and
la xbl
to a and b is
vectors(, ,
sOLUTION
- 4)
SOLUTION If a =++27-k and B =+ -7+ind the
vector
perpendicular
projection of a on b. 121
Tut Let a (al, a2, a ) and b = (bi, b2, bs)
Let a (1,3, -4) sOLUTION
Then,
b (2,1,-1) Here,
a bl bi
-31 14k)
-Si31j+4

1182
Now,
axb (azbs - aabz, asbi - aibs, abz - azbi)
i 2-
5-7-7
Vizsi
2

1 2070theSupp. Q.No. 2c a xb (-3+4,-8 +1,1-6) -(1,-7 Now, (a b). a x


Now,a. b=(T -27-F>(+-7 K)
between two unit
vectors. 1-2-1
Ife is angle The vector perpendicular to a and bisa
=
(aabs asbz, asbi -aibs, aibz a2b1) (a1, az, as)
- -

= -2

and 6, showthat l- 5| =sin 12 =


a(aabs asbi) + a2(asb1- a1bs) + a(anbz azb)
- -

1,-7,-5) aia2bs - a1bzas + bia2a3 a1azbs + a1bzas - bazas


DI-VR-(-1+1 -V
to 2061 Q.No. 4a
5. 2072 SetcQ.No. 3c
Please refer = 0

Projection of a b=
2 2071 Set CQ.No. 3
on
Find the angle between the vectors 2i-i axb is perpendicular to a.
For what value of m is the pair of vectors
Similarly, (a x b).b
T-2+4k and 27 +7+mk orthogonal? 121and i -3j -5k. (a2bs - asbz, asbi - a1bs, aib- a2b1) (bi, bz, bs) 82 2074 Set AQNo 2
SOLUTION SOLUTION Prove that (-5)
(+5)-2 5. -

Let a T-2+4k Let a 2 i - i + bi(azbs - asb2) + bz(asbi - a1b») +bs(a1b2 - azb1)


SOLUTION
0
b 27 7+mk b i-3 -5k LHS (-B)- (3-3)
axbis perpendicular to b.
The vectors a and b will be orthogonal a =22+(-1?+1 =v6 80. 2073 Set DQ.No. 3c
a a a B-B*a -BxB
O+aB-Bxa-0
ifa.b 0
b -v1+(-3+(5} -V Ifa +b+c 0, prove that
ie. (-2+ 4k). (27+77+mk) =0
or, 2-14 + 4m = 0
a.b =(2-+k).(i-3j- axb b xc =c xa.
=2+3-5 =0 SOLUTION
.. =-3
Ife be the angle between a and btnen 2a B RH.S
x

3.2071 Set DQ.No.3c In AABC, let BC =


a, CA= b, and AB =
c
83 2074 Sot BO.No.3
Find a unit vector perpendicular to each of the Cos
a.b = 0 cos
Find the area of the parallelogram determined
vectors T-A
I al |bV6y35 by thevectos f +2+3K and -3î -27 +R 12
3++2k and 2 -2+4k 21 =90° Please refer to 2069 Set A Q. No. 3c
SOLUTION 6. 2072 Set DQ.No. 3 42074 Supp Q.No.3
h-C
Let a
=3i +j+2k fab).(a -b)=0,prove that | 67 7-si andb-7-47+2k.
b 27-2+4k SOLUTION By detinition of vector addition,
Here, (a+b).(a - b)-0 show that a xb is perpendicularto a 21
Unit vector perpendicular to AB AC CB
a and b is
a . b+ b.a b.b0
axb O a.a
or, c =
-b - a
Iaxb Or, a 2-a.b+a.b -[b|2
(a.b b. a)
Mathematics-il

Solution of Basic And,


Asmita's NEB
118
B-V3P+-VT0 VECTORS AND ITS APPLICATIONS
sOLUTION Unit 6 119
Here, a b 6
Projection of
on

-6T3-5K Now,
h cos B sínB.
Cos A r1 sin A A nsin A
-7-42k a MARKS QUESTIONS BNKA X
Now, 0Q OP-(0, 0, ri Fa sin A cos B- ri r2 cos A sin B)
72057C.No.11 0Q OP = rnn (sin A cos B- cos A sin B)
Prove by vector method: Since (A- B) is the angle between OQ and OP, so
cos (A - B)= cosA cOsB+ sinA sinR
SOLUTION
Also, let OP = r1 and OQ = r2 sin (A-B)-oQ OPL
Then the coordinates of P and Q are (ri cos A, 0Q1 OP|
Let XOX' and YOY be tw
risin A) (sin Acos B- cos Asin B)
431-7|i?|-*| perpendicular straight lines reDro and (ra cos (m - B), ra 8in (T - B)) = (-12 cos B, n

axis and y-axis respectively. Let sinB). sin A cos B - cos A sin B
and XOP = A so that 2Q0P = A

n and OQ=
So, OP (ri cos A, n sin A) 81. 2061 ONo, 11E
let OP 2. Then the
sincoordn
( r i cos A, TI sin A, 0) Prove, in any triangle, by vector method that:
of P and Q are (Ti
T(6-20)-7(12 5)+ k(-24-3)
COS A,
(T2 cos B, r2 sin B) respectively. So,
i
& 0Q (-r2 cos B, rz sin B) =
(-ra cos B, r2 sin B, 0) sina A sin B sin C
b
-14T-17 -27K P Now,
sin A sOLUTION
COs A A
ri sin
T-2 Cos B T2 sin B COS A
To show x b is perpendicular to a,
we
X acos B sin B
In AABC, let BC = a, CA =b, and AB =
b.a=0. B OPx OQ- (0, 0, ri
ra cos A sin B+ Iin sin A cos B) By definition of vector addition,
haveto prove
x

OPx OQI = n n(sin A cos B+ cos A sin B) AW


Now,
t-A

B).7 O
B)A Since
0Q
(A +
B) is the angle between OP and
(-147-177 -27K).(67+37-5K) sin [7-(A +B)] =OPxOQI
=
- 84 -
51+135=0)
OPI |0Q|
This shows that xb is perpendicularto a . I1 a (sin Acos B+ cos Asin B)
85.2075 Set AQ.No. 30C OP = (1 cos A, ri sin A)
sin
AB AC CB
Find the cosine of the angle between the two (A+ B) =
sin A
cos B+cos AsinB or, c -b - a
0Q= (ra cos B, r2 sin B) 90. 2060 C.No.11E
vectors7-7-27+ 3Rand B-7+57- 2R.
12
Now, Prove by vector method: or, a + b +c
From (1)
0
()
sin (A B) = sin A cos B
-

cos A sin B
sOLUTION
-

OP.0Q = (n cos A, ri sin A). (r2cos B, sn


Given, T2 Cos A cos B+ rn 2 sin AsinB SOLUTION a xata x b * a xc =0
Let XOX' and YOY' be two or, 0+ a x b =-a x c
a T-2T+3K 1 2 (cos A cos B + sin A sin B mutually (:'a x a =0)
perpendicular straight lines representing x or, a x b =c x a
B T+3+2K Since (A B) is the angle between 0Q and 0R axis and .-(i)
y-axis respectively. Let <XOQ =
B Again, from (i)
and XOP =A so that
T-(-2P+3=y14 cos(A-B) =00 OP 200P A B. Again, = -

let OP n and OQ n. Then the


=

coordinates =
axb bx bcxb=0
-V3+2- 4 0QI 1OP of P and Q are or, a x b +0 -c x b
. (T-2T 3K). (T+37+2K)
=1 -6+6=1 i T2 (cos Acos B+ sin Asin B) P
or, a x b = b x c
11)
From (i) and (ii),
If e be the angle between a and b cos A cos B + sin A sin B Q
then, ax b bx c cxa
cos 8. 2058 Q.No. 11 a Taking modulus on each side, we have
Prove by vector method. ax bl |bx cl= le xal
cos(A B) = cosA cosB + sin A sin B BA X or, ab sin (7 C)
- =
bc sin (t - A) =
ca sin (x B)
6. 2075 Set CQ.No. 2
-

or, ab sin c = bc sin A = ca sin B


Please refer to 2057 Q.No. 11a
If a =
(1, 2) and b =
(-3, 1), find the projection sin AsinB sinC
on b
of| e9 2059 Q.No. 11 a DI,

Prove by vector method:


2
sOLUTION sin (A +B)= sin A cos B+ cos A sin D
i cos A, A sin BsinC
Given, ri sin A) and (ra cos B, rn sin B)
SOLUTION mult

respectively. So, 92 2062 Q.No. 11 a


- (0,2), B= (-3,1) Let XOX and YOY be wo reprex OP ( i cos A, n sin A) = (n cos A, n sin A, 0) Prove vectorically that:
Now, perpendicular straigh lines cos (A- B)= cos A cos B+ sin A sin B 14
X-axis and y-axis spectively. Let2 OQ-(ra cos B, ra sin B) (a cos B, rn sin B, 0) Please refer to 2057Q.No. 1la
a.b-(1, 2). (-3,1) -3+2- -1 =
and 200x = B so that ZPOQ*
VECTORS AND ITS APPLICATIONS Unit 6 121
of Basic
Mathematics-l
2067 Q.No,1
2069 0ld (Set B)C.No, 10
NEB Solution
120 Asmita's

by
vector
method:

A 8in R
H02.
Ghow that the three points whose position
105,2071O1d Q.No.10E
3 2063 Q.No. 11 triangle
that:Prove

(A+ B)
cos
Acos -8inI1, Q.No 1 Show that the area of the triangle PQR whose
refer to ModelSet
vertices are P(1, 2, 3), Q(3, 4, 5) and
USing vector method, prove in any cos
Please vectors are:
21 -j +k, Í- 37 - sk andi R (1,4,7) is N6 sq. units.
a=
ab cos c-
b cos C+ccos B 2068 Q.No.11 4-4k form the sldes of a right angled sOLUTION 4
SOLUTION method, prove
that; triangle. Also, find the remaining two angles. 14]
let BC= a, CA b
and AB -
Using
vector
C Let P(1, 2, 3), Q (3, 4, 5) and R(1, 4, 7) be three
In AABC, 2ab cos
addition, we have a2+ b -
definition of vector
By c sOLUIION
vertices of APQR. Let O be the origin. Then,
In AABC, let the position vectors of A, B andC be
SOLUTIONN

P(1,2,3
BC BA AC CA b and -j+k, i-3 - 5k and 37 - 47 - 4
In&ABC,let BC AB
a,
or, a = -c - b
respecively. Let O be the origin. Then,
OA -21 -j + Q(3,4,5 R(1.4,7
W OB1-3} -5k
OC 3 -47 -4k
OP (1, 2,3)
0Q (3,4,5)
Now,
- have AB -08-OA-7-27 -6k OR
(1. 4,7)
a, we
sides scalarly by
Multiplying both
BC -Oc-OB-21 -7+k PO 0Q-OP (2.2 2)
a.a -a .c-a.b PR -
OR - OP= (0,2, 4)
or, a?=-ac cos (z B) ab cos (7 C)
-
-

of vector addition, AC -OC-0A=*-37 -5k Now,


By definition
-

C
B+ ab cos
or, a? ac cos
acos 5+ bcos C
AB AC CB AB 1AB-VGIP+-23 (-6 =yai 2
0 XiXX
94 2064 0No. 11 triangle, that: or, c
-b - a BC BC =y2+(-13+1 =y6 PQx PR = (8 - 4,0-8,4 - 0)
Sing vector
method, prove in any 14 or, c = (-b - a? AC JAC -v13+(-3+(-5 =y35 (4,-8,4)
b c2+a?- 2ac cos Here, BC2+ AC? =6+35 41 41)
SOLUTION
( a + b2
AB2 PO PRI-4 (-84- y96 -a6
b +b2
a +2a C 90°
Area
of APQR -IPQ* PR|-sa6)
=a2+b2+ 2ab cos (t -C) Hence, ABC is a right angled triangle.
a 2 + b2-2ab cos C Now,
26sq. units.
0. 2069 (Set A)Old Q.No. 11a Cos A AB AC -1+6+30 106 2071 Old Q.No.11a
Prove by vector method that: Prove by vector method:
cosA sinB. cos (A+ B) = cosA cosB - sinA sinB
sinA cosB
sin (A B) =

4
-
-

Please refer to2060 Q.No. 11a A cos41 Please refer to Model Set II, Q.No.10
In AABC, let BC= a, CA= b and AB =c 101. 2069 Old (Set B)Q.No.9a 07. 2075 Set BQ.No. 7a
By definition of vector addition, we have Prove, analytically that the angle in a semicin Cos B BC BA 2-2+6 State and prove the sine law by the vector
method.
CA CB BA is a right angle. BC||BA| 4
or, b -a -c SOLUTION SOLUTION
Let ACB be an
B cos\44 Statement: In any triangle ABC, the sine law
or, b- (-a - cp= (a +c
angle in the semi- states thatA. sinB_ sinC
Hence, the remaining two angle
a2+ 2a.c tc? circle. Let O be are
cos Proof: Please refer to 2061 Q.No. 1la
a 2 + C+ 2ac cos (T - B)
the origin.
b2 a2+ c2-2accOs B andcos-V 108 2075 Set BQNo. Ta OR
Let OA a.Then Prove that if 8 is the angle between the vectors
2065 QNo11E 03. 2069 Old (Set B) Q.No. 11a
By using vectors, prove that in any AABC, OB -OA -a Frove and b, then a.b ab cos.
sin A sin B sin C vectorically that in any triangle. 41 Please refer to Model Set I QNo 8b OR
41Let OC= C
a b C
Then, sin Asin Bsin 6 MARKS QUESTIONSs
Please referto 2061 QNo.11a C Please refer to 2061 Q.No. 1la
CO+ OA =-¢ +a a
-

CA
96. 2066 CQ.No.11 1042070 (Old) Q.No. 11 a 103. 2069(Sot B) Q.No.10
Prove by vector method: and CB = CO+ OB = -c - a Prove by vector Define vector product of two vectors. Interpret
cos (A - B) = cos A cos B+ sin A sin B method: the vector product ot two vectors geometrically.
141 Now, cos (A +B)= cos A cos B -sin A sin B 4
Please referto20 Q.No. 11a Prove by vector method that:
Please refer to Model Set ll, Q.No. 10
=
CA.CB -(-c). (ea -c)-g-
97 2066 O.No.11 sinAsinB sin C
Use vector method prove to that, in any triangle
0( OA =OC) b C 6
ABC, a= b cos C+ c cos B. LACB =90°
Please refer to 2063Q.No. 11a
41
Mathematc-
Asmita's NEB Solution of Basic Q.No.10
122070 Set Dproduct of two vect
122
VECTORS AND ITS APPLICATIONS Unit 6 123
SOLUTION Define scalar
vector method that:
ectors. Prove sOLUTION
119.2072 SofDQ.No. 10
Vector product of two
vectors: The vecto Vector product of two vectors: The vector
(A B)=cos. cos B+sin Asin B. Define Vector product of two Vectors. Prove by
and b
-

vectors a cos
(cross) product of two vectors and b
(cross) product of two
by
First part:
Please refer to ModelSet 11, Q.No. 10
a
Vector method that in any triangle ABC,
,

denoted by a b
is defined
Second part
denoted by a b, is defined by b 1alsin Asin Bsin
x
a
[6
axb a|b| sin®nwhere istheangle refer to2057 Q.No. 11a
Please
IbI sin 0 n where 0 is the angle between a Please refer to 2069 (Set B) QNo. 10
vector
b and n is the unit Q.No. 1 and b and nis the unit vector perpendicular 120.2072 Set EQ.No. 10
between a and
1132070Supp. Define scalar
and b. product of two vectora to the plane of a and b. product of two vectors. Give the
perpendicular to the plane of a
Define scalar
b, and AB = c
vector method that rove Second part geometrical interpretation of the scalar
product
a, CA-
let BC Let XOX' and
of two vectors. In any
mutually thata2 b2+ c- 2b costriangle
In AABC, cos A sin A sin B.
cos B YOY' be wo prove vectorically
-

addition, cos(A+B) A
By definition of vector Please refer to Model Set II, QNo 10 perpendicular straight lines representing [6)
Q.No. 11OR
=
axisand y-axis respectively. Let xOP= x-so ASLUION
142070 Supp. and Q0X' B so that ZPOQ= T-(A B).
=
Scalar product of two vectors: Scalar
+
product
Define vector product.
a Find a
unit verh
of two vectors a and b, denoted
perpendicular to the plane of ä =ï+? by a.b, is
2+k. Also compute
defined by a.b lal |bl cos =ab cos 9
5 - the sine where is the
-8 angle between them. of B A X Geometrical Interpretation
angle
between the two vectors.

AB AC CB sOLUTION Let, OA a
and OB b. Let AOB
of two vectors: The
=
.
or, c-b-a
Vector product ech v Draw BE
perpendicular to OA and AD
(cross) product of two vectors a and perpendicular to OB.
or, ab +c =0. ) Also, let OP = ri and OQ= r2.
Now,
From 1) denoted by a x b,is defined by a x b= Then the coordinates of P and Q are (ri cos A, nsin
A)
and (r cos (T- B), ra sin (T B) (-T2 cos B, rz sin B).
aaax b*a c 0 |bI sin en where 0 is the angle betwe
- =

or, 0+ ax b=-a x c ('axa-0) and b andn is the unit vector perpendinl


So, OP (ri cos A, ri sin A)
(Ti cos A, ri sin A, 0)
or, a b =c *a ...i) to the plane of a and b. 04
Again, from (i)
& 0Q=(-r2 cos B, 12 sin B) (-rz cos B, r2 sin =

B, 0)
Here, a -i+j-2k Now,
abbb.c*b-0 Ti COs A ri sin A a.b l a| |bl cos
i -2j +k -2 cos B n sin B i- cosA
cos B sin A
or, a x b +0= -c xb sinB ab cos
OP OQ= (0, 0, ri r2 cos A sin B+ ri n
x
or, a x b = b x c sin A cos B)
-
(OA) (OB) cos 6
From (i) and (iüi), OPx OQI n z(sin Acos B+ cos Asin B)
= (OA) (OB cos 0)
Since n (A+ B) is the
-

angle between OP and (OA) (OE)


axb bxc=c xa OQ so (magnitude of a) (projection of b on a)
Taking maodulus on each side, we have
a b l Ib c| = |cx a 1-|?|:|i:F Sin -(A+B)] =OP*oQL Similarly, a . b (magnitude of b)
or, ab sin ( z -C) = bc sin (7 - A) = ca sin (m - B)
or, ab sinc=bcsin A = ca sin B
-3i-3 -3k | OPI |OQ| (projection of a on b)
Next part,
ra (sin Acos B+ cos Asin B)
5in
of, aA sinb B sin C Unit vector perpendicular to the plane of a a
2 InAABC, let BC a, CA b and =
AB c =
S i n (A+ B) = sin A cos B+ cos A sin B
By definition of vector addition,
is
116.2071 Set DQ.No. 10
Define scalar
product of two vectors. Prove by
T-A

110. 2069(SotA O.No.10 vector method that


61
Define scalar product of two vectors. Prove
by
axb cos (A+ B) =
cosAcosB -

sinAsinB
the method of vectors that: Also, Please refer to Model Set II, Q.No. 10
cos
(A -B) cosAcosB sinAsinB. 16 lal-v12+12+(-2 =y6 17. 2071 Supp. Q.No. 10 C
First Part: Please refer to Model Set II QNo, Define the scalar
10
Second Part: Please refer to 2057 QNo. 11a
b--2P1 -
Ife be the angle between a and D then by vector method:
product of two vectors. Prove
BC BA AC
Cos
(A+B) CosA CosB -SinA SinB [61 or, a - c-b
11. 2070 SatGO.No. 10 Sin axb
Please refer to Model Set
11,Q.No. 10 or, a =(-c -b)*
Define vector product of two vectors. Using 18.2072 Set CQ.No. 10
vectormethod, prove that:Asin Bsin Define Vector product of two Vectors. Prove by
- (c +b)3 =b+ 2b. c +

b2 2bc
14. 2071 Set ca.No.1 +
(T A) c= b2
=

Please refer to 2069 Set BQ.No. 10 Pro


ector method sin(A + B) = sinA cosB cosA cos -
+ + c2- 2bc cos A
vectors. sinB
Define vector product of two
Please refer to 2071 Set C QNo. 10
[6
vector method that
sinA+ B) sin A cosB + cosA sinb.
124 Asmita's NEB Solution of Baslc
Mathematics- VECTORS AND ITS APPLICATIONS Unit 6 125
-B.c-.2
121.2072Supp Q.No. 19
vector product of two
vectors.Usin8
or, c 2 - b c c o s (T - A) - ac cos (T - B) OA-(1,0,-1)
Define
vector method prove that
or, c-bc (-cos A) -ac (-cosB) o6- (-1,2,0)
6 bc cosA + ac cosB
or, c2
BAx
=

sin (A B) sin A cos B cos A sin


=
B. -

X
C b c o s A + a cosB.
o-(2,0,-3
SOLUTION
Vector product of two vectors: The vector (cross 126.2074 Set B Q.No. 10
Define Scalar product of two vectors. Prove
ob-,-2,-1)
Now,
product of two vectorsa andb,
denoted bya xb, vectorically
is AB-OB-O -(-1,2,0) - (1,0, -1)
is defined by axb =| sin al |b| 6nwhere Cos(A+B) CosA. CosB - SinA. SinB. (6] -1-1,2-0,0+1)
and b andn i s
the unit Next part Please refer to Model Set I1, Q.No. 10
the angle between a to 2057 Q.No. 11a (-2,2,1)
the plane of a and b.
Please refer 127.2074 SuppQ.No. 10 And
vector perpendicular to 10 Define scalar product of two vectors.
Second part 123,2073 Set DQ.No. vectors. Give vector method that cos(A - B) = cosA.cosB +
Prove by
c- Ob-o-3,-2,-1)-(20,3)
be two mutualy Define scalar product ot two
Let XOX and YOY
geometrical interpretation
of the scalar prad.
codut
sinA.sinB. 3-2,-2-0,-1+3)
perpendicular straight
lines representing
Prove vectorically that, First Part: Please refer to
Model Set II QNo. (1,-2,2) 6
axis and y-ais respectively.
Let ZXOQ =
5
of two vectors. 0
that 2QOP A B. Again, -
b=c2+a2-2ca cosB
Second Part: Please refer to 2057QNo. 11a
AB.CO (-2,2,1). (1, -2,2)
=

and XOP A so =
Part:
and OQ = n Then the
coordinates First ahd Second -2-4+2-4
letOP
=

of P and Q are
n
Please refer to
Model Set II Q.No. 10
28. 2075 Set AQ.No.10 AB AB=V-2 21=ys=3
Last Part: Define vector product of two vectors.
P 11a
Please refer to 2064 Q.No. Interpret the vector product of two vectors
geometrically. Prove, in any triangle, by vector
vector
cD
|CbI=y (-2 2=9 =3
124.2073 SuppQ.No. 10
Define vector product of two vectors, prove b thatnA sin B sin
Dy
Projection of AB on C -ABCD.
method
b [61
vector method
Please refer to 2069 Set B Q.No. 10 C
eAX sin (A-B) sin Acos B
Please referto2072 Supp Q.No.10
cos A sin B.
h28.2075Set CO.No. 10
Again,
Define a dot product. Interpret i t Projection of Cb on AB A . .
125.2074 Set AQ.No. 10 geometrically. Let A1, 0, -1), B(-1, 2, 0),
AB
Using vectors prove that C(2,0,-3) and D(3, -2, -1) are four points. Show
b=c2+a2- 2ca cos B that the projection of AB CD is Projection of AB on CD = Projection of CD on
i on
equal to

(h cos A, sin r A) and (: cos B, Ta sin 5)


. e=acos B + b cos A for any triangle ABC projection of CD on AB. Also, show that their
respectively. So,
AB.
SOLUTION inclination is cos-
OP -(ri cos A, n sin A) = (n cos A, n sin A, 0) [6] Ife be the angle between AB and Cô then.
InAABC, let BC= a, CA
=
b and AB
SOLUTION
Now,
oQ= (ra cos B, 12 sin B)
=

(t cos B, T2 sin B, 0)
r-AX First and second part: cose AB C
2 cos B 2 sinß
cos A
S i n B

X sin A
Please refer to 2072 Set E QNo. 10
A6
n cOs A sin A Next part:
ox OP-(0, 0, ri a sin
0QOPI=nn (sin A cos B- cos A sin B)
A cos B- ncos A sin B)
Given points are A(1, 0,
-1), B(-1,2,0)
C(2 0, 3) and D (3, -2, -1). Let O be the ()
we have origin. Then,
Since (A- B) is the angle between 0Q and OP, so
By definition of vector addition,

sin (A-B) = 0 0 x O P CA CB+BA


or, b = -a - c
0QI|OP|
I 2(sinA cos B-cos AsinB)
T1 2
or, --a -c=(a c +

a2+2a.c +c
sin Acos B-cos Asin B a l t c2+ 2ac cos (n - B)

122.2073 Set CQ.No.10


Define scalar product of two vectors, Prove
b2 a2 + C? - 2ac cOs B

b and
vectorically that cos(A-B) =
cos A cos B+ sin A |i. In AABC, let BC= a,cA
sin B. 61 By the definition of vector addition,
SOLUTION
Scalar product of two vectors: Scalar (dot) AB-AC BC
AB=-CA BC
product of two vectors and bdenoted by a or,
a
-

.bis defined by a . b | al lbl cos 8=ab


befween
or, - -
the two
, w eh a v e

where 0 is the angle


cos
vectors.
Multiplying each term scalarly by
DERIVATIVE AND ITS APPLICATION Unit 7 127

UNIT lim eth-E,


eh0 h
th+s
Vr+h+ dlogtanx)h-lim log(y+b-logy h
lim +h-z_]
h
D TS
DERIVATIVE AND lim
0
lh (Wa+h*)
1 h0
lim

h 0 +h+y
APPLICATIO 2 k-0
lim

5. 2062 Q.No.10D
lim fr+h-fN Find from firstprinciples, the derivative of lim
we have, f) h >0 h k
A. DERIVATIVES
lim esin(r * h) - esinr
tan-x.
soION
[4 =17'h>oh
DERIVATIVE BY FIRST PRINCIPLE OR h0 h Let f(x)= tan-lx
1 lim an (r +h)-tanr
DEFINITION r + h) = tan-1 (r + h)
yh>0 h

Put sinr =y and sin (x+h)= y+ k. sin (r +h) sinx


4 MARKS QUESTIONS sin (r h)
Then, k
= + -
sin x lim f(xt h)- f)
We have, F) h>0 1 lim
cos(r+h) cos
1. 2058 Q.No. 10
Ash 0, k>0
lim tan1 (z+ h)-tanlx yh>0L h
Then, from ()
Find from first principles, the derivatives of ean 4 drtan ) h->0 1 lim sin (r +h) cosx -cos (x+ h) sinx
sOLUTION e)h>0
lim ey*k-ey
h Put
Then,
tan-x= y & tan-1 (x+ h) = y +k
&
h0 hcos (r + h) cosr

Let! etan
x= tany (r + h) =
tan (y+k) 1 lim sin (r +h-1
h = tan (y + k) - tany
f(rH = elan(a*h lim e ( yh0hcos (r + h) cosr
Also, as h 0 , k ->0
lim elanr* - etaru
k0 lim sini
Ne have, fa) h>0 h ..) Now, from (i)
lim sin(+h-sinx lim y+k-y yh0
h cos( h) cos
= e.1 h>0
Put tanr =y & tan (r+ h}=y+k
so that k= tan (x + h}- tanr
h
(tan1)h>0h
k lim cosx.cosx
1
tan
1
cosr
1
sinrcos
Ash>0,k>0
Now, from (i)
lim h0hn 2
h>0 lim 2 sinr cosr2cosec2r
lim eyk-e k0 tan (y +
k)
dre)h-0h h tan y
-

lim Sin lim


2064 Q.No. 10
lim e
h0
e
h 0 2cos ( k0sin (y +) siny
Find from first
log sin x.
principles, the derivative of
cos (y+ k) cosy)
lim e-1 lim k
lim sOLUTIONN
k+0 k h>0 h e.cosx. I = esn".cosx Let fr) = log sin x

lim tan (r+ h)-tany


k 0 sin (y +) cosy-cos (y+ k) siny f(x+ h) = log sin (r + h)
=
e. 1.
h0 2061 O.No.10 cOs (Y*k) cosy

Sin (+ h) sinr Find from first principles, the derivative of e lim k cos (y +k) cosy We have, f) h lim
>0
flr+h)- {)
lim cos (+ h) cosx k0sin (y+ k-y)S h
lim log sin (r +h)-log sinx
h0 h SOLUTIO lim
cos (y +k) cosy dro8 Sin x) h-0 n

k0. Put sin r =y and sin (x+ h)


y +k,
=

lim sin (r+h) cos- cos (+ h). sin x Let f so that k = sin (r + h) - sinr
eh0 h cos (r+ h) cosx
f(x+ h) =e*h As h >0, k >0
lim sin (x +h-z cosy secty= 1+tany 1+x Then from (i)
cos ( h) cos We have, f() =h->0
lim f(x+ h)-f{x)
h 2063 Q.No. 10 b lim log(y+ k)- logy
d log (sin )h»0
Find from first principles, the derivative of h
= e, cOSx. CO5
= elan sec2x
lim _ F
log tanx.
2 2059 Q.No.10 SOLUTION lim
Find, from definition, the derivative of etan. 41
Put y and r+h =y+k Letffr) log (tanr)
= h0
Please refer to 2058 Q.No. 10b sothat k=+h- ( x + h ) = log tan (r + h)
Ash 0 , k->0
3Find 2060QNo.10b the lim e We have, f()=h-+0+ h h) -f{o) lim lim
fromhrst principles, derivative of esla, 14 Then from ()()h-0h
SOLUTION lim log tan (x +h)-log tanr
0og tana)h-»0 ..)
Let f{x)= esinu lim 1 lim k
Put tanr=y and tan (r + h) =y +k
fr+h) esinr +h) k0 so that k =
tan (x + h) -
tanr
yh>0 h
As h >0, k->0 1 lim sin (r+ h) -sin x
. 1 im vh-s£
e.I h+0 Now, from (i) yh0 h
Basic Mathematic> Unit 7 129
Solution of DERIVATIVE AND ITS APPLICATIONN
NEB
128 Asmita's
lim
k 0
20ms
13. 2069 (Sot A) Old Q.No. 10E lim In y+ k-iny
h
lim lim Find from first principle, the derlvative of em
h0 cosy. h >0h 14

lim COsy h+0


lim log (I* h-log sOLUTION
Let f(x) ero9z

ecos (r*h)
lim
h»0
) n

yh0 r + h)
log We have, f he hlim
»0 ffx+ b)-{a)2
lim lim
.1=sincosr = cotx d lim ecos (r +h -erosz
cos cOsY h+0 d ( t osr) ... () k-0
dr h0
8 2065 a.No 10 the
derivative of a* |4| Put cosx y and cos (x + h) = y + k
from first principles, so that k = c o s ( x + h ) -c o S x lim
Find lim
SOLUTION
= etloga
= COsy- h0|
As h >0, k ->0
Then, from ()
1hoh
a'=elhga"
Let f(r)
f(a+ h) = e )lge lim e lim Sin
dr h>0 h
lim fa+h-fa
h osy.1. cos (log ) yh0 n

We have, f(7) h^0


=

lim eahiloge- e . . .
11. 2067 Q.No. 10
h »0 h first principles, the derivat 2 cos
+ k, soFind from lim cos (x+h)-cosx lim
Put r log a =y and (a +
h) log a =y e.1.h+0 h yh>0
log a sinr'x.
that k= (x+ h) log a x
-

SOLUTION
As h 0 , k > 0
Now, from (1)
lim e*k-e
Let f(r) sin-'x
=

f(r+ h) sin- (r h)
= + e
h>0 ( )
lah>0 lim fla+h- f)
h 0
(sin
We have, f() h->0

) h->o
h
lim sin-(x+h)-sinl
n
h0
lim
) ,I
,lim (+h) loga -loga Put y = sin'x and y +k= sin-1 (x+h)
=e.lh0 so siny and x+h= sin (y k)
that x =sin
+
= ey.sinx.(-1)= -ecos. sinr sin
lim (h-loga Then, h (y + k) - siny. As h >0,k>0
14. 2069 (Set B) Q.No. 11 O
en Then from ) Find from first
16. 2070 Supp. Q.No.6
e . loga = er log. log a a" loga principle, the derivative of
esin Find, from first principles, the derivative of
lim y+k-y 4 sinx.
B2066 CcO.No.10
Find from first principle, the derivative of
d(si)h->0 h FR Please refer to 2060 Q.No. 10b
SOLUTIONN
4
log tan x 4 16 2069 old (Sot B) Q.No. 10b Let f(x) sinr
=

Please refer to 2063 Q.No. 10b k0 sin(y+k)-siny j Find, from definition the derivative of f(r + h) sin (x=
+
h}*
log (sin).
0. 2066 ONo.10 k0
lim
4 We have, P(x) h
=umf(r+h)-ft
Find the derivative of Sin (log ) from first
SOLUTION - sin1?
principles5. 4 2cos d(sin) =,mun ( +h#
SOLUTION lim Letf) log (sinr)h0 h

Let f(x) sin (logr)


f(r+ h) sin log (x+ h)
k0 lim
f(r+ h) log sin h->0
We have, f)=hlim
>0 f(x+b- h f{r) lim We have, f(x) lim f(xth)- f{x)
>0
d lim sin log (x +h)-sin logx k>0 K
drSin logh>0 h sin5
lim
Put log r =y and log (x * h) = y +k
so that k =
log (r *
h)- log x.
log sin
-) . (9
h>0

Ash>0, k>0. Put


lim
Then, from (i) sin y & sin
(
drsin log r)
=
t k -sin y cosy -siniyi-
12. 2068 Q.No. 106 so that k sin
2cosr.1.x=2 cos?
Find from first
im
principle, the
deriva Also, as h >0, k> 0
h lease refer to
Now, from (i)
2060Q.No. 1Ob
Mathematics- XII Unit 7 131
Basic
130 Asmita's NEB Solution of DERIVATIVE AND ITS APPLICATION
23. 2071 Sot DQ.No. 11OR
17. 2071 old Q.No.9b from first principle,
Find
the
derivative of
derivativa 26. 2072 Set CQ.No. 11 OR
first principles.
Find the derivative of etanr from 41
Find, fromfirst principles, the derivative of xinx.
16] yh>0
Please refer to 2058 Q.No. 10b
Please refer to 2069 (Old) Set BQ.No. 10b SOLUTION
1 lim cos x - cos(x+h)
Let (r)= xnx y h >0 h cos (x + h) cos x
6 MARK QUESTIONS Supp.Q.No.11 lim fz+ h-{()
24 2071
from first principles, the derivative of We have f) h>0 h
Find,
18 2069 (Sot A)Q.No. 11 o of 1lim
lim
n r ) h->0 +h) In(z
derivative
Find from first principle, the
161 n (cos v) +h)- xinx yh0 h cos (x + h) cos x
sin(log ) h
SOLUTION lim hin (x +h)+zln(x+ h- zBnx
Please refer to206k6 QNo. 10b Let f(r) In (cos h
1 lim
19 2070(Old) Q.No.10 first fr+ h)- In cos + h m aln(N-ha, hlnu
of log (tan a) by yh>0 | cos (x+ h) cos x
Find the derivative
principle.
[6]
We have, f() h+0
lim fx+h-ft)
h
h0 h a
Please refer to
2063 Q.No. 10b limo ( sin sin x
GO.No.11 o CosVr*h-n cosy In (+ h y cosx cos x .1-sec x
cos
cos
cos x
sec x x
tan x
20 2070 Satfirst the derivative of d n cosya)= h>

Find from
principles, 30. 2073 Set DONo.11 OR
log (tanr). 16 Put cos =y&cos x+ h=y+k Find from first principles, the derivative of
Then, k= cos y x + h - cosyx sin (log).
Please referto2063 Q.No.10b lim
In (r+ h) =1+ Inx
[61
As h 0 , k->0 h0 h
Please referto 2066 QNo. 10b
21. 2070 Set DQ.No.11 O Then, from (i)
Find from first principles, the derivative of x*.
31. 2074 Set AC.No. 11
[6] lim n (y+ -hy 26 2072 Set DQ.No. 11 OR
SOLUTION
n coy -h--0 h
Find from first principles the derivative of
Find, from first principles, the derivative of

Let f{r) = x = ebg = eriog


In cos-lx.
16 Infcos 61
t(r +
h) = elr*hlgts
*
h) lim Please see Model SetI1, Q.No.11 or SOLUTION
lim fr+ h)-f{) h>0 27 2072 Set EQ.No.11 OR Let fx)- Inçcos x/a)
We have, f() h>0 h Find from first principles, the derivative of sinx

d h0
lim er+h)log ( +h) -erlog
h
(i) k0
lim Please refer to 2070 Supp. Q.No. 6b
28 2072 Supp Q.No.11 OR
fx+h)= In cos (
By definition of derivative, we have,
Put y dogx and (r h) log (r+ h) + =
y +kso Find from first principles, the derivative of lim fx+ h- {)
log (tanx). 61
that k= ( + h) log (r+ h) - x logr

As h 0,k>0
=1.lim k
1
yh 0 h Please referto 2063 Q.No. 10b P)h0 h

29 2073 Set GQ.No.1


Now from (i)
1 lim cos y+h-cos
yh-
Find the
loglsec x
first principles the derivative of
I n cosi ha o .)
e)h0h 2sin
sOLUTION
Let f(x) = log sec x
6] h-0

lim Then f(x + h) log sec (x *


=
h) Puty cos&y+k=cos
yh>0 We have,
lim (+h) log(x+h)-zlog
= e.1 h+0 lim
fx+ h)-f{x) thatk co o
h
2h
F)h0 h Also, whenh-0, k>0.
lim zlog (r+ h)+hlog(z +h)-xloga\ or,
alog sec h0
x)
lim log sec(x+ h)-1ogseCX
h00 h
Then, from (i)
Puty sec x andy + k=sec(x+ h) m n y+ k)-Iny
d (In cos) =h-0
such that as h >0, k>0

eh0
lim
h logh ( h) 0 n
Then from (),

d log sec x)h--o


h-0 h h0 h
lim (Ni-h
h00 log(+ h) lim k/y
h-0 k/y h-0 h
e(1+ log)= r(1+ logr)
22 2071 Set CQ.No.11 OR lim lim
Find from first principle, the derivative of tanix.
16
k0 h>0 y.h
Please refer to 2062 Q.No. 10b
Mathematics- XII Unit 7 133
Asmita's NEB Solution of Basic DERIVATIVE AND IrPPLR
132 2. cos 2+x2
sin Q.No,2
lim 38 2058 derivative of roshr, 41 2061 Q.No. 26
yh-0 Findthe
(2 Find the derivative of: xn ha (2
lim -2sin
lim 2 cosh24x?
*n , sin x2+2xh+ h2. SOLUTION
SOLUTION
1 2 2
Let y
=os
hr

yh +0 yh-0 Taking log on both sides, we get,


Let y = x*osh x/a

Taking log on both sides, we get,


sin(2x+ h)h log y cos log
=hr x

=z lim Differentiating both sides with respect to'


log y- coshlogr
yh>0
1
yh-02
lim 2, cos dog y) (coshrlog) Differentiating both sides with respect to 'x
h
sin2a at. dy y) cos h dr log )* log (cos hur)
2 c5x1.2sin 2xcosx 2x ordcos hxlog x.sin hx
or dlog
x COtx? cos hx+
log x sin hx Y-coshlogr)+ log dr

36. 2075 Set CQ.No.11 OR


Find from first principles, the derivative at
xeos rcos hx+ log xsin hx
o coshlog sinh
=

a.cos x/a coSx


sOLUTION 39. 2059 .No.2a
32 2074 Set BQ.No.11
the derivative of log (tanx.)
Let Find the derivative of ero.
12
Find from definition
f(x) = cosx2 SOLUTION
f(x+ h) = cos (x + h)2
Let y = ecoshr
ay
Please refer to2063 Q.No. 10b By definition of
derivative
Differentiating both sides with respect to '
dx

33. 2074 Supp Q.No.11 OR lim fix +h)-f{x)


Find from first principles, the derivative
of sinx?.
f%)h0 h
42 2062 0No. 2
[6] Find the derivative of
Please referto2070 Supp. Q.N.6b
d
dx cos x) d(ecosh) d{cos h'). oros h- logsin [21
d(cos h-1 x) dx
soLUTION
34. 2076 Set AQ.No.11 OR
Find from first principles, the derivative of
lim cos (x+h'-coS X
h-0 h eco% h
logsinx. 16]
im -2sin2sin
-1 Laty-logsinh)
Please referto2064 Q.No.10b lim 40 2060 Q.No. 2 b Differentiating both sides with respect to '?
35 2076Set BQ.No.1O h0
Find, from first principles,
(sin x)
the derivative of In
16) lim
2 sin + + 2:xh + h2-x Find the derivative

SOLUTON
of 2ta 21 ()
sOLUTION
Letf(x)= In (sinvx)
ffx+ h) = ln (sin (x + h)})
h->0

lim
(2x + h}h
2x Lety 2tanh n)n))
By definition, we have h0 2 sin, Differentiating both sides with respect to'r
lim f(x+ h)-fx)
f(0h0 -2x sin X
Cd2 tan h"(tan sinh
.cosh coth
nd(In sina=lim lnsin(xth2-In sinx2
sinx) "h>0 -2sin
h
d tan h 43. 2063 Q.No. 2
Puty s i n x and y + k = i n (x + h?
So that, k = sin (x + h)? - sinx L DERIVATIVEUSINGFORMULA Find the derivative ofx*os * ty
1 .

2 MARKS QUESTIONS SOLUTION


Ash 0 , k0. Then (Gi) becomes
ddxn sinxa3
dtan Let y = u h (
37. 2067 Q.No.2 2 Taking log on both sides, we get,
Sec?.i
lim Iny+ k)-
h-0 y Iny Show tha Og+)_ 1 tan

SOLUTIONN Sec?
log y
cou(). logr
+
lim l o g +
seca. cos 1 Differentiating both sides with respect to '
im L.H.S.
cosr sec *
h-0 0 X sin cos3-sin
-
lim d k92| lim Y cos
limsin(x +h2-sinx2 lim
h+0 y.h
Mathematics-
Xl
Basic

6 2066ONo.28
M-oluuo
134 ASmmita's
Unit 7 135
DERIVATIVE AND ITS APPLICATION
derivative of2tan h(tan log y) dr (sinh-ix log coshz)
of dy Find the
or,-y (tanr+ secx. log secz)
referto 2060 QNo.
2b
Please on dy ogy) dr sinh dlog cosh: d(secx)ome (tan?x+ secx. log sec x)
7 2066 GQ.No.2
dr
log logcosh d (sinh-'*)
Findthe derivative of: log (sin
56 2076 Sot BQ.No. 3a
or,d =sinh-ix. d(coshx)
COshx) dcosh
dr Find the derivative ofxmhs, (21
2b SOLUTION
Please refer to 2062 Q.No. log cosh T Let y = xstnh

48. 2067 Q.No. 26 Taking log on both sides, we get,


or, log y = sinh x log x
h dy
of cos Sunh"¥
cosh sinhr+ logaosh
ay
Find the

Please
derivative

refer to2061 Q.No. 2b


odr 1 Differentiating both sides w.r. tox,
dr
92059 Q.No. 28 =(cosha)in{ sinhr'x. tanhr+ og cosh z 0og y)-inh ogs) x

206 NO. d
he derivative or (Cos h) 52. 2070(Old a.No.2 C OT
log -sinh og iog (sinh
Fingtne derivative of
(si F i n d w h e n y = sec h{tan-r).
dx (21 o, dsinh xlogx.cosh x
sOLUTION
SOLUTION logr SOLUTION
Lety-(cosh Here, y= sech (tan r)
Letyn
log
Taking on both sides, we get,
Taking log on both sides, we get
Differentiating both sides with respect to '
dy sech (tan' )}
dy
logx cosh x
logy logr. los(cosh d sech (tan' a) d(tan
logyogsinh Differentiating both sides with respect to ' d (tan-11) dx 4 MARKS QUESTIONS
with respect to '*
Differentiating both sides 57 2057 QNo. 10E
-sech (tan'x). tanh
ogy)-og oscoh (tan'7)- +
-sech (tan-1). tanh (tan-lx) nd the dertvative o atn bch
d 1+
rlo ogsinhr 53 2070 Supp. Q.No. 3a SOLUTION
Find the derivative of (lnx)sin r
loco) tog) Please see Model Set I, Q.No. 3a Lety-(inhc
dr
54 2071 Old o.No. 2 Taking log on both sides, we get,
Find the derivative of log sinh x.
Or y
dy
d= logz SOLUTION logyns losinhco h
iogsinh.2 Let y =log sinhx Differentiating both sides with respect to 'r
Differentiating both sides with respect to 'r*
1d? sinh .cosh+21.logsinh
y dr . dy
dr dr (log sinhr) Ogsinh1) d(sinha)
=

d (sinhr) dr
or, Coshx =
cot h
sinhr
cosh
5. 2071 Supp. Q.No, 3a
Find the derivative of
(secx)an,
SOLUTION
5 2065No 2 50 2069 (Set A) Old Q.No.2b Let y (sec x)ans oy drn
Find he derivative of Arc tan sin h. (2 Taking 'log' on both sides, we get
LUTION Findthegerivativeofsinh log y tanx. log seçx
lerentiating both sides with respect to 'r
Lety Arc tan sin hx Pase refer to 2064 Q.No. 2b
=
tan-1 sin hx
Differentiating both sides with respect to 'r 51 2069 01dSotB)Q.No. 2 d(logy)-dh (tanx. log sec dx

dy(tan sin ha) Find the derivative of (Coshr) n

sOLUTION
dy logy) tan d(og secr)+ log seca dy(tan
d (tan' sin h) d(sinh y ddy tanog secx) d(secn) secx. secix
d (secx) d r
d (sinhx) dx Lety (cos hx)pintr or,dy
Cosh x Taking log on both sides, we get, y dr tanr. secx. tana + secx. log seCr
1+sinhr COSn coshar sechx logy sin h'x log cos hx
Differentiating both sides with respec to'
MathematicsX
22
DERIVATIVEAND ITS APPLICATION Unit 7 137
Asmita's NEB Solution of Basic When x= 2 4y 2 4 - 8
x
T2064 Q.No,2
136 l d the points on the curve y = -3x41
x=t4
y-4
4y (-2-8 (-2P Required points are (4, 0) and (-4, 0)
: )**(** When r =-2

Required points are (00), (2,4) arnd 2,(3 the tangents


are parallel to x-axis,
121 10 2066 Q.No. 2c
sOLUTION
ind the angle of intersection of the curves
APPLICATION OF
DERIVATIVES 2081 C.No. 2 Here, y=-312+1
y
B.DIFFERENTIALS, TARGET& NORMAL Find the slope
and inclination with the . andy 2x at the point (0, 0)
=

21
the tangent ofy
-
3x -Matx= -1. 3x2-6x SOLUTION
Given curves are
2 MARKS QUESTIONS sOLUTION For tangent parallel to x-axis,d 0 . y2 2x
Here, y-3x
2057 Q.No.5 31-6x =0
1 and
inclination
with r-axis of the
dy3-4x 0 =

Find the slope 2y=2-*atx=1.


21 dr 3r (r-2) Now,
tangent of thecurve 3-4(-1) =1 r-0,2 differentiating both sides of (i) with
SOLUTION Atr -1, When x = 0, y = 03 - 3 . 0 2 + 1 1 respect to '*

Give curveis 2y
= 2-r2
to 'r
Slope (m) =1 When x-2,y =2-3.2+1 =-3 dy 2
sides with respect be the inclination
of the tangent
Differentiating both If The required points are (0,1) and (2,-3).
-axis, then y 0
At (0,0 d
2=0-2 3 2065 Q.No2
tan=1 tan Find where the tangent is parallel to the x-axis 1.e. mi = 0

dr
-
forthe curve y = x - 3 - 9x+ 13 12 Again, differentiating both sides of (i) with
SOLUTION respectto'?
Atr-1,d=
dr -1 Given, y =r-3x2-9x + 15
i e . Slope (m) - -1 2062 .No.2
Find the angle of intersection of the
dx

If 6 be the inclination of the tangent


with
-32-6x
dr -9 At (0,0 0
-1
3T
tan T
4y + 12 and y2 = 8x at (2, 4) For the tangent to be parallel to the x-axis,
X-axis, then tan 8=
=
we ie. ma = 0
SOLUTION y. Ife be the angle of intersection
Given curves are

4yx+12
haved0 then,
ie. 3x2- 6x - 9 = 0
tan6 n-m2_2-0
1+ mim2 1*2*0 2
andy2 8x ) or, - 2 x - 3 0
2 2058 Q.No. 5 the From() . 0= tan-" (2)
inclination with x-axis of or, (r-3) (r+ 1) =0
Find the slope and
tangent of:r+y?= 36 at (0, 6).
12
-2 -1,3 11. 2067 Q.No. 2
When x=-1,y =(-1)"-3 x (-1)2-9x (-1) + 15 Find the angle of intersection
between the
SOLUTION.
- 1 - 3 +9+15 = 20 curvesy =? and 6y =7- x*at (1, 1).
Given, r2 +y2= 36 sides with When x =3, y =33 -3 x32-9x 3+15 -12
Differentiating both respect to 'r sOLUTION
Required points are (-1, 20) and (3, 12)
2+2 d-0 At24 2066. CO.No.2
Given curves are
y=x? (
e. mm=1
Again from (i) Find the points on the curve 2+ y2 = 16 at which and6y 7-x (i)
the
tangents are parallel to y-axis. Differentiating (i) with respect to'r
At(0,6 d 0
2d8 SOLUTION dy 2x
Given, +y2=16 (i)
Slope (m) =0
If e be the inclination of the tangent with x- Differentiating both sides with respect to 'r At(,1d2x1-2
axis, then 6=0=tan 0°. At 24) 1 (+ y-d (16) 1.e. mi =2
then Again, differentiting (i) both sides with
G=0 If 6 be the angle of intersection,
3Find2060 O.No.5c
the points on the curve 4y = - 8 x where tan
_ mi-m2
1+ mm21+1*1
0 tan 0 220 respect to '*
ay
dr-ar
the tangents are paralleled to the z-axis. 2 0 0°
dr
SOLUTION 2063 Q.No. 2c heated soth 2
Given, 4y = x - 8 2 plate is
A circularcopperfrom 5cm to 5.06cm At(L
radius increases or the tangent parallel to y-axis, we have
410-16x approximate increase in area. dy 0 ie. m
=-4x
dx SOLUTION
Let'r be the radius and 'A' be ne
area
i.e. 00 Ifo be the angle of intersection, then

For the tangent to be parallel to the x-axis, dy


dx =0 circular plate. Given, y0
0.006
=dr 5.06 5 tan 1+ m e 0 = tan 90°
i.e. r - 4 x = 0 5 cm, Ar =

Thenfrom ()
or, x(r?- 4)
=
0 We have,
8, x+0
=(t 4)16
x =0,2,-2 A ar 6 = 90°
When r =0 4y 0-8 x0 5 x 0.06 0.61 Cu
dA 2nr dr 2n x
=

=
0.6n c
area
y =0 APproximate increasein n
area
Asmita's NEB Solution of Basic
Mathematics- X Init 7 141
138
16. 2071 Old Q.No.Gthe
DERIVATIVE ANDITS APPLICATION Unit 7 139
12 2068 QNo.S Find the points on curve y . at (2, 4)6x2- 10x 2-4
- z-067form
lim
the the x-axis,J
x cut parallel to
the y(1+)= thetangents
are
At what angle does
curve
where
[21
Please referto 2064 Q.No. 2c
=
of normal
x-axis? Slope lim cos
SOLUTION
Given, y 1 + x ) = x
17 2071SotGO.No.3 dx
0 . So, Find the points on the curve y - 324. The equation of normal at (xi, yi) = (2, 4) is
The curve meets the r-axis
where

get
y
he tangent is parallel to the x-axis.
+1 wh y-y mx
-

xi)
putting y=0 in (i),
we

From (1) Please referto 2064 QNo.2c or,y - 4 - - 2 ) 25 2070 Set CQ.No. 3E
18. 2072 SetEONO,3a 4y- 16=
x + 2
Using L Hospital's rule, lim tanx-2
1+1
-

Find the points on the circle x + y? = 16a. or,

the tangents are parallel to X-axis. whl X+4y = 18


evaluatex0x- sinx
21. 2074 Supp Q.No.3a 2
SOLUTION SOLUTION
(1 Here, r + y2= 16 . (i) Find the equation of the normal to the curve
lim tar-2 [
y 2x-5x + 8 at the point (2, 4)
I 0 1 - sin x
2r 2y=0 Please refer to 2074 Set BQN.3a
At-0 F1 22. 2076 Set CQ.No. 3a lim sec?I-1 0
with r-axis
Find the equation of the tangent to y = x-2x+4 x01-cosz
fe be the angle made by tangent at (2,4). 121
then,tane 1 tan
For tangent parallel to x-axis, dr =0 SOLUTION lim 2secx. secI tan: 9
Sin i
Given,
ie - o y=x*-2x2+4 lim 2 sec? I tan x2
dy.
dx3x24x
Sin
13. 2070 Set DQ.No.33a
I=0 lim 2sec?.sec?1 +tanz.2secz. secI. tanr)
the Then from () dy (2,4)3x 23-4 x 2=4 CosX
equation of the tangent to
curve.
Find the dat lim
0+y2 16 2sectx+ 2sec I tan? z)_2102
4 The equation of
y23-5+8at (2, 4) y tangent at (xi, yi) =(2 4) is
COs X

SOLUTIO Required pointsare(0, 4) and (0,-4) y-y1 m(x -x1) 26. 2071 Set DO.No. 3
Grven curve is y = 2 r - 5x2+ 8 or, y-4=4 (x - 2)
'? 19 2074 Set AQ.No.3a Using L Hospital's rule, evaluate:
Differentiating both sides with respect to
or, y - 4 4x - 8
Find the points on the curve x + y2 = 36 at whi
or, 4x-y -8+4 0
lim er+-2 cosxX
x-0
612-10x
dx
the tangents are parallel to the y-axis. 4 x - y - 4 - 0.
sin (2
iy
SOLUTIONN
Given, curve is x + y2 = 36
SOLUTION
At(2 4) = 6 x 2 - 1 0 x2=4 L.HOSPITAL'S RULE lim
e+e-2 cosx |e
Differentiating both sides w.r.to x
ie. siope (m) =4 2MARKS QUESTIONS 1-0 sin?x fiorm
The equation of tangent
slope 4 is
at (2, 4) and having
y- 23 2069 (Set A) Q.No. 3a lim
I-0
e+e+ 2sinx 0form
2sinrcosz torm
y-yi m(x - Xi)
or, 2x2-0 Using L Hospital's rule, evaluate: im e-e+ 2sinr
Or, y-4
or, y -4
4(r-2)
4 x -8
0 x0 sin 2x
or, 4x- y -4 = 0 SOLUTION lim e -e* Zcos
have, I-0 2 cos 2x
tangent parallel to y- axis,
we
4-y4 For the
0 ee+2 cos 01+
14 2070(Old) Q.No.6
Find the points on circle z2 + y2 =16 at which
2 cos 0

tangents are parallel to y- ais. [21 02 orm 27 2072 Set CQ.No.3


Please refer to 2066 C QNo. 2 or0y-0 lim Evaluate, using L' Hospital's rule
Lt tan aax
Putting the value of y in (), we get, x-0 tan bx
16 2070Supp.CNo.3
Find the slope of the tangent to the curve. = 36 [21
24 2069 (Set B) sOLUTION
y=+2+3x -10 at (-3,2). 21 or, x = t 6. Q.No. 3a lim tan ar
SOLUTION Hence, the required points are (6, 0) & using L lim 3-sinr
Here, y = x + 2 r 2 + 3x - 10
Hospital's rule, evaluate: x-0 0 anbrform
dy
20 2074SetBO.No. 3 cur
Find the equation of the normal of the cu SOLUTION 2 lim a secd ar _a.1
dr3x2+4x+3
dx
y 2x 5x +8 at (2,4). 0 bsecabxb.1 6
dy 28 2072SatDONo 3
d r at (-3,2) =3 x (-3)?+4x (-3)+3
sOLUTIOL I0
27- 12+3= 18 Given, Using L' Hospital's rule, evaluate:
y = 2 x -5x2+ 8

Slope oftangentat (-3, 2)- at (-3,2)-18.


ddy 6x-10x
-0 form Please reer to 2069 (Set A) QNo. 3a
121
Mathematics- X DERIVATIVE AND ITS APPLICAT Unit 7 141
NEB Solution of Basic

33 2074 Sot AQ.No,4


140 Asmita's =
0
(c -3)
29. 2072 Supp O.No.3a
Using L Hospital's
rule, evaluate:
x0 -sind
(c-1)
C1,3

butc 3 g
C 2
Here, c 1 e (0,3)
=

Using LHospital's rule, evaluate 3)


theorem is verified. Here, c-e (-1,4) butc -2 e
lim xe- log(1+x) Hence Rolle's (-1, 4)
x2 sOLUTION MARKS QUESTIONS Hence, mean valuetheoremis verified.
SOLUTION
lim xe-log(1+x)
x>0 X
( 6form x-0 fom x2
2 2069 (Set A)Q.No. 1 State mean value
3 2069 (Sot B) Q.No. 1
theorem. Interpret it
mean value theorem. Interpret it geometrically. Verify mean value theorem for
State
lim 2x-2 sinx cosx metrically. Verify mean value theorem for| the function f(z) = (* - 1) x - 2) (x- 3) in [1, 4. [6]
lim * e +

form
2x torm geom ** *x*- 6x in [-1,4.
the function f{x)
=

16] SOLUTION
x0 lim 2x-Sin 2x sOLUTION First part:
x-0 2x Statement of mean value theorem Please refer to 2069 (Set A) QNo.11
is
Ifa function f(x) Next part:
lim a+ee 1 lim 2-2cos2x
2
(a) continuous in the closed interval [a, b] Here, fr) =(r-1) (r -2) (r -3) in [1, 4
x0 b) differentiable in the open interval (a, b) r - 6x2+11x -6
0+1411. 2-2 cos0 - 2 1 . 2 . - then there exists at least one c e (a, b) Since f{r) is a polynomial function, so it is
such that f(c) = =fa) continuous in [1,4]
b- a Again, f()= 3x2 -12r+11 which exists for all
30 2073 Set CQ.No. 3a 34. 2075 Set AQ.No.3a
limx-Sn x cos X Second pat xe (1,4), so itis differentiable in (1, 4).
rule, evaBuate:0 evaluate:
Using L Hospita>'s
[21
Using LHospital's rule, Geometrically, Lagrange's mean value Hence, both the conditions of mean value
theorem says that in a continuous curve, in theorem are satisfied. So there exists at least
Please refer to 2070 Set C Q.No. 3a
which tangent can be drawn at every point, one c e (1, 4) such that
SOLUTIONN
there is at least one point where the tangent is f(c)- fla)
lim x-Sin x cos X
x0
IL. ROLLE'S THEOREM AND MEAN VALUE parallel to the secant joining the end points as
shown in the figure. But
fc)b-a
THEOREM f{b) =f(4) =4 -6.42+11 4 -6=6
2x ffa) - f(1) =13-6 12+ 11 1-6=0
lim 2 Sin 4 MARKS QUESTIONS Plc)D)-fia)
x>0 P( b-a
1. 2070 Supp. Q.No.6 bOR
lim 1-cos 2x State Rolle's theorem. Verify that the function 3e-12c11-
x0 3x2 conditions of
f(x) x (x- 3)2on [0, 3] satisfies
=

3c2-12c+ 11 -2
in the
Rolle's theorem and find c prescribed
3c2-12c+ 9=0
forn theorem.
-4c +3=0
SOLUTION C-1) (c-3) =0
lim sin 2xx First Part
C1,3
x0 3x Statement of Rolle's theorem:
But c 1 e (1,4) and c =3 e (1, 4)
If a function f() is Hence mean value theorem is verified.
>0 (a) continuous in fa, b]
31. 2073 Set DO.No.3a
(b) differentiable in (a, b) 2070 Set Ca.No.11
it
Using L Hospital's rule, evaluate: ffa) =f(b)
then there exists at least one point c e (, Last part
State Rolle's theorem. Interpret geometrically.
for the function
Verify Rolle's Theorem
lim x-Sinx Cosx 2 such that f(¢) = 0. Here, f(x) =x+ x2- 6x fxsxt-3P for x e[0, 31. 161
x0 x
Since is function, so t
Last part f() a polynomial 1ssoLUTION
Please refer to 2073 Set C Q.No. 3a Here, f(r) = x{x - 3)?, x e [0, 3] continuous in [-1, 4. First Part
Again, f(x) 3r2 + 2x - 6 which exists for all
32 2073SuPp O.No.3 x(x2 6x +
9) x
= -
6x2 +9xX Statement of Rolle's theorem:
lim In(tan
Evaluate, using L' Hospital's rulex-0 Inx
Since f(x) is a polynomial function, so XE-1,4)
f ) is differentiable in (-1, 4).
Ifa function f(x) is
continuous in [0, 3] (a) continuous in [a, b]
0 both conditions of mean value theorem are
SOLUTION Again, f(r)
=
3x2 -12x+9 which existsfor au satisfied. Hence there exits at least one c m
(b) differentiable in (a, b)
() f(a) = f(b)
lim In(tanx) xe (0,3)
-1, 4) such that f(¢) =[b)- fa) then there exists at least one point c e (a, b)
x0 Inx So, f(x) is differentiable in (0,3)
And b-a such that f(c) = 0.

=lim tanx sec?x 0-6


f(0)
f(3)=3
* 02+9
6 x 32+9x3 =0
x
0 =0 56-0
3c2+ 2c -64-(-1) Second Part
f all the conditions of Rolle's theorem are
x0 3c2+2c -6 10D satisfied then there is at least one point c e(a,
f(0) = f(3)
All the conditions of Rolle's theore 3c2+2c- 16=0 b) where the tangent is parallel to raxis.

satisfied. So there exists at leas 3c2 + 8c -6c 16=0


xsecx form
lim
c(3c+ 8) - 2 (3c +8) 0
-

x-0 tanx ce (0,3) such that f(c) =0

lim x.2secx. secx tan x +secix 3c2 12c+9 0 (c+8) (c -2) =0


x0 sec?x c-4c+3 0
Mathematics- Xl
142 Asmita's NEB Solution of Basic
fa) f(0)= 0'-2x02 +0=0 DERVATIVE AND ITS APPLICATION Unit 7 143
fb)-fa) Again, sOLUTION
y={) Now,f(e) b-a Statement of mean value theorem:
y-
-x-1"- If a function f(x) is
-4c +1- a) continuous in the closed interval [a, b]
fta t{b)
c(3c-4) =0 o)-T1 (b) differentiable in the open interval (a, b)
then there exists at least one c e (a, b) such
b &2)2-11 that P(c)-b)-fa)
Here, c-0 e (0, 2) butc=e (0,2) f(0)= {(2) a

Here, fx)= V?-4,xe (24


The function f({) =

(r-11s not defined at


x =1.
| For every value of x such that 2 Sx s4, f{r) has
y={ Hence, mean value theoremis verified. a
= So it is not continuous in [0. 2. So, the all definite value, so f(r) is continuous in (2, 4j
6State2071 Set CQ.No.11 conditions of Rolle's theorem are not satisfied and
mean value theorem. Interpret it geometrica hence, there is no number c e(0, 2) such that Also, f(x)= whih exists for all x e (2,4)
Verify the mean value theorem for the fu
unctlon f)0 f) is differentiable in (2,4)
f - (r-1)r-2)(*-3) in 1, 4 92072 Set CQ.No. 11 Hence, both conditions of mean value
Please refer to 2069 (Set B)QNo. 11
State Rolle's theorem. Verify Rolle's theorem for theorem are satisfied. So there exists at least
Last part
. 2071 Set DQ.No.11 one c in (2, 4) such that
Here, f() =x{r -3},re [0,3] State Rolle's Theorem. Interpret it geometrically the function f t ) = 2x2-3x +1 in
c) =b-ffa
x(r-6r + 9)
=
r*- 6x2+ 9x Verify Rolle's Theorem for the function
f() =r{r- 3) for x e [0,3]
f(c)b-a
f{r) is a polynomial function, so it is SOLUTION
Since
continuous in [0, 3] Please referto 2070Set C Q.No. 11 Statement of Rolle's theorem:
But, fb) {(4)=v4-4 -23
3x2 -12x +9 which exists for all Ifa function f(x) is ffa) f{2) =y2-4 =0o
Again, f(r)
=

3. 2071 Supp. Q.No.11 OR (a) continuous in [a, b] Pc)=D- fa)


re (0,3) State Rolle's theorem. Verify that the funtion
(b) differentiable in (a, b) f(c)b-a
So,fit)is differentiable
And
in (0,3) f(x)=sinx + cosx on [0, 217) satisfies the conditions of
Rolle's theorem and find the constant c prescribed ) fla) f(b)
f0) =0-6x 0+9x0=0 by the theorem. If f(x) =(x-1), show that f(0) = {12) then there exists at least one point c e (a, b)
f(3) = 3 - 6 x 32 +9 x 3 =0 but there is no number c in (0,2) such that f(c) =0 such thatf() =0.
f(0) = {(3) Why does not this contradict Rolle's theorem? 1
All the conditions of Rolle's theorem are SOLUTION Here fr)- 2r-3x+1 in d=3c?-12
satisfied. So there exists at least one First part Since f() is a polynomial function, so it is =6
ce (0,3) such that f(c) =0 Statement of Rolle's theorem: c=66
3c2- 12c+9 0 If a function f(x) is continuousi Here, c - 6 e (2. 4) butc=v6 e (2.4).
-4c+3 =0 (a) continuous in [a, b]
- 1 ) (c-3) =0 b) differentiable in (a, b)
Hence, mean value theorem is satisfied.
c=1,3 c) fa) = f(b)
Also, f() = 4x -3 which exists for allx e 1 2072 Set EQ. No. 11
State Rolle's theorem. What is the geometrical
Here, c 1 e (0,3) butc =3 e (0,3) then there exists at least one point c e (a, b)
ation of Rolle's theorem. Verify Rolle's
So, fe) is differentiable in
interp
Hence Rolle's theorem is verified. such that f(¢) =0. theorem for the function
Next part
2070 Set DO.No.11 Here, f(r) = sin x + cos x in [0, 2r)
f=1-*,x e l-11. [6]
SOLUTION
State mean value theorem. Interpret it
geometrically. Verify mean value theorem for
the function f{x) x (x-1P in [0, 2.
=
[6
Since for all values of x such that 0

f{r) has a definite value, so f()


x s
is continuos
-3-1-0
f(1)= 2 x 12-3x 1+1 =0
First Part: Statement of Rolle's theorem
If a function f(r) is
in [0, 2] (a) continuous in [a, b]
SOLUTION (b) differentiable in (a, b)
First part:
Please refer to 2069 (Set A) QNo. 11
Also F(x)= cos x - sin x which exists for au
X E (0, 21) - Hence, all the conditions of Rolle's theorem
() f(a) = f(b)

then there exists at least one point c e (a, b)


So, f(x) is differentiable in (0, 271)
Next part: & f(0) = sin 0 + cos 0 =1 are satisfied. So, there exists at least one such that f(c) = 0.

Here, f(x) = x(x - 1},x E [0, 2] Second Part: If all the conditions of Rolle's
f(2r) = sin 21+cos2=1
x(2-2x+ 1) = x - 2x +x such that f(c) = 0 theorem are satisfied then there is at least one
f0) = f(2)
Since f(x) is a polynomial function, so it is point c e(a, b) where the tangent is parallel to
continuous in [0, 2).
All the conditions of Rolle's theorem a i.e. 4c -3=0 -axis.
satisfied. So there exists at least o
Again, f(r) = 3x2 - 4x +1 which exists for a l x
ce (0, 2x) such that f(¢) = 0
in (0, 2)
COS c sin c =
0 y=()
f(x) is differentiable in (0, 2) Hence, Rolle's theorem is verified.
Sin c =
COSC
Hence both the conditions of mean value
theorem are satisfied. So, there exists at least
10 2072 Set DQ.No. 11| t(a) fI(D)
tan c =1 tan =

ate Mean Value theorem. Verify the mean


(a)
onece (0,2) such that P(c)= fa) value theorem for the function b
b- a
Here, f(b) = f(2) = 2 -2x 22+2-2
c- (0, 2r) )-y-4, xe 124 6
Hence, Rolle's theorem is satisfied
Asmita's NEB Solution of Basic Mathematics- XI
144
Put x - i n y = f(x) = sinx, we have

DERIVATIVE AND ITS APPLICATIOON Unit 7 145


y f()
ysin-1 16 2073Supp Q.No.11 flc)-o-e
laing Mean Value theorem, find a point on the b-a
rabola y (*3 where the tangent is or, P(c) -
Required pointis (: narallel to the chord joining the points (3, 0) and 7-2
(4,1). 16 or, 4ck- 10 = -1027+2)-22-10x2+29
13. 2073 Set CQ.No. 11 OR 6OLUTION

Here, f(r)=V1-3,xe-1, 1)
such that -1 s r s i , f(x)
State Rolle's theorem. Using Rolle's the.
find a point on the curve f{x) Cos2x wher
The value of x ranges from 3 to 4.

[3,4]
or, 4c -10
-
For every value of x re the So, y f(x)
=
(x - 3}, x e
or, 4c 18
is continuous in| tangent is parallel to x-axis on |-T,n].
has a definite value, so f{?) Since fx) is a polynomial function, it is
sOLUTION continuous in [3, 4].
Rolle's Theorem Again, f(x) = 2x - 6 which exists for all x e (3,4).

-2)=
which Clearly c-e (27
Again ft) If a function f(x) is So, f(x) is differentiable in (3, 4).
Hence Lagrange's mean value theorem is
exists for all x in (-1, 1) i. continuous in [a, b] Both the conditions of mean value theorem are
i. differentiable in (a, b) and verified.
So,fz) is differentiable in (-1, 1) satisfied. Hence by mean value theorem,

Also, f(1) -v1-12 =0


ii. f(a) = f{b)
there exists at least one c e (3, 4) such that 18. 2074 Supp ONo.1
State mean value theorem. Interpret it
then there exists at least one point c e
f-1) =V1-(-1)* =0 (a, h f c ) - b - L a )

such that f(c) = 0


fc) b -a geometrically. Verify value theorem for
mean
f1) =f(-1) the
All the conditions of Rolle's theorem are
Next Part
4)-3 function ffx) x(tx-1F in [0, 2],
=

satisfied. Hence, there exists at least one


Here, f{x) = cos 2x, x e l-T, T | |
or, f(©) 4-3 First Part: Please refer to 2069Set A QN. 11
For all x e (-T, T), {x) has a definite value, s Second Part: Please refer to 2070 Set DON. 11
ce-1,1) such that f(c) ==0 =
-3-3)2
f{x) is continuous in (-T, n). or, 2c-6 19 2075 Set AQNo.11
ii Again, f(x) = -2sin 2x, which exists for all State Rolle's theorem. Interpret it geometrically.
Verify Rolle's theorem for the function
xE -T, T) or, 2c-6--1
c=0e (-1,1) f(x) is derivable in (-n, T)
or, 2c =1+6
fx) = (x-1) (r-2) (x-3) in [1, 3].
6
Hence Rolle's theorem is verified. ii. Again, f(-r) = cos 2(-7) = cos 27=1 SOLUTION
12 2072 Supp QNo.11 and f{7) = cos 27=1
:cze(3,4 First two parts:
State Rolle's theorem. Interpret it geometrically. Verify lease refer to 2070 SetCQNo. 11
Rolle's theorem for the function f{x) = sin x, x E [0, m].
f(-T) =f(7) So,c 2 i s the x - coordinate of the point at Last part
Thus, all conditions of Rolle's theorem are
Also find a point in the curve represented by given Here,
function where the tangent is parallel to x-axis. satisfied. So, there exists at least one valuecCE which the tangent is parallel to the chord
6] f(x) = (x- 1) (x-2) (x-3)
(-T, 7) such that joining the given points.
SOLUTION f() =0
= * - 6 x + 1lx - 6

First part and Second part


-2sin 2c =0 Putx-iny=(x -3), we get y = ( Here, f() is a polynomial function, so it is
continuous in [1,3.
Please refer to 2072 Set EQNo. 11
sin 2c =0= sin 0, sint, sin (-T)
Last Part Again,
f(x) = 3x2-12x+ 11 which exists for all x e (1,
Here, f(x) = sin x Required point-(
3), so it is differentiable in (1, 3).
For all x e [0, 7], f(x) has a definite value, so
16 2074 Set AQ.No. 11 OR Also,
(x) is continuous in [0, n) . c0 or
2 oris the x-coordinate ofthe the Mean value theorem. Use finda ()=(1-1)1-2)1-3)=0
Again, fx) = cos x which exists for ali
point where the tangent is parallel to x-axis. State
point on the parabola f(x) = (x - 3), where the
it to 3 ) = (3-1) (3-2) (3 -3) = 0

xe (0, 7). When x = 0, f(0) = cos 0=1 All the conditions of Role's theorem are
angent is parallel to the chord joining the
fx) is differentiable in (0, 7). satisfied.
cos - 1
points (3, 0) and (4, 1). 161
Also, f(0) = sin 0 = 0 Hence, there exists at least a point c e (1, 3)
f(7) = sin n =0
First pat: Please refer to 2072 Set D Q.No. 11 such thatf() =0
Second part:Please referto 2073 supp Q.No. 11
f(0) =f(7) Wen-4- cos -1 7 2074 Set BQ.No. 11 O
or, 3 c - 12c + 11 = 0

all the conditions of Rolle's theorem -12) +V-123-43.11


are ne Lagrange's Mean value theorem. Also Or, C
2.3
satisfied. erity the theorem for the function:
Hence, there exists at least a point
Required point is (0, 1) or( t(x)2x 10x+29 in [2,71.
such that f(c)=0
c e (0, m) 2
14 2073 Sot DQ.No.11 SOLUTION -2.577, 1.423
or, cos c=0=cos5 State Mean Value Theorem. First part: Please refer to 2072 Set DQNo.1
Interpalue Next part:
Clearly c2.5577, 1.423e (1,3)
geometrically, Verify Lagrange's mean Hence Rolle's theorem is verified.
theorem for the function f(x) x(x-1)* in 10, 4p Here, f(x) 2x2-10x + 29
c e(0,) Please refer to 2070 Set D Q.No. 11
=

c
Sir
x) is a polynomial function, it is|20. 2075 Set BQ.No, 10OR
Hence, Rolle's theorem is verified. continuous in |2, 71. State the mean value theorem and interpret it
geometrically. Verify that the function ftx) = vx
c-is the xcoordinate of the point at which so, F(x)= 4x -

10, which exists for all x e(2. 7)


L.e. (x) isidifferentiable in (2,7) on [1, 4 satisfies conditions of the mean value
are
tangent is parallel to x-axis. Ce all the conditions of mean value theorem theorem and find C prescribed in the theorem.
satisfied. 6
t h e r e exists at least onec e (2,7) such that
Mathematics- XII
146 Asmita's NEB Solution of Basic

sOLUTION

8
2069 Set A QNo. 11
First part: Please refer to
Second part:
Here, f) =vx in [1, 4]
For every value of x such that 1 s x s 4 , f{x) | UNIT
in [1, or,
has definite value, so fx) iscontinuous
a

Again, fo)-x which


exists for a
c-(4
Required value of Cis
ANTIDERIVATIVES
2 MARKS QUESTIONS
f(x)
x such that 1<x<4.
is differentiablein (1, 4). 21. 2075 Set CQ.No. 1
&2061 NO3
Thus, both conditions of MVT
are satisfied.
State the mean value theorem.
Interpret LA087 QNo.3
So, there exists at least one C in (1, 4) geometrically. Verify the mean value theorem
such that for the function f(x) =
(x -

1) x -

2) (x -

3)
Prove fcosecx de =logtan SOU
f(C)-o-ha)
L, 4]. SOLUTION
b- a
First Part: Please refer to 2069 Set A QN. 11
or, f(C)= Second Part: Please refer to 2070 Set D QN. 11
cosecr dr-J sin dr Lt-JVia
sec?
1+ -
dx
2 sincw

Put tan=y

secaddy sin-'x -1-r2+C


Then, 2062 Q.No. 3a
dr
cosec xda -

Integrate: ax- (21


= log lyl+C SOLUTION
loz tanc Letl- adr
-
&/2058 Q.No. 3a
/Prove that:. si* -

SOLUTION
Put x a sind
dx= a cos0 do
J
a cose de_ a cose de
Va-a sin? a cos
&2063 Q.No. 3
ao 0+C= sin"C [21
2060 Q.No. 3 Please refer to Model Set I1, Q.No. 3b
Evaluate:
Please refer to 2058
Q.No. 3a
Mathematics-ll
of Basic ANTIDERIVATIVES Unit 8 149
Asmita's NEB Solution
14 SOLUTION
Q.No, 3
7 2070Sot D
-1 23 2071 Supp. Q.No. 3b
2064 Q.No.3
[2 Let Evaluate:a-a)-B>a. 121
Evaluate: Please refer to Model Set I1, Q.No. 3b
Find the integral 3-- [21
Put y SOLUTION
SOLUTIONN 2070 (O1d)O.No.3
ay-d Evaluate:4e (2
dy =e+C=eT+C Please refer to2065 QNo. 3a
fe
D2070 Supp. Q.No.3
12. 2068Q.No.3e Fiodthe integral l(2x
-

5) V* -

5x +1.dr 12
2065 Q.No3 sOLUTION

Put 2-5x + 1=y


Evaluate: e + (2x-5) dx-dy
Please refer to 2062 QNo.3a
SOLUTION 13 2069 (Set A) Q.No.3D I2r-5) F-5x+1dr-Jaly dy
24. 2072 Sot CO.No. 3b
Evaluate: Syn dy-C-ja-5 1c Evaluate:1-2 cos 2
Put e y 3a
sOLUTION
Please refer to 2062 Q.No. dr
Teti- 1-2cosE
e dr-dy 20 2071 Set C Q.No.3
14 2069/SetA) Old Q.No.3a Evaluate: 4 d
I- n-y C Show that: sin1+c
SOLUTION
Please refer to 2058 Q.No. 3a

tan-1(e+C 16 2069 (Set B) O.No. 3b dr d


=
h+ a (say)
.2066 ONo.3
valute: n-cossec
Integrate Please refer to 2065 Q.No. 3a Put y= x2+9 sec
dx
SOLUTION
d
16. 2070 Set CQ.No.3 b
Evaluate: + d 2
dy 2x d

h-J-3 logy +Ca 3 log(r+9)+C


3n-1
2a1+ a?- a2 Put3 tan-y
SOLUTION
Let- 2x +3 dx
+ a-a
Put y=X+a
dy = dr

- 3log(r2+9)+C*tan+G
1y--log (+vy-a)+C 3 log (+9)* tanC
where C- Ci+ C2
log (r+ a +r+a-a2)+C 1. 2071 SetD Q.No. 3
log (+a+2ax+ 2)+C
0. 2066 C Q.No, 3a
Evaluate: 2ax+ 2 ani+1
Flease refer to 2066 Q.No. 3a 25 2072 Sot DQ.No.3
Evguate: x- 2 2071 Old x dr
Q.No. 3 a Computethé integral Jsinhx -9 cosech (21
Please refer to 2062 Q.No. 3a
11. 2067 Q.No. 3a
ShowthatS a.
sin SOLUTION
lease refer 2058
to Q.No. 3a coth x dr
Evaluatee"dr Letlsinhx -9cosech x
MathematicS-l
Solutlon of Baslc ANTIDERIVATIVES Unit 8 151
Asmita's NEB
150
COsh
dx sOLUTION
cosh sinh r
JUnhr-9a r
-2.x.3+33-33+ 13 Let
sinh dx 1 1+3cos%
sinh sinh
Sinhr
-3P+4
29 2073 Set DQ.No. 3 x

Sinh?r - Evaluate:
e-3y4 2 secx d
log(x-3)+Vx-3)+2]tc secx+3
Puty = sinhr =log (x-3+Vx2-6x+ 13+ c
Then, dy
cosh x dr Please refer to 2073 Set C Q.No. 3b Secx dx
2 2074 SetB O.No. 36
1+ tan?x+3

- 3y3* 20733uppa.No.38 -sec dx


1 Evaluate:
sin 2x
(sin x+ cos x
Evaluate: -a)(x- b}dx 12
Put z
tanx+ 4
tax
SOLUTION dz = secx dx.
. c sOLUTION
Then,
sin 2x Let-a)x-b) -a) K-b6
6 2072 Set EQNo.3E Let (sin x+cos a) Ax-b) +B(x-a)
x-a) (x-b)
Evaluate: >) or, 3x A(x- b) + B(x-a)
C2sinx aosx Putxa in (i), then
Q.No.3b (sinx+ cos)2
Please refer to Model
Set 1, 3a- A (a -b) tanC
27 2072 SuppQ.No. 3b (2sinr cosx + dx Aab ta c
( s i n r + cosx)2 ar

Evalyate: (V2ax-xds [21 Again, put x=bin (i), then


(sin x + cos y + 2 sinr cos-1 3b=B(b-a) 36. 2075 Sat CONo.30
(sinx+cosx)
sOLUTION B- Evaluate:
Then, from )
2ax-xdx -
Va-a?+22x -x dx (sinx +cos
(sinr+ coSx)
dx 3x 3a SOLUTION
x-a) (x-b)(a-b)(x-a)(a-b)(x - b) Let
-Sva--2axa ) dx
J }a* Now-a)x-b d*
JVa--af dx
Sar-piTF Sta-br-a-b)r-bi
(*).¢| =x- h (say)
(secxd Let y =
e
e
Ve+1

dx
J1+tan
=

ha-aa.sin ch (sinx+ cosx)


Put 1+tan x = y
sec
5a
a-log x-a) log-b)+C
dy = e

Then,
dx.

28. 2073 Set CQ.No.36 secx dx= dy a - l a log (x- a) - b log (x-b)+C
[2
alyte a i h-J-Srdy *1C 83 2074 Supp Q.No. 3b log (y+vy+1)+c
log (e+v+1)+C
SOLUTION Evaluate: t e 21 log (e+Ve+1)+C
C- +tanx)*
C

-+C
7 2075 Sotga.No.3G
I=x+
JX*1+ tan+C
x
=x* 1+tan Please refer to 2065 Q.N. 3a Evalte: (x-11
+x-2dx
1. 2074 Set AQ.No. 36 42075 Set AQ.No. 3
SOLUTION
- d-fe-=i)dx Find the integral dx
6x +13
Evauate: 2a- 121 (2x-11
x+x-2x
Please refer to 2062 Q.No. 3a
- Sxdx- JV-idk SOLUTION
dx
Bs 2075 Set BQ.No. 9 dx
Find the
6+ 13 integral +Scos
og x*-i)c
Mathematics-l ANTIDERIVATIVES Unit 8 153
Solution of Basic
Asmita's NEB
152 dx
Jya-0 Jia (sinx +cos) sec?5 dx

2
2 sinxcosxr+ 1-1.
dx |4 tan?+ 6 tan4
-dx (sinx + cosx)2
- d
'sinx+ (sin
cos'xx ++2cosx)2
sinx cos-1dx sec2 dr

'*x-2dx -12/
C(sinx+ cosz)2-1dr Jamng-
(sinx + cosx)?
dx Put tan-y
i)2-3.c
sin-' -V1-2+C
0. 2059 C.No.11E seca dr 2 dy

cdx,
a bcosx' a<b
J (sinz +cosa)a dr

SOLUTION -x-h (say)


-log x-2)-4lo dy
Alternatively Here, aboo sec
J (sinx + cosr)2 sec?x
dx
ax

Let,
dx (secrdx
2)
X221*2*-i
T- Bx+2
Jaa
Put 1+ tan x = y
secx dx = dy
or, 2x-11= A(x -1)+ B(x +2)... (2) sec?
Putx =1 in (2), we get .

dr
2-11 B (1+2)
-3 (a+b)cos+(a-b)sin 2 sec 2 tan4
Again, putx s-2 in (2), we get,
-4-11 A(-2-1) sec dr 2062 Q.No. 11D
A=5 ...i) dx
Then from (1)
2x-11 (a+b)-(b- a) tan? -0 tan *C Integrate:| sin x+cosr 41
x+x-2x+ 2 2(1+ tan a)* sOLUTION
Put b-atan= y
Now, Here.
Here, d
sin x+ cos
b-a secdx- dy
Then the integral (i) becomes 2.2061 O.No. 11b dr

-5zdx-3/dx. Find the value v2sinxcos


5 log+2)-3 log (x-1) +C Jab- of3 sin dx
x-4 cos * 4
dr
4 MARKS QUESTIONSs
lnb+aY,c sOLUTION coSrCoss i n xsin
8. 2057 Q.No. 11
dx
-/b -y Let 3 dx
x-4 cosx dr
Integrate:a+b cosz whena
>b
4 VD+a+ b-atan +C

Please refer to Model Set I, Q.No. 7a B- 6a-yb-a tan ) dr


99. 2058 Q.No.11 3x 2sin cos-4cos- sin
1 2060 .No.11B
Integrate: 14 X,cos
Find the value of:Sn
of: dr
Sec
(sin x +cos x dx
SOLUTION 4 sin?+6 sincos-4 cos
SOLUTION sec2
Let I S n X.cos
LetI xx
J (Gin x+
cos
x
Solution of Basic
Mathematics
-
154 Asmita's NEB

I log log (y*vy-1)+c ANTIDERIVATIVES Unit8 155


+2) (r+ 3
Jmdhy(i).c|| -
log (sin + cosr *
VN(Sinr * cosx)?-. Now,
-log (sinr +cosr +Vsin21) +C
i)J* 6 2065 Q.No 11
dr
2 ydy -2cc
2063 O.No. 11b Integrate:3+4coshr + C

dr 14 1-tan
Integrate: T+sinz+ cosT sOLUTION
-4log (2)-3 log (+3) + C Alternatively
Letl-s4h 8. 2066 CQ.No. 11 LetI
dr dx
Let-Ji-sin-W dr Evaluate: 1+ Sinx+ Cosx Put 1 r cos and 1=r sin
41 So that r -2

(oo-sin coasi Please refer to 2063 Q.No.11b


92067 Q.No. 106 Also, tan =1 tan
sec h25
dr
dr 7 cosh2t sinh? sec h? Evaluate:1+2 Sin x
Please refer to Model Set I, Q.No. 3b
2 02sin~oos 0 sinx+
dx
sech? 60 2068 Q.No.11 r cos r sin 0 cosr
- dx

tanh+7 Exaluate:sin sin (- 0+J coe (r *0) dr


1an Put tan h=y
Please refer to 2063 Q.No. 11b

Put y1+ tan 1. 2069 (Set A) Q.No. 7a


Then, s e c h 5 dr = dy

dyseda Evaluate:3 sin x-dr4cos x log tan(


or, 2 dy sech2 dr
2dly secdr Please refer to 2061 Q.No. 11b 2069 (Set B) O.No. 7
Then integral (i) becomes $2 2069 (Set A) Old Q.No. 11b
. Evaluate: 2+3 co8
4
raluate:sin Sin x+ cos x
SOLUTIONN
2064 O.No. 115 SOLUTION
d
Integrate: dr 14
u n c an LetI
Let1-J23o
47 2066 Q.No.11 x+ cos x

SOLUTION
(coSY-sin xdx Integrate:&+2(k+3F dz dx
|2 sin cos+ cos?;- sin? sec
SOLUTION dr sec
(_cosx- Sinr COSX-Sin sec2 d sees dx
2 sinx cos JV1+ 2 sinxd
cos X-1r
Ag 32+B(r+ 2) (r+3):C
cOSX- Su) dr
OF, 2 )a+3)
+ 2) (+ 3*
2 +
2 tan1 tans tan
Put tany
or, - A (x+ 3)+ B (x +2) (x+ 3) +C(x
sin?+ cosx + 2 sinx cosz -1
Putx=-2 in (i), we get 4 A =
Put y =1
Again, put x = -3 in (i), we get
-

tan
C Icosx-sin x) dr 9C(-3+2) sec dr 2dy
(sin+ cosr -1 C 9 dy sec
Put sin+ And, put x -0 in (i), we get
cos yy 0 9A+ 6B+ 2C -2dy secdx
(cOSX sinx) dx =
dy or, 0 -9 x 4+ 6B + 2 (-9)
B -3
Mathematics-ll
ANTIDERIVATIVES Unit 8 157
of Basic
156
Asmita's NEB Solution
55 2070 Set CQ.No.
7a
dr sec 8.2071 Set CO.No. 7
Evaluate: 2+co8x 6 sin cos 2-5cos+ 5 sin seca Evaluate: 4
2 cosx

(5 Aan.c sOLUTION
ar
sec dr sec dx
SOLUTION
dz
tan 5 tan'+ 6tan-5 a un Let1-1-2cOSK
2069 Old Set B)Q.No 118 dx
Evaluate: e-1V16-ár-rdr Put tan y

sOLUTION
se d dy
dr sec
Let 1-/2-)V16 -6r - dx
secdr-2 dy dx

fa-21io-orFd
3 cos+sin'
3sa-3ain-co
sec
Jne-- yie-6 dr
Sec2
secd dx
3 cos2+ sin2 sec2 dy

- h * k (say) Put tan y


Put y tany
Now, dy
h 5Vi6-6 -
dr s d dy
sec
s V s - 4 3 dr seca dr 2dy

s3 (
Then,
dy
()
( tun-1
n))
logy3-134 C
tan y345y3y4
Again,
2 dx 2ta Ca 5tan+3- 30 2071 SetDO.No.7
56. 2070 Set DQ.No. 7a +C Evaluate: 1-3 sinz 4
Put y= 16 -6x - r2
dr
or, dy (-6-2r) Evaluate:
=

2sinx Alternative Method SOLUTION


Then,
Please refer to Model Set I, Q.No. 3b Put3 r cos and 5= r sin
67 2070 (Old) o.No.11b
so that r2= 34 Leti3
34
Evaluate: cosx-
sinS1n 2 dx
sir Also, tan 0 .0 tan
016 -6x-2)+Ca Please refer to 2064 Q.No. 11b
an'*cow-o singoos
Ih+h dr
25
68. 2070 Supp.QNo.7 rcos 6 sinxt r sin 0 cos r

a+3) Vi6-61-7 sin- Find the dx5 cos nzcossec$


integral 3sinx-
G06-6-j+G SOLUTION Sec d

dx log tan (r +0)+C tan-6tan+1


Let 3sinx 5 cosx
-

(16-61-192+C V ogtan (r*tan )c Put y tan


dx
where CC+Ca 1
2sno-s( dy
Asmita's NEB Solution
of Basic Mathematics-|Il
158 ANTIDERIVATIVES Unit 8 159
d Sec
secda=2dy 65 207/2 SotEO.No. 7
Then, 5cossin2*42sin ~cos se Evaluate: d 41
J-ey1-2J--( sec dr Please refer to 2069 Set B Q.No. 7a
36 2072 Supp O.No. 7a
5+5 tan25+8 tan
Evaluate dx
3sinx + 4 cosx [41
tan-3-2 +C sec2 dr
sOLUTION
tane uan1 dx
3sinx +4 cosx
31. 2071 Old Q.No.10
Sec2 dr -(-u3g).c Put 3= r cos a amd 4 =r sin a
Integrate
tan+2 tan
3 -3 1og 2-c so that r=v32+42 5 and a =

tam
SOLUTION
sec dx
-3 3 Sin x +4 cos xK
84. 2072 Set DQ.No. 7a
dx
r cos a sinx+ r sin a cos x
Evaluate1P-2 14

Puty tan2+ SOLUTION sec (x + a) dx

dx
dy-secadr Let-J-1(-2
=h*h (say) 2dy=sec?dx Putx-1 z(r -2)
sin+C=sin' x+C
24y 5 tam37C Or, dr -2-1:2-2-9:1dz
(z 1)2
dr =2z-2-22+1.
or, dr= (z-1)dz
Put y 1 -12
Alternatively
, . dx(z -1)
dy-2xdx tan-l z
Letl
dx
3sinx+4 coSx
Also,
-xdx (5 tan4
an dx

63. 2072 Set C Q.No. 7a


2

=vy+Ca-1-+C
Evaluate
J-2 dx
(-3 4
dx sec
I =h+ l2 -sin-'x+C1-2+Ca SOLUTION 4 co6 sincos-4sine se
-sin'x+1-2+Cz where C = C +C2

82 2071 Supp. Q.No.7a dx -

dz 2-3 3z-1 z
=
sec dx
Find the dx Put x-2 z(x - 3)
integre5+
4sinx 141
32-2
sOLUTION
dx - -
Z

-z
-

d ) sed
5+ 4sin x (-1)2 32-3 log z-C
AsO-23 (a-3) 2-3 (uanv-un3
Mathematics-ll
Solution of Basic
160 Asmita's NEB
ANTIDERIVATIVES Unit 8 161

sec? dx Hence-2x-15 (y +3) (y-5 72 2074 SotBO.No.7


Let(a 1)- 1*2 Evaluate:3sindxx4cos x 4
A(2z-5)+ B(z+1)
(z+1)(22-5 Pleaser refer to 2072 Supp. Q. No. 7aa
secdx (i)
8(y+3) By -5) 73 2074 Set BQ.No. 7a OR
z A(2z -5)+B(z+1)
Putting z -1 in (i), we get, A = Evaluate: a 4
Now - 2 x - 1 F dx
Again, puttingz-jin(i), we get, B= * Pleaser refer to 2061 Q. No.3a
sec dx
3,dx 74 2074 Supp Q.No. 7a
-8x2+3
Now, from ()
dx dx Evaluate: 23c05% 4

Put y-tan Please refer to 2069 Set BQN. 7a


dy-se d -J*T-5a 5. 2075 Sot AONo.7
dr

2dy sec5 dy ta lo Evaluate: sin xcos 4

2074 Sot AQ.No.7 Please referto2069 Set A(Old) Q.No.11b


dx
T2075 SetBONo 7
Evaluate:3+5coshx* Evaluate: S(2x+3)-2x-3 dx. 14
SOLUTION
-logz+1)+zlog(22 -5)+C SOLUTION Let
5
log (x+ 1) +2log (2xa-5)+C Let I=3+5dx_
cos hx I =/(2x+3) Vx-2x-3 dx
-S(2x-2+2+3)-2x-3dx
E8. 2073 Set DQ.No.7a Si(2x-2)+ 5] -2x-3 dx
dx i2x-2Nx-2x-3 dx+/5-2x -3 dx
1+2 tan2
+C
Evaluate:T+sin x+ cos x*
dx
=hta(say)
31O62-tan x

Please refer to 2063 Q.No. 11b Scosh 2sinh


h =(2x-2)V-2x-3 dx
Puty =x*-2x-3
= (2x-2)dx
7o. 2073 Supp Q.No.7a
7 2073 Setco.No.7 sec h2 =Wy dy
Evaluate:-2x2-15dx dx y2 dy
Evaluate:1-2cosx" SOLUTION y3/2
4 cos
h+ sin
h2 sec
h# 372C
Please referto2071 Set CQ.No.7a Put x= y

B2073Set CQ.No.7a OR Then -215


y-2y-15 +3)-5 dx
Evaluate: 4+ tan h2 -2x-3/a+ Ci
ty36y-5 5 And,
SOLUTION Ay-5)+ B(y+3) Puty tanh
dr (y3) (y -5)
5-2x-3 dx
LetI- 2x3-5 dy 5x-2x.1 +12-1-3 dx
or, y = A(y - 5 ) + B(y +3)
()
Put x2z Put y = 5 in (i), we get S/-1-4dx
= 5 N - 1 - 2 dx
or, 2xdx= dz 5 B.8 2dy sec h dx
xdx=
B- Then,1-4 -a-19-2-3-
Again,
Now,I- J3w-3"Jy-3 Put y -3in () y 10log (x-1+x-2x -3)+Ca
-3 A(-3-5)
or, A-
Asmita's NEB Solution
of Basic Mathematics-l
162 dy
=-2x-3)2+C+
-1)V-2x-3-10 dy UNIT
logx-1+-2x-3)+C
-2x-3p -1)v-a-3- DIFFERENTIAL EQUATIONS AND
+V-2x-3)+C, where C
10log(x-1
= Ci+C2 +C THEIR APPLICATIONS
72075Set CO.No.7
2 MARKS QUESTIONs or, sin( 1 - y yy1-x2) = sinr1 c
141
Evaluate:
2sint+3costd 13+2-Y,C
Ts13-2y .2067 Q.No.6 1-y+ yy1-x =c
dy x2+x+1 5 2061 O.No. 6C
SOLUTION
Let, (V13+2- tan Solve: dr y24 y+1 12
Solve: z?dy - ydx = 00
sSOLUTION
(2
+C
2 sint+3costat
dt
B-2* an
Alternatively,
sOLUTION
dy tX+1
Here dx y2+ y +1
Here, xdy y?dr
or, xdy = ykdx

1
- =

0D
or, (y2+y +1) dy = ( r 2 + x + 1) d r or y dy dr
dt
22sin 3(cosin Let,I=3sin t+ 4 cos t Integrating
or dr- dy =0
and 3 sin .
ec Put2 r c o s =r
3t2y=32xtc Integrating we have
dt Then, r2=22+32 =13
4.sin cos+3 cos5-3sin? sec Also, tan - 5 2 2058 Q.No.6C frdr- fy dy= c
dy + 1
Solve dxys+1
secdt ta sOLUTION
4tan+3-3tan Now,
dt
Given d
r cos sin t+ r sinecost o, (y3+1) dy = (r+1) dx or, X-y
y
c
sec? dt Integrating we get
T-y CrY
sin(t+) +y=+ x+c 2062 Q.No. 6C
Solve: rdy- ydx = 0
12
Put tan y - jcosec (t+0) at 3. 2059 Q.No. 2 b Please refer to 2061 Q.No. 6c
Solve: x dy - y dx = 0
2 2063 O.No.6C
Then,se dt- dy
-logtan (t+0)+C SOLUTION Solve: ery dr + e dy = 0
(21
Given, x dy - y dx = 0
log tan(t* tan"+c or, x dy=y dx
SOLUTION
sec dt= 2dy Here, er-y, dr + ey* dy = 0
or, dr
Now, or d r dy =o
dy Integrating, we get or, e dr + e dy = 0

Integratin8 we have
+log c
or,
log y logx+ log c
=

e+ey=c
log y =

log c x

y=cx 8. 2064 Q.No.6


Solve: x dy+ (r+ y) dr s0
2060 Q.No. 6 sOLUTION
Solve: T- dy +1-y' dx =0. 21 Given, r dy + (r + y) dr
=
0
SOLUTION or, x dy +Xdr+ y dr 0
=

(r dy +y dr) x dr =0
+
iven,1- dy+y1- yktx =0 or,

Or,dy

ntegrating, we have
Sn- x +
sin-' y sin =
Mathematics-Il
Asmita's NEB Solution of Basic
164

14. 2069er-y(Set B) Q.No. 4 DIFFERENTI EQUATIONS AND THEIR APPLICATIC


0 Unit 9 165
Solve: dr ey-*, dy
+ = SOLUTION

or, 2x +1 sin-l x + sin- y sin


Please refer to 2063 Q.No. 6c
Here d5y+ or,
si'
16. 2069 (Set A) Old O.No,6 rV1-y2(ry1-yl+yy1-z2)
Integrating we get =
sin-' c
(2x+1)dx (5y + 1) dy
+yy1 -2
ay c
Solvedye
x
Integratin& we have
( 2 r + 1) d r = (5y4 + 1) dy
25. 2072 Set CQ.No. 4a =c
2
9. 2065 Q.No 6 12] SOLUTION Solve: -1.
Solve: (y2+x) dr+ (ya*+y) dy =0 yc (2
SOLUTION Hered or, r+ *=y+y+c
SOLUTION
0
Here, (ay2 + 1) dr+ (y3+ y) dy
=

0
or, y dy = (er+ 1) dx tX-y-y =C Here, -1 ...i)
x (y2+1) dr + y (13+1) dy 1) (y2+ 1), we get
=

Integrating we have
21.2071 Set CQ.No. 4a
or,
Dividing both sides by (+ dx + ey-dy =
0 Comparing it with dy
oTd dy-0 =etx+c Solve: e ,
Please reter to 2063 Q.No. 6c 121 Py Qwe get,
=

or,

y-2e
y2 = 2 e + 2r +2c1

+2r+ c, where C = 2c1 22. 2071 SetD Q.No.4a P-1


r+1 ,Solve: +4x = 2 e LF= = e=e =
Integrating. we get 6. 2069 Old (Set B) Q.No.2 2] Multiplying both sides of (i) by x, we get
Solve: 2rydr - x*dy = 0
log (r+1)+ log (+1) log c
=

SOLUTION
or, log (r2 + 1) (r2+ 1) logc sOLUTION
(r2+1)(2 +1)=c 2xydx - xdy = 0 +4x 2 e2 or dy x)=rdr

10. 2065 CONo.6 or, y.d(x) - x*d(y) = 00


dy Integrating we have
2e2 4r
Dividing both sides by y2 dr
-

or,
Solve: 1-r dy+V1-ydr =0 (21
xy-+c
Please efer to 2060QNo.6c yd- x*d()= o or, dy = (2er - 4x) d r

11. 2066 Q.No.6C


Integrating, we havee 26. 2072 Set DONo. 4
2ez 412 Solve:+Cos
Solve the differential equation or, G - y 2 tc Solve dx 1- cos 2y 0.
(x+2y-3) dy -(2r-y+1) dr=0 121 2y 21
SOLUTION
y ear-2x2+ c SOLUTION
Here,(r+2y-3) dy -(2x-y+1) dr=0 Integrating we
get =c
23. 2071 Old Q. No. 6 a Given differential
equation is
Solve: 2xydy - y?dx =0
or, xdy +2ydy -3dy-2rdx+ydx - dr=0 x2 Cy 121 d1tcos2y 0
or, (xdy+ ydr) + d(ty)-d(1) - d[3y) - d(x) =0 SOLUTION I cos 2y
or, dry)+ d(y) - d(r) - d(3y) - d(r) =0
17.2070 Set CO.No.4E Here, 2xydy - ydx = 0 dy 1+cos 2y
or,
or, d(ry +y2 - r ? - 3 y - x) = 0 Solve: + 1 or, r d(y2) -y2 dr = 00 1-cos 2y
dx
Integrating we get or dy)-ydr
yy2-x-3y- r=
Please refer to 2069 (Set A) Old Q.No. 6c = 0
or,
ir-11+2 sin y
12. 2068 O.No.6C 18. 2070 Set DO.No.4 or,
o, d Osy
sin?y
Solve:dy - y2dx = 0

Please refer to 2061 Q.No. 6c


21
Solve:dy -yidx =0.
Please refer to2061 Q.No.6c - Integrating we have or, dy
d-coty
13. 2069 (Set A) O.No.4a 19.2070(Old) Q.No.5 dy
Coty-dr
Solve Solve:(1+)=1. 2
Solve=ex-y +e 121 y CX or, dx + tan>y dy = 0
SOLUTION 24. 2071 Supp. Q.No. 3 or, dr+ (secy - 1)dy= 0
SOLUTION Integrating we get
Here,=e-y
dr
dy
+.e Given,(+d=1 Solve -0. 121
+tan y -y=c

SOLUTION 27. 2072 Sot EONo.


or, d e . er+ ?e
Given equation is Solve:d+y-1=0. 21
or, dy + dx
or, d e r (e+ r) Please see Model Set lIL, QNo.4a
Integrating we have
dy( e + 1) dx
Or e
28. 2072 Supp Q.No.4a
or, er dy = (er + *) dr Jay-Jizd Solve:4x=2e (21
Integrating we get y= tan' Xtc -dy
,
V1-2 dy =1-
-VI-ydrd
Please refer to 207 Set D Q.No. 4a

20. 2070Supp. Q.No. 4a y2 + 29.2073 Set CQ.No. 4a


Jedy-Je+*)dr =
0

Solve the differential equation: d


or,yi-dx 0 Solve: 1+0 21
etc integrating, we have
of Basic
Mathematics-l
DIFFERENTIAL EQUATIONS AND THEIR
APPLICATIONS
54 2074 SuPp Q.No,42
Asmita's NEB Solution Unit 9 167
166

SOLUTION
Solvee*y + xer,
Comparing equation () with Py Q SOLUTION
both sides, we get weget
Integrating
Please referto 2069 Set AQ.N. 4a P sec-
Q sec?x Given ( - y ) y
5. 2076SetA Q.No.4
elan
0. Multiplying equation (i) both sides by etuna or,
Solve: rdy+ (* y)dr
+ =
)
6c
P l e a s e refer to 2064 Q.No. we get Put y vx, then
= tan-lc
x+ tan-l y
or, tan-l

BQ.No.3 + elana, secx. y =


elan", sec?x dy
or, tam1--an 36.2075 Settanx alanr ir dv dx
Solve: sec?x dx + secy tanx dy =
0,
d(y. elan)
=
etanr, sec?r dr
Integrating we have Now, equation (i) can be written as
X*Y sOLUTION
O-C

=Cl xy
-
Given,
sec?x tanx dx + seczy tanx dy =
0 y elan dfetan) + c
v
X+y
dx + secy dy)
=
0 =
elanr + c =-V
or, d1-vi-v
30.2073 Set DO.No.
eunr

[21 or, tanx (sec?x or,


y
y
Itce-lanr

secx dx + secy dy 0
=

Splve: ydx- xdy=xy dy or, dy y-y+y


Or, d r 1-v2
SOLUTON
Integrating both sides, we get,
40. 2059 Q.No. 11 b OR
Here, ydx-xdy
=
xy dy secx dx |sec?y dy =Fc
+
dy y 2 - dv

Dividing both
sides by y, we get,
tanx+tan y = C. sec? t= tan t+ C Solve:d 4 or, d

1-v2a v d
ydx-xddy 37. 2075 Set C Q.No. 4
sOLUTION
ay2-
OI,

Solved Here dr 21y


Put y = vx, then
Please refer to 2069 (Set A) Q.No. 4a

4 MARKS QUESTIONSs
dy-y+ Xdx
drv+ dy

.
or, dy
38. 2057 Q.No.11 bOR
Now, the given equation can be written as
dv v22-2
Integrating we get
v*dr 2x. vx
Integrating we get Solve: tan x+y=sec x. log
or, V+dr
SOLUTION
dy
or,-logv log x+ log + ce

Here, tan x d+y= sec x


81. 2073 Supp O.No. have dv_y2-1-2v2
or, log (vzc)
Dividing both sides by tanx, we
or, Xdr 2
Solve: (21
+cotx. y = cOsec x ...(1
o7,
SOLUTION dr 2v
dy 1+ Comparing () with Py Q, =
we get or, 2v v
dr
dx 1+x2 P cotx Q= Cosec X or l o g(
or, (1t x) dy = (1 + y )dx
or, 2v dv
dx dy LF =
ePdx =esin= elogsina sin x or, -2y2log (y)
or, 14x 1+y equation (i) by l Integrating we get
Multiplying both sides of 2y2log (cy)=0
we get log (v2+1) + logx =logc 3. 2062 O.No.11 bOR
or,
log ((v2+1) x} logC =

Sin + Cot x. sinx. y =


cosec x. sinx Or, (v2+ 1) x=c Solve * 4
Integrating& x
tan'x tanr'y =
tan-'c
or, dy
sin d r t cos x. y = dx SOLUTION
or, tan = tane
1+xy or, d (y.sin x)= dx Giveny- .()
Integrating we have o, y
y Sin x=xtc 2+ y2 =cx Comparing equation (i) with + Py Q
X-yC(1 +xy) 39. 2068 Q.No.11bOR 1. 2060 Q.No. 11 b we get

32. 2074 SetA O.No.3d OR


dy e *
Solve:cosxdy 1 Solve; tan x+y sec x 4 P--
Solve dx y SOLUTION ease refer to 2057 Q.No. 11b LF - = elog = XX
OR
Please refer to 2069 Set A Old Q.No. 6c
dy 2. 2061 O.No. 11 b Multiplying both sides of equation () by x
33 2074SetBO.No. 4 Here, cosdr +y-1 Solve: (-
OR we get
Dividing both sides by cos?x, we
have
ya) dxy 41 dy
Solve:(1+x)d1. 121 dy .0
dr*y

drt secx. y sec?x


=

Pleaser refer to 2070 (old) Q. No. 5c


Solution of Basic
Mathematics-ll
FERENTIAL QUATIONS
168 Asmita's NEB AND THEIR APPLICATIONS Unit 9 169
= log cx
or, d (ry) =x'dx or, 21og Multiplying both sides of equation () by sin x
Integrating we get, 2 dy
ean Sin
dx cos X.
y =
x sin
ay + c log cx both sides of (i) by elan or, d
x

Multiplying we have (y. sin x) =


X sinx dx
44 2063 O.No.11 bOR CX ean
.
dr tear Integrating&
4 or,
y sin
Solve: tan
x
x sin x dx+ c

(y )=+ d
dIntegrating
sOLUTION
)
- =cX
o,
we get 07, y sin x x

Here d
+
tan yela ea
Put y = vx, then
O, y sin x--x

dy Put tan-1x=z
cos x- | 1 (-cos z) da *c
d r V * r

or, y sin
becomes, -12=cry? x=-x cos x+ sin x +C
Now, equation () Then,d r - dz . y sin x =sin x -x cos x+ C
d 46. 2065 Q.No 11 bOR 52. 2069 (Set A)O.No. 7b
v+dr
Solve:+y cotx= x
OT, d
dv tan v Solve: tan xty sec x 141
SOLUTION yetanj(etan)2 + c
Please refer to 2060 Q.No. 1 OR
Or tan v I dy 53. 2069 (Set A) O.No. 7b o
COs _a
Here, drt y cot x =x

or, sin v a r
Comparing equation () with dPy =0
yetanI+ce-tanr
Solve ayd=r+y2 [4
Integrating, we get
log I +
log c we get
49 2067 Q.No.6 SOLUTION
logsin v=

P= cotx, Q=x
Solve: xdy+ (r+ y) dx =0 4
Given differential
equation is
or, log sin v=log c r
or, sin v C
LF= e = ekol r ds = e sin= elogsin = sin
sOLUTION yx+ y
Given, xdy +(x+ y) dxr =0
Multiplying both sides of equation i) by sin, or, xdy +xdx + ydx = 0

sin- weget or, xdy + ydr + rdr = 0 xy


Puty = vx, then
5. 2064 Q.No. 11 bOR Sin r dy
d +y sin x cot x = x sinhx
or,
d(ary)+d dy dv
Salve2 d(y sin x) =x sin x dx
Integrating we get,
Then, from ()
SOLUTION dv v . VX
y sin x x sin x dx+c
=
V* drx
Given, 2=Y,2 Integrating we get
Put y =vr
or, ysin x =x/sin xdx- de r xy+ C or, V+ ,d1+
dr

dy dy or, y sin x=-x cosx }1 (-cos


-

x) dx+c 50. 2067 Q.No. 10b OR Or, d r


dv 1+ V-V
v
Then, dr V** dr + C
or, y sinx= -x cos X +] coS x dx

141 or, Xdv


d 1+ v-v
Now, given equation can be written as Solve: (1- )
y Sin X-xcos x t sin
x + c
1+*y v
47. 2066 CQ.No. 11 b OR Please see Model Set II, Q.No. 7b or
51. 2068 Q.No. 11 b OR
dv Solve: (-y)d*y
or, 2v + 2xdr =v+ v2 Solve: sin ay or, vdv=
Please refer to 2061 Q.No. 11b OR x+cos x.y =
x sin x 41
or, 2 d
dy
y2- v A8. 2066 Q.No.11 bOR SOLUTION Integrating, we have

Solve the differential equation:


2
d Given, sin xd log*c
Ovv-1) dv= (1+)+y=eun
cos x.y =r
sin x
Dviding both sides by sin x,
or 2

Integrating we get
) v SOLUTION
Given equationis: (1+ )dye
y =eun
Os=x sin x () y21-log x+c)
mparing (i) with dPy Q, we get S4. 2069 (Set B)Q.No.7b
an'x
PCOS Solve:(1+x) d21y=4r 4
logc
.

sin x Q=x
or, 2 (log (v -1)-log v)= log r+ logc
Comparing equation (i) with Py-e e =
sin =
elog sin x Sin d
=
Basic Mathematics-ll
170 Asmita's NEB Solution of
DIFFERENT EQUATIONS
Put z-dz-d HEIR APPLICATIONS
SOLUTION d
or, d r 1+v2
Unit 9 171
Ther
Here,(1+ )d +21y 4 I- e dz =e+c=ei+c d-
o r d r 1 +v2
or,
Then from (i), 1+Vdv=V-a e+xe+e+C= xei +C
Dividing both sides by e, we get
=

d v
or
we get y(1+ x) = x+ cer
Comparing () withdPy
-

ye =e. dv+0
or.
i dv 83. 2070 Supp. Q.No. 7E
P andQ- ys1+ ce dx Solve: ?dy+
i 1Ldv ylx+ y)dx =
0.
41
=1+? 58. 2070 Sot CQ.No.7 of dv
+
sOLUTION
Now, I.F =eh =e T =
eh

sides of (i) by 1 + r', we get


Integrating we get
Here, dy +y(r+ y)dx =0
Multiplying both Solve: cosdy-1. -v*
logv+ log
*

(1)d2ry-4 Please refer to 2058 Q.No. 11b OR log c=(-3+1 dr


Put y = vz
..)
39. 2070 Set CQ.No. 7bo ay dv
or. b + ) - 4 Solve: - sin Then, v+ dr
or, d {y(1 +r))
=
4r2 dr Now, equation (i) becomes,
Integraing8 we get
sOLUTION
=
Hr dr + C dr
y (1+r)

or, y l l + - 4 c is, d
Given equation
sin? -

2
dv
or, v + x -v-v2
or, log (cy) 2y2 dx
dy +X
Put y=vx, then =V+ *d dv

yl+9- 2=2y2 log cy Or,


=-2v -v2
55. 2069 (Set B)Q.No.7b O
Now, the given equation can be written as
2. 2070(Old)Q.No. 11 bor or, 2v -v?I
v +dv
d v - sinv Solve: (1+) d= ay - x+1
Solved +fan 4 dv sOLUTION
4 or, av
-2v (v+ 1)x
Please refer to 2063 Q.No.11b OR Or, d r -sinv
or,V.2dr
56. 2069 (SetA Old Q.No.11bo -dv dr Here, (1+ )dy
a y -x+1 vv+1) 0
or, sin?v X
dy
Solve:cos y 1 4
Integrating we have or, (1+) d-ny =1 -x
Please refer to 2058 Q.No.11b OR Dividing both sides by (1 + X), we get Integrating we get
7. 2059 Old (Set B) O.No. 11b O S-cosecv dvJ ...)
logv-log(v +1)+2 logx= logc
Salve or, cot v = log x + c

dy,
Comparing equation (i) with +Py 0 we
get
or,
logyi1* log r= logc
SOLUTION
cot(log r+C
P1 - logc-logr
Here ..)
60. 2070 Set DQ.No.7 LF =Pd=eh= 1
Comparingequation (i) with P y Solve (1+)+2xy4* = elog ( 1 ) - x
elog (a* 1), e- = (1 +x) e or,
we have
Please refer to 2069 (Set B) O.No.7b Multiplying both sides of
equation () by
(+x) e, we have
PandQ= 81 2070SotDO.No.7b O
IF =e =
Solve:(+ y) dy = xy dx

SOLUTION
e(
O,
e0-9 cr*y)
64.2071 Sat CQ.No. 7
Here, (2+ y) dy = *y dx
dfe (1 x) y} e (1-x) dx
+ =

Multiplying both side of (i) by e? Integrating we get 4


dy Y ...0 y(d+ )er |Sove:
( or,
dr + y2 =

J(1 -x) e* dr*c SOLUTION


Puty= VX The given equation is
Or,
d y(1+ x) e-i
or,
d(y.e)-eid dy
Then.dv **d Jerds- Jre"d
Integrating Now, equation (i) can be written as
Puty =vx
dy X VX_ then dy
i)
Now, the given equation is
dvr+ v
V+Xdr 212
Mathematics-ll

Asmita's NEB
Solution of Basic
DIFFERENTIAL EQUATIONS AND THEIR APPLICATIONS Unit 9 173
172 SoLUTION

or v Here y 73. 2072 Sot DO.No. 7


Reduce the equation = y in inear form
Comparing equation
(j) with
d+ Py =

we get
r,
-V
3y' log ( hence solve it. 4
v2-2v +1
70. 2071 Supp. Q.No.7b OR
sOLUTION
Given equation is
or, v-1)2 LE =
=ed= eo 1+x dy sin
dr 4
Multiplying both sides of equation (i) by dx
or, 2dv
- 1
(1+r), we get SOLUTION
Dividing both sides by y:, we have
dY= sin 2 ..i)
Integrating we have,
a2)dr2x:y-
log+logc Put y-lz then
or, d (y. 1+)-Tdr Here, P-,Q sin x2 -y-2 dz
or y
= log Integrating we have
LF=e =er"= elogr=r -ydd
1 Multiplying both sides of (i) by x, Then equation () can be written as

ya-Tdr --(i)
or, 2x log a yxsin z? which is linear
X-y
y(1x) = tan-ir+c dry) =xsin r2 dr
a
equation.
2r (1-1) log CT
68. 2071 OId Q.No. 10 a OR Integrating Here, P-Q--1
5. 2071 Setca.No.7 bOR ay
=
lx sin 2 dr
+ c

sinr (cosx)y =
sinr cósr 4 Solve: tanr d*y= sec x
LF=P=et=ezr =
eogr' -
Solve: dr OR ay -5 2x sin x2 dr +c
Please refer to 2057 Q.No. 11b
SOLUTON
or, ay 2 Jd(cos x)+c Multiplying equation () bywe get
Here, sinr ay (cosr)y = sinx cosx 69. 2071 Supp. Q.No. 7b
Solve r2 y dr = (r3+ y') dy
or, xy 7cos 2 +c
dy Co2y=cos X (9 sOLUTION
of Here, r? y dr = (x3 + y') dy
dx Sin i
y+ cos r2=c
Comparing equation (i)
with P y =Q we get dy
or d r + y 3 y
cos Q= cos I This is homogeneous
differential equation 1.207/2 SotcQ.No.7 Integrating
P sin I
rcos
Put y = vr
Solve:dy tan x -
2 sin x 4 -logx+ c
I.F =eP =esn d sOLUTION
elog in7
=
Sin I
dV+d
Hence equation (i) becomes, Given equation is
have
Multiplying both sides of (i) by sin x, we
dy
dv V dr y tanr-2 sin x
ay
Sun dr cos X. y
= sin x . cos
ayc-log
or, v + =
OT d r - tan x y = - 2 sin x
) 74 2072 Set DQ.No. 7b OR
or, d (y. sinr) -jsin 2r dr Solve:=Y+1

Integrating we have
or.
Comparing() with dPy =Q we get Solve:dx+y+1
P=-tan x, Q=-2 sin x SOLUTION
ysinx sin 2rdr +c

or,d I.F ePd= etan d= e' elogsx =cOsr Given equation is = T


Multiplying both sides of () by cos x, we get This equation is not homogeneous
or, y sinx or,
OS
dy Puty+1=z
d - cos r tan x y -2 sin x CoS X
-

y sin x+ cos 2x =c or,vdv = Or, d (y cos x) = -

sin 2x dr Then.
Integrating Now, given equation can be written as
66.2071 Set DO.No.7 cos r= ()
y +c
Solve: y d-+ y 14] Integrating we get
Please refer to 2069 (Set A) QNo. 7b OR v log v= -logx+ logc
-3+
y cos = cos 21+ c Put, v
67.2071 SetDQ.No. 7b OR 2 2072 Set Ca.No. 7b OR dy
Solve:d1ty (1+z 19 - gx-bg) Solve: ay dy
d -

yi =
* 41 Thendv be written as
Now, equation () can

Please refer to2069 (Set A) Q.No. 7b OR


Mathematics-l
174 Asmita's NEB Solution of Basic
or, dly (1+x)} = exdx ERENTIAL EQUATIONS
Integratin& we get THEIR APPLICATION
vtd y(1+ x)*e+c e 2xy e-x=x e - 2
i.e. when Mdx +
Ndy is a perfect differential.
Unit 9 175

or. d 78. 2072 Supp Q.No.7b OR xe** dx Second part:


(y e*)=
Given,
Solve d-sin dy e)=d
Please refer to 2070 Set C Q.No. 7b OR Integratin& we get
ord 79.2073 Set CQ.No.7
Or,
ydy-xdy +5dy ydx
Or, xdx+
= -

xdx + dx
dv dv_ -dx ye-xd-5e-*+c ydy xdy ydx
-
-

dx + 5dy =
0
Or, xdx+
or, Solve xd2y x* log x ydy (xdy + ydx)
-

-
dx +
5dy =
0
Integrating, we get 14 y - 5 + cex2
logv v-=-logr+ log c sOLUTION a doxy)- do) +d{5y)- 0
or, logv+ logr-log c = Given, x dy
+ 2 y = x* log x 1. 2073 Set DQ.No.7b OR
vr 1
Dividing both sides by x, we get Solve:x d +y2=xy
or, log
Integrating we get,
ddy 2
y-x log x )
sOLUTION
y
or,
Comparing ) with ddy + Py = Q, we get
Given equation is x d y=xy 22y-x+ 5y
x+y2-2xy 2x+ 10y=C
or, log .) 83. 2073 Supp Q.No. 7b on
or, log
P-Q-xlogx This is homogeneous differential equation.
So, p u t y = vx
d+2y tan x = sin x.
4
I.F= Pdx= e = e2 log x = elog = x? SOLUTION
or, Thon =y + dv
Multiplying both sides of (i) by x*, we get Then,=vt* dx Given equation is
2y tanx =
sinx ()
y+1=ce
75.2072 Set EQ.No.7b
y-x. xlog x Now, equation (i) can be written as

v+Xdx x
dv v2x2 vx
x Comparing ) with Py -
Q
d(x y)= x log x dx we
get,
Solve: sin 4 Integrating we get
xy llog x x* dx oT,txd+ v2=v
P 2tanx, Q=
sinx.
Now, IF = ePt = e2amd
Please refer to 2070 Set CQ.No. 7b OR e2lanx d
76. 2072 Set EO.No.7b OR y og xhe dx - | | o g f d a or, x-V2 edogserx
y
Solve: sin xtcos ry=x sin X. r, dx elogsex
[Using integration by parts in rightside sec?x
Please referto2068 Q.No. 11b OR dx Multiplying both sides of () by secx
7. 2072 Supp Q.No. 7b J
or,
dx
xy log x
=
or, v2 dv =
Secxd dy secx. 2y. tanx = sec-x.sinx
Integrating, we get
Solve:(+1*y1 or, y-log x-hd
or,
or, d(y.secx) = secx tanx dx

sOLUTION Integrating we get


tanx dx +c
or, xiy = y secx= Jsec x
Given equation is (x+1)d +2y-7 O=log x+c y secx secx +c.
y= log x -6+ 84. 2074 Set AQ.No.7b
Solve (xy - x*dy = y<dx
y/x log x+c 41
dy 80.2073 Set DO.No.7 SOLUTION
Comparing 6) with d+ Py =Qwe get dy
Solve: d-2xy x. 4 -log x+ c Given, (xy -x)dy = y2 dx

P7.*TF X=y (logx or


SOLUTION 82. 2073 Supp +c) (i) is homogeneous, so put y
=

IF =ePdx e1d" e2 log (1+) =elog (1* Given equation isdy


d-2xy = x ..0 Defin Q.No. 7 The equation vx
= =

ne exact differential equation hence solve


Then,
-(1+ x dy
y+ xdx
Multiplying equation (6) both sides by (1 + x}, The given equation is in the form+Py = 4 Now, from (i), we can write
we get
where P -2x, Q= x SOLUTION
Exact
vtx v
I.F= P = g d = e Differential Vt*dx x.vx -x2
(1+d (1 +
+1) Y-(1 +*)* +1
Equation:
equatior of the form Mdx +Ndy =0 where M
A differential
Multiplying both sides of (i) by 1.F = e
beare functions of xor y or both, issaid to
dy
or, (1+) d21+)y-e we get exact if there exists a function f(x, y) such
hat: Mdx+
Ndy =d f(y).
Basic
Mathematics-l DIFFERE EQUATIONS.
176 Asmita's NEB Solution of THEIR APPLICAT
Putt Inx Unit 9 177

dt
d dt Then (1) omes,
or,
or d - d t

d vv{y-1) dt dx
Or dx V-1 Then o7, V+xd-
dv y-v2+y
t .et (-dt)
Or,
dx V-1 I= {tdt-2+a 2a Jtet dt
Now, from (i)
or, x -1-3v2
3v
+
dx
x .Inx (Inx)2 +C, where c Ci+Ca --15
--te+ Jetdt]
or,
o7, x
d -2v2
3

36. 2074 Set BQ.No.7b


Solve:(1+x)dxy 1-x. OT,
Please refer to 2070 (Old) Q. No. 11 bOOR =- [-tet - e ]
te-+ e

Integrating 837 2074 Supp O.No.7b Integrating, we get


logx=v- log v + logc
or, log x+ log v - logc=v
Solve: x+y=xy. logx log (v2+1)-log c
Please refer to 2073 Set D Q.N. 6b OR
or, logv
or, logx+ log (2v2 1/4 logc =

88. 2074 Supp Q.No.7b OR Then from (2)


or, logx (2v2+1)/4=c
or, e Solve:cos dy +y-1.
Please refer to 2058Q.N.11b OR
or.xce [:v=y/x] 9 2075 Sat ACINo.7 +ce
y ce/*
35 2074 Set AQ.No. 7b od Solve: tan Xy 1+x*cxe (+2y=ax
Please refer to 2063 Q.No.11b OR 92. 2075 Set BQ.No. 8a OR
xInxdy- 2Inx 141 B81. 2075 Set BQ.No.8a
90. 2075 Sot AQ.No.7b OR d2ytanx=sinx. 4
SOLUTION Solve dx 3xy
Solve: - SOLUTION
Please refer to 2073 Supp. QNo. 7b OR
Given, x Inxd+ y 2 Inx SOLUTION Given, B3 2075 SetCONo 70
Dividing both sides by xnx, Here, Solve: -3{y+)
dx 2xy
ddin Please refer to 2059 Q.No.11b OR
=y OTd-3xy3xy* 342075Sot CO.No 7bOR
Comparing ()withPy-Qwe get Comparing equation (1) with +Py
X s 1s the homogenous equation. So, put| Solve: y+(+1) tan
y Vx.
get Please refer to 2066 QNo. 11 b OR
PinQ Then dvddv
Now, 1.F = erax = ean = ea[put z = Inx]
ge
Multiplying both sides of (1) by e",
= eloy=2 = Inx. we

Multiplying both sides of () by 1.F, we get

Inxdy-Inx
or, dyInx)-Inx dx
Integratin8 we get,
Integrating ... ()
y.e
y.lnx -Inxdx+c
Let1-d
2 d x +C (i)
Put
Let1-d
ISPERSION, cORRELATION AND REGRESSION Unt 10 179

alculation of M.D from Median


UNIT x- Ma= |x-71
7 1

DISPERSION, CORRELATION 10
2 3

AND REGRESSION M.D from Median


=
R-
Mal--22
x- Mal =11

2061 O.No,4
A. DISPERSION ind the mean deviation from mean of the following data: 6,
8, 10, 13 and 5. 12
2 MARKS QUESTIONS SOLUTION
Given data is: 6, 8, 10, 13 and 5
Er = 6+8+ 10 + 13 +5 =42
2058 O.No.44 deviation of the following data: 100, 150, 200, 250,
300
Find the standard n=5

sOLUTION
Calculation of S.D Mean - 84
X
d'
d'
a200)
h50)_ d'2 Calculation of M.D from Mean
100 x-x =x-8.4
24
150
200 0.4
1 1.6
250
300 4 13
4.6
d=0 Ed2=10 3.4
Here, n =S
Ex-x=124
S.D ( ) =?

We have, S.D ()= hx*1 70.71


Mean deviation from mean
= x-.124-248
2061 Q.No.12 a
Find outthe mean and Standard Deviation from thefollowing data:
2059The coefficient
Q.No.4 of variation and mean of a certain frequency distribution are 50.2% and 28 Variable 5-10 10-15 15-20 20-25 25-30 30-35
respectively. Find the s.d. Frequency 2 9 29 54 L11 5
sOLUTION
SOLUTION
Here, C.V= 50.2% Calculation of Meanand S.D
Mean ()=22.8 Profit (in Rs.) Mid value () d'
d' 75)
h5)
fd' fd2
S.D (o) = ?

We have, 5-10 7.5


10-1
15 12.5 1

CV100 20
20-225
17.5 29
22.5
or, 50.222s 100 25-30 27.5
30- 3
32.3_
Or, a 50.2x
=*

100
22.8 -1145 N=110 Sfd'= 78 Efd=160
2ta 78
Mean
11.45
()=a+ h =17.5+ 1ox5=21.05
.2060 Q.No. 4
Find the mean deviation from median ofthe numbers 5,7, 10, 12 and 6. S.D (a) fd2Efd'\
SOLUTION -h*N-(N
Arranging the given data in ascending order

..
5,6,7,10,12
n5
-5x110(
5xV145 0.5028
Median (M)- Value of(2) item Value of item Value of 3rd item 5xV0.9472
= 4.87
M e d i a n (Mi) = 7
DISPE
Asmita's NEB Solution of
Basic
Mathematics-ll
CORRELATION AND REGRESSION Unit 10 181
180 For Firm B

25, 30, 35, 40 600

2062 0.No.4 data: 10, 15, 20, n


i - Rs.575
standard deviation
of the following
Find the 100

SOLUTION Calculation of S.D r= 10

100
10
100 CV(B) =x 575 100-1.74% *

225 cv(A) <C.V (B), the wages


400 distribution is uniform in firmA.
15
ho2070(Old)Q.No.3 C
20 625
25 900
coefficient of mean deviation from median of the data 5, 4,
ind the coeffici
2,8 and 6. 2
30 1225 sOLUTION

Arranging the given data in ascending order


35 1600 2 4,5,6, 8
40
x2 5075 Here, n
5
=

r = 175

n=7 Median (Ma) =Value of ( item


Here,
S.D(o)=?
()-V-y = Value of
Wehave,S.D[o) =1/ - = Value of 3rd item
item

2066 CO.No,4 of variation and mean of a certain frequency distribution are 50.2% and 228
Ma 5
The coefficient
respectively. Find the
standard deviation.
2 Calculation of Coefficient of M.D from Median
Please refer to 2059 Q.No.
4c
z-Ma l |z-51
2066Find OLNo.4
the standard deviation from the following data:

10 12 13 14
3 12 18 12 2

SOLUTION
Calculation of S.D. Ex-Ma8
10
30
fr2
300 M.Dfrom x-Mal--16
n

11 12 132 1,452 M.D from median


2,592 Coefficient of M.D. from median = =0.32
18 216 Median 5 0.32
12 156 2,028
3
4 2 28 392 . 2070 Supp. Q.No. 4b
N= 47 Ex=562 Efr2 6764 Calculate the quartile deviation from the data: (2
15,7, 25, 12, 4, 22, 19, 10
Mean )-N 47 11.96 sOLUTION
Arranging the given data in ascending order
6764 =-y143.91-143.04-V0.87 0.93 4,7,10, 12, 15, 19, 22, 25
SD-1N Here, n= 8
2064 O.No.4E given below: -( item- item (2.25)hitem
The information about the wages distribution of the firms A and B are

Firm A Firm B
2md item+ 0.25
No. of workers 500 600 (3rd item -2nd item
Rs. 75
7+0.25 (10 - 7)= 7.75
Average monthly wages Rs 586
Variance of wages distribution
In which fim is the wages distribution uniform?
sOLUTION
81 100
)m)- 6 h item + 0.75
(7h item -6th item)
For Firm A 19+0.75 (22- 19) = 21.25
500 artile deviation (QD) - 22 675
X= Rs.586
o281
=9
9
CV (A)=x 100 5 100 1.54%
DISPERRSION,
Asmita's NEB Solution of
Basic
Mathematics-l
CORRELATION AND REGRES Unit 10 183
182
ho2072Set DQ.No,
46
4 deviation
and the
coefficient of
ation.
variati.
distribution
In the of data 20,
25,30, 36, 32, 43; find
12 2074 Set DQ.No.
120, 2r*= 1530,
2x=
find the
standard
SOLUTION
starndard deviation.
21
SouI10,
TION
Given, n=10,
r 120,
Calculation of Standard deviation
S.D ()= ? 20
Ex2- 1530
Coefficient of variation (C.V)= 25 400
y)--(V-V153-14-3 30 625
6 900
We have, S.D (o) 32 1296
43 1024
A.M 112 EX 186 1849
C.V. 100*100-25% Here, n 6
S.D (o)= ?
EX=6094

Old Q.No. 4
median.
7 from We have,
3 2071
Find the mean
deviation of the data 10, 5,
6, 12,

Please referto 2060 QNo. 4b c SD () -1015.67-96i -467 7.39


respectively.and the
Q.No.4 and 50.3 standard
14 2071 Supp. of two samples
of size 50 and
100 are 54.1
and the standard
deviation of the samnl
mean sample of 17. 2073 Supp Q.No. 4b
Obtain the
mean
The
deviations a r e 8 and 7 respectively. In the distribution of two setS of data, which of
obtained by combining the
two samples. the distribution is consistent? (21
size 150 DistributionX Distribution Y
sOLUTION 50, nn=
= 100 AM 100
n
Given,
54.1, = 50.3 S.D.
o8, 027
sOLUTION
Distribution X
mtno50x 54.1+ 100 S0 =51.57
A.M 100
Combined mean (1) =

ni1
50 100
S.D 10
Again, S.D 10
n(o+d+ nloj+d) C.V ) A . M * 100 100 100= 10
We have, CombinedS.D (0)-\ Distribution Y
Where, d 54.1 51.57 2.53 A.M 90
d 50.3- 51.57=-1.7
S.D 18
508+(2.53)+ 100(72+(-127 S.D 100 g x 100 =20
50+100 CvAM*
3520.05+5,061.22 =5721 =7.56 Since C.V (X) <C.V (Y), distribution Xis consistent.
150
8 2074 Supp Q.No. 4b
15 2072 Set CQ.No. 4b FOllowing are the information aboutthe marks of two students A and B.
oftwo cities Xand Y are as followS:
Theinformation aboutthe daily temperature
X Average marks 84
Average temp.("F)_ 84 Variance of marks 16

| Variance of lemp. 16 25 EXamine who has got the uniform mark. 2


Determine which city has greater consistency
in climate. SOLUTION
City A
SOLUTIONN
City X Average (X) = 84
Average (X)
=
84 Variance (a) = 16
Variance (o) = 16 G4
4
CA)x 100 R100- 4.76%
C.VX) 100-4100-4.76% City B
City Y Average (Y) =92
Average (Y)
=
92 Variance (o)= 25
Variance ( ) = 25 g-5
O5
CV.(B) x 100-100 5.43%
cvm-100 100=5.43% Since C < C.V. (B), the city A has greater un
Since C.V ( ) <C.V (Y), the city X has greater consistency in climate.
Mathematics-il

184 Asmita's NEB Solution


of Basic
13PERSION, cORRELATION AND REGRESSIOT Unit 10 185

10, 13, . 2068 Q.No.12


data: 6,8,
19. 2075 Set B Q.No. 4a of the Weights of group
of aa gre of individuals are
the
mean
deviation from mean

deviation.
given below. Find out the mean and the
nd
Please refer 2061 Q.No. 4b
x) 1530, find the
(x) Welght (in kg) 0-10 stand
sum ofsquare
o t items

12
10-20
33 20-30 30-40 40-50
[41
20. 2075 Set CQ.No.46 sum of items (Zx)
120and the
= FrequencCy
30 15 10
1 0 , of v a r i a i o n sOLUTION
and coefficient
standard deviation
Set D Q.No.
4b Calculation of Mean and S.D
Please refer to 2071 Weight Mid value (x)
(inkg) Frequency ( d2a(25)
fd' fd2
&MARKS QUESTIONS 0-10 12
h(10)
10-20 15 33 -24 48
21 2060 O.No.12 and S.D.from the followingtable:
10-20 10-3010-40 10-5010-60 20-30 25 30
-1 -33 33
Find the mean 63 83 100 30-40 35
45
15
15 15
Wages (Rs. 15 33 40-50 10
20 40
No. of workers N 100
S.D Efd'=-22 fd2 136
SOLUTION
Calculation of Mean and fd' d2 Efd' (-22)
No. of
-a(35
d h10)
Mean )a+ N*h=25+100x 10=25-22=228
Wages (Rs)
Mid value()| workers() -30

10-20
15
33-15 18
-1
-18 18
0
)-h*N-(N10n/100- =10136-0048-1013116-1n45
20- 30 20 20
35
63-3330
68 24 2069 (Set A) Q.No. 8a
30-40 83-63 2 0 34
40-50 Determine the standard deviation and the coefficient of
variation from the
100-83=17 Efd'= 6 I2fd2=166 distribution._ following
50-60 4
N=100
Profit (in Rs.) 0-10 10-20 20-30 30-40 40-50
2fd".
Meana+x-35+100 10-356 No. of shops 8 13 16
sOLUTION
Calculation of S.D and C.V
sD)-h NNN - 1 0 /1 Profit Mid value () No. of shops d ' a 2 5 td" fd2
in Rs.) ()
=1287 h10)
10x1.66-0.0036 =10 x1.6564 -10
-16 2
10-20 15 13 13 13
is not less than the
22 2067 Q.No. 12b Also prove that the
root mean square
deviation
20-30 25 6
standard deviation.
Define 30-40 35
standard deviation. 40-50
0 20
sOLUTION N=50 Efd'--11 fd'"= 73
Standard deviation
root of the m e a n of
the square of the deviaions 2td
defined as the positive square A.M= a + x h 25+ x10 25-22 Rs.228
Standard deviation is it is given by
denoted by Greek letter o (sigma),
taken from the arithmetic mean. It is

o= orindividualseries
S.D () =hx
1 0 x 1 . 4 6 -0.0484 =10 x 1.4116 = Rs.11.88
(
= for continuous and discrete series, C.V-x 100 Rs.1188
Rs.22.8 100 52.11%
Next part
We know that root mean square deviation is defined by: 2069na the
(Sotmean
A) Oldand
Q.No.
the12
standard deviation from the following data.
where is an arbitrary number,.
a
Marks L0-10 10-20 20-30 30-40 40-50
15 16 6
No. of
students 5
Now,
OLUTION
M - p alu-)+G-a)--p+26-a)( -9+6-"" Marks
Calculation
Mid value () | No. of students()
of Mean and S.D
X-a(25)
fd fd'2
d h(10)
-o 2- - 6-aP N-g+2-a) -+6- 0-10
0

10-20 15 0
= +-a) 20 30 15 16
Since, (r- a)? 20, we have
30-40 25 16
35 12 24
s 2 o2 50 2fd'--10 2fd=68
s20 N 50
Hence, the root mean square deviation is not less than the standard deviation.
Mathematics-l
Basic
186 Asmita's NEB Solution of ON, CORRELATION AND
BSKEWNESS
REGRESSiON Unit 10 187
10
Mean a + h 255 2 MARKSQUESTIONS
xa.
h*/N-(N10
*1/s0-(50 =10 *yi.36-0.04 10 132 11.49 2063 Q.No.4D
Consider thefollowing distributio
data:
26. 2069 0ld (Set B) Q.No. 12
and standard deviation
from the
following
Arithmetic mean: Distribution a Distribution b
ind outthe mean 100
90
X 1 0 |11 12 Median: 90
3 1 2 18 12 Standard deviation 80
Is the distributio
10
tion A same as the distribution 10
B
sOLUTION Calculation of Mean and S.D. regarding the skewness ess?
fr2 SOLUTION
For distribution A
2
30 300 i=100, Ma = 90, o = 10
132 452
11 216 2,592 We have, Mo 3Ma-2 =3 x
90-2* 100 70
12 156 2,028 10
13
2
28 392 CVA) x 100100x 100 10%
14
N 47
Xfr 562 Efx2 6764 - Mo 100-70
S(A)= 10
For distribution B
Mean-N711.96
i 90, Ma= 80, o 10 =

V47-(11.96)=V143.91
-143.04 =Vo.87 =0.93 We have, Mo =3Ma -2r =3 x
80-2x 90 240- 180
SD(0V-@ 10
60
CV(B) x 100 o0x 100 11.11%
27 2074 Set BQ.No. Ba tests.
Following arethe marks obtained by thetwo students
in 6
69 6 64 81 S(B 10
A 56 72 48
B 633 57 82 63 Since C.VB)> C.V(A), the degree of variation in
74 in tests? S(A) S(B) ie. distribution A is same as
= distribution B is
greater that that of A but
Which of the student will get performance award for the consistency the distribution B regarding to the skewness.
SOLUTION .2067 Q.No. 4
In frequency distribution
Student A Student B a
of a set of data C.V. =
5%, a =
2 and karl Pearson coefficient of
skewness =
0.5; find the mean of the data.
Marks (X) Marks (YY
56 3136 63 3969 SOLUTION
5184 74 5476 Here, C.V 5%
48 2304 45 2025 2
4761 57 3249 Karl Pearson coefficient of skewness
64 4096 32 6724 Mean () = ? Si(P) =
0.5

3969 We have,
81 6561 63
EX=390 |2X2 26,042 EY 384 EY2 25,412
Here, n = 6
CV-x 100
2
For Student A or, 5
x 100
A. M(X)- or, 5x =
200
:40
26042 2071 Supp. Q.No.
SD -(x = -(65 =4340.33-4225 -115.33 10.74 t 4cC
3 tribution, the difference of the two quartiles is 20 and their sum is 70 and the median is
CV(A) Mean100- 100-16.52 % o. Find the coefficient of skewness.
ForStudent B SOLUTION 2
Given, Qu-Q 20
A.M (Y) 64 Qs+Qi 70
Ma 36
SD-1/(Y =\/-(64 =V4235.3-4096 -139.3 11.8 Coefficient
We have, of skewness Si (B) =?
C.V(B) Meanx 100 100-1844% S)QQ
Q-Q12Ma _70-2 01
Since, C.V (A) < C.V (B), A will get
performance award for the consistency tests.

N
Mathematics-ll

Solutlon of Basic DISPERSION, CORRELATION AND


188
Asmita's
NEB
REGRESS Unit 10 189

Q.No.4 150 and Mo 1.75,find the


find the Pa.
Pearsonian coef 100
or, 5= Mean
600, 2*=
31.2072SetE of50items; circle 2*
=

Mean
= 200
or,
5
Fora group
skewness. Mean-240
Xx 600
sOLUTION
Skewness (Sx) ean- Mode 40
Coeficient of S.D 9
=
Given, n =50, Mo 1.75
0.5
r = 150,

50 3 Set AQ.No,46
6 2075roup
We know that of 50 items, 2X =600, Zx
For a =
150 and Mo =1.75, find the pearsonian coefficient of
-32-12-9-1.73 skewness.
-1/ Please refer to 2072Set E Q.No. 4b 2
SD( ) /
Pearsonian
coefficient of
skewness
6 2076 Set BQ.No.4bE
ed the Pears on's coefficient of skewness when Ex
S(P= 178 0.72 =735, Ex =
28730, mode 35.25,
=
n =
20. 2
1.7
sOLUTION
5%, Ma Given,
2073 Sot CA.No.4
results. C.V. =

distribution gives the following of the distribution.


40 Ex 7 3 5
A frequency Pearson's
coefficient skewness of Ex? = 28730
Calculate Karl
Mode 39. Mode (Mo) = 35.25

SOLUTION 2 n 20

Mean ()=40 Mean ) n20 3536.7


36.75
Mode (Mo)=39
CV 5%
Karl Pearson's
coefficient of skewness (S (P)
=
? sD(o)--- 20
-
(36.75) =y14365 1050.5625 85.9375 9.2
-

Pearson's coefficient of skewness,


We have,
S (P) X- Mo36.75-35.25
9.27 0.16
CV *100
Mean
4MARKS QUESTIONS
, 5 40100
5 x40
7 2061
The
Q.No. 12 b
or, S.D 1002 median, mode and coefficient of skewness for a certain distribution
15.3 and 0.35. Calculate mean and C.V.
are
respectively 174,

S.D()=2 SOLUTION
Given,
Again, we have, S(P) = -05 Median (Ma) = 17.4
Coefficient of skewness (S) = 0.35
Mode (Mo) =15.3
Mean () =?
33. 2074 Set AQ.No. 4 1240
C.V=?
59.50 and
Find Skewness and CV. if mean, median and S.D. are respectively 56.80, We have,
SOLUTION Mo 3Mu -2
or, 15.3 3 * 17.4 - 2
Given, mean (x)= 56.80
Median (Ma) = 59.50
or, 2r 52.2- 15.3
or, 2r 36.9
S.D ()=1240 36.9
We have,coefficient of skewness (S) =* 3%-M) 3(56.80- 5950)= 0.653 -
= =18.45
12.40 Again, we have
S.D * 100 568
Again, we have,C.V Mean 12.4 100-21. 83% S

4 2074 Set BQNo.4b or, 0.35-8.45-15.3


The C.V, S.D. and mode of a distribution are 5%, 2 and 39 respectively, a or, 0.35
Pearson's coefficient of Skewness of the distribution.
a =
3.15
Org 3.15
SOLUTION 0.359
Given
C.V. =5%
S.D = 2
C.V-100 R 10= .78%

Mode 39
We have, C.V Mean * 100S.D
Asmita's NEB Solution of Basic Mathematics- ON, CORRELATION AND REGRESSION
190 Unit 10 191

Mean )a+N*h =45, 10 36.25


80
38 2064 Q.No.12a
452, 2
= 24,270 and
mode =
43.7, find the Pearson's son's coefficient of
coocc:

Er =

group of 10 items,
For a
skewness. SD()-h*1
h#N(N) 10 105-07656-10-y244- 14.
sOLUTION Here, thighest frequency is 20. The
Given, n =10 x = 452 is 30-40
class
corresponding class to this frequency is (30 -

40).
r 24270
Mode (Mo) =43.7 Somodal
f 20, fo
=
18, fa- 15, h 10
30,
Mean - -45.2 Mode(Mo)+2-fo-6xh 30
302x -18
20-18-15 10 305 10 =32.86
V2427-2,043.04-y383.6 -
19.59

sD)- -1/6(45.2) Coeficient ofskewness, S(P)


=Z-Mo 36.25-32.86
14.95 0.227
skewmess
Pearsonian coefficient of

Mo_ 0.077 2068Q.No.12


T.Consider the follov distribution:

b9. 2065 QNo 12 data: Arithmetic m e a n


Distribution
100
A Distribution B
coefficient of skewness of the 90
Calculate Karl Pearson's
above 20 above 30 above 40 4 90
above 0 above 10 Median
80
Marks
150 140 100 80 80 Standard deviation 10
Frequency 10
the distribution A same as the distribution B
regarding the of variation and
sOLUTION
Calculation of Karl Pearson's Coefficient of Skewness skweness?
degree
Mid value
f d'=*
- a(25 fd' fd'2 sOLUTION

Marks h(10) For distribution A


150 140= 10 -20 7-100, Ma=90, o =
10
0-10
-20
140 - 100= 40 -1 -40 0 We have, 3Ma-2x =3x 90-2x 100 70
Mo
20 30 25 100-80 20 10 100 10%
CVA) =x 100 100x
80-80 0 0 0
30--40
160
40- 50 45 80 320 -Mo100-70_3
N 150 Zfd'=100 Efd2 =400 S(A) 10
For distribution B
Mean a h 25+ 10 31.67 T 9 0 , Ma=80, a = 10

We have, Mo=3Ma- 27 =3 x 80-2x 90=240 -180-60

SD)=h */ N 10*/150-(150-10 xy267-044 10x223 14.9 CVB)x 100 90 100


10
11.11%
Here, the highest frequency is 80. So, modal class is (40- 50)
I=40, f 80, fo 0, f 0, h 10
= = =
S(B)=T-Mo 90-60 10 3

-fo
Mode (Mo)=l+2h-
80-
xh=40+2x80-0-0* 10 45
Since
C.V(B) > C.V(A), so the degree of variation in distribution B is greater that that of A but
Sa(A)= Sx(B) i.e. distribution A is same as the distribution B regarding to the skewness.
Karl Pearson's coefficient of skewness
SP)-M31.67-45
2 2069 (Set A) Old Q.No. 12
0.89 or a group of 10 items, r = 452, L = 24270 and mode = 43.7, find the Pearsonian coefficient of
14.93
Skewness. 4
40. 2066 Q.No.12 b " Please refer to 2064 Q.No. 12a
Calculate thecoefficient of skewnessfrom thefollowing frequency distribution: 14
2069 (Set B) Q.No. 8a
Investment 10-20 20-30 30-40 40-50 50-60 60-70 70-80
Frequency 12 18 20 15 10 3 2 Zr 110, Efr2
standard deviation.
=
1650, N =
10 and Mo =
1245, find the skewness based on mean, mode and
4
sOLUTION OLUTION
Calculation of Coefficient of Skewness Given, 2fx = 110
d'af45) fd'
Investment Mid-value ()
h(10)
fd' fx2 1650
N 10
20 2 -36 108
20-30 18 -36 72 Mo 12.45
40
40-5
35
45
20
15 0
0
0
20
0
Mean 1011
60
60-7
55
65 3
10 10
12 SD(9)V N - - / 1 0 - 1 =y165-121 -vA =663

70 80 75 6 18

Efd-70 2d'=240 Coefficient of skewness -Mo11-1245 = 2


N 80 S(P)=* 6.63
Asmita's NEB Solution of
Basic Mathematics-ll
192 DISPER
S1ON, CORRELATION AND REGRESSI Unit 10 193
44 2070Set CQ.No.fa3
8a 10 and M,
= 12.45, find the
skewness based on
mean, mode and sOLUTION

110,
2 fx deviation.
fstandard
=
=
1650, N =

(inRs)
culation of
No. of persons (Coefficient of Skewne
Wages ()
Please refer to 2069 Set BQ.No. Sa 100 fx
110 6 200
45 2070 Set DQ.No.8 20,000
120 10 660
72,600
Consider thefollowing distribution.
Distribution A_ Distribution B_ 130 8 1,200
144,000
100 90 140 1,040 135,200
Arithmetic mean 80 N 30 560
78,400
Median 90 0 Efr= 3660
10 122 2fx= 450200
Standard deviation the degree of variation and skou. Mean - 30
the distribution B regarding
Is the distribution A

SOLUTION
same as
ewnes Here the highestfrequency is10,mode (Mo) = 120
For distribution A
Efx2 --1/450200
= 100, M= 90, o
=
10
90-2x 100= 70
sD)VN 122
=15006.67-14884 V12267 11.08
ofskewness S.(P) =*Mo 122-120
-27 =3 x

We have, Mo= 3M Coefficient 0.18


10 10%
11.08
=x 100 100x 100
2072 SuppQ.No. 8a
CVA)

S(A) M _100-70 slate the coefficient ot skewness


Calculat
based on mean, mode and the standard
10 following data: deviation from the
For distribution B
= 9 0 , Mu = 80, a = 10
Wages (in Rs.) 100 110 120 130 140
We have, Mo
3Ma- 2 =3 x 80 - 2x 90 240- 180 60 No. of person 2 6 10 8
Please refer to 2071 Set C Q.No. 8a
C.V(B) =x 100 100 11.11%
o 2073 Set DQ.No. 8a
S(B)= I-M9 0 -10
60
TE T=110, Efx2 1650, N = =
10 and Mo= 12.45, find Karl Pearson's coefficient
3
Please refer to 2069 Set B Q.No. 8a of skewness. 4
Since C.V(B) > C.V(A), so the degree of variation in distribution B is greater that that of A bul
i.e. distribution A is same as the distribution B regarding to the skewness. CCORRELATION
S(A) =
S(B)
4s. 2070Supp. Q.No.8a 2 MARKS QUESTIONS
Find Karl Pearson's coefficient of skewness from the given data
14 16 .2057 Q.No. 4 b
income 10 12| Calculate r,y if Er3= 114, 2y2 442: Exy = 174.
CA
requencY 5 8 15 SOLUTION
21
SOLUTION Here, Ex2 = 114
y2=422 Exy 174
Calculation of Karl Pearson'sCoefficient of Skewness Correlation coefficient (r) = ?
Income(x)_ Frequency (f)_ tx fx2
Eay
10
12 6
500
1,152
We have,
VEVy V114422 0.79
14 5 210 2,940 E 2065 Q.No 4 b
16 112 1,792
20 100 2,000 the covariance between the variable x and y is 18 and the variances of
respectively, find the coefficient of correlation between them.
x and y are 16 and 81
[21
N= 40 Efx 568 Efx2=8384 SOLUTION
oven, Covariance between X and Y = Cov(X, Y) = 18
Mean- - 1 4 2 Variance of X = var (X) = 16
Here, the highest frequency is 15, so mode (Mo) = 14
Variance of Y =var (Y) = 81
Coefficient of correlation (r) =
S.D ()= 8564( - 209.6-201.64 y7.96 2.82
We have, r cov (X, Y) 18 18 0

yToy8x9
Karl Pearson's coefficient of skewness S.(P) = 282
0.2
282 0.071
2068 Q.No. 4bVVar (X)Vvar()
47 2071 Set CQ.No.8a
Calcu correlation coefficient between two variables
from the following data:

standard deviation
x2x11e
114, 2y? 422 and
21
ir
=
174
mean, mode and
Case refer to 2057 Q.No.Exy4b
Calculate the coefficient of Skewness based on =

following data.
Wages (in Rs.) 100 110120 130 140 2069If the (Set A) Old Q.No. 4
covarian
and y is 18, and the
variances of x and y are 16 and
No. of persons
2 6 10 8 4
Tespect
dse aely,
lease
between the two variables
find the coefficient of correlation
x
between them.
121
refer to 2065
Q.No. 4d
Mathematlcs-ll DISPERSIO CORRELATION
194 Asmita's NEB Solution of Basic AND REGRESSION Unit 10 195
Coefficient of correlation (r) ?
54 2070 Set CQ.No. 46 cOv (X, Y 8
we have, ryvar (X) Vvar (Y) 168T 49 05
IfE (X- X )3 40, X Y- Y} =
=
63 and 2(X
-
X )(Y -

Y) =

correlation coefficlenu
35, find the correlation .

between the two variables. QNo.4 4


2068
blculate the correlation coefficient between two
sOLUTION 422 and Ery 174 variables from the following data:
s = 114, 2y
X)OY- Y) =35 referto 2057 Q.No. 4b
X - R)*=40, Y0-Y)}==? 63, S(X 12
-

Please

A) Old Q.No.4
Given
of coefticient (r)
Correlation
a2002069 (Sotariance
X- NY- 35 between the two variables x
We have, correlation coetficient (r)=* V40V630.697 Ifrespectively, find the and y is 18, and
the variances of x and y are 16
VX-)*VY- Y coefficient of correlation
1
Q.No. 4b between them. and
Please reter to 2065 [2
s5 2070 Set DQ. No.4b Q.No. 4G
580 and XY 415, find the correlation coefG. 2070 Set C
=

60, X =
400, 2Y2 =

Ifn 10, EX =
60, 2Y
=

icient
between the two
variables. X
IfE (X-X) 40, Y P =

2Y- =
63 and
Z(X -

XJY Y)
Q.No. 4b 35, find the correlation
-
=
variables.
Please reter to Model Set l1,
the w o
between
coefficient
[2
& 2071 SetCQNo. 4 find the correlation coefficient
sOLUTION

Ifn 3.2
=
15, o,
=
o 3.4 and E(X - X)
= (Y- Y= 122, h
between
the two variables.
Given, (X - XJ =40, 2 (Y-YP=63, Z(X -X)0-Y)- 35
sOLUTION Correlation of coeffident, (r) =?
Here, n
=
15 o, =3.2 oy =3.4
We have, correlation
correlation coefficient
coetncien (t)= XY- ?
S(X- X) Y- Y)=122
correlation coefficient(r)=2
Y-Y =0.75 zX-VEY ¥V40y6
63 -
0.697

n ox Y
x 3.2 x3.4 42070Set DQ.No.4 D
TE n =10, EX =60, 2Y =
60, 2X= 400, 2Y =
580 and XY
S7 2072 Supp Q.No. 46 between the two variables. =415, find the correlation coefficient
Ifn 10, X =
18, IY =
25, X? =
90, EY =
120 and EXY =
65, find the correlation coefficient
Please refer to Model Set I, QNo. 4b
=

between two variables.


65 2071 SetcQ.No.4 D
sOLUTION Ifn 15, =
3.2, ay 3.4 and
Given n = 10, IX =18,
=
o,
the two variables.
2X - X) (Y -

YY= 122, find the correlation coefficient between


Y 25, X =90, [2
X Y = 65
sOLUTION
Y 120, Here, n= 15 , =3.2 o, =3.4
Correlation coefficiernt (r) = ?
X(X- X) (Y - Y) = 122
We have,
10x65-18x25 2000 . 3 5 We have, correlation coefficient (r) = 122
nEXY- XIY
24x y575 nox OY 15x3.2x 3.40.75
n X (LXPVnY2-(EYP y10x90-18 V10 120-25 5. 2072 SupPp Q.No. 4b
2073 Set DONo 4
If x-xP = 40, Ey- y¥ = 63 and Zx- x)(y- y) = 35, find the correlation coefficient btween the
fn = 10, 2X 18, 2Y
between two variables.
= =
25, 2X =
90, 2r =
120 and Y =
65, find the comelation coeficient

two variables x and y.


12 SOLUTION
Q.No. 4b Given, n= 10, 2X = 18,
Please refer to 2070 Set C
Y 25, EX2 = 90,
9 2057 ONo, 4
Calculate Iy if Ex?= 114, Zy2- 442; Ery 174. 2 Y2 120, EXY 65
SOLUTION Correlation coefficient (r) =?
Here, x 2 = 114 y 2 = 422 2xry = 174 We have,

Correlation coefficient (r) =? nEXY-EX Y 10x65-18 x25 200 =0.35


24xV575
We have, r
174 = 0.79 2 X - (2X)*Vn 2Y2- (EYP V10x90-18V10 120-25
Ey 114422 2073 Set DQ.No. 4b
60. 2066 Q.No 4 If 2(x-5
x)(y- y) =
35, find the corelation
coefficient between the
If the covariance between the variable x and y is 18 and the variances of x and y are 10
a wo 4 0 , E(y- yp =
63 and 2(x- 121
respectively, find the coefficient of correlation between them. variables x and y.
lease refer
sOLUTION
to 2070 Set CQ.N0. 40

Given, Covariance between X and Y = Cov(X,Y) = 18


Variance of X = var (X) = 16
Variance of Y = var (Y) = 81
Mathematics-ll
Asmita's NEB Solution of Basic
196 DISP CORRELATION AND REGRESSION
COLUTION Unit 10 197
4 MARKS QUESTIONS Calculation of Karl | v=Y- B(127)
(X)Blood Pressure (Y)| u=X-(48) Pearsoz
88 2059 Q.No. 12
Age Coefficient of Correlation
from the following data: 115 -25
Calculate Karl Pearson's coefficient of correlation 4 127 12
V4 uv
625 44
12 123 5
300
6 L 140 20
0
25 16 20
145
sOLUTION of Correlation 70 22 400 169 260
Calculation of Karl Pearson's Coefficient Y2
118 -20 484 324 396
AY 28 121 -13 400 81
196 180
144 168 120 -22
-6
169 36
2 64 78
S1 72 484
64 48 Here, number ot items (n)=
Zu-43 2V=-7 Eu 2587
49
154
81 Ev2 =
819 uv =1388
100 90
10 We have,
121 121 121
144
12 169 156 Correlation coefficient (r) nuv- u Ev
49 9 21
VnEu- (Zu? VnEv2 -

(v
EY = 63 X= 728 EY2 = 6511 EXY =676 8x 1388-(43) (-7)
X=70 items (n)= 7 v8x 2587-(43) 8x 819-
Here, number of
nEXY- EXEY 7x 676-70 x63 10803 (-7P
coefficient (r)=
We have, correlation VnEX-(x)P VnIY2-(EY)? V7* 728-702 7 x651- V18847 V6508 0.98

322
V196 5 8 8 0 9 5
2070 (Old) Q.No.12b
ampute correlation and interpret about the
Age of husband ages of husband
and wife given below
23220 4
9 2063 Q.No. 12
Fromthe following table, calculatethe coefficient of correlation by4 Karl Pearson's method.
Age fo wife |20 18 23 2423 26 27 28 30 202021 228 2I
6 2 10 SOLUTION
9 11 8
Age of husband
Computation of Correlation Coefficient
Age of wife (Y)
Arithmetic means of X and Y series are 6 and 8 respectively. 4 XY
SOLUTION 23
529
Let the missing item in Y-series be 'a'. Then, 400 460
22 18 484 324 396
20 23 400 529 460
24 20 576 00 480
or, 8 211+a+8+7 23 21 529 441
5 26 21
483
676 441 546
or, 40 3 5 + a
22 729 484
=5
594
a 28 24 784 576 672
Now, Calculation of Karl Pearson's Coefficient of Correlation 30 25 900 625 750
20 26
36 31 54 2X = 243
400 676 520

22
EY= 220 EX=6007 EY 4896 XY 5361
121 Here, number of items (n) = 10
10 100 25 50 We have,
8 16 64 32
56 Correlation coefficient nEXY -XEY
8 64 49 (r)
2X =30 EY= 40 EX2= 220 EY2=340 EXY 214 nEX- (EX)PVnEY-
Here, number of items (n) = 5 10 x5361-243 * 220
We have correlation coefficient (r) nEXY-EX YY 5x214-30x 40 10x 6007-2432 10x 4896 -220
150
ynEX-(X?VnY-(2P y5 220-30 5x 340-40 = 0.198

-130 V1021 V560


V200 V100=092 2071 Set DQ.No. 8a
Pearson's correlation coefficient between the two variables height (in cms)and
70 2066 CQ.No. 12 a weiobharl
Calculate the Karl Pearson's coefficient of correlation between the age and blood pressure d of 8 eght (in kg) from the data given below
patients: Height 160 162|165 161 163
Age 23 48 43 68 70 28 35 26 Weight_ 63 62 64 60 61
Blood Pressure 115 127 123 140 145 118 121 120
Asmita's NEB Solution of
Basic Mathematics-l DISPER SION,
198 CORRELATION AND REGRE Unit 10 199
DQ.No. 8a
sOLUTION Coefficient 6 2072 Sot
Calculation of Correlation u pefine correlation. Find Karl
correlation. Find Karl
Pearson's coefficient of
UV distribution. correlation of the marks of the
Height (X)9 Weight ) u
X-(163) Y-B(62) following
4
160 0

162 sOLUTION

165 2 4 stical measure


tatistic
that describes the
161 50 0 degree of relationship
elation. Two variables are said to be correlated between two variables is called
163 E2 10 2uv=5 change in the value of the other variable. when the value of one variable changes with
2 u = -4 2v0 I 2u2 18 tne

n =5
Calculation of Karl Pearson's Coefficient of
Here, number ofitems,
We have,
uX-A(40)
-20
v=Y-B(30)L Correlation
n2uv-u v_ 20 20 uv
46 -10 400 400 -400
Correlation coeficient()nu-(u ynEv-(v}* 25
30 16 100 256 -160
5x5-4x0 =0.41 50
24 10 0
V7450 -6 100
V5x 18-425x10-0 20 -22
6 60
2u =0 400 484 440
12 2v8
a 20710ldQ.Nocoefficient of correlation between x and y series from the following data
Here, number of items (n) = 5 2ui =1000 Ev2=1176 uv=-1060
Calculate the We have,
Series x Series y
15 15 Correlation coefficient ) nEuv- Xu Ev
No. of observations
s.d 3.01 3.03 VnEu2-(2u)? VnEv?-(EvF
r - ) - y)=122 5x (-1060)-0x18
SOLUTION 51000-05x 1176-(8)
Here, n= 15 5300 -0.98
Ox3.01 V5000 5816
oy 3.03
6 2073 SetCQ.No.8a
x - )(y-y)=122 Calculate Karl Pearson's coefficient of corelation
from the
Coefficient of correlation (7) = ?
formula. following data using product moment
122
We have, correlation coeindent n- yox
-oY9 ,15
x3.01 x3.03
0.89 0

4 2071 Supp.Q.No.8a sOLUTION


Using the product moment formula, calculate corelation coefficient for the following seriesof Calculation of Karl Pearson's coefficient of correlation
ages of husbands (X) and wives (Y. Y
41 4 45 48 40 42 44 x=X- X(10) y= Y- Y)
22 24 25 27 | 21 22 2 3 2 2 3
9

SOLUTION 8 -2
Calculation of Correlation Coefficient 0 9 0 0 0
X Y XY 1 10
1618 484 902 1 1
EX =50 EY =
45 x 10 y=20
4 24 1936 576 1056
Here, n = 5
xy=14|
45 25 2025 625 1125
27 1296
48
40 21
2304
1600
729
441 840 - 10,F 9
12 22 1764 484 924 Dy using product moment formula, we have,
23 1936 529 1012 Exy 14
EX=304 2Y = 164
EX=13246 EXY =7155 Correlation coefficient () = 0.99

Here, number of items (n) = 7


2y =3868 = EY 1020
7. 2073 Supp Q.No.
We have, correlation coefficient (r nEXY-EX EY Find 8a
n2X-(ExP VnEY- (EY the Karl Pearson's Coefficient ofcorrelationform the following distribution. 4
7x7155-304 x164 X10|11 18 15
7x13246-3042 y7x 3868 -1642 Y9 14 15 |16
= 229 = 0.976
306 180
of Baslc
Mathematics-ll DISPERSION, CORRELATION AND REGRESSIOI Unit 10 201
200 Asmita's NEB Solution sOLUTION

SOLUTION Iculation of Correlation


Calculation of Karl
Pearson's Coefficient
of Correlation
XY Coefficient
81 90 Y
100
9 196 19
10
121 154 21
14 1
11 225 270 16
5
324 9 4
18 256 240 25 15
16
225 16
36
15 625
484 550 Ex 20 Ly= 20 Lx?= 100 24
25 22 400 400
Here, n =5 Zy2=90 Exy- 74
20 400 We have,
20 324 252
18 196
14 576 528 Correlation coefficient () -
484
22 24
Ex2=2475 ZY2 2542 EXY =2484 nEx-(ExF VnEy-(yP
Zx=135 2y= 138 5x 74-20 x20
Here, n=8
nXY-EXY 8x 2484-135 x138 5 100-(20 5x90-(20)
We have, correlation coetficaent()VnX-(EX)?nEY2- (2Y? V8
x 2475
(135)* V8 2542-(138y
- x
1050 -042

1242 =0,87 D.REGRESSION


V1575 V1292
2 MARKS QUESTIONSS
78.2074 Set AQ.No. 8aa table calculate the correlation coefficient by Karl Pearson's method. AM af
From the following B0. 2069 (Set A) Q.No. 4b
Find the regression equation of y on x when:
X=15. 16 14
10 20 Ex 15, Ey =
25, Lx=55, 2y2= 140, Zxy =78, n=5.
14 16
SOLUTION
Given, Er = 15 y 25
SOLUTION 2x2 55 y 2 140
series.
Let a be the missing item in X Xay = 78 n = 5.
16+ 14 72+a
Then, Ex =10+ 12+ 20+ a+ Now,
n =6
A.M of X (K) =15 =3
We h a v e X ) -
5
or, 15 byx n2y-ExEy_5x5x 78-15x25 50 15 0.3
nEx-(x) 55 -152
or, 90=72 + a
Theregression equation of y on x is
y - y = bys ( T - x)

a90-72 18 or, y -5= 0.3 (x 3) -

or, y -5 0.3x - 0.9


Computation of Correlation Coefficient or, y = 0.3x +5-0.9
X X XY
90
y 0.3x + 4.1
9 100 81
10
144 144 144 81 2069 (Set B) Q.No. 4
12 12 heregressioncoefficient of y on x is 0.32. If the arithmetic means of x and y series are 42 and 36
225 300
20 15 400 [21
324 324 respectively, find the regression equation ot y on a

18 18
14
324
256 196 224 SOLUTION
16 224 egression coefficient of r on y (bay) = 0.84
16 196 256 Negression coefficient of y on x (by») = U.3-
14 EXY = 1306
ZY = 84 EX 1420 EY2=1226 A.M of r-series () = 42
EX=90 A.M of y-series (y)= 26
Here, n 6
neregression equation of y on Is
We have, Correlation coefficient y
6x 1306-90x84 2760 . 7 8 -y bys (t-
1)
=

nEXY2-EXEY ,y-26 0.32 (* -42)


VnEX-(EX)*VnEY2 (2Y)? 6x1420 -

(90) V6 1226 (84) 420 300 y 26 0.32r 13.44


Or,
4 y=0.32r+ 26 13.44
79. 2075 Set AQ.No. 8a and y from the following
da y 0.32x +12.56
Find the correlationcoefficient between the two variables
x
DISPERS CORRELAT
AND REGRESSION Unit 10 203
202 Asmita's NEB Solution of Basic Mathematics-ll
Q.No.8a
. 20728etEression
4 MARKS QUESTIONS
The re ofraand ycoefficients
means
of x
serles are 42 andon26y
and y on x are
0.84 and
0,32 respectively. If the
age on weie respectively, find two equations of lines of arithmefic
82 2070Supp.O.No.8a OR of regression
for estimating
and SOLUTION

regression. 1
of
ReBon coefficient of yxon
the line
Wing data,
compute Regression coefficient on y (b.,) =

estimate th or (by»)
estimate the most probable age of a weight
x =
0.32
30 45 50 60 (r) 42
=
1-series
Age () 5 15 A.M of
5545 A.M of y-series (y)
26
10 35 50 65
=

Weight (y) The regression equation of y onx is


SOLUTION Regression of Xon
Y byx (r- *)
of Lineof Y2 y-y=
y-26=0.32 (r - 42)
Computation
Weight (Y 50
100 = 0,32x -

13.44
Age (X). 10 525
1225
2500
or, y-
26
+ 26 -

13.44
35 1500 or, y0.32X
15 4225 y0.32x + 12.56

30
50 2925
3025 Again, the regression equation ofr on is
65 2750 2025 r-X=by (y -y)
50 2700
50
15
EXY 10,A50 EY2=13100 or,
I-42=0.84 (y 26) -

EY=
260 - -42- 0.84y -21.84
2X 205 Or
0.84y 42-21.8
+
Here, number of items (n)6 or, x=
+ 20.16
or, x=0.84y
- n
-34.17 X=0.84y + 20.16

.Required equations of regression lines are y =0.32r+12.56 andx


F- n -43.33 =

0.84y +20.16
10450-205 x2609400.85 2074 Supp O.No.8a
nEXY-X Y_6x
6x 13100-(260)2
11000 From the following pair of regression equations, find the correlation
And, b nEY2 (CYof X variables x and y. coefficient between the two
on Y is
The regression equation 4x-5y +33=0 and 20x-9y 107 =0.
X - X=by (Y- Y)
or, X- 34.17
0.85 (Y 43.33) -

SOLUTION
36.83 Given lines are
or, X-34.17=0.85Y
+ 34.17 36.83 4x-5y+33 0
or, X =0.85Y (i)
-

2.66
or, X= 0.85Y
37-2.66 28.79
20r-9y-107 =0... (ü)
When Y= 37, X =0.85 x
Let eq (i) be the
regression line of y on x and eqr
Most probable age =28.79 (i) be the regression line of r on y.
Then, from (i)
3 2071 Supp.Q.No.8a ORdata, compute the line of regression for estimating age
on weight and
4x+33 5y
From the following 4
estimate the most probable age on a weight of 37 Kg. Or,y
30 45
Age (X) 4
Weight (Y 10 50 65 55 byx5
P l e a s e refer to 2070 Supp Q.No. 8a OOR Again, from (i)
20x -9y 107 = 0
84 2072 Set GO.No. Ba and lines of regression. Find the correlation coefficients
between the u(WO Or, 20x =9y +107
Define regression or, x + 107 9
variables when by = 18 and bys =0.35 107
the
20 20 20
SOLUTION
Regression: It is a statistical device which is used to predict the value
is known. The
of one variable wne
variable whose value is known is called independent var1a
and the
ed
bys20
value of other
variable whose value is to be
determined is called dependent variable. The analysis
between the two variables is known as a
which

regressio
nalyss Here, by. bay *221 9

to describe the average relationship o, our


There must be cause and eftect relationship between two variables in regression. s of
the
We have,assumption is correct.
Lines of Whenever there shows a relationship between two variables, T
regression: hen it ther

Correlation () =Vby».
scatter diagram will concentrate around a certain curve. If the curve is a straight n by
known as the line of regression.
of y
The regression equation of y on Xis y
=
a + bx, when b is the regression coefficient
the regression ofx on y is x = b* ay, where a is the regression coeficient of x on y

Given, by =
1.8
by 0.35 2075 Set
Correlation coefficient (r) = ?
ca.No.
egression 8a and arithmetic means (X) 36, (Y)
= =
52. Find the
coefficients, 1.5, bx 0.65
=
bay
=y063-0.79 Tegressio
=
value of Ywhen X= 60. 4
Wehave,r =Vby bys =V18
x 0.35
equations X on Y and Y on X. Also, find the estinmated
18%8 K K Rg

219 NI

919 1
PROBABILITY Unit 11 207
Probability otgetting black ball
a

Number of black balls4 n total number of possible cases =


52
UNIT P(black) Total number of balls 20 P(a red 8, 9 or 10) = ?
We have,
Probability of
getting not a black ball
Pla red 8,9 or 10)-
-1-P[black) =1- - 2
PROBABILITY 2065 Q.No4
A ) =0.4, P (AUB) =
0.56, P (B) =
0.3. Are
12 2069 (Set A) Q.No, 4
From 20 tickets marked from 1 to 20, one is
drawn at random. Find the
probability that it is a
&2060O.No.4C A& B
independent?
2 multiple of 4 or 5.
A. PROBABILITY (SIMPLE CASES Two dice are thrown.
5.
Determine the probabilit SOLUTION
=
0.4 SOLUTION
2]
2 MARKS QUESTIONs a sum P(A)
of getting Here,
P(B) = 0.3
Total number of possible cases (n) = 20
The set of multiple of 4 is (4, 8, 12, 16, 20
2057 QNo.4 from a well shuffled
sOLUTION
When two dice are thrown, total numha
P(AUB) = 0.56
and the set of
A card is drawn at random nber « + multiple of 5 is (5, 10, 15, 20}
deck of 52 cards. Find the probability of being it 6 x 6 36 P(AUB) =PA) P(B) P(AnB)
-

possible cases We have, Number of possible cases (m) = 5+ 4-1 =8


2 s5 is 0.4 +0.3 - P(AnB)
(i) a red card (ii) a heart. The set having or, 0.56
a sum

sOLUTION (1. 1). (1, 2), a, 3), (1, 4, (2, 1), (2, 2), 2a PAnB)= 0.7 - 0.56=
0.4
=

x
0.14
0.3 0.12
Pla mulbple
of 4 5) ---
or

Again, P(A)-P(B)
=

52 (n) (3, 1), (3, 2), (4, 1)}


=

Total number of possible cases

of 52 cards. Number of favourable cases (m) = 10 Here, P(AnB) * P{A) P(B) 13 2069 (Set A) Old Q.No. 4c
) There are 26 red cards in a deck nce, the events A and
What is the probability that an English alphabet
(m) 26 =
Hence B are not
Number favourable
of cases
Pa sum s 5)- independent events.
selected at random is (i) a vowel (ii) a
consonant
P(red card) - 26
522 2066 Q.No. 4 [21
. 2062 Q. No. 4C Please refer to 2062 Q.No. 4c
(i) There are 13 heart in a deck of 52 cards.
So, number of favourable cases (m) 13
=
What is the probability that an English alphabet Thechance that A can solve a problem isthe 14 2069 Oid (Set B) Q.No. 3
selected at random is () a vowel (i) a consonan A card is drawn at
random from a well shuffled
P(a heart) n
13_
52 4* chance that B can solve it is Find the deck of 52 cards. What is the
a spade?
probability that it is
sOLUTION probability that the problems will be solved if (2]
2 2058 Q.No.4 There are 26 letters in English alphabet. Thee both of them try.
| SOLUTION
If A and B are two independent events with
are 5 vowels and 21 consonants. [2 Ina deck
2 Please refer to Model Set II, QNo. 8b of 52 cards, there are 13 spade.
121| n= total number
of possible 26 Total number of possible cases (n) = 52
PA)-and P(B) =5 find P(AUB). () cases
m = n u m b e r o f f a v o u r a b l e cases =
=

5 t0. 2067 Q.No. 4c And number of favourable


Define mutually exclusive events and
cases (m) =
13
sOLUTION P(vowel) =?
Since A and B are independent events, so We have,
dependent cases with example while performing
an experiment.
Plaspade)-
21
P(AnB) =PA) PB) =»- P(vowel) = sOLUTION 15. 2070 (Old) Q.No. 4
If three coins tossed
We have, (i) n= total number of possible cases = 26 Mutually exclusive events: Two or more
are
simultaneously, find the
probability of turning all head.
P(AUB) P ( A ) + P(B) - P(ANB)
m number offavourable cases =21 events are said to be mutually [21
P(consonant) = ? their simultaneous occurrence is not it| exclusive sOLUTION
possible.
For example, if a coin is tossed either head or When three coins are tossed simul
taneously,
2059 Q.No. 3c
We have,P(consonant) n m21
26 tail will occur, so head and tail are two then the sample space is
HHH, HHT, HTH, HTT, THH, THT, TTH,
Two letters are selected at random from the mutually exclusive events.
word "EXAMINATION". Find the probability
2063 Q.No. 4 Dependent cases: Two events are said to be TTT
Total number of favorable cases (n) = 8
that both of them are same letters.
The chance that A can solve a certain problem s dependent if the occurrence of one event in a
2] 1 2 trial affects the probability of the occurrence Number of favourable cases (m) =1
Find te
sOLUTION and the chance that B can solve it is of the other event in other trial. For example,
There are 11 letters in the word chance that the problem will be solved if the if two cards are drawn in succession without
Pallheads)-R *3
EXAMINATION'.
Total number of possible cases = C2
both try. eplacement from a deck of 52 cards, then the h6 2070 Set C Q.No.4
Please refer to Model Set II, Q.No.8b t of first draw will affect the result of A class consists of 60 boys and 40 girls. If two
In the word examination, there are 2 A's,
2l's and 2 N's 2064 Q.No. 4 second draw. students are chosen at random, what is the
Number of favourable cases for selecting two A Dag contains 9 red, 7 white and 4 black Dali 1 2068 Q.No. 4c probability that one is boy and one girl?
A ball is drawn at random. Find bability ard is drawn at random from a well-shuffled
letters such that both of them are A =C2 the pro sOLUTION
Similarly, number of favourable cases for both I = C2 of drawing (i) a white ball (ii) not a black Da deckof0 52 cards. What is the that is
Total number of students 60 + 40 = 100
probability a
And number of
favourablecases
for both N=
C2 SOLUTION ed 8, a red 9 or a red 10? [2
Total number of possible cases (n) = number

Required probability = 4C Total number of balls =


9+ 7+4= 20
SOLUTION of selection of 2 students from 100

Number of favourable cases (m) = number of


= C(100,2)

) Probability of getting a 2 red 8,2 red 9,


and 2 red of
and eck 52 cards, there
white ball are
selection of 1 boy from 60 and number of
1+1+1 10.
P(white) Number
= ofwhiteballs 20
number of favourable cases = 2+ 2*2
selection of 1 girl from 40 = C(60, 1) * C(40, 1)
number of balls
Total
2 2
Basic Mathematics-
of solving
the problem
208 ASmita's NEB Solution of Probability PROBABILITY Unit 11 209
We have, P (A OR We have
+ P(B) -

P(one boy and girl)


=

PA
P(A or B)
=
P(A)
Pla king
or
a queen)
=
F
king) +
P(a queen 29. 2074 Sot BQ.No. 4c
one
O P(A) P(B) -P(A)-P(B)
+
A
class
consists of 30 boys and 20 girls. If
x 40 two
C(60, 1) x C40,1)_60100
= C(100,2)
students are chosen at random what is the

SotEQ.No.4c probability that one is boy and another is


98! 21 girl?
910-- Es 2072 rolled
rolled once. What is the probability
once.
dice
Two
are
of 8 7? | SOLUTION
60x 40216 total or
2
100 x 99 33
4cg ofgettinga Total no. of Students = 30 +20 = 50
21. 2071 Sot DQ.No. simultaneos s o L U T I O N

Total no. of
possible
DQ.No.4d are tossed no. of selections
=
when two dice are rolled once, total number cases
7 2070isSet
drawn from a
well-shuffled
deck of 52 Two coins
Find the probability thas 'nd th.
cases (n) =6 x 6 =
36
of 2 students out of 50 students
A card
that it is a
King or sample space. both are of possible
cards.
What is the probability [2 heads.
The set having a
total of 8 is 50!
C(50,2)4R7 50 2x 49 1225
3), (6, 2)
(2, 6), (3, 5), (4 4),(5,
a Diamond?
SOLUTION total of 7 is Number of favourable cases (m) = no. of
two c o n s are tossed and the set having a
sOLUTIONN
In a deck of 52 cards,
there are 4 kings and
15 When
then the sample space
is simultaneously ((1,6), (2, 5), (3, 4), (4, 3), (5, 2), (6, 1))
Total number of favourable cases (m) = 5+6 11
selection of 1 boy from 30 boys and 1 girl from
20 girls
diamonds
(m) S= {HH, HT, TH, TT}
Total number of
favourable c a s e s
probability that
both are heads. 7) C(30, 1) C (20, 1) 30 20 600
4 + 13-1 = 16
For
favourable cases (m)
=1
number of Pa totalof8 or

Total number of possible


cases (n)
=
52
Total number of posSi ble cases (n) =4
Supp Q.No.4
Pfa boy and girl)=225 9
a

m
16 26 2072contains 4 white and 8 red balls. If two 2074 Supp QNo.4
diamond) =52 13
P(a king ora P[both heads)- alls are drawn at random, find the probabilityr A and B are two independent events with

h8 2070 Supp. Q.No.4 from 1 to 24 Supp. Q.No. 4b of getting


o n e of each colour.

2 PA)- and P(B) =5, ind P{AB).


contains 24 balls
numbered 22. 2071Two sOLUTION
[21
A bag random. What is the dice are thrown together. Find th
One ball is drawn at
2 There are 4 white and 8 red balls. Please refer to 2058 Q.N. c
6? of getting both odd digits.
probability that it is multiple of 4 and probability
a

SOLUTION
Total number of balls = 4 +8 =12
31. 2075 Set BQ.No. 4
SOLUTION P(one of each colour)
=
? An urn contains 4
white, 8 black, 6 red and 2
Total number of possible
cases (n) = 24 When two dice are thrown together,
number f possible cases (n) =6 x 6 = 36 No. of possible cases (m) = number of green marbles. If three balls are drawn at
The set of multiple of 4 and 6 is |12, 24
The set of getting both odd digits is selection of 1 white ball out of 4 and 1 red ballandom, find the probability of getting 2 red and
Total number favourable cases (m)=2
of *8C1 4 x8 =32 green marbles. [21
(1, 1), (1, 3), (1, 5), (6, 1) (3, 3), (3, 5),(5,1 out of 8 =4C1
Pla multiple of4 and 6) =2412 (5,3), (5, 5)
Total number of favourable cases (m) = 9
Total no. of possible cases (n) sOLUTION
= No. of selection of 2 balls out of 12 Number of white marbles = 4
Probability of getting both odd digits, P[E)=? Number of black marbles = 8
19 2071 Old Q.No. 4 fimes.
A is tossed successively
coin
2 heads and
three We have, =
12C 10! 2! Number of red marbles = 6
Determine the probability of getting Number of green marbles 2
12 32 Total
one tail. P (one of each colour)n66 33 no. of marbles =4+8+ 6+2 20
sOLUTION 23. 2072 Set C Q.No. 4c Number of favourable cases (m)= No. of
When a coin is tossed three times, the outcomes are
TT T
In rolling a pair of dice, determine the 2073 Sot CQ.No.4 selection of 2 red marbles out of 6 marbles
and 1 green marble out of 2 green marbles
H H H H T
toss
H H TT H H I T
probability of obtaining a s u m of 10. The chance that A can solve the problem is and
2
= C2x C
2nd toss
3rd toss H T H T H TH T sOLUTION
Number of favourable cases (m) =3 If a pair of dice are rolled once, then total the chance that B can solve the problem is 42430
number of possible Find the probability that the Total no. of possible cases (n)= no. of
Total number of possible cases (n)=8 is solved
?
cases (n) = 6x6=36
by A and B.
problem selection of 3 marbles out of 200
Probability of getting 2 heads and 1 tail, P(E)
=

The favourable cases of getting a total of 10m [21


SOLUTION 20!
a single throw of two dice (4, 6), (5,5) and C
We have,P(E)- 17 310
=
are

(6, 4)
20 2071 Set CQ.No.4 So, number of favourable
P (sum of 10) = ?
cases (m) =3
Here, P(A) . P(6)- P(2 red and 1 green marbles)= n1140
m 300
The chance that A can solve the problem is 3/5 Since the events
and the chance that B can solve the problem is
are
independent, 82 2075Set CQ.No.4c
/3. Find the probability that the problem is We have, P(sum of10) 612 PAand B) PA) P(B)=$* -5 If A and B are two independent events with
solved.
2
SOLUTION 24. 2072 Set DQ.No.4 ofS2 2073 Supp Q.No. 4cd P(A)=and P(B)=, find P(A U B). (21
card from well shuffled deck
The problem will be solved if A or B solve it
n
a draw of a
cards what is the probability that it is a Ki
COins are tossed simultaneously. Find the | sOLUTION
Probability of solving the problem by hes Space. Find the probability that all are
queen? heads. Given,

A P(A) = SOLUTION Tease refer to 2070 P(A)-


In a deck of 52 cards, there are + Kug
(Old) Q.No. 4c

Probability of solving the problem by B= P(B) i =


queens. P(B)-
Pfa king or a queen) = ? P(AUB) = ?

ww
of Basic Mathematics- PROBABILITY Unit 11 211
210 Asmita's NEB Solution

We have,
or, 1- PIC)inC)+P( Total number of possible cases (n) = Number of

selection of 2students
out f100 2 2064 No. 8
P(AUB)
=
P(A) +
P(B) P(AnB)
-

100!100
2
x 99,4950 P(A) and P(B) are the probabilities of the
or. PC)1 DCa98! 2! nappening of the events A and B respectively,
--P(A) P(B) If
both
both
studentsare boys. then 2 boys out ofrove that P{AUB) = P{A) + P(B) P(AnB),
studer

events la) be s e l e c t e d in C2 wavs where P(AUB) and


A & B are independent PO- 60 can P(AnB) have the usual
meanings. What will be the form of the abovee
60 =1770 formula if A and B are
r-PM--3 581 2 independent events?14
umber of favourable cases (m) = 1770 Please refer to 2057 Q.No. 8b
Pr-r-i}
P(both boys)n 4950 165
m
a. 2065 ONo8
State and prove the theorem of compound
Ai tfboth studentsare girls, then 2 girls out of 40 Probability.
4 MARKS QUESTIONS 36. 2060 Q.No.8 (b) C2 ways Please refer to 2061 Q.No. 8aa
4
3 2057 Q.No.8b
State and prove the
Theorem of
total
can be selected in

"Theorem of Total probability". 14 probability." 40! =780 4. 2066 Ca.No. 8a


the
State and prove 1 Please refer to2057 Q.No. 8 b 388 27 2
State and prove the theorem of total
Number of favourable cases (m) = 780 probability.
sOLUTION 7 2061 Q.No.8a the "Theorem 780 Please refer to 2057 Q.No. 8 b
4
two events
with "Theorem of Compoun Pboth girls)
-

Statement: If A and B are


ana prove 4950 165
respective
probabilities P(A) and P(B)State
umber of favourable cases (m)
as 2066 CO.No.8 aOR
number of| If A, B, C are three mutually
=

their the of at least


exclusive
probability of events
oacurrence
then |SOLOON selection of 1 boy from 60 and number of
denoted by P(AUB),
one of these two events, Statement: If two events A and B
independent, then
are
the probability of heir
selection of 1 girl from 40= C(60, 1) x C(40, 1) withP(A)-P(B)=P (C. Find P(A), P(B)
is given by and P (C).
P(AUB) =
P{A) +
PB) P(AnB)
-

simultaneous occurrence is equal to P(one boy and one girl) = [4


of the the n
Please refer to 2059 Q.No. 8 aa
where P(AnB) is the probability product of their individual probabilities
simultaneous occurrence of the
cases
events A and B.
i.e. P(A and B) = P(AnB) = P(A) P(B) C(60,1)4950
xC(40, 1)60x40 16
4950 33 a. 2066 Q.No. 8a OR
Proof Let n= total number of possible Proof: Let nn and nz be the total number d Five men ina group of 20 are graduates. If three
the event A
number of favourable cases to
u
favourable cases to the even
possible cases for the events A and B 9 2062 Q.No. 8 a men are chosen out of 20 at
random, what is the
V number of cases to the event
respectively. Also let their respective State and prove
"The Theorem of Total Probability ofat least one being graduates? 14
number of favourable
W
favourable cases be m1 and m2 Then, Probability" 141 SOLUTION
A and B
Please refer to 2057 Q.No. 8 b n Total number of possit cases
Then, P(A) n PB)-and P(AnB)= P(A)andP(B)= D.2062 Q.No.8 aOR Number of selection of 3 out of 20
both A Since the total number of possible cases for A lot contains 10 items of which 3 are defective. 20x 19x 18
Since the outcomes w are common to
and B, so the cases favourable to the
event the events A and B are ni and n respectively Three items are chosen from the lot at random C17 3 2 1 =-
1.14
1,140.
and since we can combine each of ni possibe one after another without replacement. Find the Since 5 men out of 20 are graduates, so
AUB are u+V- w cases of A with each of n2 possible cases of B, probability that: number of non-graduates 20- 5 15
Number of favourable cases Now, 3 men out of 15 can be selected in
Now, P(AUB) Total number of possible cases so the total number of possible cases for their ) All three are defective. 15C
simultaneous occurrence is ni:n2. In the same (i)None of them are defective. 4 ways.
u* V-W_u V
n nn
n
W =
P(A)+ P(B) -P(AnB) way, the total number
of favourable cases for sOLUTION 121
15x14* 455
3!3x2x1455
-

BB
o c c u r r e n c e of A and 0 Since the items are not replaced, so
the simultaneous
4 2058 Q.No.& mi'm2. 3 P(non-graduate) = 1,140
State and

probability"
prove the "Theorem of total
4
Now,
P(AnB) =Probability of simultaneou
P(first defective item)
10 Plat least one graduate)
Please refer to 2057 Q.No.8 b Occurrence of A and B P(second defective item) n 9 =
-P(non-graduate) 1- 228
1 9131
=

228
35 2059 QNo. 8a .Number of favourable cases. P(third defective item)
If A, B, C are three mutually exclusive events Total number of possiblecase n 8 7A class
2067consists
Q.No. 8 of 40 boys and 60 girls.
If two
2 Since the events
with PA)-P(B)=PC), find P{A;P(B),and -m2 (
are
independent, students are chosen at random, what will be the
Plall 2, 1
P(C). 14
n 1n2
three defective items)- probability that (a) both are boys (b) both
4
are

P(A) P(B) 0 Probability that none of them are detechve girls (c) one boy and one girl?
SOLUTION 1-P(all three defective items)
P(AnB)=P{A) P(B) SOLUTION
HereP(A)-P -Pro 38. 2061 Q.No.8 a OR -1-120 120 Total number of students 40+ 60 100
=

Total number of possible cases (n) Number of


=

and 40 girls he
P(A)-P(C)and P(B) =/PC) A class consists of 60 boys t will
be t
1. 2063 Q.No. 8 selection of 2 students out of 100D

P(A) and P(B)a


students are chosen at random, Wi both a
that (a) both are boys (D) be the probabilities of the
10C = =4950
Since A, B and C are three mutually
ree exclusive Probability
mutually exclusive| independ
events, soo
girls (c) one boy and one gir that: PA events A and B respectively, prove
the (a) If both students
are boys,
then 2 boys out of

SOLUTION sal me P{A). P(B) where P(AnB) has 40 can be selected in Ca ways
P(AUBUC)= P(A) + P(B)+ P(C) 100
Total number o f students = 60+ 40
"Case refer to 2061
Q.No. 8 a
Mathematics-ll

Solution of Basic PROBABILITY Unit 11 213


SotAQNo.8
NEB We have,
212 Asmita's

26! 2075contains 10items of which 3 are SOLUTION


defective.
40
38! 2!
40-78o P(4are black) C 521 A
lot
reeitems
are chosen from the lot at
random P* probability of getting a six in one trial of a die =g
Number of favourable
cases (m)
= 780
481 41
aother without replacement. Find the
x 25 x 24 x 23 46
onehility that () all defective.
three are
defective (ii) 91-*
o n e is
P(both boys) =n 4950165
girls, then 2 girls out
of 60 -52x51 x50 x49 833 anly first 4 n=3
(b) If both students a r e sOLUTION
Probability of getting exactly 3 sixes = P(3) =?
can be selected in C
ways
50. 2069 (Set B)Q.No. 8b Since
the i t e m s are not replaced. We have,
6060x5 1770 A bag
contains 5 red and 6 white balls. P(first defective item) 3 Pt)= p q
58! 2! 2 are drawn
at
random. ind the prok Twa n 10
1770 balls
Number

P(both girls)-n
of favourable
cases

*4950 165
(m)=
that
colour.
(i)both are red (i) both are of thbili
same P(second defective item) =

Fo)-*) ) 1 1
58. 2069 (Set B) Q.No. 4
of soLUTION P(third defective item)
cases (m) = number of n A dice is
c) Number offavourable
from 40 and
number Total number of balls in the bag = 5+6=11
Since the events are independent,
thrown 3 times. Getting a 2 or 3 is
selection of 1 bov C(60, 1) n Total number of possible cases numbered as a success. Find the
from 60 =C(40, 1)
*

getting two successes. probabilities of


selection
of 1 girl Number of selections of 2 balls outot
=
Plall three defectiveitems) 0 20 SOLUTION
2
P(one boyand one girl)- i When only first one is defectiv the second
* 6016 Ca1, 2)2 third are non-defective.
Here
C40, 1) xC60, 1)_40
495033 and P probability of getting 2 or 3 in one trial
4950 (i) For both red balls, of a die
P(first item defective)
2068 QNo.8
Number of favourable cases (m) number of n
solve a certain problem is ways of drawing 2 balls out of 5 =C(5, 2)
A can P(second item not defective)=
The chance that
that B can solve it is find the 10 1-p-1--
The chance be solved if they P(third item not defective) *8
chance that the problem will n 3
14 P(both red balls) Z.6 7 Probability of getting 2 successes = P(2) = ?
both try.
to Model Set II, Q.No.
8b Plonly first defective items) =10** 40 We have,
Please refer (i) For the probability of both balls are of the
Pr)= C pq-
A)Old Q.No.
2069 (Setcards 8a same colour, the number ot favourable cases

Suppose 4
drawn are at random from a (m) = number of ways of drawing 2 red balkt B. BINOMIAL DISTRIBUTION P(2)
deck of 52 cards. out of 5 red balls + number of. ways of
well-shuffled

What is the probability


that all 4 are spade? white balls out of 6 white balls
2 MARKS QUESTIONS
i drawing 2
that all 4 are black?
What is the probability 5 6
ii.
4 2061 Q.No.4.
-c62)+C(6 2-372 12 Find the binomial distribution having mean = 12
and variance = 8.
2073 Set DQNo.4
SOLUTION A dice is rolled 4 times.
In a deck of 52 cards, there are 13 spades. ,-25 SOLUTION
Getting an even number
is considered as a success. Find the
probability
Now, Total number of possible cases
n
=
of getting two successes.
out of 52 P(both balls are of same colour) = n 5 5 1 )=255 Here, mean = 12
(21
Number of selection of 4 cards
52C
np=1 2 . (i) SOLUTION
Again, variance = 8
Probability of getting an even number in one
m
=
number of favourable cases S1. 2069 Old (Set B) Q.No. 8a npq 8 (i)
number of selection of 4 cards out of 13 theorem of "Compound
State and prove the Dividing equation (i) by equation (i), we get throw
ofadice (p)=
probability".
8a
P(4 are spade) =? Please refer to 2061 Q.No. np12 91-p
We have,
13 52 2070(Od)Q.No.8a nd
Number of trials (n) = 4

State and prove theorem of compo Probability of getting 2 successes P(2) =?


13C 9!4!
P4 arespade)= 2 probability.
8a We have, p =1 q=1- -
We have,
48! 4! Please refer to 2061 Q.No. P(r) ="C. pq
From()
13x12x11*10 11 63 2071 Old Q.No.12 of compo
o u n d

P(2)+
52x 51 x 50 x 49 4165 State and prove the theorem
n12
(i) In a deck of 52 cards, there are 26 black cards probability. B6
80 2074 SetAQ.No,4G
Now, n number of possible cases Please refer to 2061 Q.No. 8a Kequired binomial distribution is dice, turning of 1 or 6 is
In 8 throws of a
= number of selection of 4 cards out of 52 Find the mean and
64. 2073 Set D Q.No.8 considered to be a success.
= 52C4

m =number of favourable cases


A class consists of 60 boys and 40 81r s

ho
*p-( standard deviation. 2
students are chosen at random.
.2066 C SOLUTION
thred diceQ.No. 4
=
number of selection of 4 black cards out of 26| probability that (i) both are boys lf =
Probability of turning of1 or 6 in a
Here, p
black cards are what is the probability
geting exactly thrown,
one girl.
C 8 a OK
3 sixes? 2 throw ofa dice =
Please refer to to 2061 QNo.
P(4 are blacks) =
?
Mathematics-
of Basic PROBABILITY Unit 11 215
Asmita's NEB Solution SOLUTION
214
Here, p5
g-1-p1-
9-1-p1-3- P()-C 1x125-35
91-p 1-5 i) P(3) ?
n n o . of trials = 8
WePr)=have
C p'q"- We have,
Mean = np =8 x n=6
Probability that non will strike the PT)= C pq-
P(0) Co
s.D vnpq-1s
P(O) = ?

We have,
6859
.P)C
P()= C pq"- B 2065 Q.No 8aO
2075 SetofAQ.No.4 distribution is 80 and
?
8000
Ifthree dices are thrown what is the
PO- )*
=
binomial

of p, the () P(1)
of getting i) exactly 3 sixes (i) exactlyprobability
a
The m e a n
the value
8, find We have,
standard
deviation

probability of a
success.
121
that exactly 2 will
1356313
strike the
P)= C pqn-*
sOLUTION
2 sixes (44

(i) Probability .

SOLUTION P(2) ? targe 9Pprobability of getting six in trial


Mean = 80 We have
P(1)C
a one
of a
or, np =80s t a n d a r d deviation= 8
Pr) C p"q"- -3 x 1083
die-
Again,

ynpq* (i)
Pa) 313 66. 2063 Q.No.8 a OR
g*1-
n-3
or, npq =64
Dividing (i) by (9),
we get
4. 2059 Q.No. 8 a OR
If 4 dice are thrown, what is probabilitv
he incidence of occupation disease in an Probability of getting exactly 3 sixes = P(3) =?
We have,
industry is such that the workmen have a 20%
50 getting (i) exactly
3 sixes (i) exactly 2 sixes and chance of sufferin8 from it. What is the Pr)= Cp
probability that out of six workmen four or more
(iii) nosixes.

We have,
sOLUTION
Here, p = probabiliy of getting a six in a
will contact the disease?

SOLUTION
P-))-1
(i) P(2)=?
We have,
p-1-q-1--S
throw ofa die =

Here, p 20% 0 0 20 P()Cpqa-


4 MARKS QUESTIONS then q-1-p-1- g1-p-1-a- P(2)
n=4 n=6
s2 2057 Q.No.8b 3ORtimes. Getting a '5' or '6' is| ) Probability of getting exactly 3 sixes =P3)=1 Probability that 4 or more will contact the 89. 2066 Q.No. 8a
A dice is thrown We have, disease p(r 2 4) = ?
numbered a success. Find
the probability of The
2 Pr)=C p qn-* We have, probability of hitting a target is
if eight
3 and (b) exactly
getting (a)
successes

4 P) =C p' q-r hitting are made find the probability that


successes.

SOLUTION
PO)- ) P(r24)=P(4) + P(5) + P(6) () none will strike the
will strike the
target.
target, (ii) exactly two

Here,
Pprobability of getting'5 or '6' in 1 trial of
(i) Probability of getting
We have,
Pr)=C pqn-
2 sixes P(2) =? =

) ) ) ) -¬)0 SOLUTION
Here, p=
41

probability of hitting a target=


adie
Then, q-1-p1-5- P--() 5N166x41
5
240+24+1 265
156253125 1-p 1-
n=8
7.2064 Q.No. 8aOR
Number of trials (n) = 3

a) P(3) = ?
6)- uppose that in a certain city 60% of all recorded
() Probability of none will strike the target= P(0) =?
We have,
P(0)= Durths are males. If we select 5 births from the
We have, (ii) Probability of getting nosixes= Pr)=C pq-
P(r) ="C p' qn-r We have, population, what will be the probability that
Pr)="C p q-7 none
e male
of them is male
(ii) exactly three ot them
P) -
(6) P(2) = ?
)11- P-)6)-11 SOLUTION 390625
1679616
We have, Given, p 60%= (i) Probability that exactly two will strike the
P(r)= C p" qh-
65. 2060 Q.No. 8 aOR is known t0
ve

1005 target= P(2) =?


A sample of 100 fuses 1-n We have,
average 5 defective
fuses. Threefuses P(r)= C. p" qn-r
P) =C are tested. What is the probability tna hen
P(O)
n=5
of them is defective (ii) exactly one =?
We have, P() (6
63. 2058 Q.No.8 aOR defective?
The probability of
hiting a target is 1/5. If Pr)= »C p" q
109375
hittings are made, find the probability that: (i) sixSOLUTION 419904
fuse 5
none will strike the
strike the target.
target (ii) exactly 2 will Here, p Probability of adefective
14
of Basic
Mathematics-l PROBABILTY Unit 11 217
Solution
NEB
216
Asmita's
1 1 2 0 -0.1969 (i) P(0)= ?
We have,
0 2067 Q.No. 8a Of its mean
and more than one defec ve) =P (rs1) P)-Cp g"-
Define
Binomial
distribution,

find the
and

probability of
getting| -1x14 6 625
P(not

variance,

three
heads
hence

in five
tosses of two
coins. 4 256+256+ 0-5 P(0)+ P(1)
10-
P6)0)11
(i) P(at most 2defective bulbs)
trial be repeated Q.No.8a on P(rs 2)=
sOLUTION Let
2069 (Set A) Old
a
distribution:
the
| 3.20% P(0)+ P(1) +P(2)
Binomial
of n trials.
Let
p be of the electric bulbs man.
so as to
robabiity
make a set
sucvess and q
of Then
If
be the probability| company are defective, find the probabilik
+ q=1.
ured by a10*5(54 -0.5443
trial such that p bulbs chosen at random
offailurein one of
ot n out
of 4 i) 2bulh
2070Set DQ.No.8
suocess out
r will be defective.
the probability
given by':
ritossed 5 times. Find the
independent
trials is
0srsn i. Please refer to 2069 (Set A) QNo, 8h ingi)two heads (1) at least two heads.
probabilíty of
Pr)=Cpq, binomial
4]
1154565
is known as ii. P(2 defective bulbs) = P(r = 2) =P(2)
distribution SOLUTION
This
P probabilit of getting head in a trial of a
distribution.

of the binomial
distribution is given 256+256+96 608
The mean 625 625
vanance by npq. coin
by np and 8a OR 78 2070 Supp. O.No. 8
Next Part
two head with two 2069 Old (Set B) Q.No.
probability of getting If 4 dice are thrown, what is the probabiliv a g1-p1-5 The
probability of man's hitting a target is
P a

coins in one trial =


getting (i) exactly 3 sixes and (ii) nosixes n=number of trials =5
If he fires 5 .
times, what is the
SOLUTION ( P(2)=? hitting the target. probability of his
Here, p probability of getting asixim PT) p'qn-7 i. exactly thrice
ql-p1- die
i. at least thrice
throw of a
=

n=5
?
PC)-C( SOLUTION 4
two heads thrice P(3) =

Probability of getting Here,


We have, enq-1-p-1-
Pr)= C p g-
()
n=4
Probability of getting exactly 3 sixes = Poj
i) P(t 22) = P(2) + P(3) + P(4) + P(5) P= Probabiity of hittinga target 4
Then
Pr))) We have,

-10 416512 45
P(r)=C p"qn- 1-p-1-
1 2068 O.No.8b O
PE)-()) TS*E =?( ) ().) ( ()
5
P(3) = ?
Wehhave,
(i) Probability of getting no sixes P(0)
=

A dice is thrown 3 times. Getting a


numbered a success. Find the probability
of
'5' or '6' is
We have, Ca+C+Ca+ C) Pr)= C p q-
14 Pr)= nc pqn-r
getting (i) 3 successes (i) exactly 2
Please refer to 2057Q.No. 8 b OOR
successes.
(10+ 10+5+ 1) =951
72 2069 (Set A) Q.No.8b PO)-()(6-1x1 7. 2070 (Old) Q.No. 8a o
If 20 of the electric bulbs manufactured ya 5. 2070 Set CQ.No. 8 F20% of the electric bulbs manufactured a
company are defective, find the probability that
company are defective, find the probability bythat (i) P(r23) P(3)+ P(4) P{5) =
*
nrocess prod0ut of 4 bulbs chosen at
out of 4 bulbs chosen at random (i) 1 (ii) at nufacturing Pdofective.
most electrical fuses of which 15% defective. Fi) at most 2 bulbs will berandom (i) one (i) zero
are
2 bulbs will be defective. the probability that in a sample of of 10 tuSOLUTION defective. 4
SOLUTION will be (i) no dete 5 5
selected at random there 213 1024 14 1024*171024*1
Here, p= 20%
20
(i) not more than one defective. Here, p 20%=100 3 9
SOLUTION 10 T0245 T024 1024 "T0 (015+1)
9-1-p1-j- Here, p = 15% 100 20
1-p-1--
n=4
106 53
1024 512
n=4 9 P(1)=?
) P(1)= ? We have, 9 2071 Old Q.No.12 bOR
We have, g1-p1-2-2 If 20% of the electric bulbs manufactured by
P(r)= C p q- n 10 Pp)=C p' q" company are defective, find the probability that
a

) P(0)= ? out of 4 bulbs chosen at random (i) 1


(i) 0 (iii) at
P(1) C ) We have, PO)-C most 2 bulbs will be defective.
4
Please refer to 2070 (Old) QNo. 8a OR
Pr)= C p qn-

(i) P(at
-4
most 2 defective
04
125625 256
bulbs) P(r s 2)
=
.
P(0) =
C
10-0
45
P(0)+ P(1)+ P(2)
MathematicS-I

Asmita's NEB Solution


of Basic
218
83. 2072 Set CQ.No. BD PROBABILITY Unit 11 219
Define Binomial distribution,
B0. 2071 Set CQ.No.3 8times. Find P(2)
is robability of gettingof() two heads (ti) at he
6
Getting 5
or

A dice is rolled
Find the probability ot two heads
in 5 tossescoin. a
leas pqh-2-42 (0.2) (0.8 x

numbered as success.
6x5
getting ) 2
s u c c e s s e s (ii) 3 successes

8 b OR
sOLUTION (i) Pl4)=? (0.2)3x (0.8) =0.2458
Please refer
to 2057 Q.No. Binomial distribution: Let a trial be.
so as to make a set of n trials, I peated
We have,

P(= C p"qn-
2074 Sot AQNo. 8B
A
1 2071 Set DinQ.No.8
that a certain city 60% of all
recorded
probability of success and q be the prop be the Company produces electronic
process that manufactures chips by a
Suppose select 5 birth of failure in one trial such that p + aDiliy
normally average 20'o
births are males, suppose
the population.
we
What is the
the probability of r success hen defective
selected atproducts.
A
sample of four
records from males ii) independent trials is given by: random and the chips is
2072 Supp Q.No.8b
are

probability that: i) three of them 4 certain characteristics, parts are tested for
P(r)=Cp" q , 0srsn what is the
m o r e than 4 are males. A certain manufacturing
manufacturing
process produce that () no chip is defective probability
one chi is
(i) one
(ii)
sOLUTION
This distribuion 1s known as binons
electrical fuses which 15% are defec0duce
of
(iii) moreetective
defective (iiin than one chip
distribution.
orobability that in a chips are defective.
thePat sample of 10 fuses
ere p-60% 100 Second Part: Pleasereferto 2070 Set DONa random there will be (1) no SOLUTION 14
4 2072 Set DQ.No. 8E
o.8b (i) not more than one defective. defective|
() P0) =?
1-p-1-j- The probability of hitting a target is found tab. Please referto 2070 Set C QNo. 8b 14We have, P(r) C prq- =

n=5 0.25. If eight hits are made, find the probabili, 72073 SetCQ.No. 8b
that ) none will hit the target (i) exactly Tf4 dice are rolled simulfaneously, what is the P(0)
) P(3)=? Wo
We have will hit the target. orobability of getting (i) exactly 3 sixes (ii( P(1) =?
Pir)= Cpq-r exactly 2 sixes?
sOLUTION We have,
4
Given, SOLUTION P) *C Pq-
P3) p Probability of hitting a target Here, p probability of
geting a six in a
25 P1) =C
=0.25100 4 throw of a die =/

?
i) P(r>4)= P5)
=

We have,
q-1-p-1 -4- thenq1-p 1- i7) Prob. of
P(r)=GPq-* n8
n 4 getting more than 1 chips defective
,343 343 () Probability of non-will hit the target ) Probability of getting exactly 3sixes P(3) =
? Pr>1)-1-PO) - P() =1-
3125 3125 We have,
P(5)- P(0)=?
625-256256 113
We have, Po)=Cp qp- 625 625
2 2071 Supp. Q.No.8 Pr) C p" q-r
The probability of a man's hitting a target is. O2074
The
Sot BONo. 8
average percentage of failure in a certaina
If he fires 5 times, what is the probability of his P0)-Co( Probability of getting 2
sixes P(2) =? examination is 70. What is the
of 5 candidates at least 3 will probability that
hit the large We have, out
two will
hitting the target (i) Probability that exactly P)= C p pass?
i) exactly thrice none = P(2) = ? qn- SOLUTION
ii) at least thrice.
SOLUTIOR
We have,
P(r)= nCr p" qn-r
:PO)-C( Percentage of pass = 100% - 70% = 30%
Let
(i) exactly thrice and (ii) at least thrice 25
X 36 216 probability of pass of a candidate 10010
Please refer to 2070Supp. QNo. 8b
ra-)) 2073 Supp Q.No. 8b
P

3
=

()p Probability ofhittng a target he 9-1-p1-j0 10


Then
hitsaprobability
re of
hitting a target is 2. If six No. of candidates (n) =-5
exactly a de, find the probability that (i)Probability of at least 3candidates will
85 2072 Set EQ.No.8b simultaneously. Wha tly
hit tare one will hit the pass P(r23) =

-1-p1-i* the target. target (ii) exactly two will|= P(3)+ P(4) + P(5)
n = 5
Four coins are tossed

the probability of getting OLUTION 14=Capy+Cptq*+CGp%ss


Here,
P(0) ?
(i) 2 heads
(i) 4 heads. haprobability of hitting atarget =0.2 ) ) **=() )(
581 243
We have,
P)=C pqn SOLUTION 1-p=1
numbers of -hitting=6
0.2 0.8 10 x
10
1000x100 *TO000 10 100000
=0.1323+0.02835+0.00243 =0.16308
Probability of getting a
head (p) =2
prob. getting
of
.
o-))-1 1024 1024 24243
p'q, r successes in n trials
91.2074 Supp Q.No. 8b
n4
P)-Cp16-1)
Out of 32 students in a class, 8 students are girls.
Then,q-1-p-1. 6
(6-1111* 0.2 x
(0.8)5 =0.3932 If 3 students are selected, find the probability
() P(2) = ? that 4
We have, i. one student is a boy

P(r)= nc p' q
220 Asmita's NEB Solution of Basic Mathematics-ll
Q.No.BE
ii. 2 students are boys and one girl.
SOLUTION
otal no. of students = 32
No. of girls =

N o . of boys = 32-8
8

24
3 2075Sotc
The

are
mean

4
and
variance

4 andrespectively.

sOLUTION
Given, mean= 4
Find
distribution
of binomial d i .

P(* 2 1).
41 UNIT 12
Now,

Pprob. ofgeting a boy


np4
And, variance 3 2 MARKS QUESTIONS
STATICS
6a sOLUTION
9-1-p-1- npq . (i) 2067theQ.No.
expression for the magnitude and the Let a be the
angle between two forces P and
Wri Q. Since R is the resultant, so
Here, n=3
Dividing (i) by (i), we get of the resultant of two forces acting at a R=
i. P(1) = ?
givenangle. 2
P2+Q +2PQ cos a - (1)
We have p Again, given that when Q is doubled, the new
np sOLUTION resultant is
P(r)= , P'q Let P and Q be given forces and R be their perpendicular to P, so we have
resultant. If angle between P and Q be a then tan 0 2Qsin a
P(1) -
P+20 cos
We have, p=1-q= resultant (R) =yP?+Q3+2PQ Cosa o, 2Q sin a
3! Also, if e be the angle made by R with P then,
P+20 cos
-1) 1 4 T
a
Substituting the
value of p in (i), we
get or, P+2Q cos a = 0
9Sina \
e taP+QCosa
0 tan-
or, cos a
n=6 Putting the value of cos a in equation (i), we get
Now,
then automatically R
=? |f 2 bovs comes
P(x 2 1)
ii. P(2)
girl also aomes P(1)+ P 2)+P (3)
+
P(4) + P(5) + P (6) R2-p2+Q+2PQ|
We have, 1-P(0) or, R?= P2+Q- P?
or, R2= Q2
Pr)=C,p q 1-- P r ) = "C p qr- P

-2
Q=R
2058 Q.No. 5a 4 2060 .No. 5a
At what angle do forces to
equal (P+Q)N and At what angle do forces equal to (P +Q) and
1-6101x (P- Q) N act so that the resultant may
be(P-Q) act so that the resultant may be VP2+Q?
P+Q7 121
1-729 SOLUTION Please referto2058 QNo.13 a OR
Let be the
angle between the forces (P +QN
a
. 2061 Q.No.5 a
92. 2075 Set B Q.No. 8 and (P- O)N. Given resultant isP+O N If a force P be resolved into two forces making
A company produces electronics chips by a
angles of 45° and 15° with its directions. Show
manufactures normally average 20%
process that
of four chips is
defective products. Sample
selected at random and
the parts are tested for P-Q P 0
that the later fore isP 21
what is the probability SOLUTION
certain characteristics, Let Pr and Pa be the resolved forces of the given
is defective (ii) one chip is
that (i)
no chip defective.
force. We have to find the later force P2.
defective (ii) more than one chip are

4 P+a We have,
We have,
Q.No. 8b
Please refer to 2074 Set A R2= P2+ Q2 +
2PQ cosa
o,P+:)2 =(P+ Q+ (P Q)+ 2(P QP -O cos
-

Q - P 2 + 2PQ +Q+ p: - 2PQ+ Q+ 2(P- QO9 cosa


0r, 2(P2 Q) cosa =
-(P? +Q
R sin a
COs - P: sin (* )
Here, R =
P, a =45°, B= 15°

acos-"| P sin 45
2059 Q.No. 5 a sin 60
The resultant forces P and Q is R. If Q is
doubled the new resultant is perpendicular to P.
Trove that Q= R.
Mathematics-l
Basic
222 Asmita's NEB Solution of
R 12N STATICS Unit 12 223
Here, 9 + 36- 18
27
90°
2062 Q.No.6
are Pand FN2
act We have,
R N3N
Find
Old (Set B) O.No. 5
the
Two forces whose magnitudes least resultant of two forces
inclined at an angle of Q sin a 2068 Q.No.5a of
on a particle in directions
magnitude and tan P+Qcosa acting at an
angle
of 45° have magnitudes 12N and 8N respectively.
135° to each other; find thee a8OLUTION
[2 is the angle esultant equal to y10 N; if one of the forces be
direction of the resultant. (where a
between P and Q the other force.
The least resultant of
SOLUTION Q sin a EN, find
two forces of

Here, P- PN Q=PN2N
or, tan P+ Q cos a magnitudes 12N and 8N is 12N -8N=4N.
3. 2069 Old (Set B) Q.No. 5
sOLUTION

Qsin a R-v10N
-?
a 135 R=?
or,pQ cos Here, P-V2N

a 45 Q-?
Aparticle weighing 5N is suspended freely from
the ceiling by
Find the tension inWeightless inextensible cord.
P+Qcos a
=
0 a
We have, or, We have,
the cord.
R =P+Q+2PQ osa R= P2 Q+ 2PQ 2
P+(P23+2.P-PV2 os135 cosa-
Again, we have 10
cos a

2)} +Q+2 v2-Qcos 45 SOLUTION


r

=PR+2P+2P2 N2 R2-P+Q+2PQcosa or,10-2+Q+N2-0


R2= P or, 122 = P? +Q7+ 2PQ or, 8 = Q 2 + 2 Q

R PN or, Q2+ 20-8=0


Also, we have or, 144 P+Q2-2P2
or, (Q+ 4) (Q-2) =0 W= 5N
Qsin a or, Q2-P2-144 or, Q-4(rejected)
W 5
tan P+Qcos a or, (Q+ P) (Q- P)
=
144
Q-2N Two forces act
or, 18(Q-P) =
144 [Using )] 4. 2069 (Set A) Q.No. 12a upon the particde. The weight
sin 135 N
is W 5 N which acts
or, Q-P=8 .i) dowrwards and the
or,
tan P N2 cos 135 P+PN2 Solving () and (i), we get
Two forces whose magnitudesare P and
act on a particle in direction inclined at an angle
2N tension T opposite to it acts upwards along
vertical line y-axis). There is no-x
P 5N, Q=13N 135 to each other, find the magnitude and the
componernt
O 2066CO.No.5 direction of the resultant. The
y-component are T and W. So, from
D= tan 90 The resultant of two forces P and Q is R. If O is Please refer to2062 Q.No.6b equilibrium condition,
doubled the new resultant is perpendicular to P.
e 90
Hence, the resultant is PN acting at right| Prove that Q=R.
52069 (SetA)Old Q.No. 5a
Find the resultant of two forces equal to 3N and 0
X =0
0
12
Please refer to 2059 Q.No. 5 a
angle with the first force. 6N acting at an angle of 120°. 21 andY,=0>T- W T-5=0»T=5N
1 2066 Q.No.5a sOLUTION 3 2070 Set CQ.No. 12a
20653 O.No.5E
At what angle of forces equal to (P+Q) Newton The resultant of two forces P and Q is R. If Qis
Given, P=3N Q=6N
At what angle do the force
a= 120° R=? equal to P+Q and
and (P - Q) newton act so that the resultant mayy doubled, the new resultant is perpendiculartoP. 0 ? P-Qact so that the resultant may beyP+Q? |2
Prove that Q= R. 2 Please refer to
be yP+Qnewton? We have, 2058QNo. 5a
Please refer to 2058 Q.No. 5a Please refer to 2059 Q.No.5 a R =P2+Q2+ 2PQ cos a
20Two2070 Set DQ.Na. 12a
2064 QMo.5G 12 2067 Q.No.5a 32+62 +2x3x6x cos 120° forces acting at an
angle of 45° have a

Forces equal to 7P, 5P and 8P acting on particle| a Find the resultant of two forces equal to 3N and resultant equal to 10 N; if one of the forces be
are in equilibrium. Fiad the angle between the 6N
respectively such that their
diagonal 9+36+3 45-18 27 2N, find the other force.
latter pair of forces. [21 perpendicularto the first force.
Please refer to Model Set 1,Q.No. 12 SOLUTION R - 27-3y3N SOLUTION
Again,
2065 QNo Given, P=3N Q=6N
tan 6=9S1n a _6x sin 120° Q/
The sum of two forces is 18 and the resultant 90° R=?
P+Qcos a 3+6 cos 120° x
whose direction is perpendicular to the smaller
the two forces is 12, find the magnitude of the
We have, -ViON
of Q sin a
forces. 21 tan p+Q cos o0= tan 90°
sOLUTION 6 Sin P-v2N
Given, P+Q= 18 ...i) or, tan 3+6 Cos Given, P-2N, R-v10N
6 Sin & 90 a 45 Q
or, 0 We have, R? = P?+Q+ 2PQ cos a
3+6 cos a he resultant is 3n3N acting at right angle
R 12N or, 3+ 6 cos a = 0
with the first force. or, (10- N2¥Q+22 cos 45
or, cos a
=
2069
Ifa force(Set B) Q.No. 12 or, 10-2++a201
P be resolved into forces making
Again, we have ngles 4 De resol
and 15° with its
two
ection; show that or, 8=Q+20
R - P2+Q+ 2PQ cos a or, Q+20-8=0

Suppose P <Q.
he latter force isP o r , (Q+4)(0-2)=0
= 32 +62+ 2x3x6x Mease refer (:Qcannotbe negative)
to 2061 Q. No. 5a 0-2N
224 Asmita's NEB Solution of
Basic Mathematics-
STATICS Unit 12 225
2072 SotEQNo.12a
21. 2070 (ld)Q.No. 5a P and Q acting
at
P:0:R-V3:1:2 es P and 2P acting at a point have the
If the resultant R of two forces
P, then prove
that
12 a Two GP. Find the angle between the two
12 3. 2071 Sot DQ.No.,
to
a point is right angle r e s u l t a n

L45-
R2=Q-P? 7p, and 8p acting
equal to 5p on g i v e nf o r c e s .

21
sOLUTION
Forces
in equilibrium.
Find the particle
angle between P= 4N

Let a be the angle between the forces


P and are
pair of forces.
latter s O L U T I O N

Let & be the angle between two given forces. We ha ve,

Please refer to ModelSet 1, QNo. 12a


Since R i s perpendicular to P, 6 = 0
We have,
R P + Q + 2PQcosa

Old Q.No.6 R2 P2+ Q+ 2PQ cos a


+

4If a2071 V4 (22+2422.cos 45


force P be resolved into two forces making
3P)2 P2+ (2P) 2. P. 2P
+
cosa

angles 45° and 15° with its direction, show


how hat 3P2
a
P2+ 4P2
+

-2P2
4P2 cos a
168242/2
isP. 4P coS
thelatterforce
or,
1
=cos 120 v16 8+16 =y40-2/10N.
And, if the angle made by R with P be 0 then
Please reter to 2061 Q.No.
5 a or, cos a
We have,
tan 6 S n a 2/2 sin 45°
Q Sin d Set CQ.No.12 a anP+Q cos a 4+22 cos 45
a 1 2 0 °

tan P+Q cos d 25Find2072 =

Q.No. 12a
the resultant and the angle subtended by it
with P when the forces and Q act at nght
23 2072 Supp
Sina Two forcesP and 2 acting at a point have the
or, tan 90 p+Qcos a
tan 90
=

find the angle between the two


angle. 12 resultant V3 P, =

Q sin a SOLUTION given forces. (21


or, Please refer to 2072 Set E Q.No. 12a
P+Qcos a Let R be the resultant of the torce P and 0
0 and a be the angle between them. Here
P+Q cos a 12a
=

or,
29 2073 Set CQ.No.
a= cos'(-P/Q). a= 90°. of two
If the resultant equal forces is equal to
coS a We have, the given force, find angle between the torces.
Again, we have 32 2074 Set BQ.No. 12a
R? =
P2+ Q2 + 2PQ cos a 121 If the resultant R of two forces P and Q acting at
R2 =P2+ Q2+ 2PQ+cos a R2 P? +Q2 + 2PQ cos 90° SOLUTION
=
or,
or, R2 = P2+ Q2
a point is at right angle to P, prove that:
or, R2 = P2 +Q2 + 2PQ Let two equal forces be P N and PNeach. By R=Q-P 2
R =VP+Q3 given resultant force(R) = P N. Let a be the Pleaserefer to 2070 (Old) Q.No.5a

or, R2= P? +Q3-2P? angle made by R


If 0 be the with P, then angle between forces.
Q sin 90 33. 2074 Supp Q.No. 12a
. Q 2 - P ? = R2
Q sin a A heavy chain has weights of 10 kg and 16 1

22 2071 Set CQ.No. 12 tanp+ Q cos P+Qcos 90° in


in
attached to its ends and hangs equilibrium
over a smooth pulley. If the greatest tension of
acting on a particle
are
Three forces
the first and e tan-1 the chain is 20 kg. wt, find the weight of the
equilibrium: The angle between
second is 90° and that between the
second and chain. [21
third is 120°, find the ratios of the
forces. 121 Hence, resultant (R) =P+Q,angle (0) sOLUTION
Then, we have
sOLUTION P?= P2+ P2+ 2P? cos a Let Wi be the weight of the part of the chain
at tan- containing the weight of 10 kg at ts end
Let P, Q and R be the three forces acting PR= 2P2 (1 + cos a)
the point O. Then,
between third and first force 26 2072 Set DQ.No. 12a
Angle Show that the resultant of two equal
forces
1-2-2cos or
T=Wi+10
20 Wi+ 10
bisects the angle between them.
cos 3 = cos 60° W= 10 kg
of the part of the
SOLUTION and Again, let W2 be the weight of 16
r
equal force a chain containing the weight kg at its
Let R be the resultant of two
P acting at an angle a.
60° end. Then,
then
R with P, a 120° T= W:+ 16
Let be the angle made by 20 W + 16
90 P
.Psin a sin a e 2073 Set DQ.No. 12 W 4
150 tan p+ Pcos a T Iwo forces have a
P and 2P acting at a point Hence,
2 sin cos2 resulta y3 P Find the angle between the w Total weight of the
chain =
Wi+ W2

360 -(120° +90°) = 150 tan 2


forces. =
10+ 414 kg
Since the forces are in equilibrium, so by 2 cos
Please reter to.2072 Set EQ.No. 12a 94 2075 SetA toQ.No.12a of
angle
Lami's theorem 2074 Set AQ.No. 12a A force equal
10N is inclined at
an

resolved parts in
Iwo horizontal. Find its
Psin 120 Q
sin 150 sin 90 of two e Find orcesresultan
their 4N and 22 N act at an ange
resultant.
30° to the
horizontal and vertical
directions.

SOLUTIHere,ONP
resultant
This shows that the
P
forces bisects the angle between u
enl.
SOLUTION
4N, Q =22 N, a =45, R=?,Q=? Here,
R 10N
Mathematics-
Basic
226 Asmita's NEB Solution of the particle. Tho STATICS Unit 12 227
Two forces uponact OR
6 N whidh acts downwards and 2057 Q.No.of13a
is W
a 30°
tension T opposite to it acts upward he resultant fwo forces P and Q acting a
The a is (2m + 1) P24Qwhen they act at an
is along
B 90-30 60°
P=? vertical line yaxis). There
There no-x Hence, the forces P. Q and R in
-a) the resultant is (2m -1) VPR+Q. are
Q=?
component.

y-component are T and


and W. :
angle equilibrium
The So, from =
equilibrium condition, Prove
that tan a
m+T 141. 2058 Q.No. 13 a OR
X = 0 Two forces P and Q act at a point. Their resultant
R is at
R=1ON 5OLUTION
right angles to P. Show that
0-0 +
1) yP* Is the
andY 0>T-W =T-6=0>T-6N Since (2m resultant ofP * R'and the
angle between the forces is
andQacting at an angle a, so
a-30 4 MARKS QUESTIONS (2m+1) P? +QOf=P+Q+ 2PQ cos a
at. (2m+1? (P7 *Q9-(P+0)= 2PQ cos a
co-
2057 Q.No. 13 SOLUTON
We have
"Lami's Theorem" or, cosa=
(P2+0 ((2m +1P-1 . () Leta be the
RSin 10 sin60 State and prove 2PQ angle between the forces P and Q.
Since R is
P Sin (a+B) sin 90
sOLUTION Again, (2m-1)yP *Q is the resultant of P perpendicular
to P, 0 0 . =

Lami's theorem: If three forces actine and Q acting at an angle (90° - a), so
at
-10-N point, be in equilibrium, each fom m-1) P? +Q2 P= P? +Q2+ 2PQ cos (90-a)
to the sine of the angle betuo or, (2m-1)(P+0)-PQ)=2PQsina
SinRSin(a8Sn
a_10 sin910*5-5N
30_ proportional
the other two.
P2+Q) (2m-1)2-1)U
Proof sin a 2PQ (i)
35 2075 Set B Q.No. 12a
Find the angle between the forces P +Q and Let P.Q and R be three forces acting at O be Dividing equation (i) by equation (i), we have We have,
P-Q such that their resultant may be VP?
+ 3Q. in equilibrium. So, (2m-12-14m2-4m+1-1 QSin a
tana (2m+ 1)P-1 4m2+ 4m+1-1 tan P+Q cos a
4m (m-1 a
4m (m+ 1) or, tan 90 =Sin
Q cos a
Given yP+3Qis the resultant of two forces m-1
P Q andP-Q Let a be between
the angle tan a m+1 Q sin a
the forces Then, using the formula, R2
P?+ =
P+Qcos a
o. 2058 Q.No. 13 a or, P+Qcos a = 0
Q+2PQ cosa, wehave Slate and prove "Triangle of forces"
P+3Q=(P+QF+(P-QP+2P +Q)(P 14
cosa
P+0+R =0
PxP * PxÙ+Px R -0
SOLUTION saa (
or, P30P 2P0 QP-2Q+Q| o, Triangle of forces: If three forces acting at a
point be
Again, we have
2P2-Q cosa
Px -Px R (:Px P=0) represented in magnitude and R P+Q+2PQ+ cos a
or, Q-P2=2(P-Q9 cosa direction by the sides of a
triangle taken in
or, P xQ =RxP order then the forces are in or, R P+Q+2PQ(
equiibrium.
or, PxQ1 =[R x P|
or, R= PR +Q- 2P
or, PQ sin (P, Q) = RP sin (R, P)
or, cosa=-5 os 12 Q-P2 = R?
where, P, Q and R represent the magnitudes
a 120
of P,Q and R; and sin (P, Q)is the sine ot 22059 Q.No 5
State Triangle of forces.
6 2075 Set CONo. 12 Please refer to 2058 QNo. 13 a
Forces equal to 7p. Sp and 8p acting on a particde the angle between P and Q.
are in equilibrium. Find the angle between the Q R P 2059 Q.No. 13
latter pair of forces. 2 sin(R, P) sin (P.Q) Buniform sphere of weight 3N rests in contact
Please refer to Model Set II QNo. 12a with a smooth vertical wall It is supported by a
Again,
37. 2075 Set CQ.No.126 string whose length equals the radius of the
P Q+R 0 rToof:Let P, Q and R be the three forves sphere joining a point on the surface of the sphere
A particle weighing 6N is suspended freely from a O, be acting
the ceiling by a weightless in extensible cord. represented magrutu
Epresented in magnitude and direction| to a point of the wall Calculate the tension in the
by AB, BC and
P, B Ùsring and thereaction of the
wall 4
Find the tension in the cord. A of AABC. Sa, AB
CCA = =

SOLUTION
SOLUTION
or, Q R =PxQ and C-R
Let us Let C be the point of contact of the sphere and

R complete the parallelelogram ABCD. Since the vertical wall Let O be the centre of the
or,
1Q -[PxQI BC sphere and oD be the line of action of the
Bynr equaland parallel, Ab- B -Q
Or, QR sin (Q R) = PQ sin (P, Q)
so
weight Also let R be the normal reaction and
or
P R_ .. (0 logram law of forces, we have T be the tersion of the string BA.

W 6N
sin(Q R) sin (PQ)
From (i) and (i) P-Ab
Now, Ab-aC
W=6N
| P
sin(QR) sin(R,P) sin(P.Q)
R
228 Asmita's NEB Solution of Basic Mathematics-l
(P+ 1 STATICS Unit 12 229
(P+ 12) cosa =2(P+ 12) 12) cosu
+
or, 4P+ Z U T I O N

OA andOB represent the two forces Pand


2P + 24 and OB
4P -

or, R =C=Co
et
or, 2P 24
acting
at the point O. Let OC be
the
Q the
.. P= 12N.
Lagonal of
diagor
OACBparallelogram
which the resultant y3Q act. Also, ZAOC-
through Also,AC -OB- :andACAC1
-0BOB/
as 2060 Q.No.13 a
Find the
resultant of n number of Hence, the forces P,Q and R are
R forces acting
at a point. coplanar by the sides OA, AC and CO of the 30AC represented
14 taken in order.
SOLUTION
Let the coplanar
forces P, Pa
... Ph
acting;
89 2062 Q.No. 13 aOR
a
at angles a, A uniform plane lamina in the form of a
N point O be inclined 2,
rhombus, one of whose angle is 120°, is
radius of the respectively to a line OX through the no ,
By question, length of string
=

Let OY be perpendicular to ox Supported by two forces applied at the centre in


sphere omponents of the forces Pi, P2, .., F The the directions of the
AB OC =OB =OD along of the rhombus diagonals so that one side
is horizontal; show that if P and
OX and OY are Pi cosa , P2 coS d2... P'n COS Since AC
=
OB, so AC =Q. Q be the forces and P be the
Hence AOBC is an equilateral triangle.
at O
and P1 sin ai, P2 Sin az,..,Pa sin Now, apply1ng cosine law in AAOC, we have greater then
Since, the forces T, R and 3N acting
are
n P 3Q. 4
in equilibrium, so by Lami's
theorem, we respectively.
Let R be the resultant or the given system of cos A
OA +(OC?-(AC2
2OA x OC sOLUTION
have, Let ABCD be a uniform
forces and let it make an angle e with OX 9-Q2 plane lamina in the
sin < ROD sin OAOE
< AOD sin < AOE Then the resolved parts of the resultant R
along OX and OY are R cos 6 and R sin 9
Or, cos30°= 2Px 3Q
form of rhombus whose one
LBCD 120. Let E be the
=
angle say
point of
T. R a, P 202 intersection of the diagonals AC and BD. The
Osin 90°sin 150° sin 120° respectively.
POo
or, 3PQ= P? + 2PQ
weight W acts vertically downwards as
shown in the figure. Let P and Q be wo
forces along the diagonals EA and EB
or, P2-3PQ+202= 0 respectively so that DC is honizontal.
(P-) (P-20) 00
=

or, T 2R=23
R/ or,
Either P=Q or P= 2Q
Hence, P equals to Q oris double of Q
T 23 N
and2R 23 .2062 Q.No. 13 a
State and prove converse of triangle of forces. [4]
R-3N SOLUTION 120
44 2059 Q.No. 13 b OR c
Two forces equal to 2 P and P respectively act on
Since its resolved parts are equal to the
Converse of the triangle of forces
If three forces
a particle. If the first be doubled and the second at a acting point be in N
sum of the resolved parts of
the equiibrium, they can be represented in
is increased by 12N, the direction of the algebraic Since the diagonals ot rhombus are at nght
resultant is unaltered. Find the value of P. 4 forces along the same directions, we have magnitude and direction by the three sides of
=X a triangle, taken in order. angle to each other, s
SOLUTION R cos 6 Pi cos a +P2 cos a2 + . . Pn cos an (say)
+

and R sin 6= Pi sin uj + Pa sin a 2 t . . . + P a sin dn= Y (5ay) BEF =90°30° 120 =

LAEB = 90° a n d LAEF = 0 6 0 = 150


Let a be the angle between two forces 2P and Squaring and adding, we get
P. Let 6 be the angie made by the resultant R Since the lamina is in equilibrium, by Lami's
R? X+ Y2
with the direction of the force 2P. Then theorem, we have
or, R=yx?+ Y2 i)
tan
P sina .-) Again, by division, we get P
2P P cos a
+
sin BEF Sin AEF
If the resultant R of the forces 4P and P+ 12
makes the same angle 0 with 4P, then
tan- Proof Or sin 120° sinl50
Let P, Q and R be three forces acting at O.
1.e.0-tan( (i)
12 and the magnitude au
et the forces be in equilibrium. Let OA =
P,
Hence, i) (ii) give OB- and
direction of required resultant. R. ob- or. P-v3
u s complete the parallelogram OACB with
6s. 2060 Q.No, 13 aOR P=3Q
If Q i6
The of two
resultant forces P and Q is R. ne magnitude of Pand as its aljacent 50. 2063 Q.No.13 a
doubled, the new resultant i perpendicularto
es.
Then, by parallelogram law ot torec Forces of 2 V3, 5, V3
and 2 newtons respectively
of a regular
(P+12) sina
tan 6 4P +(P+ 12) cos
prove that R Q.
Please refer to 2059Q.No. 5 a
P+Q-OA
once
+OB - OC ...( act at one of the angular points
towards the tive
other angular points.
the forces are in hexagon of the
From i) and (i), we have equion Find the magnitudeand direction

P sina (P+ 12 Sina


7. 2061 Q.No.13 a
The resultant of two forces P&Q is equal
t o3

,
P+Q R- resultant.
4
2P +P cosa 4P+ (P+ 12)cosua
Q and making an angle of 30° with he
Q or
d i r e c t i o

OA OB+R -0
P+12 of P. Show that P is either to , OcR-o
Or 2+ cosa 4P+ (P+ 12)cosu equa
double of Q Using (0)
Mathematics- STATICS Unit 12 231
Basc
230 Asmita's NEB Solution of P parallel to the length as shown in the
R P QQ fore and by force
Qparallel to the
base as
SoLUTION Now, f1gin the figure (). Also, let R be the
and 2 Newtons
Let the foroes 2, 3, 5, V3 AE and
R R R normal reaction.

along AB, AC, AD,


respectively act
ABCDEFas
shown inn -(P )(P )
hexagon
AF of a regular - p+Q+2P Q P+Q+2PQc ()
the higure
where P, Q and R be the magnitudes of P,OA OA
ie. AAC=uBC
R respectively. Proof
R VP+Q+ 2PQ cos a which gves the From AOACC
magnitude of Pand Q.
OA+ AC-OC
Again, let be the angle between and R.
or, 0A+ AC OC... =

Here, R- P +Q Similarly, from AOBC,


P P +P
Sinoe an
interior angle of regular hexagon
is P-R
PR cos a =P2+ PQ cosa
OBBC-OC
120
R cosa = P + Q cosa OB+ BC HO )
BAC CAD =
DAE= EAF= 30°. Again, N Adding ) and (i), we get
the resultant indlined angle 6
at an
Let R be When P is used parallel to the length, using
with AB. Now, resolving
the fores along and R-P+O OA+ OB+JAC+ BC =@+ }OOc
Lami's theorem, we have But AAC = uBC and since àAC and uBC are
perpendicular to AB,
we have
PxR- Px P+PxQ W acting in opposite directions along the same
x =2as 0+V3os 30+5 cos 60 sin 90°

V3 cos 90+2cos 120


PR Px (:P x P =0) sin (180°-a) line, so they cancel out

IPxR IP OI or sinaW So, A0A OB- OC


-21 ii0«2 (-5 PR sin= PQ sina
R sine Q sina ...i)
Sin a

sin a w ..6) 2065CMo. 13


andY =2 sin 0+3 sin 30+5 sin 60+ Dividing equation (i) by equation (), we get State and prove comverse of triangle of forces. 4]
V3 sin90+2sin 120 = Sina
When Q parallel to base, using Lami's
theorem, we have
Please referbto2062 Q.No. 13a
tand P+Q cos aa
W
2066 CONa.13aC
2.0-.s..Vi.1-2. A body of weight kg s spended by two

t a (_Q sina a sin (180- a) sin (90 + a)


strings 7 and 24 metres in length, their other
(P+Qcos W ends being fastened to the extremities of a rod of
Sin a COS a length 25 metres. If the rod be so hold that the
resultant R.
Now, resultant (R) = Y = Vs+ 3 P which gives the direction of the or, Q cos a= W sin ca body hangs immediately below its middle point
Find the tensions of the strings
=y5 75=10 N 2 2064 CNo,of13 two forces P & Q acting
at an
or,
Qcos a= W using ()] SOLUTION
The resutant
Again,tan x -tan6 angle a is equal to (2m +1) P2 +Q, whenthy cos a ..(i)
Let OA and OB be two strings such that OA =

and OB = 24 m
7m

=60 at at an angle 90° a, the resultant B|


Hence, the direction of the resultant R is Squaring and adding (i) and (i)
(2m-1)P2 +QProve that tan a =m+ p2
towards AD.
R 10 N is the resultant which act towards
P l e a s e refer to 2057 Q.No. 13 a OR W-1
the opposite angular point
1. 2063 Q.No.13a OR 2084 0.No. 13 aOR
Find the resultant oftwo forces Pand Q acting at State and prove Lami's theorem.
a point. Please refer to 2057 Q.No. 13 a
14
SOLUTION 2065 Q.No 13 a OR
4 2065 Q.No 13 length

OLUTO OVe: A - theorem.


Let P and Q be two forces acting at O and a Two forces P& Q acting parallel to i e k W 10N
and base of an inclined plane respht Stater The resultant of two forces, actung AB =
25 m
be the angle between them If R is the w*** a By question,
would each of them singly support point O in the direction OA and OB and
Here, OA+ OB
=
72 +24P 625 =253= AB
resultant of P and Q then on the plane, prove that: ented in magnitude and directions by So, LAOB 90°
-

A and HOB is represented by (A * H) Here, OC is vertical


and meets AB at C such
where Cdivides AB in the ratic A. thatAC BC.
BC
0" and AC
=
C8, so OC= AC
CLUTIO the
Since AOB
=

ZBOC= 8.
angle
the of indinatiodyd Let OB =0 then
tbe
indined plane with the hori d by
suppor
weght W placed at A is
Mathematics-1 STATICS Unit 12 233
of Basic
NEB
Solution
B) 65N Q.No.13
2069 (Sotweight
Asmita's
232
tensions along
strings the 3. 2068 QNo.13 a Or65 N is suspended of
is 71.2070 Set CQ.No. 13 a
suspended by two| For
be the
Let Ti and T: W= 10 N acting A body of weight b A body
of lengths
5m and 12m
attached to two Forces of 2,3, 5, 3 and 2N
OA and
OB. Since Ti, and
then
Ta
by Lami's
strings of length
Sm
and 12m attached e strngthe same horizontal line whose distance one of the angular points of arespectively act at
equilibrium, w
horizontal n towards ththe five Points
regular hexagon
hexagon
is 13m, findto the tension of the strings. 14 towards
same
atO a r e in points in the other
theorem distance apart is 1sm; tind the tensjon e
t
par
2068 Q. No. 13a OR
refer magnitude and direction of thepoints. Find the
resultant.
strings. the Please

Please refer to 2063 Q.No. 13a


(Set B)Q.No.13a On
s i n 90°
e) sin(180- (90-0)) 2069 resultant
sin(180- sOLUTION of fwo forces P and Q acting at 2 D 2070 Set Q.No. 13a
T:10 Let CA and CB be the strings so that CA=5 the angle between them is a. ng atA body of weight 65N is
or sin cos e 1 and CB 12m and AB = 13m. when suspended by
122 13, so LACB 90°
po
Please = reter toz0b3 Q.No.
13a OR strings of lengths 5 and 12 m attached to two
two
kg- wt
Since 52+
= =

points in the same horizontal line


10 sin e 10* B) Q.No. 13a
=

whose
Ti
13 m .2069 Old (Set"Lami's distance apart is 13m; find the tensions of the
theorem".
gwt State and prove string
T: =

10cos 0- 10*
5 m 12 m Please refer to 2057QNo.
=
13a
Please refer to 2068 Q.No. 13a OR
58 2066 aNo.13a act
T Q.No. 13a O
fo. 2069 Old (SetofB)two 73 2070 Set DQ.No. 13 a O
Forces 2 V3, 5,V3,
2newtons respectively
resultant forces
acting on a
particletate and prove Lami's theorem.
hexagon
at o n e of the angular
points of a regular 90°+6
e
be
at right angle to one of them and its
Please refer to 2057 Q.No. 13a
the five other points.
Find the 180°- e be half of the magnitude of the other,
towards
of the resultant. 41 mahat the ratio of the
| 74 2070 (Old) Q.No. 13 a O
magnitude
and direction show thal greater force to the
If the resultant R of two
13 a W=68 N focus P and Q inclined to
refer to 2063 Q.No. smalleris V3.
D 2:
Please 4one another
at ay given angle, makes an angle 6
Let the line of action of weight W 68 N
S2066Q.No.13a OR theorem Triangle of produced to meet AB at E Lt
sOLUTION with the direction of P, show that the resultant of
State and prove the
on
ZACE 0 so that 2CBE = 0. Let Ti and Tibe
leta be the angle between the two forces P and Qne torces P+R and Q acting at the same given
forces for three forces acting
at a point. angle, will make an angle 8/2 with the direction of
the tensions along the strings CA and CB P<Q. The magnitude of their resultant is P+1R
Please refer to 2058 QNo. 13 a respectively. Since the three forces Ti, Ta and
60 2067 Q.No. 13a W 68 N acting at C are in equilibrium, so by
and the resultantis perpendicular to P. SOLUTION
into two forces making Let the forces P and Q be
If a force P be resolved represented by ODA
with its direction; show Lami's theorem, and OB of the
angles of 45° and 15° 68 parallelogram OACB
that the latter force is
Sin
sin BCD sin ACDSin ACBACB respectively. Also let the resultant R be
represented by OC.
T 68 Q Let
us produce OA to D
making AD OC so =

SOLUTION sin (90 +0) sin (180 0) sin 90 that OD represents the force P
of the R Now,
Let Pi and Pz be the resolved forces complete the parallelogram ODEB. Then thhe

P
given force P. We have to find the later force OT, cOs Sin - 68 diagonal OE represents the new resultant of
68 cos 6 and T2 68 sin 0 the forces P+Rand Q2
Q sin a
=

We have, Ti =

So, tan 90°


BC AC5 P+Qcos a
B

Butcos -A 1 and sin AB 13 Qsin a


0
P Hence, T=65x60 N P+Qcosa
or, P+Q cos a = 0

Ta65 3 25 N R D
Pa =sina Again, we have,
s i n (a + B) 84. 2069 (Set A) Q.No. 13a Here AD OC
The resultant of two forces P and Q actinga R2= p?+
Here, R = P, a = 45°, B = 15° Q2+ 2PQ cos a Also, AD CE

a is equal to (2m+ 1)
VP2+Q.Whenth CE - OC

P2
P sin 45
P. angle
act at an angle (90° -a) the resultant is (am
-P+2P0(Using ( COE
Again, CEO
4CEO
=
DOE
Sin 6
P+Q2.Prove that: tan a m+1'
m-1
,Pp+Q-2P 2COE = DOE

ButAOC=0
Please refer to 2057 QNo. 13a OR
4COE= DOE =;
85 2069 (Set A) Old Q.No.13a 7,Or, 4P2
Q 4Q2-4P?
State and prove Lami's theorem. 302
61. 2067 Q.No. 13a OR Please refer to 2057 Q.No. 153a & 2071 SetcQ.No.13 a
two forces P and Q
The resultant of acting at an
State and prove converse of the Triangle of
forces.
14 66 2069(Set A)Old Q.No.13a o tivelya
angle a is equal to (2ma) 1) VP+Q*. When they
+

Please refer to 2062Q.No. 13 a Forces of 2, V3,5, 3, 2 newtons respe act at an angle (90° the resultant is (2m
-

1) -

at one of the angular points of a regued nn-1


22068 CNo. 13 VP+Q3.Prove that: tanam+1 141
Find the resultant of two forces
the angle between them is a.
P and Q when
14
towards the five
magnitude and the direction o
Please refer to 2063 Q. No. 13a
other posulta
Q:P-2:3 Please refer to 2057Q.No.
13 a OR

Please referto 2063Q.No. 13 a OR


SoMrtion of Basic Mathematics
NEB
234
Asmita's
Since the forces P, Q and R acting restively. Since Tu Tz and 68 N acting at C STATICS Unit 12 235

Q.No. 13
OB and OC are in equilibrium,
um, 5o albyong, On, equilibr
so by Lami's
theorer
2071 SetDof
in
equal to
two forres P and
resultant
Q is theorem lami's are

Let the forces 1, 2, 3, 4 and


the
The

V3Q and makes


direction of P;
an

Show that
angle of
P is
30 with
either equal to Q
P Q
sin BOC sin 2 COA sin ZAOR 17m7 along, AB, BC, CD, DA and 22
ABCD respectively.
newtons act
AC of square a
8 m 15 m
or is doable of Q. P
sin2Asin 2 B sinsin 2C
2C
the forre P andQ 13a
Let OA and OB represent 2072 Set CQ.No.
parallelogram OACB, | 80
Two men carry a weight 50N supported
Let us complete the 180-0 90 0
the resultant
then the diagonal OC represents strings; one string is inclined at 30yh
i e OC =V3Q.
Since OACB is parallelogram vertical and other at 60", find the tension W 68 N
law in of each
so AC
OB =Q APplying cosine
=
instrin SOLUTION Tz
D
Now, resolving the forces along and
AOAC, we have
Let CA and CB be perpendicular to AB, we have
two strings and CD be th
ine of action of weight 50 N. Let Ti andT
a BCD sn ZACD sin ACB 1cos&+2 cos 90+3 cos 180+4 cos 270P+
Tz 68
the tensions of the stringS CA of sin (90+ 0) sin (l80-0) sin 90 N2cos 4
respectively. Since T, Tz and w ac and
cos0 and T2 68 sin 0. -1+2x0*3 (-1)+40+22
=

are in equilibrium, 50 by Lami's theoremat T 68


have
But cos BA 17 and sin @=AC and Y 1
sin
=

P+2 sin 90+3 sin 180P+4 cos Z0


OAP+(OC-(AQ
T-68 17 60 N 22 sin 45
cos 30 2 OA OC
1 x0+2* 1+3x0+4x
a R30-Q 30
Th-68 32N (-1)+2/2*
2 Pv30 IfRbe the
resultant, then R= Y
Hence, the forces are in equiibrium.
or, P?- 3PQ+202=0
or, (P-P-20)=0
D 2073
A
Set DONo 13
W 50N P body of weight 100 kg is suspended
by two
P=Qor P= 20
Hence P equals to Q or is double of Q. Hence the required tensions are 60 N and 32
things 7m and 24 m in length; their other ends
sin ZBCD sin LACD sin ACB N. being fastened to the etremities of a rod of
7.2071 Set DQ.No. 13 a OR length 25m. If the rod is so held that the
body
T T2 50 2072 Set EQ.No. 13a hangs immediately below its middle point, find
State and prove Lami's theoremn
o r sin 120° sin 150 sin 90°
A body of
weight 65 N is suspended by two the tensions of the
strings
Please refer to QNo. 13a
2057 s 0V strings of lengths 5 and 12
4
m attached to two SOLUTION
Ta 2071 Old Q.No. 13 50sin 120 = 25N,
points in the same horizontal line whose
Let OA and OB be two strings such that
State and prove Lami's theorem. T n 90 1 distance apart is 13m; find the tensions of the
OA 7m and OB 24 m
Please reser to 2057QNo. 13 a 50 sin 15050 =25 N string,
9 2071 Old Q.No.13a OR and T2 sin 90° 1 Please refer to 2068 Q.No. 13 Or a
B
42
O is the circumcentre of the triangle ABC. Forces
Hence the required ternsions are 25N3N Z Supp Q.No. 13a
P, Q and R acting along OA, OB and OC are in Forces of 2, V3,5, 3,2 N respectively act at one
25 N.
equilibrium. Show that
the angular points of a regular hexagon
81. 2072 Set DQ.No. 13a Owards the five
sin 2 sin 2 State and prove Lami's theorem.
other
a gnitude and the direction ofpoints.
Find the
the resultant.
TON Please refer to 2057 Q.No. 13a lease refer to 2063 QNo. 13
a
Let O be the aircumcentre of AABC. Since the 2072 Set DQ.No.13a OR &2072 Supp Q.No. 13aOR W 100ON
angle at the centre O is the twice the angle at A bodyof weight 68N is suspended by oate and prove Lami's theorem. By question AB= 25 m
the circumference, so
strings of length 8 m and 15 m respectivey ease refer to 2057
Q.No. 13 a Here, OA OB=7+24-625 25= AB
the other ends of the strings are attac
2073
207
Set CQ.No. 13a So, 2AOB=0*
points in a horizontal line 17 apa
m
fIxed Blate and Here, OC is vertical and meets AB at C such
the tensions of the strings. triangle of forces.
ease refer to 2058 Q.No. that AC BC
sOLUTION
CA and CB be the strings
so that
CA
CA
2073 Set CQ.No. 13a OR 13 a Since,
Let
CB 1 5 m and AB = 17 m.
ABCD is a s LAOB 90 and AC =
CB, so OC =
AC BC.
square. Forces 1, 2 3, 4 and N2
DAewtandon's act at
=
Let 20BC-0 then BOC=8.
ZACB
Since, 8+ 152= 172, so
Let the line of action of weg
a
pointi in directions AB, BC, CD, Let Ti and Ta: be the tensions along the strings
BOC 2A *a respectively. Find the resultant OA and OB Since T, lh and W 100 N
2COA = 2B produced to meet AB at then
s o that 2CBE = 0. Let Ti CA
au acting at O are in equilhbrium, by Lami's
andAOB= 2C and
theorem
ensions along the strings
Mathematics-l
of Basic
Asmita's
NEB
Solution
STATICSUnit 12 237
236 of action of the
W SOLUTION
Let the forces 2, 4 and 6 units act as e
et the line
produced to
weight 45N|
meet AB at D. Let LACD = 0 so03.2071 Supp. Q.No. 14 OR
sin 90°
(0
-

0)) to the sides BC, CA and AB of an Paal 0. Let Ti and


-

sin(180- 6)
sin(180
an equilalera) 2CBD- be the tensions Let force F make angles a and
ß with
OY respectively. Prove that if directions
that
100 triangle ABC respectively. along the rings CA and CB respectively. OX and
and Fy
T Sinc Tz and 45N acting at C are are the components of the force in the directions
F
sin 6 cos of Ox
100 x
5= 28kg
-

wt equilibrium, by Lami's theorem, and OY respectively then


100 sin
=

T =
2 45 F. sin F sin o
CEin ECA sinACB sinaB) sin(a+) Also, if Q is
100 * 25 96kg-wt doubled
100cs
T:=
=

T2 45 and the new resultant is


perpendicular
Now, resolving the forces to P, prove that Q = R, where R is the resultant
Q.No. 13a OR along si +0)sin(180° -0) sin 90 of forces P and
9 2073 Set D of two forces Pand Q acting at
perpendicular to BC, we have Q. [6]
Find the
resultant
them isa.
the angle
between
x =2. cos 0° + 4. Cos
120+6.cos 240° sin45
cose sOLUTION
point when Let F be a given force represented by OC. We
a 13 a OR
=2x1+4x( cose, T2 45 sin 9
=

refer to2063 QNo. Ti= 45


Please
BC 4 0 4 have to resolve F in the direction of OX and

S0 2073 Supp Q.No. 13 Here, coseAB 50 OY such that XOC =a and LCOY
into two forces making = 2-2-3= -3 B. Let =

force P be
resolved usdraw AC parallel to OY and BC parallel to
sine AC
If that Y =2. sin 0°+ 4. sin 120° + 6.sin 240
a
directions, show
angles 45
and 15" with its AB 50 *a OX so that OACB is a parallelogram. Then
F = OA and F, = OB. Also,
41 2x0+4x
the laterforceis/P. Ti 45x =36N

Please refer to 2067 Q.No. 13a -5-3


-3 2-45 x=27N
1 2074 Set AQ.No.13a Law of forces. 4 Let R be the resultant force making an angleg
Triangle
8. 2076 Set CQ.No. 13a OR
State and prove
to 2058 QNo. 13a
with the force 2 units
Please refer - k theorem.
Then, State and prove ^ 41
32 2074 Set BQNo.13a R VX+Y2
Please refer to 2065 Q.No. 13a OR AC OB, OAC = r -(a + ) and LACO = B.
Lamí's theorem.
State and prove By sine law, we have
P l e a s e refer to 2057 Q.No. 13a -3)2+ (-V3)2 6 MARKS QUESTIONS
OA A OC
83 2074 Set BQ.No.13a OR =V12 2v3 units 100.2070 Supp. Q.No. 14 sin ZACO sin a sin (T - (a * B)
is
weight 10kg
of suspended by two
A body and tan 6 = = tan 210° (:X&\ Two forces equal to 2P and P respectively
act on
their other ends or,
strings of lengths 7m and 24m, of rod
a particle. If the first be doubled and the second sin B sin a sin (a+ B)
fastened to the extremities of a
being the body are both -ve) increased by 12N, the direction of the resultant F sin
25m. If the rod be so held that and
length find | Hence the resultant is 23 units at an ang is unaltered. Find the value of P. [6 sin (a+ B)
hangs immediately below its middle point, 13b OR
the tensions of the strings. 4 of210 with 2 units force. Please refer to2059Q.No. F sin a_
Please refer to 2066 C Q.
No. 13a or
98. 2075 Set BQ.No. 13a OR ho1.2070 SuPp. Q.No. 14 OR rysin (a+B)
Let P and Q be two forces acting on a particle, Next part
A body of weight 45 N is suspended by tw
S42074 Supp Q.No. 13a 40 cm respective whose directions include an angle of at40 or t) Let a be the angle between two forces P and
and 2 N respectively act at strings of length 30 cm and Q. Since Ris the resultant, so
Forces of 2 V3,5, V3
one of the angular points of a regular hexagon
and the other ends of the strings are attached Derive the magnitude and direction of their
R2= P2+Q2+2PQ cos a
the extremities of a rod of length 50 cm. The resultant. [6 (i)
towards the five other points. Find the the new
is so held that the body hangs immediate! * Please refer to 2063 Q.No. 13a OR Again, givern that when Q is doubled,
of the resultant. 4 resultant is perpendicular to P, so we have
magnitude and direction below its middle point. Find the tensionso 102 2071 Supp. Q.No. 14
Please refer to 2063 Q.N. 13a 20 sin aa
strings. Frove that if three forces acting on a particle are tan 90P+20cos a
95 2074 Supp Q.No. 13a OR in equilibrium, then each is proportional to the
P and Q acting at SOLOON 2Q sin a
Find the resultant of two forces
a point when the angle between them is a. Let CA and CB be the strings
so that CA sine of the angle between the other two. AlSo, |or, 0 P+20 cos a

40 and AB = 50cm. DOdy of weight w is suspended by strings of or, P + 2 Q cos a = 0


cm, CB cm
=

Please refer to 2063QN. 13a OR ength 3m and 4m attached to two points in the
50cm -
n e horizontal line whose distance apart is Sm.
96 2075 Set AQ.No. 13a D Tind the tensions
or, cos a 20
A body of weight 68N is suspended by two A along the strin8s. the value of cos a in equation (i),
40cm First Part: Please refer to 2057 Q.No. 13 a Putting
strings of lengths 8 m and 15m respectively and 30cm we get
the other ends of the strings are attached to two T2 Second Part:
fixed points in a horizontal line 17m apart. Find 180°-
T 90°+ lease refer to Model Set I, Q.No. 13a
R=P+Q+2P02
the tensions of the strings. 14
Please refer to 2072 Set D Q.No. 13a OR or, R= P? +Q3- P2
or, R = Q

87 2076 Set BQ.No.13a E 46N


Q=R
Forces 2, 4, 6 units act at a point in directions
parallel to the sides of an equilateral triangle
Here
304 + 402 = 900+ 1600 =2500 = 50
taken in order. Find their resultant. 14) ACB = 90"
STATICS (CcONTINUED) Unit 13 239
C. It is supported in
p r o p

by a force of 1 N
a
norizontal
acting vertically . 2061 Q.No..6
UNIT p o

upward
s i t i o n

at A. Let
DC x . Forces
act along
equal
to 3, 5, 3 and 5 newtons
the sides of a square taken in respectively
Then,
find their resultant. order,
1N

STATICS (CONTINUEDI -B
SOLUTION
Let the forces 3, 5, 3, 5 Newtons
act along the
sides AB, BC, CD and DA
respectively of the
.2058 Q.No.6 a -2m square ABCD. Also, let length of side = a
2 MARKS QUESTIONS Define a couple. What do you mean meters.
by. arm of a SN
couple?
2057 Q.No. 6 and the moment
of a couple. SOLUTION
Define a couple
of a couple mathematically. Couple x AC
Express the moment
Two equal unlike parallel forces not or, 3
x
DC=1
havin. 2)
the same line of action and acting on a or, 3 *x*IX{X*
SOLUTION body are said to forma couple. ngi or, 2 x = 2

Am of the Couple x1
Arm of a couple Now, the forces 3N along AB and 3N
he arm of the couple
is the perpendicular Hence, the distance of the prop from the
along
of action of the two The arm of a couple is the CD forms a couple of moment
distance between the lines perpendicuiar centre ofbaris
1m.
distance between the lines of action of the
forces forming a couple.
forces fornming a couple.
two 2060 Q.No.6a 3xa= 3a Nm
Moment of a Couple Again, the forces 5N along BC and 5N along
Find two like parallel torces, acting at a distance of
DA from a couple of moment 5 xa=5a Nm
2058 Q.No.6 25m apart, to a given force of
which are eguivalent The resultant of two couples is a couple.
A straight uniform rod is am long, when a load 3ON, the line of action of one being at a distance of
Hence, the resultant of all forces is equivalent
of 5N is placed at one end it balances abouta
point 25cm from that end. Find the weight of te
50cm from the given force. 2 to a couple whose moment

road.
SOLUTION (3a+ 5a)Nm = 8a Nan
Let P and Q be the two like parallel forces 10. 2062 ANo. 5E
sOLUTION acting A and B at respectively. Let the
Let AB be a rod of weight W N. The weight w Define moment of a force about a point. Givee
P resultant R act at C.
the geometrical meaning of the moment of a
The moment of a couple is the product of the N acts at the middle point C of AB. Ifa load of
By question, force about a point.
5 N is placed at the point A, then AB will be 12
magnitude of one of the forces forming a balanced at D where AD = 25 cm. Here, Please refer to Model Set II, Q.No. 15
couple and the arm of the couple.
Mathematically if the forces P, P are AB 3m=300 cm. So, 11.2063 Q.No. 6
Find the two unlike parallel forces acting at a
measured in newton and the arm p in metre 300
AC 2150 cm P R distance of 12 cm which are equivalent to a force
thenthe moment ofcouple isgiven by P p. C D AC - AD = 150 - 25 = 125 cm AB 2.5 m, AC =
50 cm =
0.5 m.
R=30N of 20N, the greater of the two forces being at a
2 2057 QNo. 6 Since the forces are parallel, Then, CB= AB- AC distance of 6 cm from the given force. [21
Replace a force of magnitude 50 kg wt by two 300 cm 2 . 5 -0.5 = 2m

like parallel forces one at a distance of 2m and Since the forces are like parallel, so
sOLUTION
P C B Let P and Q (P> Q be two unlike parallel
other at 8 m from the given force. (21 forces whose resultant (R) = 20N. Also let P, Q
-25cm>125cm
SOLUTION and Ract at the point A, B and C respectively.
Let P and Q be two like parallel forces acting
5N WN
Here, CA = 6 cm, AB = 12 cm and BC =12 +6 =18 cm

at A and respectively. Let R 50 kg wt be


B =
Now, we have
given resultant force acting at C. By given, 5 W P 30x 06N
AC= 2m, BC = 8m. Then,
AB = AC+ BC = 2+8 = 1 0 m
CD AD =
24N and Q=256N R 20N P

lence, the required forces are 24N and 6N.


Since the forces are like parallel, s0
o 2060 Q.No. 6E 12cm
2 m 8m Define arm of a couple and the moment of a 6 cm

W=1N
Couple.
R 50 kg lease refer to 2057 Q.No. 6a Q
5A uniform
2059 Q.No. 6b 2061 Q.No. 6a
CAB bar 4m long and weighting 3N p
in horizon a
A
traight
of 5N is uniform rod is 3m long. When a load
over a
prop and is Pported pwards
placed at one end it balances about a .
position by force of 1 N vertically
a
point 25cm that end. Find the weight the
P=8x 5= 40 kg wt he other end. Find the distance of the pro rod. rom

Q=5x 2= 10 kg wt the centre of the bar. 12 or, P o-


Please refer to 2058 No. bb
SOLUTION P 30N, Q =
10N
Let AB be the uniform bar of leng yerd
weight 3N acting through D. Let itpa
of Basic
Mathematics-
Asmita's NEB Solution
240 Since the forces are lke parallel, so STATICS (CONTINUED) Unit 13 241
20 cm - resultant Ract at where AC
Let the =
6m
12.2064 Q.No.6 long. When a load
m.
and R
= 9N. R P
Q=30+20 50 kg wt
uniform rod is 3 m
Also,
A straight
end, it about a
balances
Gince the
lorces
are like parallel, so
of 5N is placed at one

from that end.


Find the weight ot R
AC AB
point 25 cm 6 Cm- 12 cm
P 15 kg wt R
the rod Q 10 kgM
Please refer to 2058 Q.No.6b R P+Q= 15+10 25 kg wt R = 9N

13.2065 Q.No6 3N
passes
Also, or, AC= 16 cms
A uniform bar 4m long and weighting
Over a prop and
is supported in
a
horizontal

AB AB Hence the resultant is 50 kg wt and it acts at a


of 1N acting vertically distance of 16 cm from the
force point at which the
position by
or. 10
a
distance of

upwards at the other


end. Find the AC weight of 30 kg wt acts.
bar.
the centre of the 6X9 22.2070 Set DQ.No. 12 b
the prop from
refer to 2059 Q.No. 6b
Q=183

Please or, AC- Find two like


parallel forces acting at a distance
of 2.5 m
2066 CQ.No.6What do you Hence, the resultant is 25 apart, which are equivalent to a given
Q=3N

14. mean by arm of a kg wt


acting aa force of 30N. The lines of action of one
Define a couple. (2 distance of 8 cm from the force 15 ko wt 20. 2069 (Set B) Q.No. 12b being at
force of magnitude 48 kgwt by two distance of 50 cm from the given force.
a
couple? 2
refer to 2058 QNo.6a 17. 2068 Q.No.6a Please refer to 2060QNo. 6a
Please Find two unlike parallel forces acting unlike parallel forces, one at a distance of 2m 23.
15. 2066 QNo.6 distance of 12cm which are equivalent to a at a and other
at 8m trom the given force.
2 2070 Supp. O.No. 12 a
48 cms in length, rests If two
A straight weightless rod, of 20N, the greater of the two forces heie sOLUTION
like parallel forces are 16N and 12N, find
in a horizontal position between wo, PEES Let P and (P >Q) be two unlike parallel
their resultant
acting at a distance of 90 cm from
being| distance of 6cm from the given force,
placed at a distance of6 cm peg apart, one forces whose resultant is 48 kg wt. Let the
the greater force and the distance between the
of 2 kg is Please refer to 2063 Q.No. 6b
at one end of the rod, and a weight forces P, Q and R = 50 kg wt act at the points
forces.
the
from the other end. Find
suspended
pressures on the pegs.
18. 2069 (Set A)Q.No. 12 A, B and C respectively. sOLUTION
A straight uniform rod is 3 m long When a load forces P
sOLUTION of 10N is placed at one end it balances abouta
Here, CB= 8m and AC = 2m
Lettwo
B
16N and Q 12N act at A
=

Let AB be a straight weightless rod resting Since the forces from unlike parallel forces, so and respectively. Let R be the resultant
between two pegs at A and C so that AB =

48| point 25 cms from that end. Find the weicht af acting at C.
cm and AC = 6 cm. Then, the rod. R 48 P Since the forces are like parallel,
CB AB- AC=48 -6=42 cn SOLUTION R=P+Q=16+12 28N.
Also, by question AC= 90 cm, AB = ?
Let PandQ be the pressure on the pegs atA Suppose AB is a straight uniform rod whose
and C. The system is in equilibrium when a
A 90 cm
length is 3m 300 cm. Let the weight of he
=

weight of 2 kg be suspended at the end B. So, rod be W newtons. Then the weight W acs at
2 kg wt is the resultant of two unlike paralel the middle C of AB. If a load of 5N is placed
forces P and Q at the point A, then AB will be balanc 16 N 12 N

about the D where AD 25 cm. Hence


point
=

CD= AC - AD = 150- 25 125 cm. We have,


Since the forces are like parallel, so Q

300 Cm
or, 90
2 28
AB
D 12 m B P 8x8 64 kg wt or, 12AB = 28 x 90
25 cm 125 cm Q 8x 2- 16 kg wt 28x 10 cm
12210 cm ==
2im
2lm
21. 2069 Old (Set B) Q.No. 5b AB=
1N WN Oparallel forces of 30 kg wt and 20 kg wt are | 24. 2071 Set D Q.No. 2
a t a distance 40cm apart. Find their Two unlike parallel forces, the greater of which
Sultant and its position if forces are like. ratio of the
or, P=x 42 14 kg wt W is 75N, have a resultant 25N. Find the
SOLUTION distances of the resultant from the component
21
and Q-x 48 16 kg wt Let R be the resultant of two like paralel| forces.
orces P 30 kg wt and Q = 20 kg wt. Let P, O SOLUTION
and Rbe acting at Let P and Q be two unlike forces (P> Q)
16. 2067 O.No. 6a W 2N
Here, AB 40 cm
, B and C respectively.
at A and B respectively.
Let the
Find the resultant of two parallel forces of 15 kg 19. 2069 (Set A) Old Q.No.6
=
acting
S the forces form like parallel forces, so
p o i n s

resultant R act at C.
wL. and 10 kg wt acting at a distance 20 cm apart Two like parallel forces P and9d as
R 25N, Q= ?
By question, P= 75N,
=

in the same direction. be


40 cm
12) 18m apart. Ifthe resultant force Since the torces are unlike,
SOLUTION a distance of 6m from P, find Q.
Let P 15 kg wt and Q= 10
the points A and B
kg wt be acting at SOLUTION p a r a l l e
knw
l

two like
respectively. Let R be the Suppose P and Q be P
30 kg
resultant acting at C. Civen AB 20 cm. =
at the points A and
B such tna w R Q 20 kg wt
aeng
Mathematics-ll
Solution of Basic
242 Asmita's NEB

STATICS (CONTINUED) Unit 13 243


12x 24
xint p must be at distance of a
32. 2074 tBQ.No.126
P-18N R=36N
The from the end A from which (8 2) mA uniform beam AB is 8m long and weights
+
Q-R p
18 N ie. 10m.
Suspended.
weight25kg
from A
Weights of 10kg and 25 kg suspended
a
are
of 20 kg 1S and B
respectively. At what
point must
12¢C the beam be
R 25 N P 75 N
9. 2072 Supp Q.No. horizontally?
supported so that it may
rest
R-P-Q 1818 parallel
P forces
Q act at points and
2
or, 25 =75 Q
or, BC AC 18m.
Wart,if the resultant force be 9N and acts at SOLUTION
6 m from P, find Q.
Q= 50 N a point 2 4m
Also we have SOLUTION
AC: BC 1:1 Lettwo.like parallel forces P and Q act at the
point A and B. Let the resultant 9N acts at C on
27. 2072 Set CQ.No.12 ABsuch that AC 6m. Then BC
Two like parallel forces P and Q act at oints 18-6-12 m.
or. 18 Since the forces are like parallel, so
m apart, if the resultant torce is 9N and
acts ata
10 kg
5 kg 25 kg
distance 12 m from Q, find P. 6m 12 m B Let AB be uniform beam of length 8m and
a
or,

AC: BC=2:3
sOLUTION weight 25 kg. Also let the weights 10 kg & 25
Let two like parallel forces P and Q act at the kg are suspended from A & B
9N respectively.
25.2071 O1d Q.No.5 point A and B. Let the resultant 9N acts
at C
P Suppose that the beam AB is supported atD
forces of
40kg wt and 10 kg wt are on AB such that BC = 12m. so that AB remains
Two parallel horizontal. Here C is the
acting at a distance 40 cms apart. Find their Then AC = 18- 12 6m. BAC AB midpoint of AB.
resultant if forces are unlike. 2 Since the forces are like parallel, so Let AD x,
A 6 m Then, CD AD -AC =x -4
SOLUTION 12 m_
From last two ratios, we have BD AB- AD = 8-x
Let two unlike parallel forces P and Q (P> Q)
be acting at A and B respectively.
Now, taking moment about the point D,
resultant R act at C. Sinace the forces are unlike
Let 10x AD+25 x CD 25 x BD
9N
Q=3N or, 10x + 25(x -4) 25(8-x) =

parallel, so
Q 10 kg wt or, 10x + 25x- 100 200-25x
30.2073 Set DQ.No. 12 or, 60x = 300

Two like parallel


forces P and Q act at points
or, 18m Of,X60 =5
300
40 Cm apart, if the resultant force be 9N and acts at
a distance 6m from
From first and last ratios, we have P, find Q. [2 Hence the beam should be supported at a
Please refer to 2072 Supp Q.No. 12c distance of 5m from A.
R P 40 kg wt 1.2074 Set AQ.No. 12b 3 2074 Supp O.Na. 12
R P - Q = 40 - 10 3 0 kg wt P=6N Forces equal to 5N, 2N, 5N, 2N act along the like parallel forces P and Q act at points 18
Again, we have
28. 2072 Set DQ.No.126 hH ASideCB CD and AD of square ABCD
is 3m. Find the moment of the couple
a
m apart; if theresultant force be 9N and
distance 6m from P, find Q.
acts at a

A uniform beam AB is 16 m long and weighs 30 [21


AB kg weights of 20 kg and 50 kg are suspended that will give equilibrium. Please refer to2072 SupP.Q.N.12
from A and B respectively. At what point musi SOLUTION 34. 2075 Set BQ.No. 12
the beam be supported so that it may e e torces 5N along AB & 5N along CD from A straight uniform beam Im long, when a load
or, 30CA = 400 in anticlockwise direction. Hence the
horizontally? Couple
moment of this couple - 5N x AD
| point 30 cm from
of 24 kg is placed at one end, it balances about a
that end Find the weight of
CA 135 sOLUTION 5 N x 3m = 1 5 Nm the beam. [21
AB 1S a uniform beam whose weighnt a
Hence, the resultant is 30 kg wt acting at a through the middle point C such fthe force 2N along CB &2N alongHence
AD sOLUTION
AC CB = 8 metres. When the weignle couple
athe moment in clockwise direction.
distance of 13 cm from the of this couple
greater force. be suspended from A and
B and 50bekg that the bean
-2N x
3nm =-2N x AB= -6Nm
26, 2071 Supp. Q.No. 12 Deam supported at D so 30 cm 20 cm 50 cm

If one of two like


parallel
forces and their may rest horizontally, let CD=X. 24 kg W
resultant are 18N and 36N Taking moments about D,
ratio of distances of the respectively,
find the Suppose AB is a ro. Let the weight oft the rod
resultant from the be W kg. Then, the weight W kg act at the
forces. middle point C ot AB. Ita load 24 kg is placed
2
9OLUTION Now,
SN at the poùnt A, then AB will be balanced about
Since P and Qare like sum Ot
ot moments
i of the couples point D, where AD =30em. we
Then CD 20
parallel, so 15N
ISNm -6Nm
the
Since the torces are parallel, have,
50 9Nm =
cm.
20 50 ence of the couple that will give
20 AD + 50 CD 50 DB equilibrioment
quilibrium =9 Nm.
or, 20 (8 +t x)+50x=50(8- x)
Or, 16+ 2x + 5x = 40- 5x
STATICS (cONTINUED) Unit 13 245
Mathematics-ll

Solution of B a s l c
244
Asmita's NEB
SOLUTION
-2P 0 - tan 180 or, Ta43T
like
parallel forces and R Then tan0x
Let P and
Q be o be any point Rb
i The tension of the wire attached at C is W3
their resultant. Let 0 180°
times that of the tension of the telegraph wire.
o plane. Let us draw a line hrough 0 an
perpendicular to the lines of actions of
Auain, Jet Xthe
a t DE
resutant
and CD =a.
R AD at E such
cuts
Now, taking moment | 2 . 2063 Q.No.14 a
or, W
Q and R to
meet them in A, B and about E,
We have Prove that the
algebraic s u m of the moments of
W36 kg respectively. Since the forces are like c
P x DE+
Px EA -

Px CD =0 any two like parallel forces about a point in their


4 MARKS QUESTIONNS so
parallel, -P *x*F* (a -X)
-Px a =0 1s equal to the moment of their resultant force
or, Px Pa =
0 about the same
R P+ Q -Px + Pa
-

point
-

or,
Please refer to 2058 QNo. 14a 4
35 2057 Q.No. 14a of
the moments
andP. AC = Q. BC
... () or,-2Px =0
algebraic sum
Prove that the
w o forres, meeting
at a point
a b o u t any
of
x =0
DC.
43. 2064 O.No. 142
any to the
moment
The resul tant actsS along A light rod of
in their planethe
is equal
141 length 72 cms has equal weight
point
their resultant
about
same point
69. 2060 Q.No. 14 a attached to it, one at 18 cms from one end and
Provethat the algebraic sum of moments of two the other at 30 cms from the other end. If it
is
sOLUTION t w o torces
and R be their intersecting forces about any point in their plane uPported by two vertical strings attached to its
and Q be fig () ig (1)
Let P ends and if the
In fig ) , the algebraic sum of the moment of is equal to the moment of their resultant about string can not support a tension
resultant draw OCP
Let us P and Q about O greater than theweight of 50kg what is
Let O be a given point.
of action of Q at the same point. 14 greatest magnitude of the equal weights? the
parallel to P to meet the line
and AB P AO +Q BO
= P(AC+OC) Please refer to 2057 Q.No. 14a
+ Q (OC BC)
C Let AC represent Q n magnitude
40. 2061 Q.No.13 a OR SOLUTION
magrutude Now,
complete the OC + P AC -Q BC
representPm -(P+Q) Find the resultant of like parallel forces. Let AB be a light rod of length 72 cms. Let W
paralelogTam ABDC. R OC using (i) be the equal
resultarnt R of P and Q about O. Please referto Model Set I, Q.No. 14 weight suspended through Cand
Then AD represents the moment of R D such that AC 18 cms and BD
=
30 =

Join OA and OB. In figure (ii),


The algebraic sum of the moments of P and 0
41. 2062 Q.No. 14 a Also, CB = AB - AC 72 - 18 = 54 cmns
cms.

The wire passing round a telegraph pole is


about O Now, taking moments about B, we have,
horizontal and the two portion attached to the
= P AO-Q.OB
P ( A C + OC) - Q(BC - OC)
pole are inclined at an angle 60° to one another. -72 cm-
The pole is supported by a wire attached to the
OC
=P AC-Q BC + (P +Q) middle point of the pole and inclined at 60° to 1 8 Ccm
= R
OC using (i) the horizon; show that the tension of this wire is
about O.
moment of R 3 times that of the telegraph wire.
fig ()
37.2059 Q.No. 13 a sOLUTION or,
50 x AB = W x CB
50 x 72= Wx 54 + W
Wx DB
x 30
Find the resultant of two like parallel fores
The point O may lie outside the ZBAC in fig Let C be the
acting on a rigid body. midpoint of the telegraph wire
AB. Let tension of the
or, 50 x 72 84W
(i) and inside ZBAC in fig (i). The moment of
lease reter to Model Set 1, Q.No. 14 telegraph wire at A be
about O is 2 AOAB and is positive in both
Ti and tension of the wire
attached at C be T2 W kg
figures. 9 2061isC.No. 14 Resultant tension of the two
wire attached at A portions of the 44. 2065 Q.No 14
The moment of Q about O is 2AOAC which is
ABCD square along
a
AB, CB, AD and
magnitude o
positive in fig () and negative in fig (ü) equal forces, P act. Show that the Forces equal to 3P, 4P, 5P act
along the sides AB,
their resultant is equal to double of a *2* TI
In fig () cos 2T1 cos 30 2T, x=TBC
the
and
CA of an equilateral triangle ABC, ând
The sum of moments of P and Q about O
= 240AB +24OAC
component and acts along DC. Draw BE perpendicular to DC. magnitude, direction and line of action of
the resultant.
SOLUTION Now, taking moment about
B,
2AADB+ 240AC (. AOAB =
AADB)
Let equal forces P act along the sides AB, SOLUTION
2AOAD AD and DC of the square ABCD. Resovg Let a be the side of an
equlateral triangle
the forces along and perpendicular to CD
moment of RaboutO A50. ABC. Resolving the forces along and
in fig (i), have perpendicular to BC, we have
The moment of P and Q about O B T
=
240AB 240AC -

24ADB+ 2A0AC (. AOAB = AADB) P


240AD
monent of R about O.
3P
D o
36. 2058 Q.No. 14 a B 4P
Prove that the D 460
X= -P - P = -2P
algebraic sum of moments of two
like parallel forces, about any Y = -P+ P =0 Tiy3x AB-
point in their
,T3x AB =TiTi 4P cos 0 +5P cos 120° +3P cos 240°
x
BE =0 X
plane is equal to the
moment of their Let R be the resultant.
about the resultant Then = 2P x
BC sin
same
point.
14
R - X 4 Y = V-2P)+ 0 ,
Ti3 AB -T, 30 - 4P 1+5Px (). ()
Hence, the resultant is double of P .
E De the angle made by R wilth CD
4P-8Px-o
of Basic Mathematics-I STATICS (CONTINUED) Unit 13 247
246 Asmita's NEB Solution
- 1 x0+V
Y 4 P sin 0° + 5P sin 120°+3P
sin 240° 2070 (01d) Q.No. 6
Let R be the resultant. Then, -3the The resultant of two like
parallel forces is 12N
4Px0+5Px 3Px
resultant. Then and it acts at a distance 2
R-+Y-Vo+ (3P =yP Let R
be
force equal
meter from the
to 8N. Find the larger
Let R be the resultant. Then Let 0 be the angle nmade by the resultant
with R-X+ Y2-V(-1)+ (5= 2 resultant from the
distance of
smaller force. the
Again,Jet R make an angle 6 with AB. Then, 4
Y=yo+(y8
=
Py3 N BC. Then,
SOLUTION
R- inclined at an angle
with

If the resultant R is tan6-


Y-Y= o= tan 90° N3 tan 120 Let P and Q
(P> Q) be two like parallel forces
BC, then acting at A
and B
respectively. Let R be the
90° 0 120° resultant force
acting at C.
tan t a n 90 Let the resultant force meets the side C inD Itant is parallel to AC. Let
the P 8N, R =12N, AC 2m,ByBCquestion, =?
cut BC at D. Draw Since the forces are ike
90°
N is in the
where CD=x.
moment about C, have
resultant
AB and AC.
perpendicular parallel, so
Hence, the resultant R =PV3 Taking
PxCM- Rx
we
CD = 0, where CM is perpendicular
from D to
Taking moment about D, we have
Q-R-P=12-8 4N
to BC Now, we have
direction at right angles at D where DN 0
MD -1
=

cut BC produced to AB. 1 A 2m


Let the resultant
m o m e n s about D,
we
0
-V3Pxx=0 MD- DN
=

CD x. Now, taking
= or, Px BC sin 60° or,
CD sin 30°
BD sin 30
=

have r,
0 or, BD= CD
5P x DE-3Px DF= P 8N R 12N
0 Hence, the resultant bisects BC at D.
x) sin 60°
=

3Px (a +
or, 5P
x
xsin 30- or, x=5, where a is side of equilateral triangle

or. 5Px3P (a*3) 49. 2069 (Set A) Old Q.No. 14a


ABC is an isosceles triangle whose angle A is or, 1
or, 5x=3a+ 3 x
47. 2067 CQ.No. 14a 120 and forces of magnitude 1, 1 and 3 N act
Define parallel forces. Deduce the resultant of
forces. along AB, AC and BC, show that the resultant or, 8 2BC
two like parallel
or, x bisects BC and is parallel to one of the other BC 2m
SOLUTION sides of the triangle.
CD
3BC Parallel Forces (41 52. 2070 (Old) Q.No. 14a
If two forces act along paralel lines, they are Please refer to 2069 Set 'A', QNo. 15 Forces equal to 3,4,5,6 N
the sides of a respectively act along
5 2066CaNo.14E s u m of the
moments of
said to be parallel forces. 50.2069 OldSet B) Q.No. 13 square ABCD taken in order, find
the magnitude, direction and line of
that the algebraic
Prove Aman carries a bundle at the end of a stick which action of
Next part their resultant.
at a point, about any is placed over his shoulder. If the distance
any two forces, meeting Please refer to Model Set I, Q.No. 14
the moment of
point in their plane equal
is to
their resultant about the same point. [4] 48. 2068 Q.No. 14 a
between his hand and shoulder be changed, how SOLUTION
does the pressure on his shoulder Let a be the length of the side of the
72cm has equal weights change? 4 square
Please refer to 2057 Q.No. 14a A light rod of length ABCD. Let the forces 3, 4,5 and 6N act
from one end and the SOLUTION along
attached to it, one at 18cm AB, BC, CD and DA
46. 2066 Q.No.14E the sides AB, other at 30cm from the
other end; if it be Let AB be a stick of length I. Let C be the respectively. Resolving
the forces along and perpendicular to AB, we
Three forces P, 2P and 3P act along two vertical strings
attached tois position of the shoulder. If W be the weight of
ABC of supported by the bundle have
BC and CA of an equilateral triangle ends and if the strings cannot support
a tension
suspended at the end B and P be
direction and line of the pressure due to the hand at A, then the
side a. Find the magnitude,
greater than the weight of 50kg
what is the
action of the resultant. 4 4 shoulder is pressed due to the resultant of P 5N
magnitude of the equal weight? greatest and W. Let R be the reaction on the
sOLUTION shoulder
sOLUTION Csuch that AC N
Let the forces P, 2P and 3P act along the sides x(say). =
where
Let AABC be an isosceles triangle Since the forces form like
AB, BC and CA of an equilateral triangle
of
ZACB =30° parallel forces, so 225
LA =
120°. Then ZABC anu
side a, the forces along 3N
along and Now, resolving
Now, resolving the forces have
perpendicular to AB, we
perpendicular to BC we have
X=3-5-2
R
Y=4-6-2
P Let R be the resultant. Then,
3P
PW
CB AC RAB
R=XY =V-2+(-2y =2/2N
If the resultant R is inclined at an angle 0 with
AB, then
2P C D s 120
t a n - = 1 =lan225*
X = 2P cos 0° +3P cos 120+ P cos 240°
X =1 xcos 0°+y3x cos 150°+ 1 xCOs R
X ' Xand Y are both negative)
Since
-2P1+3P ( -1-1s().1() be lea Constant, so R depends on x. R will 0 225
BA produced at E
atwhen x will be greatest. But the Let the resultant R cut
2P greatest value « is 1. Hence R will be least where AE x. Taking moment about E, we
=

1- when the distance between the hand and the


xsin12
have
Y =2P sin 0° +3P sin 120° + P sin 240°
shoulde DA -6 x
AE =0
xsin 0°+3xsin 150*1**
x
is 4x BE+5
0°+]

and Y 1 equal to the length of the stc


Mathematics-l
Basic
248 Asmita's NEB Solution of STATICS ONTINUED) Unit 13
13a 249
+
x)+ 5a 6x =
0 56. 2076 Set CQ.No. of magnitudea SOLUTION
4(a parallel forces
-

or,
Two like Let P and be
points B of a.ndo
end A and Q like
x=5a areacting at the parallel forces
and B. Let their resultant acting at A
of length t. If two opposite forces each
. .

P + Q be
o then Then, acting at .
added to P and
53. 2070 Supp.Q.No. 133a
s
act along magnitude areaction at the new reen Proy
wts respectively of
Forces
of 6, 5, 4, 3 kg ABCD taken in order. Find
-

that the line tant will M 1


the of a square
sides
line of action b e displaced through distance
a
the magnitude, direction and xcos 0° X*
1 +V3 COs 150° +1x cos 120P
their resultant. P+Q
sOLUTION
-1x1+
sOLUTION wts respectively act
Let P and Q be two like parallel forces ak
Let the forces 6, 5, 4, 3,kg and DA.
at the points A and B respectively. Let R
AB, BC, CD
along the sides
Resolving the forces along
and perpendicular
their resultant whose line of action pa
through C. Here, AB l and R =P+0.
1-- or,

to AB, we have C and ) 1x sin 0° +y3 sin 150°+1 sin 120


x
x or, BC-P p+O

-10 1 If the
through
force P is moved
a distance
parallel to itself
x to A, then
the resultant
-
Let R be the resultant. Then
acts at D where
AA1 x =

P+S R R Q-S Then


45 Since the forces form like parallel forces, so, R-+Y-V-13+ V3j=2
Again, let R make an angle 0 with AB. Then, X

X=6-4 - 2 Y = 5 - 3 = 2 an8- -tan120


Let R be the resultant, then 0 120°
P
R=X =y2+22=W2kg wts
P Hence, the resultant is P+0
parallel to AC. Let the
Let be the angle made by the resultant R| CB=p+o resultant cut BC at D. Draw
with AB, then When the forces ^ and-S be added to P and 0 from D to AB and AC.
perpendicular BD AD BD-AD
Q
respectively, let R' be the new resultant whose Taking moment about D, we have
tan =1=tan45 P AB
OT. BD P+Q
line of action passes through the point C. 1 MD -1 DN =0
45 AC.
Now,
or, MD- DN = 0
or, BD-p+Q AB
Hence, the resultant is parallel to R' (P+S)+(Q-S)=P +Q ot, BD sin 30° = CD sin 3 0 . .

E where
Let the resultant cut AB produced at or, BD=CD Now, the required distance
the side of Then,
BE x. Let a be the length of
=

Hence, the resultant bisects BC at D. CD BC BD


square ABÇD.
E, have 8.2069 (Set A) Q.No. 15 on P P
Now, taking moment about we
P+Q AB-P-Q A:B{using () and (i)]
Find the resultant of two like
-5 BE + 4x BC+ 3 x AC= 0
or, P9 parallel forces.
P
or, -5 xx+4xa+3 (a+x) =0 6 P+QAB- A1B)
Or,-5x* 4a+ 3a+ 3x = 0 "B-+S) Please refer to Model Set 1, Q.No. 14
or, -2x= -7a CBP+Q
Now,
53.2069 (Set B) Q.No. 1 P
P-O AA
or, x orces equal to 3p, 4p and 5p and along the sides
CC C'B- CB
AB, BC and CA of an
equilateral triangle ABC, P+Q
a
B E = w h e r e a = side of a square
F find the
magnitude, direction and the line
action of the
resultant. or2.
6
2070 Set DQNo. 15
54.2071 0ld QNo. 13 b Please refer to 2065 Q.No. 14aa
Define moment of a force about a point. Prove
Prove that the algebraic sum of the moments of 6 MARKS QUESTIONS 0. 2070 Set CCQ.No. 15 that the algebraic sum of the moments of the
moment of two ike parallel forces about any
any two parallel forces about any point
in their
57 2069(Set A) Q.No.15 Define 1moment of a force about a point. Prove
planeis to the moment of their resultant
equal whose angleNad that the point in their plane is equal to the moment of
about the same point. 14
ABC is an isosceles triangle
120° and forces of magnitudes, 1,1 andya inters algebraic sum of the moments of two | their resultant about the same point. [6|
eCting forces about any point in their
plane FirstPleasepart
Please refer to 2058Q.No. 14a
and BC; show
that the resui equal to the oment of their resultant refer to ModelSet 1QNo. 14 OR
along AB, AC one ofthe
r side
otne
the same point. about
55.2071 Supp. Q.No. 13a DIsects BC and is parallel to
Case refer to Model Set II Q.No. 15
Second part:
Please refer to 2058 Q.No. 14a
Forces 1, 2, 4, 5 kg wts act along the sides of a of the triangle.
square taken in order. Prove that their resultant SOLUTION he 2070 Set C 15 on 53.2071Sot GONa 1
is parallel to a diagonal and find where it cuts isosceles triang and Qare like forces p, 2p and 3p act along the AB, sides
the side along which the first force acts. 14
Let AABC be an parallel S e parallel forces. If P is moved Three
to eitself equilateral triangle ABC. ind
ZABC ZACB =30 lf
through distance x, show that BC and CA of an
LA 120. Then the
a
Please refer to Model Set I, Q.No. 14 OR forces ao heresultant of .the magnitude,
and line of of direction action
NOW, resolving
have
P and
tF an Q moves aa distance
Qmoves distance p+Q the resultant
61
perpendicular to AB, we
14a
P l e a s e refer 2066 Q.No.
6
Mathematics-l
F be a force and p be the STATICS (CONTINUED) 13 251
of Baslc
250 Asmita's NEB Solutlon
istance of the point O from AB then the momen
T h u s
perpendicular A C
Fx
F OM = F xp
x ON nent
O is B
84.2071 Set CQ.No. 15 ORunlike parallel forces. or, Qx CB =S x OC..(ii)
about

ofF Varignon's Theor


resultant of two we have
Find the
end of a stick
75 From(i) and (ii), The algebraic sum of the momennts of any two
bundle at the CB
Px CA =Q*
camies a
A man
on his
shoulder. What forces about any
point in their R
cm long which is placed
his hand and to the moment of their plane is fig 6)
Q
should be the distance
between
pressure on the A equal resultant about In fig ), the fig (i)
order that the
Pand Q aboutalgebraic
the same point. sum of
shoulder, in ie. C dividends AB externally in
in inver
inverse tatio O the moments of
times the weight
of the Proof: Let P and Q be two forces and let R be
shoulder may be three [61 of the forces.
ultant. Then the following tw P AO Q. BO
bundle? Second Part
Let the bundle be placed at the end a.
P(AC OC)+Q(OC- BC)
SOLUTION be two unlike parallel
A of a
stick AB 25 cm which rests on the shoul
arise
Case I:When Pand Qrmeet at a point (P+Q)OC+ P AC-Q BC
Let P and QP> Q) A and B
R OC
acting at
the points
at C and the man holds at the other
end et O be a given point. Let us draw c using
moment of R about O. )
foroes the forces be
Let W be the weight ot the bundle and P hoB
on a rigid body. oarallel toP to meet the ine of action of O at In figure (ii),
respectively
the lines AF and
BI. Join AB. the shoulder. So, the 1et AC represent Q The algebraic sum of
represented by and opposite pressure
on
in magnitude and AB about O
the moments of P and
Q
introduce hwo equal BC ? represent P in magnitude. Now, complete the
=

Let us BA and AB PSince3W, - P-AO-Q-OB


to S along the forces are parallel, so parallelogram ABDC
forces each equal
two forces being equal P(AC+ OC) -
P AC-Q BCQBC-OC)
These Then AD represents the resultant R of P arnd
respectively.
upon the system.
have no effect
and opposite
forces be represented by
AD and = R.OC (P+Q) OC
Let these
the parallelogram
AFED and Join OA
and OB. [using i))
moment of R about O.
BG. Complete D
AE and HB be
BIHG. Let their diagonals
to meet at O. Through 0,
draw OC
P :3W
68. 2072 SetD a.No. 14
produced W Define
BI to m e t BA coplanar forces. Forces equal to P, 2P, 3P
parallel to AF or be produced and S at A W P
and 4P act
along the sides of a square ABCD
Now the forces P BC AB
produced at C. AE. Let its taken in order, find the
have a resultant Ri represented by
R
and the line of action of magnitude,
direction
be transferred to O.
point of applicationforces and S at B have
In
W.3W
or BC75 the resultant 16
the same way, the Q
BH Let its point
SOLUTION
resultant R represented by or, BC = 2 5 Coplanar Forces
transferred to O. Let the A
of application be fig () fig () system of forces whose lines of action lie on
force R: be resolved, into components Fana Distance between the hand and the shoulder The point O may lie outside the the
directions. is 25 cm. <BAC infig
same
plane are called coplanar forces.
parallel to their original () and inside 2BAC in fig (i). The moment of Next part
P about O is 2 Let the forces P, 2P, 3P and 4P
65, 2071 Set DQ.No.15 AOAB and is positive in both act along the
Prove side AB, BC, CD and DA
Define moment of a force about a point. figures. respectively.
that the algebraic sum of the moments
of two The moment of Q about O is 24OAC Resolving the forces along an
perpendicular
intersecting forces about any point in theirplane positive in fig (i) and negative in
which is to CD, we have
is equal to the moment of their resultant about fig (i)
In fig((i)
the same point. The sum of moments of P and about O
Q
P l e a s e refer to Model Set II Q.No. 15 2AOAB 2AOAC
66.2072 Set C Q.No. 14 2AADB+2AOAC (. AOAB= AADB)
Three forces P, 2P and 3P act along the sides A 2AOAD
BC and CA of an equilateral triangle ofside
P
moment of R about O
find the magnitude,
direction and line ofactou In fig (i),
DA The
of the resultant. moment of P and Q about O X= 3P- P =2P
Please refer to 2066 Q.No.14a 2A0AB -2A0AC Y 4P-2P = 2P
Let 0 be the angle made by R with AB, we
67. 2072 Set CQ.No.14 OR 2AADB+2AOAC (.AOAB=AADB)
Similarly, Ra is resolved into components Q
moment. State and prove Varignon 2AOAD have
and S each acting at O. Now the two equal Define
and opposite forces S and S at O balance each
moment of Rabout O. tan9 1-an 45
other. Finally, we have the resultant P Q SOLUTION ase Il: When P and Q are
and Q be like parallelparallel
45°

acting along CO ie. along the direction of Moment forces and R be Hence, the resultant is parallel to CA. Let the
greater force. of a force about given pul
a ur resultant. Let O be any resultant cut CD produced at E where DE = x.
The moment
thefonce point n tne
Since the triangles OCA and EDA are similar, the product of the nitude of intfron DerLet us draw a line through O arnu Let CD a . Taking moment about E, we have

ne perpendicular
distance of
the po
Pedicular
torces P, Q to the lines of actions of tne P xDE+ Px DA+2Px CE =0
and R to them in A, B and C or, x+1 xa* 2(a+ x) =0
the line of action of the force. espectiv t
meet
OC O R y . Since the forces are like parallel or, -2x-3a
or CA P+Q
andP. AC Q BC
or, Px CA =Sx OC.(i) ()
Again, since the triangles OCB and B!H are
ie. DE-CD
similar,
M
252

~)o72
~
Asmlta's NEB Solution of Basic Mathematics-II

set Ea.No.]
Find the resu
]
Itani of two like parallel forces .

.. Please refer to M0del Set I, Q.No.


§;:2072Set E Q.No.15~
14

ABCD is a square; along AB, CB, AD ~nd DC


[6) -- Moment
The momcnl <:f_a force nho ut a given .
the product ol the ma gnil'u d e o f U,e f P01111 is
the perpendicular distance of the Otce and
the line of action of the for.:e . Jloh,t frol\\
T~ms if F be a fore~ and p be th<c> perpendi
distance of th_e pomt O from AB then the cular ~
DYNAMICS
UNIT 14
equal forces p act; show that the magnitude of 1110
-1 1 • 1 :■i:H..i•@is=•i!@
their resultant is equal to double of a~y
components and acts along DC. l l
of F about O ,s F " OM = F >< p
Geometric Meaning of the Moment of a F0
Last Part
111in1

rci pt·iH•Mih•MM
-- or, (I+ 12) (t - 2) = 0
Either I= -12 (not possible)
... Please refer to 2061 Q.No. 143 or 1=2
Please refer to 2058 Q.No. 14a
~ Q.N_o. 5 a)_ 1=2s
!7:1 . 2012suppa.No.~ [_7. 2074Set B Q.No.14! .__ A train moving with a velocity of 360 km/hr has
~ 2073 Set D Q.No.12d

--
Define moment of a force about a point. 'What
Define moment of a force about a point St the uniform acceleration 40 m/s2. Obtain the
does it represent geometrically? Prove that the
algebraic SUl1'l of the moments of two
intersecting {orceS about any point in their plane
prove Varignon's theorem for two int~te ~d
forces. est111g
6
distance covered by the train in½ minute. [2)
An aeroplane land
of 1~ Ian/hr. If th: :!e a
rum"'.ay with velocity
the rate of '15m/slc. fin vel~ty slows down at
... Please refer to Model Set II Q.No. 15 11
is equal to the moment of their resultant about the aeroplane bef, d ~e distance covered by
the same points. [6) Ifs. 2074 Supp Q.No. 1~ Initial velocity (u) = 360 km/hr BB ore commg to rest. 121
.. Ple.ase refer to Model Set n, Q.No. 15 Define moment _of a force about a point. Prove 360 x 1,000
that- the algebraic sum of the moments of tw =~=lOOm/ s
~ 2073 Set C Q.No. 1§ like p~llel forces about any point in the:
Here, u = 108 k m / h60>< r =60~ m/ s =3-0m/ s
Acceleration (a)= 40 m/s2
Define like and unlike parallel forces. A man plane 1s equal to the moment of their resultant v =0m/s
carries a bundle at the end of a stick which is 1
about the same point. 16!
Time (t) = 2 minute = 30s a= -25m/ s2
placed over his shoulder:, if the distance between ... First part: Please refer to 2062 Q.N. Sa s=?
bis hand and shoulder be changed how does the Distance covered (s) =?
Second part: Please refer to 2058 Q.N. 14a We have, We have,
pressure on his shoulder change? [6]
vz = uz+ 2as
l79. 2075SetAQ.No.1 ~ ½at2 = 100 x 30 + ½" 40 " 302
mJmiD P and Q (P>Q) are two like parallel forces acting
s = ut + 0 r, ()2 = 302 + 2 · (-25) . s
First part Two parallel forces are said to be
at A and B. Show that if they interchange = 3,000 + 18,000 = 21,000 m = 21 km 0 r, 50s= 900
like when thev act in the same direction and
they are said· to be unlike when they act in
positions, the point of application of the ~ 2069 Old (Set B) Q.No. 6ai s=
50
900
=I8m
opposite direction. resultant is displaced a distance P + Q AB.
f.::..2 j6j A ~ar moving with a velocity of 15ms·t has a
uniform acceleration of 2ms•z. If it moves fo 25 Required distance covered= 18 m..
Next part ... Please refer to Model Set II Q .No. 13a sec, find its final velocity. r 12I ~ 2015.S.C Q.No.12d

--
Please refer to 2069 (Old) Set B Q.No. 13b
fr3. 2073 Set D Q.No. 15! ~o. 2075 Set A a.No. 15 o~ mmmmi A ~ar moving with a velocity of 20 ms-• has a
Three forces P, 2P and 3P act along the sides AB, Given, u = 15 ms-1 a= 2 ms-2 uniform acceleration of 2 ms-i. If it moves for 25
Forces equal to P, 2P, 3P and 4P act along the BC and CA of an equilateral triangle ABC_of se c, find the final velocity. [2)
t= 2.5 S V =?
sides of a square ABCD taken in order. Find the
side a; find the magnitude, direction and the h;e We have,
magnitude, direction and the line of action of
the resultant. (6)
of action of the resultant. II V = U + at = 15 + 2 >< 2.5 = 20 ms-I Given,
'- Please refer to 2066 Q.No. 14a --- ~ Required final velocity is 20 ms-1. Initial velocity (u) = 20ms-•
- Please refer to 2072 Set D Q .No. 14
174. 2073 Supp Q.No.1 4! ~1. 2075 Set Ba.No. 14! entsof 0 072 Supp a.No. 12~
Acceleration (a) = 2m5-2
Time taken (t) = 2.5 sec
Prove that the alge braic sum of the ~o~ ihe~ A cycli 8 t travelling with a velocity of 72 kny'hr
accel Final velocity (v) = ?
Define parallel forces. Deduce the resultant of
two intersecting forces about a point in uant dista~::tes at the ~te of 4m(sz until it describes a
two parallel forces. (6) We have,
plane is equal to the mome nt of th eir resu [6\ - - o f 48 m . Ftnd the hme tilken. [21 V = U + at
,_ Please refer to Model Set I Q.No. 14
about the same point. 15 = 20+ 2 ><25
Ifs. 2073 Supp Q.No. 14 oij '- Please refe r to Model Set 11 Q.No.
u -_ k 72 x 1,000
=25 ni- 1
Geometrically interpret moment of a force. Also Here, /
~2. 2075 Set Ba.No. 14 o]I 72 ·m / h r = ~ = 20m s
1 2 4 ,11 SI~ t·i·{:fi·l 11¼ili•ltF'
state and prove Varig.non's th.eorem.
... Please refer 2072 Set C Q.No. 14 OR
Define the mom ent of a force . forces • 1 ~ ;1'~n a=4 m/sl
2057 Q.No. 13 ~
!76. 2074 Set A Q.No. 14!
wls act along the si d es of a s~uar:rallel 10'
15 s =48 m ~
order. Prove that th e ir res ultant P lde a!or.l
th 5 I= ? If <l. b, c be the space described by a particle
D efine the mom ent of a force. Prove that the diagona l and find where it cuts e I~ We h,we, . du
ring the p11,, q11, and~ seconds of its motion
algebraic s um of the mom ents of two p,1ralld which the first forc e acts. 14a QI< res pectivdy, prove that:
force s about a point in their plane is eqU.ill to the '- Please refe r to Mod el Se t 1 Q .N°· s"' lit +2a
'I t2
a(q
- r) + b(r - p) + c(p - q) = 0. [4)
moment of their res ultant abo ut the same point.
16) Or, 48 ,. 20 . I + !
SIii
Let u and f be the initial velocity and the
Or, 48 "' 2. . 11 uniform accelera tion of the particle
Or 20t + 2tl espectivdy. By question, we have
r
, tl i- 'I0t - 21 • Q
f Basic Mathematics-II
254 Asmlta's NEB Solution o DYNAMICS Unit 14 255
v,Jc hilve,
' " u + at fg 2066 C Q.No, 13 ij
V .,o+ £11 If a: b, -c be the space described by a particle
or, "• v ... (i) dunng the p,., q,. and i<" seconds of its motion
or, a =u + (~)r 2
.. (i)

[ 20so ·Q.No.1HJ
11 · "' (
p.gaiJl,
respectively, prove that
a (q-r) + b (r-p) + c (p-q) = O. (4)
Similarly , .__ vi "'u2+ 2as -.. Please refer to '21'157 Q.No. 13 b
2 ,.()2+2fs1

(~) ..(ii) A body moves ~or 3 se~ond.s with ~ constant ~- . 2066 Q.No. 13 ij
=u+ -2 f acceleration dunng which 1t descnbes or,"- v2
24 30 ... (ii) If a: b, c be the spaces described by a particle
metres the acceleration then ceases and dufbi SI "'2f
the ne~ 3 seconds, it describes 21.60 metre g during the pd', qd' and i<" second of its motion
=u+ -2r-
2- 1) f
.. . (iii) for BC respectively, prove that
( Find the initial velocity and the acceleration. j· V ='U + at
14

'I
r~~- = a(q - r) + b(r - p) + c(p - q)
lBIIB
Let u and a be the initial velocity and u.nilorni
or, 0
· b -f
"'!,-
, _y_
fti
... (iii)
a(q-r)+b(r-p )+c(p-q)=O
-.. Please refer to '21'£J7 Q.No. 13 b
fi
4• 2069 (Set A
l Old Q.No. 13ij
[4)

ii
={ u+!?.£2- 1-¼} (q-r) {u + + ¥}<r-p) +
acceleration respectively. For first
s =24.30m
35
'
.. d vi "' u2 + 2as
: 0 .,v2-2fs2
Prove that for a particle moving with unifonn
acceleration 'a' in a straight line is
We have, v2 .
1.?f; J.4} (p - q) [using (i), (ii) and (iii)]
{u+

i
u (q _ r) + (2p _ 1) (q - r) f + u(r - p) +
=
1
ut + 2 at2

or, 24.30 = ux3 + !2 a x 32 1


S2 "'Zf
Adding (i) and (ii),
V V
...(1v)
a= 1811 + t' where s is the space described in

1i+ 12 =1+r t sec. and s' during the next f seconds. (4)
1 or, 6u + 9a = 48.60 -.. Please refer to 2059 Q.No. 14 a
j (2q - 1) (r - p) f + u(p-q) +2(2r - l)(p-q)f
2u + 3a = 16.20 .. (i) &s. 2070 (Ord) Q.No. fl ij
Let v be the velocity at the end of 3s. or, 1=v(7+i) (': ti+b=t)
u(q-r+r-p+p-q) + ½f {(2p-1 ) (q-r) + (2q-1) (r-p) A body moves along a straight line with uniform
v =u + at= u + a x 3 = u + 3a t acceleration. The !,ody coven a distance of 18 m
+ (2r-1) (p-q)l which is the initial velocity for next 3 s. or, v=11 ...(v) in the first three seconds and 22 m in the next 5
For next 3s, t = 3s, s = 21.6m, a= 0 7+r seconds. Find the velocity at the end of 10
0+ ½f (2pq _ 2pr - q + r + 2qr - 2pq - r + P + Wehave, · seconds and the distance covered in 1~ serond.
Again, adding (ii) and (iv)
2pr-2qr-p + q) 1 vl v2 [41
s=ut+ at2
1 2 St+ s2=2f + 2f ~
I 2 fx o = 0 = R.HS.
or, 21.6 = (u +3a)x3
1
+2 x Ox 32 Let u be the initial velocity and a be the
fr. 2059 Q.No. 14 ! or, u + 3a = 7.2 ..(ii)
uniform acceleration of the body. For the first
3 sec, s = 18 m, t = 3s.
Prove that for a particle moving with uniform
Solving (i) and (ii), we get

I
We have,

acceleration l in a straight line f = t + t'


1t1
u = 9ms·1 and a = -0.6 ms- 2
Initial velocity (u) = 9ms·1
Acceleration (a)= -0.6 ms·2
-~{t~tf (H) [using (v)]
s
1
= ut+ 2at2
1
or, 18 = u -3 + -a · 32
where s is the space described in t secs and s' !· 2062 Q.No. 13 bf
=--ti_ _
2
during the next t' secs.
!EIEl)
(4) A railway train goes from one station to anot~er
movmg. dunng . the .
first part o f th e J.oumey withff
2(½+}) 2u + 3a =12 .. (i)
Let v be the velocity at the end of 3s. Then,
Let u be the initial velocity of the particle. uniform acceleration f, when steam is shut ?h v = u + al = u + 3a
Then and the breaks are applied, it moves wt~e or, t2=2s(½+}) =2s(:/f') This will be the initial velocity for next 5s. For
1 retardittion fl . If 'a' be the distance between . . next 5s, givens= 22m.
s= ut + at2
2
-- ~
stations, show that the time the train takes is,

()
We have,
-- t = ~
1
t=u + 21 at
5
... (i)
✓ (4] s = ut+ 2at2
fl ~64 Q.No. 13 ij .
Again, let v be the velocity at the end of time t d ' b, c, are the spaces described by the particles 1 a .S'-
or, 22 =(u + 3a) . 5 + 2.
U1en v = u + al which is the initial velocity for !E!PJlr+I C At tlW uring the pta q111 rd' seconds of its motion
Let the railway train goes from A_to · . u111 :especlively, pr~ve ;hat: or, 10u + SSa = 4-1
the second part of the journey. So, for second . t Bis maxu 11 .. .(ii)
part of journey, we have time of journey, the velocity a . . L ,t ti bL' ~q - r) + b(r - p) + c(p - q) • 0. (4] Solving (i) and (ii), we have
1 and the velocity a t A and Care ,:e~~ tJ~e tin11' ~ s e refer to 2057 Q. No. 13 b
.1 3m/s2
u= 33 msama-5
/
s' = (u + at)t' + at' 2
2 the time taken from A to B and 12
taken from 13 Lo C. Then,
p;~ 2084 Q.No. 13-b/ 2065
Q,No 13 5
s' 1 "lho1 fo, a paoid, mo•ing wi~•iln Velocity at the end oflOs
or, t' = u+at + at' ... (ii)
2 AB+ BC = a 33 m/ s,
and ti + 12 = t (suppose)
u=5 a --~
5 m/ s2, 10
s,
Subtracting (i) from (ii) . . d be tl11 '
Also let AB .. s1 and BC = St an II icceJer .
v=?
s' s •hon a in a straight line a "' t +t ' 1


veloci ty al B. Where
t'-t .
Qur111 s 1s the space described In t seconds d s'
an (4.)
" Pt~ th e next t1 seconds. ·
~c refer to 2059 Q.No. 14 a
M thematics-II
Asmita's NEB Solution of Basic a
256
we have,
I! DYNAMICS Unit 14 257
We have,
u I ,4'1set C Q.No. 1
=_ll+ (~) xJ0=2.6 nt/ S o. b, c be the sfaces described by a particle ~ - 2074 Set A Q.No. 1ij
V = U + al 5 5 £
or, I u J{ ~, tlte p•h, q , rh seconds of its motion A bus starts from station A and stops at station
. iance covered in 10'k second J11r!J1!iively, prove that: B. The Velocity increases uniformly till it
D1s -2 1000 1000 x 9
• 10s,
=sm33 / a ="7nt/ s·, 1 = 100 = 100 rcspe ) + /J (r - p) + c(p - q) "' 0 [61 reaches maximum velocity v and then decreases
u s, :, p(I/ ~ r se refer to 2057 Q.No. 13 b uniformly . Show that the time taken by the bus
s. = ? 9 .. pJ~ea~ ~~~=.: ==;;=;t ------ - - -
= 90 seconds= 1 minute 30 seconds ...-,::::::::::071 Su • Q.No. 1 to run from A to B is ~ where x is the distance
We have, 1 2
• tarts from station A and stops at station
S.
21-1
=u+-2 -a W:t·i·I :ti-ilf iii Ul~f1 t,us 8velocity
,.• fhe • •
increases um'formI y ti'II it reaches
between the two stations.
... Please refer to 2CY71 Supp Q.No. 15
(6]

~o =¥ + cxrl)(f) =t-s=s 33 19 14 ~ 2070Set CQ.No.1 ~ .._


A railway train goes.from one stati~n to another
moving during th~ first part of the Jo~mey With
IJ,

1111Jfo
1 111 11111 velocity v and then decrease
fill!% rmlY show that the time taken by the bus
' 2x
run trom A to B is -; where x is the distance
2"071 Sec A Q.No. 1◄
pi.
A point moving with uniform acceleration

I
= 28m uniform accelerat ion a; wh~n st~am 1s shut off
10
t,eiween the two stations.
describes in the last second of its motion of ls
I ' &6. 2070 Set D Q.No. ij]d
If a, b, c be the ~
during the P'"-'l'"•
respectively, prove that - 0
=ds .bed by a particle
of its motion
and the brakes are apphed, 11 moves with
retardatio n at. If s be the distance between the
stations, show that the time, the train takes
~
'Let
the point C.
.
the bus reaches its maxllllum
.
velocity at
(6) the whole distance. If it started from rest, how
long was it in motion and through what distance
did it move, if it described 15 ems in the fust
second?
(4} (6)
a(q-r) + b(r- p) +c (p57-Qt- 13 b -~
's'\Jaa t . [6] AI-
I I IDJBI
.. Please refer to 205 · 0
·
t1, S1 C B
.. Please refer to 2062 Q .No. 13 b Let s be the total distance an'd t be the time
6z_ . 2075 Set B Q.No. 13g .
E 2070 Supp. Q.No. .1ij Now, from A to C taken.
.
from rest, moves with urufo~ 9• Let, t "' t,, S = S1
A car ~ g d desa:ibes the first kilomete r m By question, distance covered in last second
A body starting with initial velocity of 15nysec, We have,

-
accel~bo n ~ ·1 now moves with uniform moves with a uniform accelerat ion of Sn\fsecz. 9 9s
3 mmutes. 1
will .1 take to describe
v=u + at =25 X S=25 ... (1)
velocity, how long i. What is the velocity after 10 sec? v=O+ ah
l [4J ii. How far will it in 10 sec?
If a be the uniform acceleration and u be the
another kilometer? . V
initial velocity then u = 0.
iii. What will be its velocity when 1t has ti=; ... (i)
Distance covered in last second
11cm 1 km traveled tom? And,
What will be the distance moved in the 10~ 21-1
~ [6]
vi= u2+ 2as =u+--a
2
... (2)
second? or, v2= 0 + 2as1
u=O 1
t = 3 min.utes B!I& v2
s1=i;;
and s = ut + 12
2 • .. (3)
FllStpart Given, u = 15 m/s a= 5 m/s 2 ... (ii)
Substituting (2) & (3) in (1) , we get,
u =O
i. t = 10 sec Again, from C to B
= 1 km= 1000 m
v=? Let t = ti, s = s2 u+ (2t-l)a = 9 [ ut + 1 at2]
25 2
e; 1)a =i
=3minutes 2
We have, We have,
= 3 x 60 seconds = 180 seconds
v =u+at= 15+5x1 0=65m /s v =·,u + at
We have, or, O+ [o.1+½a t2]
ii, t= 10s O= v + (-a)ti
1
S = ut + 2at2 s=?
12=-
V
We have, a ... (iii) or, -2t-1)
- a= 9 at-,
or, JOOO=O xt+
1 '
x ax]8()2 1 And, ( 2 50
2 1
a=ut+ 2at2= 15x10+ 2 x5x 102-- 4QOm
v2 = u 2 + 2as or, S0t - 25 = 912
or, 1000 = 16200a
iii. S"' 10m or, 0 =vi+ 2(-a)s2 or, 91: - SOt + 25 = 0 ~ - - - -
1000 5
a =16200 =81 v=? vl -(-50) + ✓ (-50)2-4.9.25
S2=- or, t
2a ... (iv) - 2.9
Again,
We have,
V = u + at v2 = u2+ 2as Adding (ii) and (iv) 50 ±40
=-1-s-
v2 = 152 + 2 x 5 x 10 vl vl
5 SJ+ S2 =-+-
=0+ x1 80 or, v 2 = 325 2a 2a Ta.king +ve sing
81
50+-IO
100 v ==.../325 5'{0,
m/ s. . "'?
2v2
x"'"i;=~
yl
(" .· total distance = .r) =ts
=9 iv . Distance traveUcd in 10th scco nd (s,o)
t -= 10 s yl = 5 sec
Second part a=c - Again, taking -v~ sing,
100 We have, .r
u "'gm,s ec 50 - 40 .
2t-1 Again from (i) and (iii) t =-18-
61 "" u +- -a
a= J km= 1000m 2
=? 2>< 10 - 1 _lQ = ~ < 1 (not possible)
s10 .. 15 +--- - - x 5 >=62.5 rn --is 9 . .

["uf]
2 .
Tot 1 . 2v 2v Again, the, point describes 15 cm m the first
i.l time taken = -;;- = vi/.r . ,, X
second,
.. ~
V
DYNAMICS Unit 14 259
Asmlt•'• NEB Solution of Basic Mathemattc1-II
258 point 0 f projection, total displocen,c111 11 6.4"' 64 -176.4
11 or, 0 = 4 - 10 x t
ill I S~'CS, ( )- 0 vr, ; u ,. ·t 76.4 - 116.4
1 Here, u = 9.8 ms· 1, g = 9.8 ms-2
s = ut + 2 at'
We have,
of, 6U = 60 t =¾s
or, _ 2,Q = 10
Again, we have
or, 15=0>< 1+½>< a" 1' h = ut - ½gt
2 or, u - 6 y 2 = ul -2gh

or, 15= 2
a
or, 0 = 9.81 -
1
2 x 9.8t2 t&!#.NO,~
. u = 10::::m
9
:::::::/: = = s , - - - - - - - - ~ or, 0 = 42 - 2 x 10 x h
or, 20h = 16

I, a =30cm / ="'
Required distance,
or, 4.912 - 9.8t = 0
or, 4.9t (t - 2) = 0
• ne is projected vertically upwards fr
A sto
1001
. h om
the
of the tower w1_t a velocity just sufficient to
h _ _!&
-w=sm
.1
art'/ it to 78.4m. Fmd the velocity of the 51 ne
1
Either4.9t= 0 ⇒ t = 0 !'· 2070 Set D Q.No. 12 cj

--
s =ut+ 2at2 ~ith which it is projected. (g = 9.8mfs2)
which gives initial time (not required) 121 A b~ is projected vertically upwards with a
.. please refer to 2062 Q.No. 5 b
= 0><5+½ x3()><5-' or t= 2 veloaty of 40 m/s. Find its velocity and position
I= 2sec ~(Set~) Q.No. 12~ at the end of 3s. (g = 10 m/s2). [2 )
= 375 CIJl5 A body is pro1ecte~ vert1call~ upwards from the
lz~ 2075 Set C Q.No-__H • •
~E 2062 Q.No. 5 ~ - fool of the tower w1_th a velocity just sufficient to
A paint moving in a straight line with urufo~ A stone is projected vertically upwards fro 1 Given, u = 40 m/ s a = g = 10 m/ s2, I = 3s
foot of the tower with a velocity just suffj/ lbe cart'/ it to 78.4m. Fmd the velocity of the stone
accderation cfeseribes a and b metres 111 1 v=? h=?
with which it is projected. (g = 9.8 mfsl)

-
. . ternls at wne h and b seconds. cany it to 78.4 m. Find the velocity of tht:5 1o
121 We have,
.. Please refer to 2062 Q.No. 5 b
successn•e . lll _ . l(bb _ a!y_
6
with which it is projected. (g = 9.8 m/s2). ~;j V = U - gt= 40-10 X 3 = 10 m/ S
gjos9 (Set Al Old Q.No. 6~
Prove !bat the a<relerabon IS t,f:2{b + t:z) · lJ mmm A ball thrown up vertically upwards returns to
Again, we have
1
Initial velocity (u) =? h = ut-zgtl
the thrower after 6 seconds. Find its position
Let u be the initial velocity and f be the v =Om/s
after 4 sec. (g = 10 m/s 2 ) (2) 1
uniform acceleration of a point moving in a h = 78.4m or, h =40x 3-
We have, ~ 2 x1Qx3l
straight hne. =75
1 v 2 = u 2 -2gh Total time taken = 6s
Then.a = u t, + 2f t, 2 or, 0 = u2 - 2 x 9.8 x 78.4 h= 75m
So, time o f ascent= time of decent=%= 3s
or, u 2 = 1536.64 ps. 2071 Set D Q.No. 12 d
or, ~=u+1.
t, 2 ft, . (")
I
u = ✓1536.64 = 39.2 m/ s When the ball is thrown up, A ball is thrown vertically upwards. with a
The velocity at the end of time ti is u + ft1 v = 0, t= 3s, h = ? velocity of 30m/s. Find the time taken by the ball
2064 Q,No. 5-ij
-;:::lz7=
_
to reach the ground again. (g = 11hn/s1)
(" : V =U + at) We have, (2)
A ball thrown up vertically returns to lhe = U - gt
This will be the initial velocity for the next Ii thrower after 6 seconds. Find the velocity with
V BIB
or, 0 = u - 10 x 3 Given, u =30 m/ s
sec. which it was thrown up. (g) = 10 mfs2) . (2] u =30m/ s a= g = 10m/ s2
Now,
] 2 amiml Again, we h ave Let the time taken to reach the maximum
Since time of ascent = time of fall, so time to
b = (u + ft1). ti+
2f ~ 6
v2 = u 2 -2gh
or,O = 302-2x10xh
height be l
At the maximum height. v= 0
b J reach the maximum height = 2 =3s.
h =45m
or, ; = u + ft, +
2f ti . (ii)
Let H be the position of the ball after 4s. So, I= 4s.
We have,
v= u- gt
Subtracting (i) from (ii), we have Let u be the initial velocity.
v=O We have, or, 0 = 30 -10><1
b a l J
i; - ; = ft, - 2ft1+ 2tt1 We have, H ut - !2 g ti = 30
t= 3s
= 4 - 1 >< 10 X 42 = 40 JO
bh -ati J J =u-gt
V
X
2 Since, time of fall = time of ascent
or, ~ = ·. In 4 s, the ball w ill be (45 - 40)m i.e. 5 m below Here, time of fall = 3s
2 ft 1 +
2t ti or, 0 = u - 10 x 3
So total time = 2 " 3s = 6s
bh -at2 f ;==u= =30m/ s ..-- - lhe highes t point.
or, ~ = 2 (li+ti) !ze. 2066 Q.No. 5 ij §J.oss (Set B) Q.No. 12cj es.2072 Set C Q.No. 12 cj

--
ht A ball is thrown vertically upwards at a rate of
( = 2(bti -ah) A stone is dropped from a balloon at a hei~he ~ ball thrown up vertically return to the thrower 40 ms· 1• Find the time taken to attain the
lili(l1+ Ii) 116.4 m above the ground and it reache~ tht ~ter 6 secs. Find the velocity with which it was maximum height. (g = 10 ms· 2) [2)
ground in 6 sec. Find the velocity with whic 121 ~ro~~ ~
8. MOTION UNDER GRAVITY balloon was rising. .___l'lease refer to 2064 Q.No. 5 a
Here, initial vek.city (u) = 40ms· 1
mmmmJ whell t)1e ~ 9 Old (Set B) Q.No. &ij Accderati011 = a = g • 10m-l
fidt·i·i:fi·i 1J4iit·JfM Let u be the velocity of the balloon ping ball is. th rown vertically upward at a spee d f
4nis-i Let the time ta.ken to reach the maximum
. I of drop o
gs. 2058 Q.No. 5 ij slon e was dropped . Al the 1nstan . of 1he tin · Ftnd th e ma xi mum he ight reached and the
ie lake ['I
height bet
A body is projected vertically with a velocity of
a stone from the balloon, the veJOCJIY
Uie stone, ~ n t o a ttam , .
thts heig ht. • Here, v = 0
{
9.8 ms-1, how long It takes to return to th ba lloon is eq ual to the velocity 0 We have,
of projection? (g ., 9.8 018 _2) e point Taking upward direction posJbve, Civc" V = U - gt
·, " "' 4ms-1
mD1S1J . 121 1 h .. ? or, O • 40 - 10t
-h '" ut - at2
Let tJ,e bo.dy take I secs to return lo lhc oint
2 Wl' havl'
V ,
t " 4s

of p roJection. As the particl e ret urn ,: U1 e 1 "' u - gt


or, - 116.4 a u x6-
2 x9.8Xfr
Asmlta's NEB Solution of Basic Mathemattcs-11
260 As time of falling is greater lhl\1' 1 DYNAMICS Unit 14 261

p1. 2074Set B Q.No.1 2 cl te


·aiu,ot be less than 1s. So t cam,ot be s, So t
~• equal lo ~be the time taken by stone and T be th lilf·iWJ•MH•Mm
If a ball is projected vertically upward at. a r:i,c 1,et t ken by sound. e
of 40ms·', find the time taken to attain (2)
gS· 1J1ne ta 2 38 & 2072 Set e Q.No. 1~
maximum height. (g = 10 sm.:) 5
So, t=2 S
fhen, t + T "'49 9
= ...
(i) Abodyis.
vel .
d
proJecte vertically upward with
.,. Please refer to 2072 Set C Q. No. 12 c. . the depth of well, then
Now, putting the value oft in (i), we get Jfh is 1 1 bo:Ct~ u a~d t seconds afterwards another
§ 2075 Set B Q.No.12g
z
Jt 41) t + gt =
2 2"9.812 = 4.9t2 ... (ii)
y is proJected similarly with the same
· fro the top of a tower
A body falls from rest m d f'nd
1
the h _=s(~f =31.25m velocity. Show that they meet a height 4u2 - g2t2
and after 5 sec it reaches the groun . · f the 8g
......,.,,, l 'ty of the body and height o
:,uuung ve OCI
tower. [g = 9.8 2
J nvs
(2)
~ 2061 Q.No. t3 ij
A body falls from rest from the top of a t
---- Maill,
h"'ur( ·:u=~)
from th e point of projection after
g-2 secs
(.!! .!.)
=352.8T ... (iii)
mmmlE 16 ower
froll1 (ii) an~ (iii), we have
from th ·
body. e instant of projection of the second
Here, and during the last second it falls th of the
Initial velocity M = O m/ 5
25 352.ST = 4.9t
whole height. Find the height of the tower. -- (6]
Time tal--en (I)= 5 sec. -11..2=...!...12
(g = tOJIV'sec 2) T"352.8 t 72 Let two bodies meet at a height h after 11 secs
g = 9.8 m/ s' .,. Please refer to 2058 Q.No. 13 b 14] from the instant of projection of the second
Striking ,,elocity M = ? Now, from (i)
body.
Height of the tower (h) = ? &1. 2063 Q.No. 13 ij - 12 38
Then, for second body,
We have. fur falling body, A stone is dropped fro~ the top of a tower .200 m t+'72=9
h- 1 2
V =u + gt
=O+ 9.8 x 5 = 49 m/ sec.
hig~ and at the same time another is projected
vertically upwards from the ground with . or,
~=~
.72 9
-u1t- 2 g1t ... (i)

And, velocity of 50 mfs. Find where and when the iw: or, 12 + 72t .,. 304 = 0
and for first body,
1
1 will meet? (g = 9.8 m/s 2). [4] or, 12 + 761 - 41 - 304 = 10 h = u(t+ h)- 2g(t + lt)-2 ... (ii)
h =ut + 2 gt2
amiml or, (t+ 76) (t-4) = 0
1 Equating (i) and (ii), we have
= Ox5+ Let two stones meet at a height of h from the t= -76, 4
2 x9.8x52 bottom or H from the top after t seconds. Since t = -76 is not possible, so t = 4
1 2 1
utt- 2 gt, = u(t + h)- g(t + tt)2
= 122.Sm t= 4s
2
So, h + H"' 200. Also, given u = 50 ms-1. Then, 1 1 1
~l&t·i~J:ti·i 1iiili·lfH 1 Now, putting the value oft in (ii), we have
2
or, ut1- 2 gt1 =ut+ut,-
2 gt2-gth-zgtf
ps. 2058 Q.No. 13 ij h = ut - 2 gt2 ... (i) h = 4.9 x 4 2 = 78.4m
1
body falls from rest from the top of a tower ~ 2073tSet·C .Q.No. 13ij or, gtt1 = ut- gt
A
and H
1
= 0 + 2 gt2 ... (ii)
2
16th A body falls from rest from the top of a tower
and during the last second it falls of the - g!
25 Adding (i) and (ii), we have or, git -u-
whole height. Find the height of the tower h +H= ut
and during the last second it falls ~ th of the
2
(g = lOms-2).
ml!!iBl
[4] or, 200 = 50t
200
whole height. Find the height of the tower. (g =
!Oms·Z) (4]
or, It=(!-½) sec.
t= =4s Putting the value of It in (i), we have
Let h be the height and t be the time taken to 50 " Please refer to 2058 Q .No. 13 b
descend the height h. Then,
1
Now, from (i) Li 07 3Siipp Q.No. 13b oij
h = u(!-½)-h (:-½)
2

1 Astone is dr
h =2 gt 2 [·: u=O] h = 50 x 4 - 2 x 9.8 x 4 2 hi opped form the top of a tower 200 m
~ and at the same time another is projected 1 (u
2 1 1
1 u
ut ut 1)
= 2 X lQ X t2 = 5t2 ... (i) = 200 -
78.4 ::~c_ally upwards form the ground with a =g-2-2g g1-g+4
= 121.6 m Wil;c•ty of 50ms·1 • Find where and when the two ul ut u1 ut ,&!:
6
Again, Thus, the two stones meet at the height 121.

h, = (21; 1) g = et; 1
) 10 =5 (21 - 1)
m from the ground afte r 4s.
&2. 2067 ·Q.No. 13ij
-
1, teet? (g = 9.8 ms-2)

~ J e f e r 2063 Q.No. 13b


~ t B Q.No. 13b)
Astone.--==..:.:::~-
(4) =g-2-2it2- 8
- ~ .&!: - 4u2 - gltl
- 2g - 8 - Sg
[·:u=0] A stone is dropped from the top of a tower :zoo Pdt
. . h . projecle
high and at the same lime anot er 18 •h a
high a ~s dropped from the top of a tower 200m
ve11ka~ at the same time another is projected Hence, h=
4u2- gz~
8g
By given, hi = ~ h vertically upwards from the ground wit the Veloctt/ upwards from the ground with a
or, 5(21-1)
16
=25 . St2
velocity of 50 ms· •. Find where and when
two will meet (g = 9.8 ms· 2).
141 lvill Ille of SOms·1• Find where and when the two
1, Pl et? (g "' 9.8 ms·2) [4)
andh = (i-½) s

or, 16t2=50t-25 "" Please refer to 2063 Q.No. 13 b ---- ease refer to 2063 Q.No. 13b §. 2074 Supp Q,No..12~
or, 16t2-50t + 25"' 0 &3. 2071 Old Q.No. 14 a! d of A stone Is projected verticall~ u~wards ~~m the
or, 16t2-40t-10t + 25"' o 800 foot of the tower with a velocity J~St suffloent to
A stone is dropped into a well and the "
8t (2t - 5) - 5(2t - 5) = o carry it to 78.4 m. Find the velocity of the stone
or, . . . 4 .t secondS· If with which it is projected. (g = 9.8 m/s2) [2)
or, (2t - 5) (81 - 5) "' o its strlkmg the water is heard in 9
.1 Pifll_l tlte .,. Please refer to 2062 Q.N. Sb
5 5 the velocity of the sound is 352.8 ms · [41
1"' 2or
8 depth of the wl!ll. (g = 9.8 ms· 2)
,,
riI I
262 ,urn1t■'• NEB so1ulk>" o

~=;.;.o_._5_
A particle slides doWII a .
.
f saslc

.,,11M•i:1#••2!·1b

n inclined plane 30 m.
~ m s·•.
a velocity of "IJ300V-'
-
M1thern■t1cs-ll
the ball when it travels a distance
(g-lOm/s z].

--
Here,
Initial velocity (u) = 25 m/s
of ~ SIi\

Inclina tion of the plane (0) = 30•


Distance covered (I) = 22.5 m
111
01

~
,.
~
, I" 9.8
1 ===: =:=:= .----- -
1·,19,, ~6_:.:":.:,,
.o.No.12
II
cie slides from the rest 39.2'{3 m in
t, pa.rt! down a smooth Inclined
11 ds
,e'1~uJate
g1 f . r .
4
P1ane.
the an e o me mation of the plane
~ 2073 S.t C Q.No. 12ij
A Particle
DYNAMICS

10 m Ion slid es d o~n a smooth inclined plane


Unit 14

Find the~ a~d a~quues a velocity of 10 ...Ji ms·'.


.,. Pl mchruition of the plane. (g = lOms-2) (2)
ease refer lo '2072 Set D Q .No. 12c
~ 2075 Set A Q.No. 12d
263

-er
long and acqu1~ f the plane. (g • 10ms·2). (2) Velocity of the ball (v) = ? lg
° vA body · pro1ected
·
I find the inclinatio n
EID V= ~ .,/3 ID/ S
g=10m/ s 2
We have,
v2 = u2 - 2(g sin 0) 1
~.8 JIV's2J

~ the particle slides from the rest, 50 u ~ 0


(2)
1
:
1
:t
• 15
0~
up ,;m inclined pJane with a
25 m/s. If the inclinatio n of the plane
will . onzon be 30", what length of the plane
Given. u = 0 m/ s.
g = 10 ms-2 or, v2 = (25)2 - 2 x 10 >< sin 30° x 22.5 after 4 s«? (g = 10 mfs2) 121
J= 30m. 1,.39,2..(3
e=' 1
or, v2 = 625 - 2 X 10 X 2 X 22.5 1,,45 Here,
We have, g"9.8 m/s2 . . - ?
Angle of mclinatiOn (0) - . u =25 m/s
v' = uz+ 2(g sin 8) I or, v 2 ,. = 400
0= 30
(~)2 = 0:+ 2 x 10 x sin9 x 30
·. v =20m/s We have, 1= 22.5 m
§. 2070 Supp: Q.No. 12 ij .__ I" ut + ½(g sin 0) 12 v =?
300-{3 = 600sin8
A particle, projected from the bottom g= 10ms-l
or, . 300..f1. - ..f1._ . 60•
smooth inclined plane with a velocity 0 /~
1 1
or, 39.z.[3 =Ox t + 2 >< 9 .8 x sin 0 >< 42 We have,
or, sin 8 = 600 - 2 - Sill
nys, is just carried to the top in 4 sec; find ~! v2 = u2 - 2g sine. I
e=60° i nclinatio n of the plane to the horizon and 1~ or 39.z.[3 = 78.4 sin 0 =2S2-2 x 10>< sin30"x 22.5
§:: 20&5Q.No 5~ t he length of the plane. (g = 9.8m/s 2) . Ill or, , ~ - ~ - . 0
= 625 - 2 10 !2 22.5
A particle slides down an inclined plane ~ m Given. u = 19.6 m/s t = 4s sin9 = 78 .4 - 2 - sm 60 X X X

long and acquires a velocity of 1o,/i m/sec. Fm d 8 =? 1= ? = 625 - 225 = 400


0=60°
the inclinatio n of the plane, (g = 10m/sec2) l2) v=O
Since the particle is projected up, so we have, [::2072 Set D Q.No. 129
V = '20m/ sec
BIB v=u-gs in0t
A particle slides down a smooth inclined plane GIJl"i·i!li·i'liilllU~1

--
Given. u = Om/ s 1= 20m 10 m long and acquires a velocity lo-,/2 ms-1.
r, 0 = 19.6 - 9.8 x sin 0 x 4 !so. 20l6 Q.No. 13 ij
v = 1o-.{i m/ s g = 10 m/s 2
0
Find the inclinatio n of the plane. (g = 10ms·2) (2)
0 r, -19.6 = -39.2 sin 0 A particle slides down from rest from the top of
e =' 19.6 1 . 300 a smooth plane of height 1962 ans and
We have, or, sin O= = 2 = sm
392 Here, final velocity (v) = 10\[2 m/ s inclination 30" with the horizon. Divide the
v' = u' + 2(g sin 8) 1 0 = 30• Length of inclined plane (I) = 10 m plane into three parts so.tllat a particle at the top
or, (1 o-.fi>2 = 02 + 2 x 10 x sin 0 x 20 of the plane may describe each part in equal
Again, we have Initial velocity (u) = 0 g = 10 ms-2
or, 200 =400 sin 8 v2 = u2 - 2(g sin 0) I Inclination of the plane (0) = ? times. (g = 981cm/sec2) (4)
1 We have,
or, sin 0 = = sin 30° or O = (19.6) 2 - 2 x 9 .8 x sin 30° x 1 !1mlJB
2 1 vi = u2 + 2(g sin 0) I Let 1 be the length of inclined plane and t be
0 =30" or, 0 = 384.16 - 2 X 9.8 X X / 2 or, (lo-{2) 2 = 0 + 2 >< 10 >< sin 0 >< 10 the time taken for each part Given h = 1,962
!

m
LJ,~,
~1. 2066 C Q.No. 5 or, -384.16 = -9.8/ or, 200 = 200 sin 0 ems and 9 = 30°. Also, u = 0 m/ s. Let r, y, z
A body slides down an inclined plane 39.24m. 384.16 or, sin 9 = 1 = sin 90• be the length of 1", 2'-1 and 3rd part
long and acquires a velocity of 19.6 mfsec. Find or =9.8 :. 8=9()•
the inclinatio n of the plane. (take g = 10m/sec 2)
(21
.. I =39.2m ---
Aball is
2 Set E Q.No. 12ij
.
U?
~- 2071 SetC Q.No.12 :g 'th l Yeloci
. r pro1ected up a smooth plane with .
A ball is thrown up an inc me d plane WI di)' ~ the h ~ 25 m/s. If the inclinati on
of the plane to
Given, I = 39.24 u =Om/s th I
v= 19.6m/s 9 .,1 vel ocity of 14.7 mfs. Where will e ~ec~nat\a,1 Whe/~zon be 30°, find the velocity of the ball
g = 10 m/s2 the ball be 4.9m/s? Assume that ~
th
he plane to the horizon is 30°- (g - ·
11jl~
1
~"snv
travels a dis tance of 22.Sm. (g .. 10 m/s 1)
z 30'
We have, oft
-- (21
v2 =u 2 + 2(g sin 0) I BIB Now,
Given h
or, (19.6)2 = 0 + 2 x 10" si.n 0 x 39.24 He re, u ., 14.7 m/ s v = 4.9 rn/s , ll = 25 m/ S O = 30
g = 9.8 rn/ s
2 sinO =1
or, 384.16 = 784.8 sin O 0 = 30 0 Ivehav~" 22.S m g = 10 m/ si v =?
1962
. 384.16
or, sm O = 784 . "'sin 29.3° (approx) I=? e have, VI ~ l or, sin 30• = -1-
8 Si nce th e ball is thrown up, w Or, v1 ,. 11 ,- 2(g sin 0) I
25 2 1 1962
0 = 29.3° v 2 = u2 - 2(g sin 0) I or · - x 10 " sin 30° x 22.5
.
or, (4.9)2 =- (14 .7) 2 - 2 >< 9.8 >< SIil 30
0 ,. 1
• VI " 625
or, 2 =1
~2. 2070 Set C Q.No. 12 q 1 llt
I
-2>< 10x-x2 2s I = 3924.
• VI " 2 .
A . ball is p_rojected up a s mooth inclined plane or, 24.01 .. 216.09 - 2 x 9.8 >< 2 >< I 400
with velocity 25 m/s. If the inclina tion of the
or, 24.01 - 216.09 = -9.81 "' 20 111/s
pla ne to the hori zon be 30°, find the velocity of
or, -192.08 • -9.81
I Mathematica-II
' NEB solutlon of Sas c
264 Asrnlta s

Again, 1 . 0 12
I"' u\ + 2 (g sin
! " 981 " sin 300 " (3t)2
l~~~-=-:----:----:-0-_RI
A bo y slides down a smooth plane
len h is tOOm and height 20m. Find ~ho11
~
velf ity of the body when it reac~es the ~~t tht
UNnl5
or, 3924"' 0(3t) + 2
1 lx 9i:
or, 3924 =2" 9S1 "2
f the plane, (li) time taken by it to rea ht%
: ttom of the plane (iii) velocity of th c bth,
DYNAMtcs (CO NTIN UED )
3924" 4 .!§. :ter 4 seconds. [g =10m/szl e Ol4y
or, t2=m79" '9 [SIii& 61
, 11!.li:
..~-ili
..iA"'.. r::i•liMi-i•i•i§NIMA•iq
"..,~"'...
This is the equation from which we define an
Let a be the inclination of the plane. Then, absolute unit of force. The unit of force is the
4
:. t "':;;s 20 1 . ~ 062 Q.t,h); 6 force Which produces an acceleration of 1
Length -of firSl part (x) tana=100=s m/ s2 when it acts on a mass of 1 kg .
· t nt force of ION acting on an object
0118 .[ 20t5 Q.No ••

--
f,. c \ 8 velocity from lSms-1 to Sms-1 in 2
"'½gsif18!2 .; reduC: ~ind the mass of the object.
ecoJ\ : (2) The pull of the earth on a body is 49 N. If the
~ 1 acceleration due to gravity is g "' 9.8 mfsec2• Find
"'½" 9sl" sin 30°" (fY .c the mass of the body. (2]
1 16
i. U= 0, V =?
~= 15m/s
Given,
t=2s
v =5m/s
F=lON
mma
1 We have, vi= u2 + 2gsina./ Given, F = 49 N
=2"981 "2"9 1
m=?
g = 9.8 m/s2
or, vz = 0 + 2 " 10 " 5 " 100 = 400 We know that
-436ans ) m=?
I.en~ of first two parts (x + y v=u + at We have,
v = 20 m/sec
or, 5=15 +a · 2 F =mg
"'½g sin a (2t) 2 ii. Let the required t_ime be t sec.
We have v = u + g sma. t
or, -1O"'2a or, 49 = m x 9.8

=½" 981 x sin30° "4"


= 1744ans
(~Y ,
20 = 0 + IO
or, 20 = 2t
X
I
5 )( t
or, a= -5
Retardation = 5 m/ s 2
We know that
f = Mass x Retardation
49
or, m =9.3= 5 kg
Mass(m) =5 kg
Length of second part (y) t = 10 sec.
,iii. Let v be the velocity or the o dy after 4 seconds
b or, 10 = Mass x 5 ,~- 2066 C-Q.No.'6 !
= 1744 -436 =l30Sans 10 Find the velocity of a 4 kg shot that will just
Length of third part (z) Then,
v = u + g sina.t
Mass"'
5 = 2 kg. penetrate through a wall 16 ems thick, the
resistance being 4 metric tonnes weight [2)
= 3924 - (x + Y) I ~ 2064 Q.No. 6 aj
= 3924 - 1744 "'~ans or, v = 0 + IO x 5 x 4 ml&!
Show that Newton's second law of motion gives
2J•O the measurement of a force.
Resistance force = -4 metric tones wt
v = 8 m/sec. (2) = -4 x 1,000 x g N
mm Let 'a' be the retardation Then,
Second law of motion F=-ma
The rate of change of momentum of a body is (')f, -4 X 1,000 g = -4 · a
proportional to the impressed force and takes or, a= 9,800 m/ s2
place in the direction in which the force acts. Let 'u' be the velocity of the shot which can
just penetrate a wall of thickness 16 cm= 0.16
Suppose a force F acts on a body of mass 'm'
m. Then
for time 't', and changes its velocity form u 0 ! O= u2 - 2as
~· The change in momentum of the body in or, o= u2 _ 2 x 9800" 0.16
lime 'I' is mv - mu.
or, u2 = 2 x 9800 x 0.16
Therate of change of momentum is mv ~ mu u = 56m/s
By Newton's second law o f motion, ~ 2066 Q.No. 6 !

--
Fc:c ~
A. body of mass 1 kg is falling under ~vity at
f ms"I What uniform force will stop
t the rate o 28 · "2) (2)
Or,p(( ~ It in 0.1 second? (g = 9.sms
l
l;i1~ ~~nl(e in velocin• Given, m =1 kg
a acceleration a a (say) u = 28m/s
1"heref ~
o,, p ~ k ore F ex: ma t"' 0.1 s
if IV Illa, Where k is a constant. v"'O
nu11 e ,ta ke tn "' 1 kg and a = 1 m/s 2, the f"'?
lh/errica1 We have,
re ore value o f f = 1 if we take k = 1.
, v "' u-at
~ " tna or, 0 = 28 - a " O.
-
DYNAMICS (CONTINUED)
Asmlta's NEB Solution of Basic Mathematica-II ~cQ,N6. 12 m,,, Unit 15 267
266 - 1
I, io11 of mass 50 kg is falling fro111 a cert .
bodY . brought to rest after striktn ain We have F
28 Given, m = lOOo kg or, 150 x 12, - x t "' m(v - u)
or, a =-=280
0.1 ft1e1gl1I •9 Ith a speed of Snys. If the res· gt the
F • 2500 N or, 18000 O- m(350 - 300)
U 'F' be the force retarding the motion, then by i dw d I SOON fl is ance -mx 50
a=? ~o~:ol the groun s , nd the duration of
Newton's second law of motion.
We have,
:. m,,,~
F-mg=ma 10~1acl· - 50 "'3600 kg
F = n,a (2]
or, F = mg+ ma = 1 x 9.8 + 1 x 280 = 289.8 N
2500 = 1000 x a '~
~ . of a t,ody (m) = 50 kg
[i 2075 SetA .
289.8 A cart. Q.No.12~
=10 kg wt =28.98 kg wt 2 1,1~:ai velocity (u) = 5 m/ s "h IS P\IShed on
Wit an av fri . I
a=~= 2.5 m/s a clionless Slll
= 29 kg wt (approx) (tu al veJocity (v) = 0 m/ s cart With erage force of 20N for 5 ooth plane
fin
§2070(0lcl)Q.No: 6g •:I, ~ :istar'ce force (F) = -500 N
mass 50 k . seconds. If the
d the velocity g '.s at rest in the beginnm
~- 2067 Q.t<lo. sij A car of mass 1000 kg is brought to re 1 ~
A bullet fired into a target loses half its velocity ouration of contact (t) = ? acquired by the cart g,
applying a breaking force of 2500N. Fin~ by -- . (2]
after penetrating 6 ans. How much further will th We have,
average retardation. Here
it penetrate? (2) ... Please refer to 2069 Old (Set B) Q.No. Ge ' 11
2e
f ""
~ F=20N
t
tmBlml
Let a be the retardation. Then.
ff" 2070 Set,C Q.No..12.~ . .._ ' _wo-_fil t "' 5 seconds.
A cart is pushed on a frictionless smooth 1 m "'50kg

(~Y ~~:
or, _5()()- t

--
with an average force _of 20N fo~ 5 seconds. ~f u=o
=uL2a -6 cart with mass 50kg 1s at rest m the beginn·1 or, -S(X)t= -250 v=?

or,
u 2-
=u2 -12a
find the velocity acquired by the cart. it I" 0.5 sec
~supp.·Q.No.12cj
We have,
4 Given, F = 20 N t =5 s F =7(v-u)
3u2 A bull~t ~f mass 25gm moving 250m/s
m = 50 kg u = 0 m/ s
or, 12a= 4 ~ne~ttng mto a tree ~nk _and is then brought 50
or, u2 = 16a ... {i)
v=? to rest in 0.02 seconds. Fmd rmpulse of the force or, 20 =s (v-0)
Weh'ave,
Let (6 + r) ems be the thickness penetrated
_ m(v-u} -on the bullet. (2) or, 20 = 10v
before coming to rest. Then, F - t v =2m/sec
0 = u 2 -2a (6 + r) 50 (v -0} . 25
or, 0 = 16a - 2a (6 + r)
I _
or, 20 - 5
Given, ,m = 25 gm= 1,000 kg = 0.025 kg 513f'i-i!fi•l 11iill·U~i
or, 2a (6 + r) = 16a
or, 6 + r =8
or, 50v= 100 u=0m/s H9. 2059 Q;Ncx.ru ij
v=2m/s v=250 m/s
A body of mass 1 kg is fallin
.. r=2an
:. It will penetrate 2 cm further. !13. 2070 Supp. Q.No. 12 cl ,Impulse of the force (I) = ?
We have, ·
the rate of 28 ms-t. What is the g ~der gravity at
will stop it in (i) 0.1 sec (") 20uniform force that
A bullet of mass 25 gm moving 250 m/s
lf. 2067 Q.fllo.-6111 penetrates into a tree trunk and is then broughl Impulse= _ip(v - u) = 0.025 (250 - 0) = 6.25 k m s Instead of falling under u . ~ (g = 10 ms-2).
State Newton's second law of motion hence to rest in 0.02 seconds. Find the distance of [2073 Supp Q.No. 129 g / moving at the rate of 28 ; ; ~ ~ if the ~ody is
define a force. [2] [21 !ind the mass of line, what will be the fo n~ a b~nzontal
&e1.mi.ll1n of the tree-trunk. weights98N ( • two cases? rce reqwred m above
... Please refer to 2064 Q .No. 6a an ob1ect which on earth
- · g = 9.Sms•l) m.1l!mml (4]
Is.2068 Q.No. ij4 25
Given, m = 25 gm= l,OOO kg= O.o25 kg
121
A constant force of ION acting on an object Weight Given. m = 1 kg
o ob-
" f u=28m/s
reduces its velocity from 15m/s to Sm/s in 2 u =Om/s Ject on earth (F) = 98N
g 9.Sms-2 t=0.1 s v=O
seconds. Find the mass of the object. (2] v = 250m/s We have,
.,. Please refer to 2062 Q .No. 6 a t = 0.02 s Mass(m)=?
Wehave V = U + at
We have,
.
~- 2069 (Set A) Old Q.No. sij hu,g , or, 0 = 28 +ax (0.1)
A bullet of mass 2 kg is fired from a gun of mass
FX t = m(v - u) or, 0.la = -28
or, F x 0.02 = 0.025 (250 - 0) ~, 9g,, n, x 9.8
100 kg with a velocity 250 m/sec, find the recoil -28
6.25 or, a =0,1= -280
velocity of the gun. ~, 111"~
(2) or, F = 0.02 9.8 = 10 Retardation = 280 m/ s2
[B!lllll -'· F=312.5 ~' lOk If F be the uniform force applied in upward
We know that, ~Us g
Again, we have, F = ma direction to stop the body. Then,
Mass of bullet x muzzle velocity = Mass of the
or, 3125 = 0.025 x a 10kg. F-mg=ma
gun>< recoil velocity
312.5 or, F = m(g +a)= 1(10 + 280) = 290 N
or, 2 >< 250 = 100 >< recoil velocity a = 0.025 = 12500 (ii) u = 28 IIIS" 1, s = 20 ems = 0.2 m. v = 0, a = ?
500 na change the velocity of a
. . Recoil velocity = 100 = 5 m/ s. Also, we have We have,
cond i~t;aig~t line from 300 to 350 vi = ui + 2as
v2 = u2+ 2as nunutes. Find the mass of
[o. 2069 Old (Set B) Q.No. 6g or, (250) =•02 + 2 X 125()0 XS
(2)
or, 0 = (28)2+ 2 x ax 0.2
A car of mass 1000kg is brought to rest by 62500 lieie r or,
_:fill:_
a. - 0_4 - -1960.
applying a braking forces of 2500N. Find the 6=
25000 "'2.5 m ii- ~ v,3SQ n"' /lSON, 4
average ret;trdation. 121 :. The distance of pcnctratiQn of 11,c tree irui l sec, I = 300 m/ sec, Retardation = 1960.
= 2 minutes Again,
2.5m. F = m(g +a)= 1(10 + 1960) = 1970 N
= 2 >< 60 sec= 120 sec
11
Asmlta's NEB Solution of Basic M1th1metlc•·
268
If we take m .. 1 kg ond a - 1 II .., ,nf - m g+ xg • 2mf - 2xf ~ DYNAMICS (CONTINUED) Unit 15 269

lf the body is moving in a h<,rizontal line, i1111m:ric11l value o f F • 1, if we lok~ sl, lh1, or, 111 ~ [usln~ (i))
k•1 2073 Set D 0.No. 13]
Thcrdore zx( • 2mf - mf ~
there is no componen t of m g. , -1-
mass of S kg falls 3
F - ma or, , (g .., 2f) = mf m from rut and is then
(i) F = ma = 1 x 280 = 280 N de brought to rest
This is the 1c-quation from which We 1g f
(ii) F=ma=l x 1% 0 = 1960 N or, hnd. Find the atJy penetrating 30 cm into some
absolute unit o f force. The unit or f hne ~n verage thrust of the sand on it.
~o. 2060 Q.No. 14 b OR! fo~e which produces an accelera~~ce is lh, :. ; '" g+U
A shot whose mass is 40kg is discharged fromth• m/ 2 when it acts on a mass of'\ kg. on o( I ~ 71 set D Q.No.
13 ~ Please refer to 2069 (Set A) Q·No. 13b (4)
700kg gun with a velocity of t40ms·•. Findsto e
5
5· ~ of mass 400 kg fires a shot of fil_:
constant force which acts on the gun would 4p jg 2069 (Set Al Q.No. 13bi -------- i\ gun . f 200m/ t· mass 3kg,
s, ind the constant force
: 2074 Supp 0.No. 13ij
A shot of 2 k · d"
with a vel?c1ty o

--
it after a recoil of 6.4m. 11 A mass of S kg falls 300 cm from rest and kg With g IS. ISCharged tJy a gun of-. 400
which acting on the gun would stop it after a
wi:

--
brought to rest by penetratin g 30 cm int!'80th,n ~elOCtty of 800 m/s. Find the constant
ama
Momentum of the sh0t = m v = 40 " 140 = 5600 sand, find the average thrust of the sand. 11\, recoil of 2.s meters. (41 fore,
c Woa]d be required to stop the ftalil

Momentum of the gun = MV = 700V


111!11111 l4J
M omentum of the shot = mv = 3 x 200 = 600
oftheguni ni¼ sec.
141
We have, Suppose v is the velocity of the body Wh . Momentum of the gun = MV = 400 x y
momentum of the shot = momentum of the gun falls 300 cm from re st. Then u = o, h = 301 _en 11
We have, Let v be the recoil velocity of the gun. Then,
or, 5600 = 700V We have, Momentum of the shot = Momentum of the
.(i) Momentum of the shot = 2 x 800
or, V =8 v2 = u2 + 2gh
gun Momentum of gun = 400 " v
Let a be the retardation . Then. or, v2 = 2 " g " 3 or, 6()()=400V We know that
0 = V2 - 2as v2=6g ... (i) 3
:mentum of the shot = Momentum of the
or, V2 ~2as
or, 82 = 2" a x 6.4 [using (i))
The velocity given by (i) is reduced too Wh
sar: V=z
ilie body goes to 30 cm = 0.3 m into the Again, let a be the re tardation. Then, or, 2 x 80() =4()() xv
or, 64 = 128a lf a is the retardation , then Q=V2 - 2as v = 4 m/ sec
64 Q2 =v 2 -2" ax 0.3 • or, V2 =2as I -
or, a = 128 = 5 Given, t = 1 4seconds = ~ seconds
~ _&&,_
Now,
F = ma = 700 x 5 = 3,500 N
a =2 "
0.3 - 0.6 - 10g
Let T be the average thrust of the sand on lhe
or, (if= 2 " a " 2.5 m = 400 kg
Let a be the retardation Then..
9
121. 2063 Q.No. 10 ORI body. When the body is penetrating into !he or, a=wm/s 2 O= v-at
State N ewton'li Laws of motion. Show that sand, then the forces acting on the body are: V 4 16
Now, or, a=7=~= s
Newton's second law of motion 'gives the (i) the weight 5 kg of the body acting
(4) F=ma
measurem ent of a force. downward s. 4
9
(ii) a force TN of the sand acting upward.
-
Resultant upward thrust= (T - 5g)
or, F = 400 " 20 = 180 U F is the required constant force to be
t • Part applied, then
By Newton's second law of motion, we have F = 180 N
Fust Law of motion: Every body continues in
T _ mg = ma ~6. 2072 Set C Q.No. 13ij F= m x a= 400 x ~= 80 x 16 = 1280N
its sta te of rest or o f uniform motion in a ::,

straight line unless compelled by some or, T _ Sg = 5 x 10g Sta~e laws of motion. A body of mass 50kg
external forces to change that state. 55
=== =,--- --- I falling from a certain height is brought to rest iibf·i·i fi·i'liiil•Ui1
Second Law of motion: The rate of change of • -i==T~=,,;;;,;k;gg~-w;;,;,t . mg th e ground with a speed of S ms-1.
after strik'
!32. 2013 Set C Q.No. 1◄
If
to the ~3. 2069 (Set B) Q.No. 13ij
m om entum of a body is prop ortional d the.resistance force of ground is SOON, find the
impressed force and takes place in the State Newton's
laws of motion. Show lhil
01 (41 State Newton's Laws of Motion. A bullet of mass
.,.ur;~on of the contact.
direction in wruch the force acts. Newton's second law gives the measurement
Third law of motion: To every action, there is a force.
Ill t
st Part: Plea se re fer to 2063 Q.No. 14 b OR
' ;~ond Part: Please refer to 207 1 Set C Q.No.
10g is fired from -1 gun of mass 3kg with a
velocity 300 kmh· 1• Find the velocity of recoil of
(6)
an equal and opposite reaction. ._ Please refer to 2063 Q .N o . 14 b OR - - - 1 the gun.

2..i Part !24.


2069 Old (Set Bl
Q.No. 14ij
f Pr01
.1
~ 2 Set D Q.No. 13tij mlllll
Suppose a force F acts on a body of mass 'm' A balloon is raising with an acce 1era
fon
1
- First Part: Please reier to 2063 Q.No. 14 b OR
bailooi State la
for time 't', and changes its velocity form u to defi ws of motion. Use Newton's Law to Second part;
that the fraction of the weight of t~e rder- (4)
out of the balloon tn o "' ;~ an abs olute unit of force.
v. The change in momentum of the body in which be emptied
time 't' is mv - mu.
f Ill ~_5: re fe r to 2063 Q N,,. 14 b OR
Here, m.iss uf bullet = 10 g
Mass oi gun= 3 kg= 300 g
1
The ra te o f change of momentum is m v ~ m u double the acceleratio n is g + Zf . ~ 12072§et E Q.No. 13ij Muule velocity = 300 kmh-
a e Ne Refoil velocity = ?
1hen_il Newto , Wton's laws of motion. Prove that
By New ton's second law o f motion, 1111B 0 0 11
,
Of the ; s second law provides the measurem,m t Wt!h.Wt!,
If 'm' be th e mass o f the ball d }iitirl
m v - mu "' Pie orce. (41 Mass oi bullet x Muzzle velocity = Mass of
weigh t = m g a nd R be tJ,e up war
F oc - - t - - e refe r lu 206-.1 Q .N,,. 14 b O R
~ the gun x Recoil velocity
m(v - u) force, then , 2012 Su
>. ball Q,No. 13 or, ,o " 300 = 3CXXl • recoil veloci ty
R - mg = mf
1 frOll ~
or, F oc t
•· (i)
1 1h1t thoo n Is ri s ing with .i accc lt-ration f. Prove Ra.'Oil veloci ty = t kmlr'
v - u change in velocity · R = mg + mf
Bui-1-= time •acceleratio n • a (say) If x be th e m ass to be ta kcJl uu p''''
w move u lvhich e fraction of the weight of the balloon
no •nu st be emptied out of the ballo0Jl i.n
Therefore F oc ma balloon, so tha t it m a y 0 f
rder to d
o r, F = kma, w h e re k ls a constan t. ao:.clcrll t:ion 21. Th en, '- (l Du b le th e acce hiratlon is g + 2i • 141
R - (m - x)g '"' (m - x)2f lea.s,
~ rc for tll 2llt,9 O ld (Set 8) Q.No. '14,i
270 Asmlta'a NEB Solution of Basic Mathemattc• · 11
DYNAMICS (CONTINUED) Unit 15 271
2 2 2
u sln a ( ~ - u Bin' <L)
B PROJECTILES ., 4 2g 2g 2g or, h =x-.L =~
x+ y x+y
fi&i·i~J:fi·i 11iill·i~ti ~ (u'-u 2
sin2 Ct) h• .2i'.....
x+y ,

-
j:,3. 2059 Q.No. 6 aj ,.4 2g 2g
lf u and a be the velocity and angle of proj~tion
of a projectil e, then find the time of flight. [2) =
2g {u' ;;1'a + ----..1._
16 -4
u' sin 2 a · u' cos2et
(2g) 2
~ 2061 Q.No, 1{_§
If R be the horizon J
its . ta range of a projectile and h
greatest height. Prove that it.9 initul velocity
The time taken by the pl\ijectile ID reach the
horizontal plane ~ugh the point of
4sin~ cos'a •
g
;;-:-IL_)
Bu 2sinl a u2 2 sm a cos a u 2 sm 2a is✓2g(h+~)
16h · [4]
projection again can he obtained from =~ = R = LH.S
(u a+~)
1 2
g ._ Please refer to 2IJ57 Q.No. 14 b
1 sin
h = usina t - 2i;t' 2g 2g 2g ~Q.No.14ij !0- 2062 Q.No. 1·4 b of!
Wnen the particle stnkes the h orizon again, h
= ✓u' (sin'a + cos'a) = u
fJOIII apoint on ~he ground at a distance 'r from If_R be the horizontal rmge and T be the time of
= O, then the foot of a ve~ca_l wall, a ball is thrown at an flight of .i projectile, show that
1 gle of 45° which 1ust clears the top of the wall lana-~
O= usmo t - 2ef Hence, u = 2g ( h + 1~~) alld afterwards strikes the ground at a cfialance 'y' - 2R w h ere a 15
· the .angle of projection

:: the other side. Prove that the height of the wall .. Please refer to 2058 Q.No. 14 b
usm a-½ gt) = 0 !36. 2058 Q.No. 14 ij .__

--
or, t ( ~1. 2063 Q.No.14 ij
2u SID Q
If R be the h~riz?ntal range and T, the lime
01
11-f+r· (4)
A projectile thrown from .i point in .i horizontal
t = Oor1 = - - - flight of a proiectile, show that
g p~ane comes bade to the plane in 4 sec. .it a
Bui t = 0 means projectile is a t the point of tan a= g:!2 h . h
2R, w ere a 1s t e angle of projection. Lei u be the velocity of projection Given a = dist_anc~ of ~ in front of the point of
141 P~ection. Find the velocity of projection.
pro,ection. 45'. Then the horizontal and vertical
~ component of u are u cos 45° and u sin 45° i.e. (g -10 m/s1).
141
2
Henre, the tune of flight = u sin a
g Ii u be the velocity of projection, then u u
{2 and {2 respectively. am
,ju. 2071 Old •Q,No. 6 ci horizontal range (R) =~
g
and Let u be the velocity of projection and a be the
Let I be the time taken by the ball to reach the angle of projection
A particle is projected at an angle 75° to the 2 U sin U · top of the wall. The horizontal distance in this Given, Time of flight (T) = 4s
horiron with a velocity of 2943 cmfsec. Find the time of flight (T) =--g-'
time taken by the ball is x. Horizontal range (R) = 60 m
range on a horizontal plane. 12) We have,
Now,
~

g(T)'
fu
Gwen, a = 75° 4 u 2 sin 2 a T =~
g ---g,-
u = 2943 cm / s
L.H.S. -- ~
2xuxsin a
R= ? 2R u 2 sin 2a 2u2 • 2 sin a cos a or, 4 =--1-0- -

--x--y-
We have,
2----
g usin a = 20 .. (i)
R =~ sin a Again, we have
g =~ = tan a= R.H.S. u2 sin2a
R=---
(2943) 2 sin 150"
10 = 43306245 cm @
1. 2059 Q.No. 14 b OR! Then, x =u cos 45° • t = 4 . .Ji . t
g
u1 • 2 sin a cos a
43306245 A particle is projecte d with a velocity u. If the or, 60
= ~ m=4330.62m greatest height attained by the particle be H, I=~
prove that the range R on the horizontal plane u (u sin a ) • 2 • u cos a
or, 60
g
ki'ft,i·i!fi·l 1 Jiilt·Jf~1 through the point of p ro jection Is Now, if h be the height of the wall, then
20•2• u cosa
j:ls. 2057 Q.Ho. 14 ij
If R be the horizontal rmge of a projectile and h
R = 4 ✓H (~-H), 141
h = (u sin 45°) t -½ g12 or, 60 - 10
or, u rns a = 15 ... (ii)
[using (i)]

mmmmJ =U • -l. X ~ !. X 2t2 Squaring an,I adding ~) anJ (ii), we get


16 greatest height, prove that its initial velocity is
Let a be the angle o f projectio n.
../2 ll -2g u2 u1 sin1 o. + u1 cos1 a = 201 +. 152
ig ( h + 1~~ )- [4) We have, =x -!:&
u' ... (i)
or, u1 =b25
u =25 m/ s
u' sin 2 a
!EmmlZl Grea test height (H) = 2 g and J-l~n.:e, th~ vel<Xity of projection is 25 rn/ s.
Let _a be the angle uf p rojection and u be the
Again th
'
.
e ho rizonta l range=
u2
g @2. 2064 Q.No. 14 ~
. u' sin 2a
1.rutial velOCJ ty uf the projectile. H un wn ta] range (R) = ~ I.e. X+ y ., ~ A stone is thrown horizontally with velocity
0 g ..{2gh front
We have, g realeb t heigh t (h ) = ~ Now, r, ui,. g(x + . the top of a tower of height h. Find
2g Su bst11 y) . . .(ti) where it will strike lhe level ground through the

h orizontal range (R) = ~


g
R H.S. a 4✓H ( ~ - 1-1)
h "x- -21L
Uhng the v;1lue of ui in (1), we have foot of the tower. What will be its striking
velocity? (4)
g(x + y)
. --~--1;;· 11_ .. ~~~~t·
\';;
DYNAMICS (CONTINUED) Unit 15 273 ~
272 Asmlta'• NEB Solution of B■slc M1them1t1cs-ll ~ .No, 14 -
Let y be the dis
~ 2066 C Q.No. 14 ij ------- e. the velocity and direction of projection of level. tance of B from the original
IDIID1! A particle Is projected with a velocl fl~d t,lch passes In a horiz ontal direction . a
By question, u = 20 m/
A projectile is any object wi th a given initial greatest height attained by the partic1~· If lht ,i,ol ~e toP o( a wall which is 250 m olf and Just Horiz.ontal com s, u. ~ 30"
velocity and moving along a path J ctero1ined prove that the range of R on the hori:z: be II. oterl • 9.8 ms-2) 125
4 ponent of u = u cos a = 20 cos
by gravita tio nal hi!\."<' acting on ii and 11\rough the point of projection Is: onta( Pl•~t 111 111gha·~~ refer to 2066 Q .No.14 b
.. r1e·~ .:,;;,;;:::::==;;--'---=----
11 30" = 2(J X 1}- ]OV3
=
frictio nal resistance of the air. The path along
4& a.Ho. 14
w hich the pI'O)ectile moves is called the
trajecrot)' . The point from which the particle is R=4~ (fs-H) [4[
1 · t,orizontal and vertical components f
'f)te
trial veJoc1
·ty of a proi· Iii o the
ec e are u and V
Ver tical componen t of u = u sin a - 20
1 - sin 30"
projected is called point of projection . The ... Please refer to 2059 Q .No. 14 b OR =20 " 2= 10
angle a between the horiwntal plane through :,pectlvely. If R be the range and the H the
For the horiz.onta] mobon, we have
~ - 2066 Q.No.14 ij ------ h tt. d 4H V ""'r,,
the point of proja..--tion and direction o f the
Find the_velocity a1~d direc~on of projectto gieaiest heig ta ame , prove that: = ij R
141 ""'43 ~ 1°"3 t
projection is called angle of projection. The
shot which passes m a honzontal directto n ?r ii ,. please refer to Mo del Se t II, Q .No. 14
l= Ss
time taken by the particle to come back to the
over the top of a wall 250 m off and 125 n )1151 The Vertical distaoce covered
horizontal plane again is called the time of
(g = 9.Sms""2) Ill high, ~ e t Al Old Q.No. 14ij 1
flight. and the distance between the point of
~ [4] rojectile thrown from a point in a h . y =ut-2g12 = 10x 5-½ x 10x 52 =-75m
projection and the point where the particle "iapne comes back to the plane in 4 :;i:z:ontal
strikes the horizontal plane again is called the Let u be the velocity of projection of p . f cs at a Since y is nega.tive, so it strikes the .le
distance of .60 m m ront of the point of wall 75m below the original level opposi
horiz.ontal range. making an angle a with the horizon. a shot
proJectlon, fmd the velocity of projection.
y u 2 ~ - 2070 (Old) Q.No. 14 ij
1g=10 ms-) (4)

!kb_ lb
Fmd the velocity and diredi
f!!JSl
Let u be the _velocity of projection and a be the
shot which passe . h . on of projection of a
s m a onzontal diredi
over the top of a wall which 15
. 50
.
on rust
angle of proJechon. 25 meter high (g _ meter off .md
::,
u H
.
Given, Time of flight (T) = 4s
mmJEI - 9.8 mr2). 141
Horizontal range (R) = 60 m
0 ° ucosa M _X Let u be the velocity of projection of a shot
-2som~

-~rs~-"
We have,
Maximum height (H) = 125 m making an angle a with the horizon.
Next l'Mt
H ori:z:ontal range (R) = 2 x 250 = 500 m T = g~

fu
Let t be the time taken by the stone when it
We have, 2 x u x sin a
u 2 sin 2 a or, 4 10
H=~
usin a =20 .. (i)
u 2 sin 2 a Again, we have
or, 1 2 5 = ~ ... (i)
R= u 2 sin 2a
2 +-- 50 m +
and R = u sin2a g
g Maximum height (H) = 25 m
or, u2 • 2 sin a cos a Horizontal range (R) = 2 x 50 = lOO m
60
or, 500 = u2 sin 2a .. .(ii)
g We have,
X g or, 60 - fusin a ) · 2 · u cos a u2 sin2 o
Then,h=O+½g12
Di viding (i) by (ii), w e get g H =2g
u 2 si.n 2a or, 60 = 20 • 2 x u cos a u2 sin2 a
125 _ 2g 10 (using (i)) or, 25 =2 g ... (i)
t=~ 500 - u 2 sin 2a or, ucosa =]S ... (ii) u2 sin 2a
g Squarmg
ui . 'and a d d.m g (1). a nd (1i),
. we get andR =~
Also, r =ut=..,/2ij, x ~ = 2 h ] u2 sin2 a 5111 2a + u 2 cos 2 a = 202 + 152
or, or, ui = 625 u2 sin 2a
Let v be the strikmg velocity of the stone 4 =~ x u 2 - 2 sin acosa or, 100 =- - g- ... (ii)
when it strikes after falling a height h _ ' . U=25m/s
or, 1
= sin a Dividing (i) by (ii), we get
Then, cos a .,,,.2:_l__ence
~ • th eve 1oc1ty
. o f projection is 25 01/s.
u 2 sin2 a
v 2 = u2+ 2gb or, tan o. = 1 = Lan 45° ~ I d (Set B) Q.No. 14b! ~ - 2g
= 2gh + 2gh = 4gh ; ('l-,Jgh)2 a =45° all is th - 10 0 - ~
towards a tal row~ ~rom th e top of a building
V = 2{iji Su bstituting the va lue o f u. in (i), we have
Velo,c11y of t I bmldmg 50 "1/3 rn away. T he Initial
~3. 2065 Q. No 14 ij 125
= u 2 sin 2 45° horii ontat he ball is 20ms · 1 at 30° a bO\'e the 1 U' Sill'O.

ff_R be the horizontal range and T the time of 2 X 9.8 ltv,1 Will ;hHow far a bove or below its origina l
or, 4 = 2 g • u' x 2 sin a cos a
flig ht of a p rojectile, show that: or, 125 - ~ . . . . . e ba ll stri ke the oppos ite wa.11? (41 sino.
- 19.6 or, ~ =1
ta na-8:C
- 2R' w h ere a is
· t h e angle of pro jection. (41
or, 125 xl 9.6 ~ ~2
41t
buil _
the ba I I thn,w n fw m \he top A of d
or, tan a= I = tan -t5°
... Please refer to 2058 Q .No. 14 b a =-15'
or, u2 .. 4,900 awaydui g 'st n' kt' <1 nnther tall building 5vv
.• i;,3 m
Substituting the value of a in (i), we have
' ~t B dfter t se.:unds. ·
u -70m/s
274
Asmlta'• NEB Solution of Basic Mathamat1c1-II

®· 2071 Old Q.No. 14 ij


DYNAMICS (CON71NUED) Unit 15 275 f
~

~[S"
u2sin245°
A ball is thrown by a player from a h ~ and a be the velocity and e of
25 "' 2 X 9.8 meters, at an angle of 30° with the ho . eight or~ L,CI ·eetiofl respectively. angl
_ u2x 1.J1
.~fuu
velocity of l8ms·1, is caught by anothrtzon With
or, 25 - 19.6 the height of 0.4 meter from the grou:~ Player i~
or, u2 =25x19.6x2 apart were the two players? (g. 9.8 ms-; ltow fl!
...
P""
or, u = 980 ) (41
u =14'{5 m/s Given, a = 30° 180m
The velocity of projection= 14 {s m / s. So, the horizontal and th
~ H
a s=ucosa -t
~1. 2070 Set C Q.No.1.i] components of velocity of projecti e Verlieaj or, 180 = u cos 45• . t
T = 4s
Find the velocity and the directi_on of p~jec~on 30° and u sin 30° respectively. on are u cos
R = 58.8 m or 180-2!!.
of a shot which passes in a honzontal direction A ball thrown from a height of 2m , ' - -{2 ...(ii)
just over the top of wall which is 250 m off and caught by a player at a height of O~s lo ~ 5Y given, . -
Fr~m (i) and (ii), we have
fi,.ale of flight (T) - 4s
125 m high.(g= 9.&avs1) (4] ball has to descend 2 - 0.4 = 1.6 m . · · So the -65 = 180-4.912
j-ioriZOntal range (R) = 58.8 m
• Pleasereferl02066Q.No.14 b Taking upward direction as positive 0 r, 4.9 t2 = 180 + 65
'We have We have, 0 r, 4.912 = 245
\!2. 2070 supp. Q.No. 13 ij 2usina 0 r, 12=50
A ball is pro;ected at an angle of 30° to the r=~ I=..[sos

...
horizon md land on the surface of height 10m 1 1 Again, from (ii), we get
which is 1JN3
m alQy from the point of or, usina=2Tg= 2x4x9.8=19.6
projection. Find the velocity of projection and it:s ... (i) u -,=180
usina=19.6
striking velocity on the surface. (g = 10nys 2) (4]
Again, we have
E u 2 sin2a or, u = 1so:J2 _1so:J2
N ..[so - 5-[2 = 36
Given, angle of projection (a) =30° R=~
u =36m/s
Greatest height (H) =10 m or, u2 sin 2a = Rg
Horizontal range (R) =20\[3
L..-_ _ _ _ _ _..l.0.4m
or, uz , 2 sin a cos a = 58.8 x 9.8 ~ · 2073 Supp Q.No. 13ij
180m 58.8 X 9.8 Fi n~ ~e velocity and the direction of the
Velocity of projection (u) =?
Striking velocity (v) = 7 -h = u sin 30° t or, u2 sin a cos a = - -2- - prOJection of a shot which passes in a horizontal
direction just over the top of a wall which is 250
1 or, u2sin a cos a = 288.12 ... (ii)
We have,

H=2g
u 2 sin 2 a
-2gt2
1 1
Dividing (ii) by (i), we get
288.12
.
m off and 125 m high. (g = 9.8 ms-')
Please refer 2066 Q.No. 1-!b
14 I

u cos a= 19.6 = 14.7


or, 10 =~
uz sin2 30•
or, -1.6 = 18 X 2 X t- 2 X 9.8 X 12 ... (iii)
~- 2074SetAQ.No.13bO~
Fin d the velocity and the direction of the
or, -1.6 = 9t- 4.912 Squaring and adding (i) and (iii), we get
2 or, 49t2 -90t-16=0 u2 sin 2a + u 2 cos 2a = 19.62+ 14.72 pro jection of a shot which passes in a horizontal
or, 200 = uz x (½) or, (t - 2) (49t + 8) = 0 or, u2 = 600.25
direction just over the top of a wall which is 250
mo ff and 125 m high. (g = 9.Sm/s') (41
u=2o{2m/s. t=2,49
-8 or, u = 24.5 m/ s
Hence, the velocity of projection is 24.5 m/s.
... Please refer to 2066 Q.No. Hb
We have, Rejecting the -ve value oft, we have t = 2s
~- 2072 Set E Q.No. 13b oRj
!9. 2075 Set A Q.No. 13ij
R = u cos a T, where T be the time of flight. If x be the horizontal distance apart between lfR be _the horizontal range and T, the time of
Wilb what velocity must a body be projected at
or, 2o-.{3 = 2o{2 •cos 30" • T the two players, then
:~ an~~ ?' 45° from the top of a tower 65 m fli ght of a projectile, show th.it tana = ~
or, 2oV3 = 2o{2 -l T x = u cost 30°1 = 18 x f x2 = 18'/3 m gh, if 11 1s to reach a point on the ground 180 m
~base of the tower. (4I ..
whe re a is the angle of projection.
Please refer IO 2058 Q.No. 1-'b
(41

T =...Ji Distance between the two players is 1 ~


Let v be the velocity with which it strikes the
154. 2071 Set C Q.No. 13 ij . Let .u be the velocity with which a body be I~!·1-(fi·.Uliill·UM
grow1d at an angle 0 with the horizontal, lhen
~- 20lt (Sit Al Q.No. 1!
A stone is thrown horizontally with veloc•I)' pro1ected and t be time taken by the body to
V COS 0 = U COS a = 2o{2. · COS 30° nd r~ach the grow,d. Now, taking upward
-./2gh from the top of a tower of height h. fi The
horizontal and the vertical coDtponents of
where it will strike the level ground through lhl direction positive, we have nitial velocity of a projectile are U and V. If
= 2o{2 -~ = 1tr{6 . • striking the i
the range and H, the greatest height
f oot of the tower. What will be its 141 -h -- .
U Sill a . t _ 1 g t2 R be
2 attain ed, p.rove that •
r
and (6)
velocity?
v sin 0 = u sin a - g-T
= 2o,[2 · sin 30° - 1o,[2
• Please refer to 2064 Q.No. 14 b , _ _ _ - 01
' • 65
1
= u su'1 45° . t-2X 9.812 4
(a) _!!
R
=ij (b) (i =~

=2of2 -½-1of2=0
@6. 2071 Supp, Q.No, 13ij
A body thrown from a point in a hooi : 1 a
4 st
, 0 tal
or, -6S =~ ..Ji - 4.912 ... (i) .. ease refer 10 Model Set II , Q.Nu. 14
Pl
plane comes back to the plane In c 1100.
Squaring and adding, we ha ve distance of 58.Sm from the point of prol~ 1!141 ~ e particle hits at a distance of 11!0 Ill (rum
v2 = (1tr{6)2 + 02 9 811 e base of the tower, so
Find the ve locity of the projection. (g'" · s
v = 1o,[6 m/s
Asmltt'• NEB sorutton of s,slc Mathematics-II DYNAMICS (CONTINUED) Unit 15 277
276
-=~==
s.==2==0==12==s==u==p==
P'= .1
a=:.N;=o==;[4]r-
=; ------- ~
lo::{ 2069 (5et Bl Q.No. ~ Fron1 a point on the ground at a dlstan or,
t,45,. 2 )( 9.8 f!f 20eO Q.No. i]
~
~ A car covers a dista

I
. tal range of a projectile and h the foot of a vertical wall, a ball is thr:e ll fron 1
If R be thehh?ghnz;i°nprove that its initial velocity is si112<:t "' 9.8 frlctionat force. U ~: of 50m in 5 secs against a
is greaiest el • angle of 45° which just clears the top of:: at an
and afterwards strikes the ground at a di e Wall
or, 1 4000 watts, find th metier
of the engine is
121
2g ( h 1~~) .
+ (6) on the other side. Prove that the heigh ance y
. 2l'....
st
tofth~
2
or, si11 a."' 2
1
-
Given,
e ona1 force.

.. Please refer to 2057 Q.No. 14 t, wall is x + y . siJ1a."':.{2 Distance covered = 50 m


161
g
'
2070 Set DU.No.]! .
:ectil thrown forff\ a point in a honzontal
i!.:
. - Please refer to 2060 Q .No. 14 b
~7. 2073 Set C Q.No. 14 OR! cosCJ. _·c-m
,,,..Jl-sin2 a Time taken = 5 s
Power = 4,000 w
A 1com:S back to the plane in 4 s~- at a
~stance of 60 D\ in front_ of the ~int_ of
. . Find Ille vel0<1ty of proJection.
----
If R be the horizontal range of a projectiJ
is greatest height, prove that its vet e ~nd h
oc11y Is
='\J~(iY frictional force (F) = ?
We know that, .

P"2~!;;)
(g Please refer to '1()69 (Set A) Old Q.No. 14b
(6)
2g(h+!~) ·
=J.l=~=}i Power_ Wor~ done_ Force x distance
time time

- we have, or, 4,000 = F x_ 50


!3. 2071 s.t D Q.No. 14J2~ Please refer to 2057 Q.No. 14 b
u2sin 2a u 2.2 sin a cos a ::,
A cannon ball bas the same range R on a F=400N
~8. 2073 Set D Q.No. 14' R"'~ g
horizontal p.lalle for two different angl~ of 1
fts. 20&1 a.N«sY

-
A projectile thrown from a point in horizonta 1 1
I
ro ·ection. ff H and ff1 are the greatest heights
i /w plane comes back to the plane in 4 secs
1 (9.8)2 )( 2 X Jf 1?- A car of mus 1000 kg. moves up an incline of 30?
0
paths for which this is possible, prove
tbatR2=J6Hffl. (6)
distance of 60 m from the point of projec~I a
Find the velocity of the projection. (g = iouJs~j
9.8 9.8m ;ra c?nslant speed of 20 m/sec.. U the frictionai
~rce IS _21XX>N, calculate the power developed by
0•;.::r,e.
I Let a and 13 be two differer.I angle of
projections having the same range R Then, R
• Please refer to 2069 (Set A) Old Q.No. 14b
~ 9. 2074 Supp Q.No. 14'
161
p ·i:fi·Uliiit•Hf1
(g = 10uysec2).

Here, mass (m) ;. 1,000 kg


(2)

= ~ a n d R= u2sin2jl ffl061:Q.No. 5 ij Angle of inclination (a) = 30"


g With what velocity must a body be projected at
g Ca)culate the power of a pump which can lift 300 Velocity (v) = 20 m/ 5
an angle of 45° from the top of a tower 65 m

I ---=---
u' sin
g
u sin 21}
2a 2
high, if it is to reach a point on the ground 180 m
from the base of the tower. [6]
kgs of waters through a vertical height of 4m in
10secs. [g = 10m s· 2) (2)
Friction al foICe (F) = 2.000 N
Force acting on car (F)
or, sin 2a =sin 21} - Please refer to 2072 Set E Q .N . 13b OR !!il.l!Ifl Component of weight down the plane +
Since a -., /3, we must have Mass of water (m) = 300 kg frictional force
21} = 180" - 2a ~o. 2074 Set B Q.No. 1~ mg sin a+ 2,000
From a point on the ground at a distance x from Time (t) = 10s
or, 13 =90°-a 1,000 X 10 X sin 3() 0 + 2,()()0
the foot of a vertical wall a ball is thrown at an Height (h) = 4 m
u 2 sin2 a We have, Power o f the pump 1
'Ve have, H = ~ • angle of 45° which just clears the top of the wall = 1,00J X 10 X 2+ 2.00J = 7,000 N
and afterwards strikes the ground at a distance y = Work done Weight x Height_~
u 2 sin 2 ~ u 1 sin 2 (90° - a) Tune We have,
on the other side. Time t
andH1 = 2g 2g Power = Force " Velocity = 7,000 x 20
_ u2 cos2 a Prove that the height of the wall is -!;y. (61 300 X 10 )( 4
=--10--'- = 1,200 W
=140,00J W =140 KW
- 2g
... Please refer to 2072 Supp. Q. No. 14 [7&. taa Q.NQ.S ~
A pump having a power of 294 w pumps water

-
u• ·4 sin 2 a cos2 u.
~1. 2075 Set C Q.No. 140Rj :""'a 2059 Q.No. 6 d at the rate of 90 liters per minute. Find the height
gl
A pa.r ticle is projected with a velocity u. lf the ~ car is_ moving at 36 kmh-1• What velocity will to which the water is raised. (g = 9.8 m/s2, 1 litre
u 2 sin2 u u 2 cos2 a greatest height attained by the particle be H, ouble its kinetic energy? (2) of water= 1 kg) (2]
=4X4 X ~ X ~ =16.HH'.
prove that the range R on the hori~ont~l pla~: ~
through the
I½, 2072 Set C Q.No. 1 ij point of pro1ect1on
Here, v = 36 k m / h r = ~ = 10 m/s
Power= 294 w
If R be the horizontal range of a projectile and h
its grea_ test height, prove that its initial velocity R= 4 ✓ H(ii- H) .A body is projected ~it~ If _ 60 x 60
th e mass of the car is m kg, then its Mas.s of water ejected per second {f)
is✓2g ( h + 1~~) . K E-l
a velocity of 9.8 m/sec an d rises upto the hei~~I 1 3
· - 2 mv 2 = 1 m • (10) 2 = 50 mJ
9() X
(6) 2.45 m. Fi nd the horizontal range . 2 =fiokg =2kg
lfitsKE · doubled, then new K.E = 2 x 50 mJ
~ OOo · 15
• Please refer to 2057 Q .No. 14 b B!mmJ 1 Height (h) = ?
OR Th~n mJ and let its new velocity be n
~5. 2072 Set D Q.No. 16 OR!
First part: Pl ease refer to 2059 Q.No. 14b We have,
Second part:
1 • Workdone ~
Describe motion of a p rojectile. A stone is Power = Time taken = t
Velocity of projection (u) = 9.8 ni/sec. 2mv,2,, 100
thro wn horizontally with velocity from -J2iit Ot
, v11., 20()
m
the top of a tower of height h. Find where it will
strike the level ground through the foot of the
G rea tes t height (H) = 2.45m.
H o rl w nL'.1 1 range (R) = ? Vp , . ~
~200 = 1o,[2
or, 294 = {f} ~ g• h
tower and also find the striking velocity. We ha ve,
~l!tj uire .- 3
... Please refer to 2064 Q.No. 14 b u 2 sin 2c,, ct velocity= 10\[i. m/s or, ,94 "2" 9.8" h
H .. - -
2g
(
f Baalc Mathematics-II Unit 15 279
Asmtt••• NEB so1ut1on o DYNAMICS (CONTINUED)
278 At the point A so

~fJI
294 )( 2 K.E. s 0 or, lOOx X= 10
or, h=9,SXJ P.E. = mgh
X'= 20 HP
K.E+ P.E= 0 + mgh = mgh
. h-20m
~ Required HP of the engine %20 HP
ff 2071 Old Q.No. 6 ij
A car is moving at 36knv'h·
What velocity will
[2)
At the point C
Let v, be the initial velocity of the bod
point C. TI,e.n. v, 2 = 2gx. Yat the
E2064 0.No.14 b o~
double its kinetic energy?N 6 c 1 1 ~:rce ilcla on ii body, prove t~t the change in
2 B F C
9
.,. Please refer to 205 Q. o. K.E. = mv, =2m - 2gx = mgx d c tnergy of a body is equ1 to the work
2 one by the force.
[ii. 2072 Set E Q.No. 1 B
ofa pumpw
hich can lift 300
. 0
P.E. = mg(h - x)
J\I the point A
1 .:.__ Please refer to 2058 Q.No. 14 b OR [4I
Calculate the power vertical height of 4m m 1 K.E. + P.E = mgx + mg(h - x) = mgh _.!.mu2=- m -02=0
t(.E. -2 2 it_2065 0.No 14 b O~
kg of water through a (2) At the point B
2
secs. (g = 10 m/s ) When , P.E. = m(g sina) 1 = mg (I sin a) = mgh Slate and prove the Principle of Conservation of
Let v be the velocity of the body Energy.
reaches the ground. Then, 11 K.E + P.E = 0 + mgh = mgh
mlmi1\il . h=4m t=lOs At the point D . .:: Please refer to 2057 Q.No. 14 b OR l4l
Given, = 300 kg
Ill vi= 2gh
Weight (faro.' due to gra~; ty) = mg 1 1 Let v, be the velocity of the particle at D. The @!. 20M Q.No.14 b §!
K.E= mv 2 = m ·2gh= mgh
2 2 acceleration down the inclined plane is g sin A bullet loses 1/'Jllf' of its velocity in passhlg
= 300 " 100 = 3000 N
a. Also, v2 = 2 g sm a · x = 2gx sin a thr
We have. P.E=O ough_a planic. Find how many such unifonn
Weightx Height 300}><4 = 1200 watt 1 1 . Planlcs 11 ~ould pass through before coming to
K.E+ P.E=mgh+ O=mgh K.E = 2mv2 = 2 m x 2gx sm a = mgx sin a
Power - Time 10
The sum of K.E and P.E of the freely falling rest assumtng the retMcution to be uniform. (4)
fig 2073 Set C Q.No. 12q body at any instant is same (i.e. mgh) and P.E. = mg (/ - x) sin a !Ema
. , large a force is required to cover a distance hence it is constant. K.E + P.E. = mgx sin a + mg (/ - x) sin a Let u be the velocity of penetration. Let x be
0
:C 12) = mg (/ sin a) = rngh the thickness of each plank. The velocity of
~ if the tot.I work done is SOOJ? ~1. 2osa Q.No. 14 b oRj At the point B plank after penetrating the first plank
!il!IIIII "The change in kinetic energy of a body is equal Let v be the velocity at B. Then u 19u
Here. distance covered (d) = 80 m to the work done by the acting force". Prove this vi= ()2 + 2(g sin a) · l = 2g / sina
=u-20=20
Work done (W) = 800 J statement. [4]
Force (F) = 7 BID
1 2
K.E=zmv = zm
1
X 2glsina=mgn sina)= Change in KE =lmu 2 -lm
2 2
(l9u)
20
2

We have, Let F be a force applied on a body of mass m. mgh


W =F•d Let a be the acceleration. TI,en, 39
P.E=mg x O= 0
800 =F><SO F=ma
=
800 mu 2

K.E+ P.E = mgh + 0 = mgh


Let s be the distance covered by the body We have, work done by the resistance force F
=800 = 10N Hence, the sum o f K.E and P.E is constant =fxx
80 when it changes its velocity from u to v. Then, throughout the motion. 39
Required force = 10 N v2 = u2+ 2as F xx= 800 mu2 ... (i)
!3, 2062 Q.No. 14 ij
or, v 2 - u 2 = 2as
1 An engine pumps 746 liters of water per minute Suppose that the bullet passes n planks before
2 coming to rest. So, the total distance pas.5ed by
@0- 20.57 Q.No. 14 b 0~
Initial K.E of the body = 2 mu from a well through an average height of 60m.

--
Find the horse power of the engine if 50% of the the bullet is nx.
Define work, power and energy. Prove that the 21 1
Final K.E of the body= 2 mv power is wasted. (1 liter of water = 1 kg., g = 10 1
sum of the k.inetic and potential energies of a nysl) (4) Change in KE. =2mu 2 - 0 =2mu2
freely falling body remains constant throughout 1 1 l
Change in K.E of the body = 2 2
mv 2 - mu Work done= F x nx
the motion. (4)
Mass of water (m) = 746 litres = 746 kg or, F x nx =½mu2
lB!.im 1 _
= 1 m (v2 -u2) = m . 2as - m -as
=(ma)s
First Part: 2 2 . [·.· 1 litre= 1 kg (given)) 1
Work: The work done by a force is defined as = f x S = work done by the force Tune (t) = 1 minute= 60s F ><.r= ~mu2 .. . (ii)

the product of the force and the distance Height (h) = 60 m From (i) and (ii), we have
~2. 2061 Q.No. 14 b OR! f erav,
moved in the direction of the force. tion o en ., Horse power (HP) = ?
State the principle of conserva . of a body We have, .lmu2=~mul
Power. The time rate of work d one is called [41 2n 800
lllustrate it with the consideration
Work done Power Workdone ~ or, 78n = 800
power. So, power = ~ . sliding down a smooth inclined plane.
- lime taken = t 800 10
Energy: Energy of a body is defined to be its or, n " 78 = lO'jg
liml!JB 1·desdown a
61 d heigh!
7-16 X lQ X 60
capacity to d o work. Suppose a p a rticle of mass m
1 60 ~7. 2066 C Q.No. 14 b oRj
Second Part: smooth inclined plane AB with AB = anl1orizOP· = 7460 watts
h which is inclined at a n angle u. to Lile An engine pumps 746 litres of wat~r per minute
Suppose that a body of mass m is
Horse power= .?.:!£Q HP = 10 H P from a well through an average. height ~f 60 m.
initially at the point A which is a t a h . 7-16 Find the horse power of the engine if 50 Vt, of the
height h from the grow,d B. Let the Th en, sin o. = T ⇒ h = / sm a. poiol S'
d get the ince 50 % o f the puwcr is wasted. So the II
erficic1 · f
body start falling from A and C be • • SO"' power is wasted.
Let the pa rticl e i;tart from A an (1 litre of water= 1 kg, g,. 10m /sec2) 4
the position of the body a t any
le . icy o en gm e 1s (lllO - 50) %"' ·" ·
D at any instant suc h that AD "' x. 5() t l be th e req uired HP of th<! engi ne. Then, .,. Please refer to 2062 Q.No. 14 b
instant such that AC = x. Then Then BO '-' 1-x \ orx .. 10
BC= h-x
• -:j
Mathe matics -II 1
Asmita 's NEB 5<)1utk>n or Basic
280

Qij
"8· 2067 Q.No. '"' 200
A ballet of '.1'~f 500
is fired into a t.u-get
S:-•. lf the m.an of the
!llDli&1ZI
Let v and V b;e the velocity
gun respec tively .
Mome ntum of the shot = mv
of the sh
01 ill)~
Wehave,
= -18()(X)() m
wordone = F " s2

k done = chang e in KE
DYNAMICS (CONTINUED)

~ P - Q.No. 1Jb ~
• P.lrticle i1 allow ed to Ii
Unit 15 281

'
move, find the loss a smoot h
: free t IJlclined plane. Show thats hde dawn
with a ~oot y
targrl is 4.8 kg md in • o
(4) Mom entum o f the gun = MV w~rsi = -180000 -~ .. . (ii) of its kineti c
and J>OtentiaJ ener ·es t_ e SWn consta nt
of lcinetic energ y by the unp.a d We know , ~vidi ng (i) by (u), we get throag hoat its moti gi LS always
of &un {4J
Mom entum of the shot= mome ntum F•S1 ~ Pi on.
gm = 0.2 kg p:;; -"' ease refer to 2061 Q.No. 14 b OR
~ ~ o f bullet (in_! = 200 or, mv=M V = -1800 00m
of M = 500 :s
m MV
!! €.w2 Set C Q.No. 13b O~
Velocit...- bullet
Mass of the targt>l (M) =
Let \ . be the ,-etoaty
4.!~ comb inatio n
or, v = ~
Again , we have
... (i) SJ
;;= 9 Also
. l
State the pri
nap e of conservation of
~ove that the sum of. the i,:__~
nergy.
-.-uc and
. . sJ : s2=1 1:9 J>OtentiaJ energies of a movm g body ~
ediate aftel" the impac t. E = total kineti c energ y - constan
supp. a.No. 13 b o]
.
. - ,_ of conservatio n of linear ftj!o !4I
unm the pnnaf
Using 'tt" 1 1 5 - throag hoat the motio n.
or, E = 2 mv 2 + 2 MV2 · ~.rtide is slide down a smoo th inclin ed , __
P1411e.
mome ntum.. .
f ·ts kin etic
. 1 M 2 V2 1
1'r-- th
SboW that e sum o I
and Potential y: r J...L
m.- = (m .. MJ\' Principle of Conservation of Energ •vuue r
0£, 0.2 X 500 : pl + 4.8)\ oT, E =2 m ~ + 2 MV2 IUSing (~J eoergjes alway s const ant throu ghout
its motion. theact ion 0 f
a conservative system of forces,
(4) the sum of the kinetic and potenl iaJ ener .
E =½MV2 (~+ 1)
100 OR
or, V =5= XJm/ s .. Please refer to 2061 Q.No . 14 b of a DlOving body remai
ns cons ~
or,
oM - throughout the motio n
KE belae impact.=½mv= =½ 0 2 " (500)2
x ~ 1 01d a.No. 1, b
that the z;on d Pa.rt: Please refer to 2057 Q.No.
1-t b
E =½MV ( M: m)
2 Define work, row:e r and energ y. Prove y of a
= BXXJ J OT,
the kineti c and poten tial energ
5111D of
1 ant throug hout ~- 2074 Set A Q.No. 1~
= 2 (M + m) V2 or, 2mE = MV2 (M + m) fret)y falling body remai ns const
KE afa impac t [4]
2mE the motion. ~ ~article is allowed to slide down a smoot h
1
= 2 (4_S + 0.2) X 2()2 or, V2 = M(M + m) .. Please refer to 20.57 Q .No. 14 b OR mclined plane. Show that the Stllll
of its kineti
s constan;
j1.2071 Set C Q.No. 13 b 0~ and potent iaJ energies is alway
= ½x .5 ><(20}2 = 100'.)J V = ✓Mf::m) find the H.P. of an engin e which
can travel at
oft in 200 the
throught out its motion.
._ Please refer to 2061 supp Q.No. 14 b OR
141
J = 2 4{XXJ J Hence , gun recoil s with a · velocity therat eof14 4kny'h r up an inclin e
Los5 of K.E = 25aX) J- l(XX) being 15 metric ;ons .[01.2075 Set A Q.No. 13b O~

--
mas, of the engin e and load
ps. 2068 Q..No. 1-' b 0lf ✓M ::m) · etc. being 15 kg that the
md the resist ance due to frictio n Define K.E. and P.E of a body. Prove
[4) lo the work
If a farce be applie d on the body,
prove that the wright per metri c ton. (g = l0m/sec2). change in the K.E. of a body is equal [41
is equal to 2070 Set C Q.No. 13 a oij done by the force.
chmg e in kineti c energ y of a body
Bm l
. 34 .
f

(4] two planks in


the work done by the force. A bulle t passe s throu gh Let a be the angle of inclin
ation of the
b OR is 1200 m/s and First Part
.,. Please refer to 2C68 Q.No . 14 succe ssion whos e initia l veloc ity inclined plane with the horiz on
penet rating each of a body
loses a veloc ity of .200ny's in PotenliaJ Energy: Potential energy
jgo. 2069 (Set A) a.No. 13b o,J plank . Find the ratio of the
thickn ess of the Then sin a =...l.. is its capacity of doing work by
virtue of its
Prove that the 200
Defin e work done by a force. offer the same state or position and is measured
by the
is equal to plank s assum ing that they
change in kinetic energ y of a body (f] " 1000 = 40 m/s amount of work which it can do in
changing
(4) a verag e resist ance. Again, 144 km/h r= 144
the work done by the force. 60 X 6()
from its actual position to some
standard
14 b OR
Fust Part Please refer to 2057 Q .No. As the engin e move s up an inclined
plane, the
position. It is denoted by P.E anJ
given by
Q .No. 14 b of first
. plane and 52 be
Secon d Part Pl.ease refer to 2058 81 be the thickn ess resolved part o f the weigh t of the
engin e and
-Let
- P.E. = mgh, where m = mass of the body
OR the thickn ess of secon d plank . lhe resista nce force act down
an inclined
g = acceleration due lo gra vity
Here, u = 1200 m/s,v = 1000 m/s plane.
1\11. 2069 (Set A} Old Q.Ho. 14b o,! tons = 15 •
h =height
booy is its
that the sum cf 21 1 2
Mass of the engin e (m) = 15 metric Kinetic Energy: Ki.neti c energy of a
Defin e work and energ y. Prove Chan ge in K.E 2 mv - 2 mu l(XX) ,, 15000k n.
the kineti c and poten tial energ
ies of a freely
The£ _g capacity lo do work by virtue of its motio
v be its
fallin g body at any inswt t is cOD5t
a.nt. (4) 1
(1) resol~rces acting o n the e~gin
e are: . If m be the mass of the body and
b OR =2m (v2 - u2) ed part of the weigh t of the
engin e velocity, then
- Please refer to 2057 Q.No. 14 d
orl own an inclin ed plane I
~ 2. 2069 Old (Set B) Q.No. 14b =! m(HXXJ2 -12002) =- 220000 rn K.E, =2[\IV1
princi ple of 2 N
What do you mean by the " lltg Sin a= 15,00 0 x 10 x _!_ = 750 Second Part: 2058 Q.No. 14 b OR
its validi ty for a Also, work do ne = F " SJ lb) res18 200
coiue rvatio n of energ y? Verify I4J n
We kno w that " 15 ~ance force due to frictio Ho2. 2075 Set c Q.No. 13~
body fallin g under gravit y. a target with
.. Please refer to 2057 Q.No . 14
b OR work done = chan ge in K.E " JS • g Wt per metri c ton A buJM of mass 100 g is fired into

-
= - 2200000 ... (i) R · l0 )( 15 N = 2250 N a velocity of 500 ms·•. The mass
of the target is
~3. 2010 (Oldl a.No. 1, b orl F )( S1
esulta t f e down an free to move; find the loss of
from a gun of Agajn, l11c1in n ° rce ac ting o n the engin 4 9 k and is [4]
A shot of mass m is projec ted u .. ,ooo m/ s, v = 800 m/ s Weh ect plane "' (750 + 2250) N ..
3000 N. kineli~ energy by the impact.
gener ates a p ave,
mass M by an explosion which 1 2 !mu
2 -vvVV)
gun recall s with -- mv - 2 01ver ., p
kineti c energ y E. Show that the Chan ge inK. E 2 orce x veloc ity = 3000" 40 '" lAMJU Here,
2mE ~C(fUlrect 120000 . 0.1 kg
14) 1 ()02 _ 100()1) .. 160.86 lll = 100g =
a veloc ity M(M + m) . 1
.. 2m (v 2 - u2) .. 2m (8 H .P of an engi ne = ~
M =4.9 kg, v =500 ms·•

L
Asmlta's NEB Solution of Basic Mathematics-II ~ DYNAMICS (CONTINUED)
282 We know that, ~ Unlt15 283
Let V be the velocity of the con,binatiol\ Momentum of the shot ~ monientui ~part: ..
inetic Energy: Kinetic, energy of a bod . .
L!.!!: 2074 Supp Q.N
A bullet o. 14 ~
iJIUnediate after the impacl . combination " ()f lhe
J( acity to do work by virtue of it., Ill _YIS tis succ J)ilsses through
Then. by using the principle of conservabon or, nw = (m + M)V cap be the mass of the bod lose esslon. Its initial velocity _two planks in
of linear momentum. 20 ( 20 380)
If rn . th
otion.
Y and v be its thr«! a veloeity of 200 m/:
~ m/s and it
we have, or,
1000 " 200 = lOOO + 1000 v velodty, en Ihle~ each plank.. Find thetn ~etrating
the sa ess of the planks, a58Umin ratio of the
mv=(m+ M)V ,,,,)mv 2
400 ((,i;;, 2 ._ PlIlle average resistance. g that they offer
or, 0.1 " 500 = (0.1 + 4.9) V
V =lOms-1
or, 4= 1000 v potential Energy: Potential energy f fil ease refer to 2070 Set C Q.No. 13a OR (6]
V=lOm/s . its capacity of doing work by '"~-a body
I<E before the impact = ½m,,,
I5 ' ti d ' •u•ue of tis - 2075 Set B Q.No. 15!
1 1 20 tale or pos1 on an 1s measured by th
Initial K.E = 2 mv2 = 2 x 1000 " (200)2 "" 400 l a. A rocket expels
1 :inounl of work whi~ it can do in chan e the veloeity of th':9 at _'he rate of 0.4 kwa. U
= )( 0.1 X 5002
2 Again, fro!D its actual position to some gmg force produced by ~ is 400 m/s, what rs tbe
= 12500) 1 1 400 pasition. It is denoted by p E and s ~ d b. A particle is . e roc~et?
Final K.E=z(m + M) v2=2" 1000" (10)2=2oJ p.E. = mgh, where m = mass ~f the i,;;ven
by m/s, at an an:i:;;:. wtth a velocity of 49
KE after the impact = ½(M + m)V2 Loss of K.E by the impact = 400 J - 20 J = 380 J g = acceleration due to gravity y the tune of fligh to the horizon, find
h=height c. A pump ha . t and the range.
= 1 (4.9 + 0.1) x102 j1os. 2071 Set D Q.No. 14' - water at th vmg a power of 392 W pumps
2 Define potential energy and kinetic energy of a
Second Part: Please refer to 2057 Q N
~ -~Ub e rate of 100 litres
1 Find the height to which p~ mu_iute.
= body. Prove that the sum of the KE. and P.E. of a {g = 9.8 m/s'- l li water IS raised
2 x5 x100 freely falling body at any instant is constant. [6] §!)o73 Set D Ci.No. 14 oRj -- , tre = 1kg) (6]
= 2.50 J. air: mo~ing at 20m/s, imping on the
Loss of KE by the impact =12500 - 2.50
amm 800 kg of
vanes. of a WI~dm1ll every second. At what rate
a. Here,

..
. = 12250]. First Part
Potential E~ergy: P~tential energy of a body
is its capaoty of domg work by virtue of its
in kilowatt 1s the energy arriving t th
winchnill? What is the maximum mass ofa watere Expel rate (7) = 0.4kg/ s
that could be pumped each second thro gh
state or position and is measured 'by the Initial velocity (u) = 400 m/ s
vertical height of 2.5 m?(g = 10 mfs?) u [~
ft o3. 2069 (Set Bl Q.No.14a oij amount of work which it can do in changing Final velocity (v) = 0
A car of mass 2000 k.g moves up an inclined from its actual position to some standard Force produced by a rocket (F) =?
plane at an angle 30" to the horizon at a constant position. It is denoted by P.E and given' by Mass (m) = 800 kg We have, ·

-
-.,eecf of ZJ)m/s. Uthe frictional force is 2000N, P.E. = mgh, where m = mass of the body Velocity (v) = 20 ms-1
-F =7(v-u)
calculate the power developed by the engine g = acceleration due lo gravity We have,
(g = 10 nys 2 ) . (6) h = height 1 1 = 0.4 (0-400) = -160
Kinetic Energy: Kinetic energy of a body is its K.E. = 2mv2 = 2 " 800 x (20)2 = 160000 J F =160N
capacity to do work by virtue of its motion. b. Velocity of projection (u) = 49 m/ s
Here, mass (m) = 2000 kg
If m be the mass of the body and v be its Work done = 160000 watt = 1 ~ kw = 160 kw Angle of projection (a) = 30"
Angle of inclination (a) = 30°
velocity, then Time of flight (I) = ?
velocity (v) = 20 m/ s ~gain, let M be the maximum mass of water Range (R) = ?
Frictional force (F) = 2000 N
Force acting on car (F) = Component of
weight down the plane + frictional force
1
K.E. =2 mv 2
Second Part: Please refer to 2057 Q.No. 14 b 0!..-
r:
that could be pumped each second through
vertical height of 2.5 m .
or, 1 en, work done = mgh
We have,

T
2u SU\U
g
2 X 49
9.8
X sin 3()

= mg sin a + 2000 1106. 2072 Set D Q.No. 1ij 600<JoJ=M><l0><2.5 1


= 2000 " 10 " sin 30° + 2000 Define energy. State principle of conservatjon ~f 2 X 49 l<
2
1
:. M"" 160000
energy. Also prove that the sum of the ldn~llc ..,,____ 10 x 2.5 = 6400 kgs·• = -9-.8- = 5 sec
= 2000" 10 x 2+ 2000 =12000 N . • b Ody remains
and potential energy of a m?vmg 161 Again, we have
We have, constant throughout the motion. ~Supp Q.No.1~
Power =Force" Velocity = 12,000 x 20 e the Prin . R - ~ _(49)lsin(iJ'
Aho prove t ciple of conservation of energy. - g -
= 240000 W = 240 kW BB is its capacity of Potential hat the sum of the Kinetic and
9.8

hD4. 2070 Set D Q.No. 14 oij


A bullet of mass 20g is fired horizontally into a
Energy: Energy Of a body
doing work . . Under
Principle of Conservatio_n of En~rg~i forcei,
tona1illl t:~ergies of a moving body remains
~ flnt p ughout the motion. [6)
2401
= - -.-
xf
- = 212.!Sm
con:;= Please refer 2072 Set D Q.No. 15 98
suspended stationary wooden block of mass 380 the act10n of a conservative sys~c~I energi,~ c. Please refor to Model Set 1 Q.No. 12c
g with a velocity of 200 oys. What is the common art: Please refer 2057 Q.No. 14 OR

-
the sum of th e kine tic and pot~n I co11Sllllll
velocity of the bullet and the block if the bullet o f a m ov in!', body reniau
15 ~.
"'fine
4
SetBQ
.No. 15 O
[u. 2075SetCQ,No,1~
is embedded into the block? Find the loss of KE 'lie energy St Define kinetic energy and potential energy with
by the impact. {g" 10oys2) 16 j througho ut the motion. . N 14 p IJl tgy. i\lso · ate principle of conservation of examples. A shot of mass 'm' is projected from a
Next part: Please refer to 2057 Q. v. ~ 4 Potenr prove that the sum of the kinetic gun of mass 'M' by an explo~ion, w~~
l1 07. 2072 Supp a.No. 14 ~ . of a bodY.
~in, to lal energies of a moving body generates a kinetic ene.rgy E. Find (1) the imtial
20 , " stant throughout the vertical motion. velocity of the shot (ii) the velocity of the gun. (6)
Here, mass of the bullet (m) =20g = D efine kinetic and potential cnergte~ 311 d tbl
1000 kg 0 fiil1 r 161
Prove that the s um of the 1<in~t l~il,g b dJi
380 Stc ;;: Please re fer 2072 Set D Q.No. 15
Masso f the wooden block (M) = 380 g • 1 kg potential e nergi e s of a freely a I 011
000 remains constan t throughout the inotloll• art: Please refer 2057 Q.No. 14 OR
Velocity (v) = 200 m/ s
16
,1c M1them1ttc1-II
284 ,umltll'I NEB s01ut1on of Bl

or, E =2
(M+ 1)
ltv1v2
IS,l lil m
UN IT
First part: Please ref-er to
2071 Set D Q.No. 14
1 (M m)
+
SecOJld part:
Let v and V be the uu
. 'tial velocities of the sh
ot . or, E "2MV 2
----;; -
imE" 'MV2 (M + m)
LI NE AR PR OG RA M M IN G
and gun respeclivelJ~t = ntv or, _w§ .-,
Mornentwn of the E MV or, V2 = M(M + m)

'.!J-~•~•~ ,._r. ,.".r.i.-■ U•l•


Momentum of the gun
We know, shot~ momentum of gun
.-. v = ~ •.
Momentum of lhe ,·
or, mv=MV gun recoils with a velOCity
MV ..(i) H~ 4 2 68 0.No.3
or, v -=-;- '\jtJ(M+m)· ·
GtaP
h the half plane given by: y - x ~
1
. wehave
~total).-irletic energy AJld, initial velocities of the shot,
[2\
- -·v en inequ ality is y - x 2:
1
1
or, E =2nw2+ 21'
1 N2 MV
==-;- : i:- ,~line~ y - ,- 1

o := ~
V
1 ~+l MV ' (using (i)]
or, E =2n:t nt2 2
,,~~ (;:; Ill) fu;{co,O) as a testin g point , we get
I -~
,,~
, o-02: 1 (False)
Hence, the half plane deter mine
Now, We draw the graph .
d by the given inequality does oot
contain origin .

y-x= 2

.. ~ 2067 Q.'No.

We have,
4x+3 ~ 2x-1
or, 4x-2x ~ -l-3
3d
Find the solution set of 4x + 3 > 2x
- - 1.

x~-2
Y'

121

or, 2x~-4
-3 -2 - 1 I 2
or, ~ ~1
or, X~-2
~ olutionset (x: x ~ -21
~
~nn ine the half plane given [2]
by the inequ ility 2.t-Y < 2.
The · .
T~e given inequality is lr - y < 2 _
corres ondin , boW1da1 line IS 2I
• ., 2
1
0
LINEAR PROGRAMMING
286 Asmltll'• NEB Soiutlon of Basic M■them■tlcs-11 Unit 16 287
Q.Ho,3
Take (0, O) as a testing point, we get e graphically : x ~ y and %~-y
2 x O - 0 < 2 (frue) y [2]

I.
O'•ph ~ ~ """. bound
Hence, halfcontain
the not plane determined

~
Ii D-
But
- .· 3<j
it does the boun by ~e Y'_ .
...,,.line.gtven
So"""'mequality is on the sa'dotted
d,aw d, . M
of testing pomt

" ' ~••""• "' of 2x. 1 > b + 3


a,y line.

We have,
2r -1>4I +3 ce, the solution set is
I
⇒ 2x - 1 + 1> 4I + 3 + 1
⇒ 2r>4r +4
2C:No. 3cl
rmine the half plane given
. by the ine quality. 21'. y < 2, graphically. [2)
⇒ 2r-4I>4I - 4x +4
I ⇒ - 2r > 4
⇒ 2r< - 4
2r 4
⇒ 2 <-2 [2)
I .y<!:2x-1
⇒ .r <-2 -1 0
/.r: x<-2, xe9!/
' The
= correspond in .
uations of boundary line is:
LBlll!J!
raphicallysh ow the solutiono f:r-y-3> 0.
y
,
O~ 2 >< 0 - 1 (g
The correspo din boundary ,~ x-y - 3 =0 ·
line i.s n g true)
point (0, 0) in y 2: 2x - 1, we have

#
3
y

or--+ -~~ ~[_-_ ___.aX


X'
Taking (0, 0) as ates . .
O- O- 3 > ting point,
0
i.e. · 3 > 0 (false)
by the hall
So, the . plane determined
given in"" .1,
not contain th -......,_u=ty does
0). The e testing point (0
inequali solution of giv~ ,
in th ty is shaded as sh
e graph below: own
Y'
Ir 288 Asmrta'• NEB Solution or e..1c M1them1tlc1-II LINEAR PROGRAMMING Unit 16 289
:la~2C)66~Q~:N;o.~B::J________I' - - - - - -- - - - - - - - - ~ • t . A Q.No, 18
,, :o••d the feasible region determined by th
-

1:: Detennine the half plane given by the inequa ity a•• e 5
; ... zy~12,.r+ys ,x,y:.?:0
e foU~-1 •
v,. ng1nequalities:
y2:-x. 121
~ rresponding equations of boun-t2-·
[2)
~rresponding equation of given inequality is: ~~ -, lin ~~

3t +:Z)' "' 12 ... (i)

'I
fi-x I~ 1~3 . _1
Taking testing point (1, 0) 111 y 2: -x, we have x..,._~~~ct;~~zj$:f.b!--
t + y"'S
, .. o
.. . (ii)
... (iii)
O 2: -1 (true) )(
y,,. 0 :...:..----.....
12 . .. (iv)

~f T: I
So, the graph of y 2: -x is the plane region
containing the point (1, 0)

Y'

- rr:•yr:
raking testing point (0,0) in 3.r + 2y s 12. we get
/10. 2067 Q.No. ,j
Solve paphically: x - y s Oand .r:.?: 0 [2] 3 " o+ 2 " 0 s 12 (true)
y
The equations of boundary lines of given inequalities are:
x=O
.r-y=O .(i) 1
x=O .. . (ii)

From (~ raking testing point (0,0) in r + y s 5, we get


/; ~ I ~1 O+ OS 12 {true)

I Taking testing point (1, 0) in .r - y


have
I - 0 s O (false)
s 0, we x---i::--~+-'r½--+--,~+--+-_.....,
-3 -2 -1 5 }
From (iii) r = 0 which is y-a.ris.
x ~ Ogives the right half p)an~ing y--a.ris.
From (iv) y = 0 which is x-axis.
So, the graph of .r - y s 0 is the plane region
y ~ 0 gives the upper half plane including .r--a.ris.
not containing the point (1, 0) y-x=O y
From (ii) .r = 0 is y-axis and x 2: 0 gives the
right half plane including y-axis. y•

-lij.2111GJ1o.u
Determine the half plane represented b y the inequality y - .r :.?:1

Taking testing point (0,0) in y - r ~ 1, we get


0 - 0 ~ 1 (false)
y
(2]

y-x=1
-3 -2

~ t--+--+--+-- + -....- X y=O


1 2 3 4

-2 Y'
-1

l• 2y e easible region determined by the inequalities (21


So, the a h _ .r Y'
gr p y
· . .
~ 1 is the plane region wi thout contairun . ..
-- ' g thn., or1g1J1. ~ SlO,x+ y S6,.\', y~O.

'Ince0
nesponct ing t!quations of bow1dary liIies are
Alfflllll'S NE8 soiutton or Saale Mat11ematlce•II LINEAR PR0GRAM111NG
290 Unit 16 291
... (iii)
. (i)
r+2y • 10 ... (iv)
.. (ii)
_r+ y• 6 ... (v)
. (iii)
.r• O
. (iv)
y =O

f T:·~r:: 3
Taking testing point (0,0) in x + 2y s 10, we get
6

3 ~ O+ 4 " 0 ~4 (true)
0
y . testing point (0,0) in 3.r + 4y s 24 we t
raJong ,

'flle graph of 3x + 4y ~ 24 conlaim origin.


ge

0 + 2 XO$ ]0 {true) frolll (ii) y = O which IS r-aXIS.


y 0 gives the upper half plane containing x-axis.
2
fro!ll (iii) y = 4 is the line parallel to .x-aris and through the point
raJdrlg testing point (0,0) in y s 4, we get O4
(' )
o~ 4 (tr11e)
Taking teSting point {0,0) in r + y s 6, we get fro!ll (iv) x = 0 which is y-axis.
o+ Os6 (true) x:2: o gives the right half plane containing the y-axis.
From (Iii) r = 0 which is y-axis. from (v) .r = 7 is the line parallel to y-axis and through lhe point fl, O)
y
.r 2: Ogives the right half plane including y-axis.
From (iv) y = 0 which is r-axis.
y 2:0 gives the upper half plane including r-axis.

0(0.4)

y=4

-4 -3 -2 -1 0
-1
0
-·f!ic
x•-..._-1-__,..--1.,__~--'"~.,;;,a,a;;;.;;.;;i:.;,;;,;;,j,~~ ~i,.....:,,._~-1 ---L- x
2 3
v=O ACT.O
II -
3x+4y=24

1=7
Y'
~ required solution is shaded in the figure.
l 2070 Set D Q.No. 16 aj
Dnw the graph of the following inequalities. (2]
~ S 6 , 2x+y~8, y~O.

The corresponding equations of boundary !in


es are
x+ y "6

--
.. . (i)
'l' i- y 8
'"' .. . (ii)
H4. 2070 Set C Q.No. 11 ,j y" 0
Draw the graph of the following inequ.litie11: 121 F ... (iii)
roll\ (I) x + y - 6
3.r Hy s 24, o $ y si4, o ~ .r 1. .s
- . . 6
The boundary equ11tiol'lll of given ineq UJ1lities ar~
3x+4y "' 24 0
... (i)
Takin .
y =-, 0 .. .(ii) 0~ Og tesung point (0, O} ln x + y ~ 6, we get
S6 (true)
292 Asmlta'I NEB Solution of BIiie Mathemellc1-II W.EARPR 0GRAMMING
UnH 16 293
. 011 set C Q.No. 18
So, the graph ofx + y :S 6 is the plane re ·on .. 2
Again, from (ii) 2x + y =
8
gt rontammg the origin. 1 petennine
~
the feasible re
2t + y 93, x + 2y :s: 10, x, y r;n of the foll~,.,.,_.,
ff
_!_
1,
/s
-..,,
I• }
/o J
point (0, O) in 2l' + y 2: 8, we get
11" _,.pundlng "'""""' 01
Zt + Y "' 8
x + 2Y "' 10
... (i)
. (ii)
bounda .
ry lines are
-~
(2)

Taking teshna
X"' D (iii)
2 X O + 0 ,!: 8 (false) y•D . (iv)
So, the graph of 2x + y 2: 8 is the plane re . .
~12x•ya•
gion without containing the on·gm.
I
~-
From (iii) y = 0 which is x-~.,;< .

y 2: 0 gives the upper half plane including x-axis.


X _ I~ - I~
faJcjJlg testing point (O .
2" 0 + 0 S 8 (true) ' O) Ill 2x + y s 8
A ain, from i x + 2 = ' we get
10
X 0 10
5 0
ra1'illg testing point (0
0 + 2 X O$ lQ (true) , 0) in X + 2y $ 10
from (~) x = 0 is y-axis , we get
x 2: 0 gives the right half .
And, from (iv) y = 0 whi plane includin .

I
Y2: 0 gives the ch IS x-axis g y-ans.
upper half plane includin
Y gx-ans.

x+y=6

fil From
(4, 0), (6,the
0) and ,(2 4)ABC
figure .
'"P""""'Y· + Y=8
15 . the feasible region where th e 2xrooroinates of A, B and C are

. 207881ft: Q.No. ••i


. pphially the solution set of the inequabty:
Determine . 2x - 3y s 6
(21
-
Y'
correspondin y ·
The
boundary line is g equation of
region is shaded in the graph.
2x-3y =6

I;. I I~
6, we get
~2
Taking testin .
1
.
_
g pomt (0,0) in 2x - 3y S
-~ y , -,,ry :s; 24 x ._.,, ca
..,..-qww•"
the feasible region determined by the foll . . (2)

2x0-3x0 < 6( The co


3x + 4yrrespond · equations of boundary lines are
So - true) = 24 mg
' the graph of 2x
plane region c o ~ 3y s 6 is the y,l':, 2 ... w
g the origin. "1 .. . (ii)
... (iii)

5 6
l'illj 0
3 x 0ng
+ 4testi11g
>< · (0, 0) in 3.r + 4y s 24, we get
point
0 S 24 (true)
.- I

294 Asmtta'• NEB So1Ut10n al Basic Mathematics-II


Unit 11 215
From {ii) :r s 2
ConatraJnt. .r +
U 1~ I~ I.
e
~ 2ys7,.r,y~o
~corres ponding equation of bn.....:a_ . (2J
Taking testing point (0, 0) m :r 2 2. we get - 7 ---"'41)' lines are
0:? 2 (false) ;x+2y- ·- - (i)
From iii v =1 x'"O ...(ii)
X 0 ...(iii)
y=O
2y=7

ro[I: I: I
Taking testing point (0, 0) in y 2 1, we get
0 ;>: 1 (false)

10
D
raking testing point (0, O) in .r + 2y :s; 7, we have
o+ 2 x O:s; 7(true)
So, the plane region determined by .r + 2y s 7 is .
From (ii) x = 0 which is y-axis. the plane region C<>ntaining the origin

And. r 2 0 gives the right half plane includin -axis


From (iii)y = 0 which is x-axis. gy ·
..,.,,,~"" C----'-- y=1 And, y :? 0 gives the upper half plane including .I-axis.
x -+--+--+ ---f"l-+ --+--+-- +-+--+- -+--"1'- "+--+-- x
y

0
11 ·
3x + 4y = 24 .. 7
x =2

-fis.2071 Supp. Q.No. 16!


D e t ~ graphically the solution set of the inequalit y x - Sys

Given inequality is x - 5y s 5
The correspon ding equation of boundary line is
.I- 5y = 5

I; I~ I I~l
5. (21

Y'
Taking testing point (0, 0) in x - 5y s 5, we get
0 - 5 " 0 s 5 (true) ~Set D Q.No. 16i
~ the graph of the inequality: lr- 3 ~ Sx- Y·
(2)
The graph of x - Sy s 5 is the plane region containin g the origin.

Given • ·
lllequality IS
4
or,
or
.3;
3x 3
SS.t -y
Sx-3x-y
3
' ~ -H-3
..e~rrespo nding equation of boundary lines is lt -
JC -5y = 6
~

X ~ - - - - +c
l'a~· . 3 l
2 - ~~e5 1ing point (0, 0) in 2r - y 2 -3, we get
~ - 3 (true)

Y'

Jll_l
to c""...:d UH£AA PROGRAMMING
Asmlta'• NEB Solution of BHIC Malhem• ttu-ll 0 and y 2: 0 means we have - · "'• er the re . . Unit 16 297
296 ~i
y gion tn first quadrant only.

X
X'

So, the graph of given inequality is the plane region


containin g dle origin

-
2072 SIil E Q.No. 6!
IZ2. Delmnirl 1 · f•
o meq
uali ·
ties:
e the fe.uible region bounded by the following system
(2]
r+ ys6, 2r+ y 2:8, y 2:0.

Please see 2070 Set D Q.No. 16 a


x·--- --1-- -

-
• . .•
123. 2072 Supp Q.No. 1611! followin g mequaht ies: -4
Find the vertices of the fusible region determined by the
(2]
2x + y S8, x + 2yS10 =d x, y 2:0.
y=o
-4
Please see 2071 Set C Q .No. 16 a

-
y•
124. 2t173 Set C Q.No. 1611! (2]
ts 3x + 2y S 48, x + y S 20; x, y 2: 0. From the figure OABC is the feasible re · h th ·
find the vertices of the feasible region under the constrain
0), (8, 12) and (0, 20) respectively. gion w ere e coordinates of 0 , A. Band C are (0, 0), (16,

The corresponding equations of ~undary lines are: !_2073 Set D .Q.No. 16!

3x+2y+ 48 ... (i) Find the feasible region determin ed by the inequlities

x + y =20 ... (ii)


.!:.! S 8, X + 2y S 10, x, y ~ 0.
(2)
---
x= 0 ... (iii)
~ see 2071 Set C Q.No. 16 a
y =O ... (iv) ~Sup p Q.No. 16~
From (i), 3x + 2y = 48 lbe Procedu re of solving a linear programming problem
by the gJaphlc~ method. (2)
-

i. :e procedure of solving a linear programming problems by the


ii E l'll\ulate the given linear programm ing problem into the
graphical method are as foUows:
mathemat ical fom11f necessary .
ts (inequali ties) into the correspon ding equations.
ill. /Press the constrain
d oblained in (ii).
Taking testing point (0, 0) in 3x + 2y s 48, we get ~- Dlll l:he feasible region from the graphs of the equations
eterini th . .
3 X O+ 2 X OS 48 (IJ"ue) 1
. Bva( ne e vertices of feasible region.
YI. F!rt Uate the value of the objective function
at each of the vertices of the feasible region.
From (ii) x + y = 2D
~ 0
Ptitna1 solution

I: I: I: I
•,v?4le
o tAQ.No. 1 (2)
· 3y s 3·
~ ne graphica lly the solution set of the lnequabty x -
~llni

Taking testing point (0, 0) in x + y s 20, we get 'l'he cor


~ res onctin , ualion o.f boundary line is x - 3y • 3
0 + 0 s 20 (true) 0 3
From (iii) x ~ 0, which is the equation of y-a.xis. . (0 1) and (3, 0).
Sci, the ho~~ O
From (iv) y = 0, which is the eq uation of x-a.xis.
1
ary line passes thro ugh the poUlts ' -
(
298 Asmrt.'• NEB Solution of Basic Matflemettcs-11 LIHEAAPROQ
RAMMING Unit 16 299
Taking testing point (0, 0) in x - 3y S 3, we get ~ o r responcting equation s of giv ,
O - 3 >< 0 s 3 (true)
half plane region containing lhe origln as 8
~ + y "' 6 .. . (!~ en ineqUalities are
So, the solution set of given inequality is the closed how11 y"' -2 .. . (u)
below. :: O ... (iii)
., 0 ... (iv)
x-Jy=J
Y ~rolll j,x+ -6
0 6
3 0
2 fakiflg testing point (0, 0) in X + y S 6
0 + 0 ~ 6 (true) ' we get

-
jza. 2074 Set ea.No. 1si So, it contains origin.
Shade the feasible rqion under the constraints Frolll ii , x - = - 2
2x + y s40,x +2ysso , x.y .?0 121 0 -2
2 0
The rorresponding equations of boundar y lines are faking testing point (0, 0) in X _ y > _2
o-02:-2 - ,weget,
2x+y=f 0 ... (i)
X + 2y=50 .. . (ii) 02:-2 (true)
x =O ...(iii) So, it contains origin
y =O ... (iv) From (iii) x = 0 which is y _ axis
From (i) 2x + y = 40 From (iv) y = 0 which is x - axis

/; I: I: I
Taking testing point (0, 0) in 2x + y S 40, we get,
2 x O+ 0 s: 40 (True)
And, x 2: 0, y 2: 0 means we have to Consider the
v
. .
region II\ first quadrant only.

From (ii) x + 2y = 50

I; I~ I~ I
Taking testing point (0, 0) in x + 2y s: 50, we get
0 + 2 x Os: 50 (True)
From (iii) x = 0 which is Y-axis
And x 2: 0 gives the closed right half plane. x' --t-- HH~ -
From (iv) y = 0 which is X-axis
And y 2: 0 gives the closed upper half plane.
x+yal!Ji
y

Y'

1. 2075
Detenn· (21
- e graphic ally the solution set of 2x + y 2 z. x 2 0,
y 2 0.

X'
l'h ecorres o .
~-- P ndmg equation s of boundary lines are
<.11+ Y"' 2
~"0 ...(i)
y" 0 ... (ii~
Y'
_ _ _ _ _ ____.
.:..d:..._ _ _ _ _ _ _ _ _ _ _ _ _ _ _ _
·b=le=r=e=gio=n=is=s=h~a.:..de
i:Th~e==Fe:::::as=·1=
l21
129. 2074 Supp Q.No. 16.j 1
ities 3x + 4y s 24, Osy S 4, 0 S x ~ - 2
Find the feasible region determi ned by the inequal _
_ ____ __ lak· o
.N.:.:...:1.::6a:__ _ _ _ _ _ _ _ _ _ _ _ _ _ _
c.,.==='P:=':l'='ease===re=fe;;r::::,to~2fJ7;;;;0;Se;.:l:....:C:..Q~
lllg te -tj
h O-+- 6 ng point (0, O) in 2x + y 2 2, we get.
121 0~ O~ 2
!30. 20711 Set A Q.No. 16ai 2 ..
ttes x + y s 6, x - y .!: -2. x ~ 01 Y .!: O.
'11. (fctlse) . without containing the onglll.
Find the feasible region determi ned by the inequaJl 'ne SOI .
y 2 Is the closed plane region
From ("hon set of 2x + 2:
•I) x .. 0 Which is y - axis.
Asmlta'• NEB Solution of Basic Mathem■tlca-11 y LINEAR PROGRAMMING
300 Unit 18 301

and, x 2: 0 is the clo_s ed right half ~lane.


From (iii) y = o which 1s the x - axis.
and. y 2: 0 is the closed upper half plane.

x·_..1..--+--j---f--\ ""'f' X
y =O -2

-2
2x + y =2

-'l
Y' Y'
}ience·the common regi~n ~ quadrilateral AOBC, . .
132. 2075 set C Q.No. 16! further, in order to maxmuze and minimize the giv~ded on the graph.
Shade the (easible region bounded by x + Y S 6, x, Y .!! 0. 12)
pix, vl .. 9x + 7v;
are - Vertices F l:t, vl •9:r+ 7v
The (111115J)Onding equations of boundary lines
. Remub
A(0,3.5) F = 9 x O+ 7x 3.5=24.s
x+y=6 ... (i)
x=O ... (ii) toio,ol F-o
Minimum
y=O ... (iii) 014,o F=9x4+7x0=36

Froll1 x+y =6
ccs,1) F 9x5+7xl=52
Maximum

I: ~ I~ I
Taking testing point (0, 0) in x + y s 6, we get
Maxunum value 52 at E (5, 1)
Minimum value = 0 at O (0, O)
~ll Q;No'M:4b!
0 + 0 s 6 (true) . . . . .
So, the region determined by x + y s 6 is the closed half plane contauung the ongm. Find the extreme values of the function G(.r. y) defined by G( ) • lQr
Y polygon given by the inequalities. % + 2y ~ 20; % + y < l6; % > 0: y~ ~ + 15y over the convex
(4)
from (ii) X = 0 which is y-axis. .. - - • -
And, x 2: Ogives the closed right half plane.
From (iii) y = 0 which is x-axis.
And y 2: 0 gives the closed upper half plane. x=O Linear
Boundary line Points Testing point Result
ineaualities
(0, 10)
r+2y.520 x+2y=20 (0,0) 0~20 Cfrue)
(20, 0)
(0, 16) (0,0) Oi 16 Cfrue)
r+ y _516 X + y = 16 (16, Q)
..
The 11\eqUalities x ~ 0; y ~ 0 indicate that the C'OllllllOI\ region (1.e. solution set) lies on fust quadrant
-2 -1 Now we draw the graph.
X +y= 6
y=O The solution set is shaded on the graph which is q,uadrilateral OABC.
y

§16t·i·i:fi·i1i¥iit-U?i
?3. 2056 Q.No. 1#1
Mwmize and minimize the function F(x, y) • 9x + 7y. 14)
Subject to constraints x + 2y ~ 7; x - y ~ 4; x ~ 0; y~ 0 .

...=---------- ----- ---,- ---


in
Linear
··
Points T esting points
Result

X ~ Q ! x+Y "' 16
lhe d (0 l0) respectively.
4, 0 (0, 0 4
COordi~tes of O, A, B, C are (0, 0), (16, 0), (l2, ) l!Jl '
The inequalities x ~ 0, y 2: 0 indicate tha t the common region lies on the firs! qu ad rant.
302 Asmlta's NEB Solution of Basic Mathematics-II

Again,
Vertices G-l0x+ 15v
Remarks
Minimum
~
1
io61 ·
an: minimize the function F ,., 34.t
, r,ta~l~::nd 1 ~ x ~ 3
,♦ Y ~
- LINEAR PROGRAMMING

+ 6y subject to the constraint


Unit 16

sx+y~l,
303

G =10 x 0+15 x 0-0 (4)


0(0,0) ualities
A (16,0) G = 10 x 16 + 15 x 0 =160 _
Maximum
G = 10 X 12 + 15 X 4 = 120 + 60 -18~ oint
B (12, 4) x+ y ~1
C (0,10) G 10 x o + 15 x 10 =150 (0,0)
:. Maximum value of G = 180 at (12. 4) x+y:S6 x+y=6
"'2"::!;Minim~ ___
-~-~ u~m~v!al~u~e~otfG~-~0~a~tl(0~,~0)L_ _ _ _ _ _ _ _ _ _ _ _ _ _ _ _ _ _ _ (0,0)
x=1
ps. 2058 Q.No. Ub! subject to constraints y • x ~ l, (0,0
Maximize and minimize the function F 9,. + 40y _ x=3 02:1
(0,0) 0:53
y-x5~25x5~ ~
ml!IB Testing point Result
Linear ineg_ualities Boundarv line Points
(0, 1) .(0,0) 0~ 1 (False)
y-x>l y-x= 1 (-1,0)
(0,3) (0,0) 0~3 (True)
y-x:53 y -x=3 (-3,0)
x>2 x-2 - (0,0) 0 > 2 (False)
5 - (0,0) 0<5 (True)' !
x<"S X
y

I'

x=3

The common region is shaded on the graph, which is quadrilateral ABCD. The coordinates of A, B,
Cand Dare (1, 0), (3, 0) (3 3) and (1, 5) ·respectively.
Vertices F= 34x+ 6v Remarks
y-..r=3
A(l,0) fa:34x1+6x0=34 Minimum
B(3,0) F "' 34 x 3 + 6 x 0 = 102
Y' x=2 x=S r(3, .~) F = 34 x 3 + 6 x 3 = 120 Maximum
lQ.{1, 5) F 34x1+6x5 64
Hence the common region is -the parallelogram ABCD which is shaded on the graph.
Here, the coordinates of A, B, C, D are (2, 3), (2, 5), (5, 8) and (5, 6) respectively. :. ~e lllilXimum value of F = 120 at (3, 3)
lnod
r er to maxuruze . . . the fun c tion F = 9x + 40y, we ,have
. . and IIllIUIIllZe eminimum value of F "' 34 at (1, 0).
Vertices F-9x + 40y Remarks
A(2,3 ,
8 (2,5
P-9x2+40x3 - 138
F - 9 x 2 + 40 x 5 - 218
Mini[llum
~
--lllize: F '"SOx + 15y, subject to x + y ~ 60, Sx + Y ~ lOO, x ~ O, Y O
(4)

-
>
C(S,8 F-9x5+40 x 8-365 MaxiD)um
D(5,6J F-9 x 5+40 x6-285 •

Testing point Result


Hence the maximum value - 365 at (5, 8) and Linear Points
~ualitles Boundary line 0~60 (frue)
Minimum value= 138 at (2, 3). (0,0)
x+y~60 x +y"'60 (0,60)
136. 2059 a .No. 14ij (60,0)
(0,0)
o~ 100 (frue)
Maximize and minimize the function f(~ y) "'9x + 7y ~~100 Sx + y 100 (0, 100)
Subject to constraints ;r + 2y ~ 7, :r • y ~ f; ;c ~ O; y ~ 0 141 (20, 0)
---- ll;in ..
equalities x ~ o, y~ oindicate that the solutl
·on set lies in the first quadrant.

=,;,;;;~~=:=:=:=:.-"
'31. 2060 Q.No. 14ij ----------- -------
Please see 2056 Q.No. 14b
((
4
Maximize and minimize ,F = l0x + 15y subji;ct to ;r + 2y ~ 20, ,c + y ~ 16, x ~ o, y ~ o
• Please see 2057 Q .No. 14b
LINEAR PROGRAMMING
the upper half plane con•~1- ,_g . Unit 18 305
304 Aamlta'a NEB Solution of B11lc M1th1m1tlc1-II ogive8 -..wi .t-axlS

y i (iii) y .. 4 is the line parallel to .t•a.tis and ~


y
ffO~ testiJlg point (0,0) in y S 4, we get OUgh the point (0, 4)
1,~g
s4 (trlle) . .
0 ll1 (lV) X"' 0 which ls y-axis.
ffO . es the right half plane contalnJng th e Y·axls
0 giv 1hr ·
1i (v) x .. 7 is the line parallel to Y·axls and
pro01 y OUgh the point (7, 0)

SO)

x=O

Sx+y= 100

is the quadrila teral OABC.


Henre the solution set is shaded on the graph which
The co-ordin ates of vertices 0 , A, Band Care
(0, 0), (20, 0), (10, 50) and (0, 60) respecti vely.
x=7
Again, • Y'
Remark s e.
Vertices F = 50x+ 15y The required solution is shaded in the fi
0(0, 0) F = 50 X0 + 15 X0 = 0 Vertices Value ofF = 16x - + 40
A(.20, 0) F = SQ X2() + 15 XO= 1000 0(0,0) F = 16 x O- 2 x O+ 40 = 40
= 50 X10 + 15 X50 = 1250 Maximu m
B(lO, 50) F A(7,0) F = 16 x 7 - 2 x 0 + 40 = 152
q o, 60\ F =50x0 + 15 x60=90 0 3
Maximu m value of F = 1250 at the point E(l0,
50). s(7, ¾) F =16 X 7-2 "4+ 40= 150.5

i40•.2063 Q.No. 14bl 8


Graph the followin g system of inequalities
to find maximu m and minimu m of the objective
c(~, 4) F =16 x j'- 2 x 4 + 40 =74.67

-
function
F=16% -2y+40 F = 16 x 0-2 ><4+40=32
[4] .M D 0,4
3% + 4y ~ 2', 0 ~ y ~ 4, 0 ~ X ~ 7 t ·value.:.:..:.
·~,-..... of F = 152 at (7, 0)
.!:.'.... :!..__ _ _ _ _ _ _ _ _
_ _ _ _ _ _ _ __
~=32at(0,4)
The boundary equation s of given inequali ties
are

~•~= 24 ... w ;:ar th e followi ng systems of inequalities and find the vertices where they exit
t
{41
~~20 ,x+y ~ 16,x~0
y =O ... (ti)
y=4 ... (iii)
. (iv) •
'l'hecorres pondmg .
x=0 .t boW1dary lines are respccbvely
... (v) +2y"' 20
x=7
HY "'16
From (i) 3x + 4y = 24 hQ

I; I: I: I
Taking testing point (0,0) in 3x + 4y :, 24, we get
N Y"O
01v,

Bounda ry lines
Points
(0, IO); (20, 0)
Testing
oint
(0, 0)
Result
Qs 20 (true)
Os 16 (true)
3 x O+ 4 >< 0 ~4 (true) x+ 2y •20 (0, 0)
(0, 16); (16,0)
The graph of 3x + 4y s 24 contains ori gin. .r + y ~ 16
= 0 which is x-axis. x • 0 i.e. y-axis
From (ii) y
y "' 0 i.~. x-axis
~-\~2;;;,~.: ~;
306 Asmlt•'s NEB Solution of B■ -tc Mathematics -II
"
Unit 16 307
f\10, 1•
,
~ • a.doe8 a JI near fnequalJty differ from the 11 __ _
I h f
110W 11111 value o t e unction 41(r, y) • l6x- . . _ equation? 0 .
J!11~1111 2y + 40 Sllbject to L tttnrune the Dlaxinnun and
~ "-"+4ysu, 0
~ . part: An equation of the form ax+ by ,, c s x s 7, O s y s 4.(4)
flf8 1 is called a linear equation If '•• .
·
·• .(1) Where a b
1s replaced by ' , c are cons tants and
_rlableS th x and y are
~a 11 alily- < or s or:> or :> . (.)
iJ1e<l f .... Please see 2063 Q.No. 14b - m 1 ' en it is called linear
J"eJCI a,.,
,4'a.No, 14 -----
j!!d the extreme values of the obje(tive fu1l(tj --------._
FIJI .,. 2x + y ~ 20,
A .. r .!: 3, y.? 4 on lO:r+ lSy su'-'--io
~~
c--1ra·
vu" mis:
,+ 2y:,,&1,
x =O
~,respond ing equations of boundary Jin
l4l
The -2.5 ... (i) esare:
X ,+2y =20 ... (ii)
y=O 2t +3y - . (iii)
0 (0 0)
X + 2y = 20 ,~ .. . (iv)
y~4
x+ y = 16 frolll i X + 2 = 25
The a,mmon region is shaded on the gra ph w hich is the quadrilateral
AOBC having the 0 25
coordinates with the vertices A (0, 10), 0 (0, 0) . B(16, 0) and C (1 2, 4). 125 0

-
ju. 2111 Q. No. 14 ij Taking testing point (0, 0) in x + 2y s 25,
(41 Taking testing point (0, 0) in 2x + y s 20, we have
!Jetel:mine the extreme value of the functions F(x, y) = x + y + 100 subject to the constraints: we have 2 X O+ 0 !> 0 (true)
y - r .!: 1; y - r S 4 and 1 $ x S 6 0+2 x o s 2S(true)
from (iii) x = 3 From (iv) y = 4
The corresponding equations of boWldary lines are
}" - X =J ... (i) [; I ~ I ~ l; I~ I~ I
)' - X =4 ... (ii) Taking testing point (0, 0) in x ~ 3, we have Taking testing point (0, 0) in y ~ -1, we have
X =J .. . (iii) v-x=1 O~ (false) 0 ~ 4 (false)
y
x=6 . . . (iv)
From (i) 0
y - x =l !:c 7

I~ l~ I~
1

Taking testing point (0, 0) in y - 1 ~ 1


0 - 0 ~ 1 (false)
From (i)
y- x=4

Talcing testing point (0, 0) Ul y - x :S 4, A( ,0 - 7 5 - 5 -~255 2.5 {>r+2y= 25)


we get
-5
0 - 0 5 4 (true)
-7.5
From (iii) x = 1 which ib the line
parallel to y-axis and thro ugh the point x=
(L~ y
From (i v) x = 6 which ili the line parallel to y-axis and th ro ugh the point (6, 0) , titu1tes of A, B, C anJ Dare A(3, .J), B(S,
0- 0~ 1 (false) Fron, th . Y'

Value of F = 10x + 15v


4, c5 e hgu re A BC D is the feasible region where tbe ~oon
Vertices ' l O a nct D 3, 11
A(l ,2) P = 1 + 2 + 100 "' 103 Vilue of F = 10x + 15
8 (6,7) F = 6+ 7 + 100 = 11 3 V ertices F"' 10 X 3 + 15 X -l = 90
q6,10) f .. 6+ 10 +l 00 .. 116 A (3, -l) F"' 10 X 8 + 15 X 4" 140
D/1,5) F ., 1 + 5 + 100 "' 106 B(8, 4) F • 10 X 5 t 15 X 10 = 200
Max. va lue of F = 11 6 a l (3, 7) C(S,10) F"' 10 X 3 + 15 X 11 " 195
Min. val ue of F =103 a t (1, 2) ~a~. Va t D 3, l 1
~In. value .. 200 a t (5, 10)
ue"' 90 (3, 4)
a t
LINEAR PROGRAMMING Unit 16 309
Asmltl'• NEB Solution of Saale Mathematica-II
308 ~•=i~#i•I•
-;::~=--==-a.~No:;:=.=:;1';";4]::r----- - - - - - - - - - - - - - - - -
Find the maximum and the minimum values of the obf ective fundlon.
F • 16l'-2y + 40subje<ted to 1r + 4yS 2'- Osy S'- 0 sxs7. · (4)
d, maximize p • x + 3
.. Please see 2063 Q .No. 14b t11III y subjttt to CONtnints
~ ' x+ys4,x-ys1 ~o
riNi·i):fi·111iili·i~~i ~ and 5 be the non-negative slack Variabl 'x , Y:a: 0. {4]

~- 2074 Set AQ.JiiO,.!,I


~,!,1iJl+ the
i,r •
following form:
r ,. 4
es. Adding the slack variables
, we can express the given
A small industry manufactureS neclclaces and bracelets. The combined number of necklace 1
bracelets thllt it can handle per day is not more than 24. Each bracelet takes 1 hour of labournd ""'yY+s=l
make and each nedcLll'e takes a half hour. The total number of hours of labour avallabl d to1 71,- ,
not exceed 16. If the profit on the necklace is Rs. 80 and the profit on _the bracelet is Ra. 50e H °' ; ... 3y .. p
.., y + r + O.s + 0 .p = 4
many of each product should be produced daily to maximize profit? · W ::> : _ y + o.r + s + o.p = 1
iml8!il -x _ 3y + o.r + O.s + p = 0
Let no of neddaces = x SiJnplex Tableau
& n o. of t,racelets=Y X r RH5
If p be the total profit lo be maximized, then given I.PP can be written as 1 0 4
0
MaxiJDiZ.e P = 80x + 50y subject to the constraints 1-1 0 O 1
1
X +y S24 -3 t
-1 O 1 o
O
x+½ys16 i.e. 2x +ys32 Here -3 1 negative entry. So, column second IS
4 is the most .
. the p1Yot column.
x2 0, y2 0. Now, 1 =4 and :i = -1 (We shouldn't take negative ratio)· So, 1 IS. the pivot
. element
'The corresponding equations of boundary lines are
x+y = 24 ... (i} Applying Ri ➔ Ri + R1 and R1 ➔ R, + 3R1
2x + y = 32 ... (ii}
r RH5
x=0 ... (iii) 1 0 0 4
y=0 .. . (iv) 2 0 1 0 5
From i , x+ = 24 2 0 3 0 1 12
X O 24 Since~ the entri~s ~ the last row are non-negative, the optimal solution is obtained.
24 0 The optimal solution 1s
Taking testing point (0, 0) in x + y S 24, we get O + 0 S 24 Max. p = 12 at x = 0 and y = 4
(true)
6 MARKS QUESTIONS
So, the solution set of x + y s 24 is the closed plane region
containing the origin.
A ain, from ii , 2x + y = 32
118.'-'20691Set A) Q.No.1ij
0 16 Using simplex method, Ma.rimize Z .. 7:rt + 5.r.z ·
(6)
~ c t to Xt + 2x2 !'. 6; 4x1 + 3x2 S 12; :rt, %2 :i!: 0
Taking testing point (0, 0) in 2x + y s 32, we get 2 x O + O :5:
32 (true) Let ~ and X◄ be non-negative slack variables. Adding the slack variables, we can write the given
So the solution ~t of 2x + y s 32 is the closed plane region LPP m the following form.
contauung the ongm. + 2x2 + XJ = 6
l'I
From (iii) x = 0 w hich is y -axis. 4x, + 3x2 + X4 = 12
And x 2 0 gives the closed right half plane. z., 7x, + Sx2
From (iv) y = 0, which is x-axis. ::) Xi + 2x2 + X3 + 0 . X4 + 0 . Z ,. 6
And y 2 0 gives the closed upper half plane.
~l + 3x 2 + Q • X3 + X4 + Q • Z = 12
From figure OABC is the feasible region where the coordinates of o A B& c are (0 0), (16, O),
- Xi - Sx2 + 0 . X3 + 0 . x4 + Z .. 0
8, 16 & 0, 24 res ectivel . ' ' ' Simplex tableau
Ver tices Value of P -= 80 x + XJ
z RHS (b)
Basic Variables X1 Xl 6
0(0,0) = 80 x O + 50 x O= 0 2 12
A(16,0) = 80 x 16 + 50 x O= 1280 4 3 0
B(8,16) • 80 " 8 + 50" 16 • 1440
-7 -5
C 0,24 = 80 " 0 + 50 " 24 • l 200 t 6
The maxim~ val ue of pis Rs. 1440 at (8, 16) Ii . fi t colUJJII\ is the pivot column. Since 1 = 6,
e req6 wred no. of n ecklaces & bracelets that shoul d be p rod uced daily to maximize the profit
So, 8th& ere, - 7 is the mos t ne ga tive entry in the last row. So, rs
are 1 respecbvely . 12
4 "'3 and 3 < 6, so 4 is the pivot elemen t.
LINEAR PROGRAMMING Unit 18 311
310 Asmlta's NEB Solution of BHlc Mathamallc1-II X2
XI
3
1 2 RHS
Applying Rt ➔ i Rt 2
)1 X2 .l) X4 z RHS(b\ 3
-7 13
Basic Variables 0 6 -5 12
- X_; 1 2
3/ 4
1
0
0
1/ 4 0 3
t
X1 1 _7 is the most negative entry in the last
0 0 1 0 .
-7 -5 11ere, 12 row. So, 5eeond I
< 6, so 3 is the r• I
nivot eenient co umn IS the pivot colu-- s·
AppIyi·ngRI ➔ R1- RtandR, ➔ R,+7& x., z RHS(b)
13 4 3 -2
;,," .. ' I
== 6 and 4 . 3 ~uL IIICe

Basic Variables .,·, X2


1
X4
-1/4 0 3 - 3 1
~pplylng Rt ➔ 3 Rt
0 5/ 4
.\'.3 1/ 4 0 3
1 3/ 4 0
x, 7/4 1 21 X2
0 1/ 4 0 Xl .XJ
l'.j
the solution is optunal. 1 1/3 RHs
Since all the entries in the last row are non-negative, so 2/3 0 0
3 2 0 13/3
Max. value of z = 21 w hen x, = 3 and x2= 0. 0
-5 -7 0 12
~9. 2069 (Set Bl Q.No. 1j i\ppJying Rz ➔ Rz - 2Rt and RJ ➔ RJ + 7Rt 0
Using simplex method, maximize Z = 7Xt + Sx2
16)
Subject to: .rt + 2xl :S 6, 4Xt + 3xz :S 6, xi, x:i ~ 0 X2 %3 l'.j
Xl
~ 2/3 1 1/3 O RHS
Let .r~and l'.4 be the no!Hlegative slack variables. Then the given LPP can be written as 5 0 -2/3 O 13/ 3
1
x, + 2.Q + .r~= 6 -1/3 0 7/3 10/ 3
4x, +3.r2+.r., = 6 91 / 3
't
7x, + Sx2= Z .
⇒ .r, + 2rz+ .r; + 0 . x., + 0 Z=6 'J'hiS is not .the optimal solution as the last row contains negative entry. Again, first column is the
. ,!U2 .!Qil
4.r, +3.r2+ 0. X3 + .r. + 0 .Z = 6 .
- 7.r, -5.r2 + 0 .r; + 0 x., + z = 0 P
ivotcolumn. Smce, 2/3 =6.5, 5/3 =2and2<65 3 IS the pivot element
· , sot
Simplex tableau
%1 l'.2 %3 %4
0
z
0
RHS
6
Applying Rz ➔ R2 s3
1 2 1 X2
1 0 6 .Xt %J 1'.j RHS
4 3 0
0 0 2/3 1 1/3 0 13/ 3
-7 -5 0
1 0 -2/ 5 3/ 5 2
i
Here, - 7 is the m ost n egative entry. So first column is the pivot column. Since t = i=
6, 1.5 and
-1/3 0 7/3 0 91/ 3

Applying Rt ➔ R1 - ~ R2 and RJ ➔ RJ +½Rz


15 < 6, so 4 is the pivot elemenL
RHS
. 1
Applying Rt ➔ i R2 1 3/5 -2/5
%1 %2 %3 z RHS 0 -2 5 3/ 5 2
1 2 0 6 11/5 1/5 1 31
0 0
1 4 3/ 2
1 3/ 4 0 Since all the entries in the last row are non-negative, so the optimal solution is obtained.
-7 -5 0 0
Max. z = 31 at x, = 2, x2 = 3.
Applying R1➔ R, - R2 and R3 ➔ R3 + 7&
x1 x2 XJ Xt · Z RHS [2070 Set D Q.No. 18'
0 5/4 0 -1/4 O 9/2 Using Simplex method,
3 4 0 14 O 3 2 Max. P = 50 Xt + 80.x2 (6)
0 . _ 1/4 0 7/4 1 21/2 ~ectto.x, + 2x2~32; 3xt +4.r2~IH;x,,.n:l!:O.
.
Smee all the entnes lJl the last row are non-negative, so the optimal solution is obtained .
21 3
Ma.r Z = 2 at x, = 2 aruix2 = 0. Let .t3 and X4 be the non-negative slack variables. Then given LPP can be written as
:1 + 2.t2+ XJ = 32
t

-
!50. 2070 Set C Q.No. 1~ (6) + 4x2 + .\'4 = 84
Using Simplex method, Max. z c 5.r1 + 7.r2 x, + 80.r2= p
Subject to 2x1 +3.r2:S 13; 3x1 + 2x2S 12; .r1,x2 ~ O. ~ x, + 2:
Jx Xi + XJ + 0 , X4 + 0 . p = 32
-~ + 4xi + 0 ,.\'J + X4 + 0. p = 84
Let x1 and .r., be the non-negative slack variables. Th.en the given LPP can be written as Xi - 80x2 + O. x, + o..n + p = O
Simplex tabJe;au p RHS
2x1 + 3X2 + Xl = 13
3x1 + 2x2 + .r., = 12
X1 x X, -~,r'~ --~~- t-~1:322-
- 1~ - - ~ r n2 ~ - - - ~1- - O M
5x, + 7x2 = z
⇒ 2x1 + 3.r2 + XJ + 0 . X4 + 0 . Z = 13 3 4 0
3X1 + 2x2 + 0. I l + X4 + 0 . Z = ] 2 -so -80
- Sx1 -7x2 + 0 . x1 + 0 . .r., + z = 0 't
,' .. ~ ~·-·..... ,, -~ ... ·•·.:-. ~
,
312 Aamlta'a NEB Solution of Baile M1th1m1tlc1-II 'f),l8 ls not the optimal solution as I LINEAR P R ~
• 4 8 ast row contains th MING Unit 16
Here, -80 is the most negativt! entry in the lllst row, 80 second row Is the pivot column. Since pivot column. SlllCe215 • l0, 8/5 •
5 and 5 < e negative entry. A . . 313
5 10, so 8/5 is th . gam first column is th
32/ 2 16, 84/ 4 = 21 and 16 < 21, so 2 is the pivot clement.
E

Applying R, ➔ ½R,
,4pplylng R2 ➔ 8 R2 e p!Vot ernenl e

" ( ...
:l't
"''2 X) x, p RHS ~s 1 ~ l
1/ 2 0 0 16 1 0
1 1/ 2
3 1 0 84 -8 0
4 0
-50 -80 0
Applying R, ➔ R, - 4Rt and R., ➔ R, + SOR,
0 0

RHS
= 2
,4pplying R1 ➔ Rt - 5 Rz and RJ ➔ RJ + SR2
x,
1/ 2
Xl
1
X!
1/2
X,

0 0 16 "0 1
7/'2!5
1 0 -2 1 0 20 1 0
1280 0 0 -1 8
-10 0 40 0
i sinCe the last row contains all non-ne a~
This is not the optimal solution as the last row containS negative entry. Again, first column is the ax. = 160 at x = 5 and y =2. g ve en , so oP . . 160
lion IS Obtained
pivot point oolumn. Since f J
= 20, 11 2 = 32 and 20 < 32, so 1 is the pivot element.

..
U1lng simplex method
Applying R, ➔ R, -½ R, and R., ➔ R, + tOR, Maximize/"' 15x, + 10x2
1ubject to 2xt + x2 SlO [6]
xt Xl x:i X• RHS Xt + 3x2SlO,
0 1 3/2 -1/2 0 6
Xt,%2~
1 0 -2 1 0 20
0 0 20 10 1 1480
Sinoe all the entries in the last row are non-negative, so the optimal solution is obtained. Let X3 and X• be the non-negative slack Varia .
Max. P =1480 at x, = 20, x2 = 6. LPP in the following form. bles. Adding the slad variables we can .
21'1 + x2 + x1 = 10 • wnte the given
152. 2070 Supp, Q.No. 1aj

-
Using the simplex method, maximize r, + 3x2 +LI= 10
p • 20 r + 30 y subject to constraints lSx1 + 10x2 = f
2r + Sy s 20, 2r + y s 12. x ~ o. y ~ 0 ⇒ 21'1 + X2 + XJ + 0 . X4 + 0 . f = 10
(6)
X1 + 3X2 + 0 . XJ + x 4 + 0 . f = 10

Let r and s be the non-negative slack variables. Then the given LPP can be written as
-15x, -10x2 + O. x 1 + o. X• + f = 0
2x + Sy + r = 20 . Simplex tableau
2r+ y+s=l2 x, X2 XJ X.
20r+30y=p 2 1 RHS
⇒ 2x+5y+ r+O . s +O . p =20 1 0 0
' 1 .3 10,
2x+y+ O. r+s+O .p =12 0 1 0
-15 lQ
-20r - 30y + 0 . r + 0 . s + p = 0 -10 0
Simplex tableau
t 0
X
2
2
w r
1
0
s
0
1
p
0
0
RHS
20
12
Here, -15 is the .
5 <10. So 2 is the pivot element
10
most negative entry, so first column is the pivot column. Since - = 5 !Q= 10 d
2 • 1 r an
- 20 -30 0 0
i Applying Rt ➔ ½R,
Here, ~3(J is ~~ most negative entry. So, second colwnn is the pivot column.
Xt X2 X)
~s
Since 5 = 4, 1 = U and 4 < 12, so 5 is the pivot element.
! 1/2 1/2
Applying R, ➔ ¼R, -15 -----:3=-:--_ _ _..:,0::,.__ _ _.!,__ _ _~--+--~1~0_ _
Appl in -1.0 0 0
IX y r 8 p RHS y g R2 ➔ R2 - R1 and RJ ➔ R.J + lSR,
2/5 1 1/5 0 ~ ~ ~ ~s
0 4 x:i
0-----=---+--=5=---
2 1 0 1 0 12 1 ~-----'1=2'----~1/~2_ _ _......;=
- 20 30 0 0
Applying R2 ➔ R2 - R, and R, ➔ R1 + 30R,
1 0 o 5 2 -1/2 1 5
O -5/2 15/ 2 0 75
X r 8
] RHS
2/5 1 5 0 This is n 0 1 . t .
0 4
0 -1 5 1 the Optimal solution as last row contains negative entry.
0 8
-8 0 6 0 5
t 1 120 Again, second column is the pivot rolumn. Since 2 .. 10, 5~2 = 2 and 2 < 10, so 5/ 2 is the pivot
element. 11
314 Asmtta'a NEB Solution or Buie M•th•m1tlc1-II 1 LINEAR PROGRAAfMING
J\PPlylng Rt ➔ R1 - 3 Rz and R:i ➔ R:i + l0R2 Unit 16 315
2
Applying Ill ➔ 5 Ill X
RHS 0
Xt
.\'I .\'2 0 5
0 1 O -1 -1/J O RHS
1 1/ 2 1/2 0 2 6
2 5 0 0 I 1
0 1 -1 5
0
75 , ce all the entries in the last row are 10 o 3
0 -S/ 2 15/ 2 1. .
t :~ u ,, 345 at x = 3 and y .. 6. non negative. So the oP"--' 345
"""" solution IS oh+-.,__.
. ro~~=====7 .':~';;j1 -.!..-..::::__
-½ R2 and R, ➔ R, + 5/2 R2 ~ P P · Q.No. 18f ~ -iea.
Again, app lying R, ➔ R,
RHS
6
• ~sl:::he simp_lex method. DWamiu

0
~ 1
~ -1 / 5
,is2/5
-VsO
O
4
2
80
I i
• 4l' + Sy subJect to
+ Sy S 25, 6r + Sy S 45, r ~ 0, y ~ o.
1
~and sfollowing
0 0 7 1 . ..
Since all the entries in the last row are non-negative, so o ptimal solution is obtained. be the non negative slack v . bl 161
LPP in the form: aria es. Adding the slack Variables we .
Max. f - 80 at :n - 4, x2 - 2.
2l' + Sy + r =25 can wnte the given
154. 2071 Set D Q.No. 1!
Using simplex method, 6x+Sy+ s=45
4x+Sy =p
MmmmU= 25r+45 y
=> 2X + Sy + r + 0. s + 0. p = 25

-
subject to r + 3y S 21
2r+ 3yS24
6x +Sy+0 . r+s+O . p=45
:r, y ~O [6) -4x - Sy + 0 . r + 0 . s + p = 0
Simplex ~blua
the given X r
Let I and s be the non-negative slack variables. Adding the slack variables we can write
l.PP in the following standard form:
2 rn RHS
25
6 5
r+3y+ r= 21 0 45
-4 -5 0
2r + 3y+ s = 24
25r+ 45y = U t
Here, -5 is the most negative entry in the last So .
⇒ r + 3y + r + 0 . s + 0 . U = 21 . row. second column IS the pivot column. Since
25 45
2r + 3y + 0 . r + s + 0 . U = 24
-25r - 45y + 0 . r + 0 . s + U = 0
5 = 5, 5 = 9 and 5 < 9, so 5 is the pivot element
Simplex table
r u RHS Applying R1 ➔ ¾R1
1 1 0 0 21 X y RHS
2 3 0 1 0 24 2/5 [I] 1/5
- 25 -45 0 0 0 6 5 0 45
t -4 -5 0
column. Siner
Here, -45 is the m ost negative entry in the last row. So, second column is the pivot Applying Rz ➔ Ri- SR1 and R, ➔ R, + SR1
21 24 X y RHS
= 7, = 8 and 7 < 8 so, marked 3 of first row is the pivo t element.
3 3 ~ 2/5 1 1/5
Applying Rt ➔ 3 Rt
1
- rn-2 o ~ ~
0 1 25
r y u RHS
A · . 5 25 ~ . _ £ . .
1/3 1/3 0 0 7 gam, first co lumn is the pivot column. Since 215 =2 ,4 =5 and:, 2
< , so 4 IS the pivot element.
2 0 1 0 24
- 25 -45 Applying Rz ➔ ! Rz
4 RHS
Applying R2➔ R2- 3R1and RJ ➔ RJ + 45R1 y r
U RITT x
r 0
0 O 7 2/15 1 1/5

==
1/3 1 1/3 1/ -1
O _ O -1 / 4 25
0 -1
-2 O 1
-10 0 15 0 t 31 5
t th
first column ts ''
Applying R1 ➔ R, -¾ Rz and RJ ➔ Rl + 2Rl RHS
This is n ot the o p timal solutio n as the last row con tllins negative en try . Again, r s 3
7 3 -1/10
3/10 5
3 "'11, 1 ~3 and 3 < 21, so1
p ivot column. Since 1 ; is the pivot elem ent. 0
0 -1 / 4 1/ 4 1 35
0 7/ .5 0th lution is optimal.
S1.1\ce all 0 gatlve so e so
M ' the en tries i.n the las t row are non-ne '
ll)( p "'3S a t X = 5, y = 3

., \.
)( + 2y + s = 50 LINEAR PROGRAMMING
316 Aamlta's NEB Solution of BHlc Mathamatlca-11
-1-3Y
! ~t - 5)( -1- r + O·S -1- 0.z = 40
~6. 2072 Set C Q.No. 1 maximize z • 7" + Sy &ubJect to
Solve by Simplex method, the LP problem to ? 'J,'/,+Y -1-0•r-1-s+0·z=·SO
0
.r+ 2Y s6, 4.r+ 3y s 12, x, y ~ - [6] ~+'l.Y3y-l- O·t + 0·S + z = 0

---
BIR ,s~ - bJeau
x
18 _ _ _ _ _ _ _ _ _ _ _ __ Je" ta
~P~I;;ea;:se~see~206~9~(Se~t;A~)~Q~.N~0:::_·_:::__ _ _ _ _ 51111P Y

-
-=1<7 2072 Set D Q.No. 1! 2 R.HS
... find the optimal solution of z • 7x,. + Sx1 subject
to

. Sunplex
c: • Using meth Ou, 40
-, ~6,4x,. +lx:,~12., .l\,1'.2~ 0• [6]
XI +2.>. -5 -3 50

18
t
Please see 2069 (Set A) Q.No.
_5 is the most negative entry, first col .
p. 2072 Set E Q.No. 1j 50
tunn IS the pivot col . 40 SO

-
Using Simplex melhod,
f1ere, wnn. Smee 2 = 20,
1 = 50 ar.d
50 50 2 is the pivot elemenl
Maximize F • 5.r - 3y, subject to 20" ' l
(6)
lr+ 2ys6,-x- + 3Y ~-4,x, y~O Applying R, ➔ 2 R,
X
Given mequalities are 1/2 1 R.HS
3r+2y:S 6 1 2 20
0/ 2
-x+ 3y24 ~.r- 3y :S 4 -5 -3 50
0
. the given
. bl es, we can wnte
2
.r, y r and
Let
· Add.ing the sI ack vana
· b!es.
0 5 be the non-nega tive slack vana Applying R2 ➔ R2 - R,, RJ ➔ RJ + SR,
· X
LPP in the following form R.H5
1 1/2 1/2
3x+2y+ r =6 20
and 0 3/2 -1 / 2
x-3y+ s=4, 30
0 -1/2 5/2 100
5.r-3y = F
~ 3.r + 2y + r + 0 . s + 0. F = 6
t
.r - 3y + O. r + s + 0 . F = 4 The solution is no t optimal as the third row contains negative
entry.
-5.r + 3y + 0 . r + 0 . s + F = 0 1 20 30
Simplex tableau . d 1umn 1s .
. th pivot = 4, = 20 and 20<40, so 3/ 2 is the pivot
Again, secon co e co umn. Since 112 312
s F RHS
.r y
1 0 0 element
2
1 0 2
-3 0 Applying R2 ➔ 3 R2
-5 3 0 0
X R.H.S
t 0 20
So, first column is the pivot column Since 1 1/2 1/2 0
Here, -5 is the negative entry in the last row. 2/ 3 0 20
0 1 -1 / 3
6 4 · eIemen t. 100
3 = 2, 1 = 4 and 2 < 4, so 3 IS· th e pivot 0 -1/2 5/2 0
1 . R1 ➔ R, - 1 Ri, RJ ➔ R, + 1 Ri
A lymg
PP 2 2
Applying R, ➔
3 R, RHS R.H.S
y F X
.r 0 10
0 0 2 O 2/3 -1/ 3
2/3 1/3 20
0 4 0 1 -1 / 3 2/ 3 0
-3 0
o o 7/2 1/ 3 I no
-5 0 0 · al solution is
. so the optim obtai.nro. Max. z = 110 at
s·mce all entries in the last row are non-negative,
Applying R2 ➔ R2 - R, and RJ ➔ RJ + SR, RHS
r F ~ l0and y 20.
.r
1/ 3 0 0 [6)
1 2/3
~~tCQ .No.1 ij . +:oost2;xi,.'(l~O by5implexmethod.
1 0
0 -11 / 3 -1 / 3
10 , p axinuze z = Sx, + 7x2 subject to~,+ 3X2 ~3. 3xi
0 19/3 5/3 0
so the solution is optimal.
~:=ee 2070 Se t C Q .No. 18
Since all the entries in the last row are ,non-neg ative, --- ~ DQ.No.1ij 161
Max.. value of F = 10 al x "' 2, y = 0. ..__ _ _ _ _ _ _ _ _ _ _ _ __ __ Ssing the simplex method. Maximum z. • tSx, + tOxJ
;::!:;::9.~2;;;07~2~S~u;pp~Q. o.=i1~~.::...::__.:::.L..__::
~ N=
, p~biect to 2x, + "1 ~ 10, x, + 3Xl ~ 10, x1.X2 ~ O.
Using simplex method, maximize z • Sx + 3y ~ see 2071 Se t C Q .No. 18 - ----~
+ stO,x,+3,u ~O,x1,X2~ 0.(6)
subject to 2x + y S 40, x + 2y S 50, x, y ~ 0 ·2073su QN 1
' o. + lOxJ subject to 2xi X2
.\p 1
IBID& ' Pi p Y Simplex method to maximb. e z • lSxi

Let rand s be U1e non-nega tive slack va riables. Then given


LPP can be written as ease refer 2071 Set C Q .No. 18
2x+y +r = 40
~-======;;---------------- _ LINEAR PROGRAMMING Unit 16 319

----
318 Asmlta's NEB Solution of Basic Mathematics-II
'S Jex me od to maximize z .. Sx + 3y sub·
upp. Q. No. 18 Jed to 2x + y s 40 + 2y
~3. 2074 Set A Q.No. 18 01!) .x SSO;x,y~O. [6)
(6)
Use the simplex method to maximize p • x + y subject to constraints .
x+ 2y S6, 3x + 2y S12. x::!:0, y::!:O. od, maximize p .. SOx1 + 80x2
t D Q.N. 18 subject to x1 +2x2s 32, 3x1 + ~
mil!IB SM,xJ,Xz~O. [6)
Let r and s be the non-negative slack variables. Then the given LPP can be written as
x+2y+r=6
maximize U = 2Sx + 4Sy subject to x +
---- ---
3x+ 2y + s = 12 t D Q.No. 18 3ys21,2%+ 3ys24,x, y~o.
x+y=P [6)
⇒ x+2y+r+O.s +O.P=6
3x + 2y + O.r + s + o.P = 12 od, minimize W = 3x + 2y b'
-x - y + O.r + O.s + P = 0 vsiJIS • SU Jettto2x+y~ 4,x+2y~4,x,y ~O. (6)
~ Augmented matrix formed from the constraints
Simplex Tableau

ITI
X
1 2
2 0
p
0
RHS
0
0
6
12
0
•-U In
~ ... .
~.~~fuo c...
.

-1 -1 0 0
t
There are two equal negative entries in the last row. So, we may choose any one of 1 st
as pivot rolumn. Let us choose first rolumn as a pivot rolumn
6 U
or 2nd column
N=U ~I~)
Now, the correspondin g dual problem of the given LPP is
Since =6& 3 =4and4<6.
1 Max. z =4y1 + 4y2
So 3 is the pivot element subject to 2y1 + y2 ~ 3

Applying~ i Rz p RHS
y1 + 2y2~ 2
y1, y2~0
s
X y
0 6 1n1roducing the slack variables x,, x2 ~ 0, the problem can be restated as follows:
1 2 0
0 4 2y1+ y2 + x, + O.x2 + 0.z = 3
0 1
1 3 y, + 2y2+ O.x1 + x2 + O.z = 2
3 - 4y1 - 4y2 + O.x1 + O.x2 + z = 0
-1 -1 0 0 0
Initial Tableau
Applying Rz ➔ R1 - Rz &t R3 ➔ R3 + Rz Basic variables Vl Y2 X1 X2 z RHS
X
RHS
3
~
X1 1 1 0 0
0 2 2 0 1 0 2
0 X2 1
0 0 1 0
-4 -4
0 - 0 4 t
. 2 .
Smee 3/2 = 1.5, 1 = 2 and 1.5 < 2. So 2 is the pivot element
0 0 ! 4
3
Applying R1 ➔ ½R1
This is not the optimal solution as the last row contains negative entry. RHS
=? =1.5, 2; 3 = 6 and 1.5 < 6, so 1is the pivot
X2
Basic variables x, 3
Now, second column is the pivot column..5ince / 3 ! 1 2
element 2 2 0 2
3 0 0

Applying R1 4 R1 -4 -4
Applying R2 ➔ Ri - R1 and R, ➔ R1 + 4R,
X r p RHS RHS
x, Xl
0 1 3/4 -1/4 0 3/2 Basic variables ~
2 3 0 1 3 0 4 1 ! 2
-1/3 0 1/3 4 2 2 1
0
2 1 0 2
Applying Rz ➔ R2 - 3 R1 and R3 ➔ Rl + 3 R1 0 6
x r s p RHS
- 1/4 Q 3/2 0 -2
O 1 3/4
0 -1 2 1 2 O 3 t
O O 1/4 1/4 O 9/i ~~e-Ul 1 12 1 1 3 . .
1/2 aa 3,m =3 and3<3,2 is pivot.
Since all the entries in last row a re non-nega tive, so the op timal sol1,1tio
. . n 1s _ ne d ,
, o btai
9 3 A.pp) . 2
Max. P =2 when x = 3 and y =2 Ying R1 ➔ 3Ri

,L
320 Asmlta's NEB Solution of BHIC Mathomatlcs
-11
Basic variable s \'l )( t )( 2 1, - RHS
\' l
1 3
l 0
y,
2 2 2
'I 2 1
0
y, 0 -3 3 '.\
0 -2 2 0
COMPUTATIONAL METHOD
R, - l2 R,, R, ➔ R, + 2Ri

~~m
Applvin g R, ➔
.
Basic variable
,, x, X2 RHS
2 1 4
-3 0
3 3
1 2 l -:-- --- ---form.
--- --- --- --- ---
_ __:Y~"- + --- -;~ - -34 0 3
4
3
20
eral 3733s into decimal
0 0 3 3 . 3 \'21
This is the op timal solution as all the en tn·es in last row are non-negative.
l
4
2()
t,.tax.Z =3 at y1 ---&
3 ,V2 =-
3 ,=83 X 3 + 82 X 7 + 81 X 3 + So X 3 3
20 i
. _i = 1536 + 448 + 24 + 3
i.e. min W =3 at x, = 3 , X2 - 3 = 201110
.;
. al numeral 2E4B into decimal fOIDI.
[21
Convertt---e-------- r-~ -7- -2-, -=c =J =::Jo~ :J
1

- .
omt I. ; ~ 4 B
- 16~ X 2 + 162 X E + 161 )( 4 + 160 )( B
2E4B16 = 8192 + 3584 + 64 + 11
= 1185110
6 2070 Set C Q.No. 16 ij h decimal form.
. Convert the decima l numera l 1503 to exa [2]
B!ID
Remain der
16
16
E503
93 F
16 3 D
0 5
150310 = 5DF1b

[jo10 Set D Q.No. 16 ij


Convert the decimal number 3058 to hexadecimal form. [21

Ima
Re m ainder
16 ~058
16 191 2
16 11
F
0 B
~ 1 o "'BF210
~ =p=p~
.Q
~.=N=o.=1=s~ij, - - - be( [2\
~ e r t 110011 2 to the decimal num '

Position 5 4l
Bina 1oi.nt 1 , , x 1 + 2°" 1
1l (X)111 =, 25 o+ -
)( 1 + 2~ )( 1 + 2l XO+ 21-
"- 32 + 16 + 0 + 0 + 2 + l
Unit 17 323
322 Asmlta's NEB Solution of Bnlc Mathematics-II

-
ber 1438 into hex.ad.....,
""'Dial f0nn.
~ - 2071 Set C Q.No. 16 b!
Convert the decimal number 2567,o to O<'t.tl fonn 121 ~ .. 001100011 (a group of 3)
(2)

1~ 1 .. 11oom1
Rem ainder -0110 0011 ( a group of 4)

:1~7 7
8 40 0
(' : o110i. bi~ OO'lh = 3i•)

8 5 0 umeral 3058 t o ~

;,,=25o;;
-~ 7~
0 5
,o~500~7~•==:'=':,"_ _ _ _ _ _ _ _ _ _ _ _ _ _ _ _ _ _ _ _ _ _ _
_ ~J]~~_lfil_r______;;_-.;__:__
D Q.No. 16 b
t
__________
fonn.
121
fr. 2071 Set D Q ,No. 16 bj . o number '7593 into hexadedma) fonn.
Co.o vert the hrxdecim,al number ABS,. to the decunal number. (2)
DI
~ Remainder

[rE~~exa -~-~e:2!:·~t ~~-:: "'"."-:-: -:-:'."~'l-..-~A!..--.-.J---~---"---~~-3


~-b~
AB5,., = 162" A ... 16 B ... 16° 5
1 X
16f93
16 474 9
X
16 29 A
= 256 x 10 + 16 "
= 2560 + 176 + 5
11 + 1 " 5
16 1
0
D
1

-
- 2741 10
759310 = 1DA916

-
jB. 2071 Supp. Q.No. 161!
Convert the binary number 1010001100C'2 into the octal number. (2) @j74 Set A Q.No. 161>!
Convert 2811, into the binary number.
(2)
101OOJ11 OOC'2
010 100 011 000 (group of 3) Weknow,
2430. 216= 00102
Bi6=101h

[.Lt F] 116=0.00h
2Bl 16 = 0010 1011 0001

-
0002 = '-"' =10101100012
@- 'l1J72 s.t C Q.No. 16ij ~7. 2074-Set B Q.No. 161>!
Convert the decimal number 3159 into hexadecimal form. [2] Convert the hexadecimal number 22ft, in to binary form. [2)
B!mml
Remainder We have,
216=00102
16 ~ 197
16 7 F16= 11112
16 12
0 C
5
r :. 22F6
---
= 0010 0010 1111
= 10001011112
ifil, Supp Q.No. 16ij

-
:. 31591o= C571,
(21
Ho. 2012 Set D Q.No. 161>! Convert the decimal number 3058io to hexadecimal form.
[21
Convert heudecim.aJ number 70At6 into binary form. ~ refer to 2070 Set D Q .N. 16b
~ S e t A Q.No. 161>! [21
We know that ~onvert the hexadecimal numeral AB516 to decimal {onn.
716 =01 112, 01• =00002, A1• = 10102 ~ efer to 2071 set D Q .No. 16b
(70A)1• =0111 00001010 = 1110000101 02
~ t C Q,No. 16ij (21
j1 1. 2072 Set E Q.No. 161>! onvert th h dedn1al form.
[21 ~ e decimal number 452610 to exa
Convert the decimal numeral 1503 into hexadecimal form.
-;:-=P=l=.ea=se=r=ef=er=to=W=O=Se
:::;--t_C_Q.::.._N_o_._16_ ;b:.__ _ _ _ _ _ _ _ _ _ __ , . . - - - - - - ~ - - 16
4526 Remainder
l1 2. 2072 Supp Q.No. 161>!
[21 16 282 ➔ E
Convert the declmaJ number 1503 into hexadccl mal Conn . 16
._ Pl.ease refer to WO Set C Q.No. 16 b 16 A
➔ 1
1
324 Asmrta's NEB Solution of Basic Mathematica-II

l=M:J~i#ill•l¢■,§l=illl•l•
Fl,V·i·i fi•lliiill·Wfi
e_1. 2075 Set B Q.No. 16ij ----
If f(O) = -1 and f(S) = 1, how many steps of the bisection method will be required to find
. 3
3
3
3.1:ZS
3.25
3 25
·
3.5
b
4

3.125
3.1875
m•~
3.5
2
3.25 -1
-1
-1
-0.?343
6
2.25
0.5625
Unit 17

f(m)
2.25
0.5625
-0.2343
325

•pproximation to the root of f(x) accurilte to 0.257 an 3.1:ZS 3.1875 3.15625 0.5625
IB!IID 121 3,15625 3.1875 3.17188 -0.2343 0.16015
0.16015
3,15625 3.17188 3.16407 -0.038o9 0.16015
-0-03809
Here, -0.038o9 0.06079
3,15625 3.16407 3.16016 0.06(]79
f(0) = -1 and f(S) = 1 -0.038o9 0.01131
So, the initial interval = [O, SJ 3,16016 3 .16407 3.16212 0.01131
-0.01339 -0.013M
Length of initial inten•al = I b - a I = I8 - 0 I = 8 tJ~e, I f(m) I = 1-o.~1001 "'0.00103 < 0.01 0.01131 -o.00tm
Here, error (e,) = 0.25 So, the required root 1s 3.16212
We have, ~072 set D Q.No. 17
2
~
21 ~E 1
· (] ·ng the bisection method find the root of th .
,s.1 f d ;=als
p)aces o ec-· ·
.
e equation x> + x - 4 -- 0 m (1, 2) coiled to two
!2' <0.25
-
8 -Let-f(x) = x2 + x - 4 141
:::::, 0.25 :5 2' Here, a = 1, b = 2
32:52' r(t) = l2 + 1 - 4 = -2
25:52' f(2) = 22 + 2 - 4 =2
5 :Si Since f(1) and f/2) have opposite signs, so there is a root between 1 and 2
i~S ""."" a+b
Required no. of iterations (steps) = 5 or more. a b m "'-2- f(a) f(b) f(m)

!·l1't'·i·J:fi·l1 1ffiit•iifH
- 1 2 15 -2 2 -0.25
1.5 2 1.75 -0.25 2 0.812>
!ZZ. 2070 Supp. Q.No. 17 a! 1.5 1.75 1.625 -0.25 0.Sl25 0.2656
1.5 1.625 1.5625 --0.25 0.26.56 0.0039
Use the Bisection method to find the solution of the equation x - 2-x = 0 in the interval [0, 1),
accurate to within m·3• (4) 1.5 1.5625 1.5313 --0.25 0.0039 -0.1238
1.5313 1.5625 1.5469 -0.1238 0.0039 -0.0602
IIJll&l 1.5469 1.5625 1.5547 -0.0602 0.0039 -0.11282
1 0.M9 -0.0122
Let f (x) =x - 2-x = x -T, 1.5547 1.5625 1.5586 -0.0'282
1.5586 1 .5625 1.5606 -0.0122 0.0039 -0.0039
1 1.5606 -0.0039 0.0039
f (0) = 0 - 20 = -1 1.5625
The values of a and b are same to two places of decimal in last row. Hence. the appro=te root to
1
two places of decimal is 1.56.
f(l) = 1 -
2= 0.5

--
[_2073 Set C Q.No. 17a! .
Since f O x f(1 = (-1 x 0.5 = --0.5 < 0, a root lies between O and 1.

b m=-2-
a+b
f(a) f(b) f(m)
Apply successive bisection method to fmd the root of the equation x1-4x- l = Olymg betwi 4
~
and 2 correct to two places of decimal
0 0.5 -1 0.5 -0.2071
0.5 0.75 -0.2071 0.5 0.1554 Let f(x) "' x3 - 4x - 1
0.5 0.75 0.625 --0.2071 0.1554 - 0.0234
f(1) = 11 - 4 X 1 - 1 a= -4
0.625 0.75 0.6875 -0.0234 0.1554 0.06657
0.02172 . f(2) = 2' - 4 >< 2 -1 "' -1 in betw\.'\'Il 1 and .,
0.625 0.6875 0.65625 --0.0234 0 .06657
-0.00081 ~ f(1) and f(2) have same sign, so there is no any root ly g
0.625 0.65625 0.640625 --0.0234 0.02172
6. 2074 Set A Q.No. 17ai 1 = 0 in the interval (1, 2) coned to three
Here, I f(m ) = 1-0,00081 I = 0.00081 < 10·3 = 0.001
llse the bisection method to find the solution of xi - x - l41
So, required root is 0.640625.
~3. 2071 Supp. Q.No. 17ai
(41
"-msof decimals.

Find the solution of x 2 -10 = 0 using t he bisection method with a= 3, b = 4 and e = 0.01 .
~ t f(x) a: XI _ X _ l
B!EI
Let f{x) = xi -10
rt: f\l)"' 1' -1 -1 "' -l
s.l 2 - 2 - 1 = 5 1 & 2.
Here, a = 3, b = 4, E = 0 .01 ll)ce f(l) & f(2) h as opposite sign, a root lies ootween
f(3) = 32 - 10 = -1
f(4) =42 -10"' 6
Since f(3) x f(4.) < 0, a real roo t lies between 3 and 4.
326 Aemn•·· NEB Solution of BHIC M•th•m•tlc•-11 b Unit 17 327

Now, 1 2
a+ b f(a) f(b) f(m) f(m)
ll\ C _ 2 _ 1.5 ; 1.75 -4
a b 2
~
5 1.75 1.875 -2.125 -2.125
1 .5 -1 1.875 2
1 2 0 .875 1 ,75 1.8125 -0.39o6 --0.39()6
1 .25 -1 .-0.29688 2
1 1.5 1,75 1.8125 1.7812 --0.39o6 0.7167 0.7167
-0.29688 0 .875 0,22461
1.375 0.1418
1.25 1.5 0 .22461 1.7812 1.8125 1.7969 -0.39o6 0.1 418
1.3125 -0.29688 -0.05151 --0.1296
1.25 1.375 0 .22461 1.7812 1.7969 1.789() -0.1296 0.1418
1.34375 -0.05151 0 .08261 0.0048
1.3125 1.375 t.7890 1.7969 1.7929 -0.1296 0.0048
1.32813 -0.05151 0 .08261 0.01458 -0.0625
1 .3125 1.34375
-0.05151 0 .01458 1,7'1]9 l. 7969 1.7949 -0.~ 25 0.0048
1.32032 -0.0187 --0.0'292
1.3125 1.32813
0.01458 1.7949 1. 7969 1.7959 -0.0'29'2 0.0048
1.32422 -0.0187 -0.0021 --0.012J.
1 .32032 1.32813
0 .01458 t.7959 1.7969 1.7964 -0.0121 0.0048
-O.(XB6
1.32618 -0.0021 0.00622 7964
1 .324.22 1.32813 1.7959 1.7 1.7961 -0-0.003(, 0.0048
1 .3252 -0.0021 0.00622 0.00206 O.~
1.32422 1.32618 1.7961 1. 964 .003(, o1XXJ6 --0.0014
1.32471 -0.0021 0.00206 -0.00003
1.3252 Since a and b have same value upto 3 plac --0.00}4 01XXJ6
1.324.22
1.3252 1 .32496 -0.00003 0 .00206 0.00101 es of decimaJ
' 50 lhe=:-;;;;;::i;;:~~
::l:,.----1
1.32471 0.00101 ~ (Set B) Q.No. 191 U'I required root is 1.796.
1.32471 1.32496
Smee a & b are same upto 3 p la ces of decimal
-0.00003
the req uired root is 1.324. - Show that the equation f(x) = x3 _ x _ 4 has
find the positive root correct to 3 places of d ~t positive root ,1J1d IISing tht mtthod . .

-
127. 201, Set B Q.No. 17! • Please refer to 2069 (Set A) Q.No. 19 ttimals. of buection,
~
ShoW that the equation x 3 - x - 4 = 0 has two negative roots and one positive root and find th
positive root correct to 3 places of decimal by successive bisection method. 14~ ~070 Set C Q.No. 19I
Fin~ the root of th~ eq~ation x3 _ 21' _ 5 = 0 )yin
decimals by successive bisection method. g betwttn 2 ,1J1d 3 correct to three plilC.eS of
E ll& S,!& (6)
Given equation is x~ - x - 4 = 0
Let f(x) = x~- x-4 Let f(x) = x3 - 2t - 5
f(l) = l ' -1 - 4 =-4 Here,a=2,b=3
f(2) =2l - 2-4=2 f(2) = 23- 2 X 2 - 5 = -1
Since f(l l and f(2l have opposi te sirns, a root lies between 1 and 2. f(3) = 3' - 2 X 3 - 5 = 16
a+ b f(a) f(b) f(m) Since f so one root lies between 2 and 3.
a b
m=-2-
a+b
-4 2 -2.125 b m =-2- f(a) f(b) f(m)
1 2 1.5
-2.125 2 -0.3906 ~ ~
1.5 2 1 .75 3 ~ 5.625
1.875 - 0.3906 2 0.7167
1.75 2 2.5 2.25 -1 5.625 1.8'Xl6
-0.3906 0.71 67 0.1418 0.34.57
1.75 1.875 1.8125 2.25 2125 -1 1.8'Xl6
- 0.3906 0.141 8 -0.1296 --0.3513
1.75 1.8125 1.7812 2 2.125 2.0625 -1 0.J-157
-0.1296 0.1418 0.0048 --0.(nl.l
1.7812 1.8125 1.7% 9 2-0625 2.125 20938 -0.3513 0.3-b7
-0.1296 0.0048 -0.0625 0.1671
1.7812 1.7969 1.7890 2-0938 2.125 21094 -0.00!4 0.3-b,
-0.0625 0.0048 - 0.0292 0.0~
1.7890 1.7969 1.7929 2-0938 2.1094 2 1016 -0.(nl.l 0.1671
0.0048 -0.0121 0.0352
1.7929 1.7969 1 .7949 -0.0292 2-0938 2.1016 2J1177 --0.00!4 0.0790
0.0048 - 0.0036 0.0139
1.7949 1.7969 1.7959 -0.0121 2-0938 2.0977 2 0958 -0.0034 0.0352
0.0048 0.006 0.0028
1.7959 1.7969 1.7964 -0.0036 2-0938 2.0958 2.0948 -0.(nl.l 0.0'139
0.0006 -0.0014 -0.0028
1.7959 1.7.964 1 .7961 - 0.0036 2,0938 2.0948 2.0943 -0.~ 0.0028
1.7964 -0.0014 0.0006 - 2.0943 2.0948 ..0.0028 0.0028
1.7961
Smee a and b have same value upto 3 places o f decimal, so th e required root 1s 1.796. The values o f a a nd b a re same to three places of dl'Cirnal in last row. H~nce, the approxi.mate root to

6 MARKS QUESTIONS ~ laces of decimal is 2.094.


~ etDQ.No.1 ~ .
t of the equation xJ - 4x + 1 " 0 lying

-
Appl . - d th
~8. 2069 (Set A) Q.No. 1~
Using m et hod o f bisection, find t h e root of the equation x' _ x _ 4 = Q lying between
1
aR
d

I
ii ~
b Y•ng the m e thod of successive bisection, fin
een 1 and 2 correct to 2 places of decimals.
e roo · [6)

correct to 3 places of d eci m als.

let, f(x) = x' - 4x + 1


Given eq ua tion is x' - x - 4 = 0 f(l) = 1' - 4 " l + 1 "' -2
Let f(x) s x' - x - 4
Sine f(2) = 2' - 4 " 2 + 1 = 1 . . between I and 2
f(l)=P -1-4 = -4 e f(l) a nd f(2) ha ve opposite signs, a real root lies
[(2) "' 2l - 2 - 4 "' 2
Since f(l) and f(2) have opposite s igns, a root lies betwee n '( a nd 2.
328 Asmlta's NEB Solution of Batie Math1m1ttc1-II

a b
a+b
m • -2-
f(a)
f(b)

t
·-
f(m)
-1 .t i ~
:".::;..;;,,::::::=====::::;--D_Q .
.::..:.:...N.:.:o::..
e root of h
) with error lesa t~ e 'Cjll.llion:
19
.:: n l0-2.
[6J

-2
1 2 1.5 1 -0.6406 !section, find th
1.75 -1.ti25 cimals. e root of the, .
1.5 2 1 0.0918 Cfllalionx1-2ic
I 1.1175 -0.6406 Q N 19~ - 5 = 0 lying heh¥een 2and 3
1.75
1.75
2
1.875 1.8125 -0.6406
0.0918
0.0918
-0.2957
-0.1073
•~~=====;----.;:~·...;o:..:..·.:...
-0.2957 [6J
I

1.8125 1.875 1.84..",7 0.0918 -0.0092


1.8594 -0.1073
0,0411 . f(x) • xJ _ 3x _S •
1.8437 1.875 0.0918 . 0 has only one . . root
1.8672 -0.0092 Vslng bisectio n method , find a root IJI
18594 1.S75 0.0411 0.0172 {2.3) correct t0 P0s1tive
1.8633 -0.0092 3 Places of deci
1.8594 1.8672 0.0172 0.0038
1.8614 -0.0092 ma.l.t. [6)
1.8594 1.863..", 0.0038 -0.0026, ttere, f(x) = x' -3x -8
-0.0092
1.8594
1.8604
1.8614
1.8614
1.8604

S= the values of a and b are same for 2 places


-0.0026 0.0038
of deamal, so reqmred root 1s 1.86.
- ttere,
-- f(x)
Now,
has 1 change in sign. So, it has
only one P0Sitiv-e root

~2. 2071 Set C Q.No. 19! 1 b


[1, 4) within an accurac y of 10· (6] f(;i)
Find a n,ot of .m-equa tion .r! + r - 4 .. Oin the interval f(b) f(m)
2 3
E!a l 2.5 -6 10 0.125
2
Here. f(r) = r ' + x - 4 -6 0.1 25
2.25 2.5 2.125 - 3.3594
a = t , b =4 -3.3594
2.5 0.125 -4.7793
f(l) = P + 1 - 4 = - 2 2125 2.3125 -4_7793
£(4) = 4' +4 - 4 = 64 2.5 0.1 25 -2.5710
lies between 1 and 4.
2.3125 2.4063 -25710
Since f (1 ) and f(4) have opposite si=, a real root 2.4063 2.5 2 4532
0.125 - 1.2857
a+b f(b) f(m) - 1.2857 0.125
m=-2 - f(a) 2.4532 2.5 --0.5958
a b
-2 64 14.125 - 2.4766 2.5
2.4766
248l3
--0.5958 0.125 --0.2395
1 4 25 --0.2395 0.125 --O.t:682
-2 14.125 3.1094 2.4883 2.5
1 25 1.75 2.4942 --0.0582 0.125
-2 3.1094 -0.0254 O.O.U9
] 1.75 1.375 2.4883 2.4942 2 4913 --0.0582 0.0339 --0.0115
Her-e, I f(m ) I - 1-0.0254 -
- I = 0.0254 < 0.1 2.4913 2.4942 24928 --0.0115 0.0339 0.0120
So, the require d root is 1.375. 2.4913 2.4928 2 4921 --0.0115 0.0120 0.0010

-
24913
P3. 2071 Set D Q.No. 19! equatio n: [61 2.4917
2.4921 2.4917 --0.0115 0.0010 --0.0032
Us ing the b isection method , find a root of the 2.4921 2.4919 --0.0032 0.0010 --0.0021
less than 10-2
f(r) = 2.r3 - Sr + 2 = 0, betwee n 1 and 2 with error 24919 2.4921 2.4920 --0.0021 0.0010 --0.(XX).5
24920 2.4 921 -0 (XX)5
· 0.0010
Let f(x) = 2x3 - 5x + 2 Since thal
of daima.l. So, the required root is 2A92.
H ere, a = 1, b = 2 filo14 S e v u es of a and bare same upto 3 places
· upp Q.No. 1~
f(l ) = 2 x l' - 5 x l + 2 = -] . . .
Applying the m e th O d o f success ive = lying
f(2) = 2 X 23 - 5 X 2 + 2 = 8 between bisection, find the root of th, equation x3 _ 4x + 1 0
1 and 2. 1 nd [6]
Since f(l ) x f(2) = - 1 x 8 = -8 < 0, a root lies between ,. Please f a 2 correct to two place of dedma.Js
.
a+b
m = -2- f(a) f(b) f(m) ~ e er to 2070 Set D Q.N. 19
. 2~75 Set A Q.No. ~
a b .
-1 8 1.25
1 2 1.5 Find the ..
x- 4• 0 comet to 3 places of decimal with error less
1.5 1.25 -1 1.25 -0.34375 " thilll o.oofos ihve root of the equatio n x.J- (61
1 0.32421
1.5 1.375 -0,34375 1.25 Please ·
1.25
1.3125 -0.34375 0.3242 - 0.04052 ~ refer to 2069 Set A Q.No. 19
1.25 1.375 . 2075 Set C
1.34375 -0.04052 0.3242 0.13397 Q.No. 19 O
J.3125 1.375 Find th
1.32813 -0.04052 0.13397 0.04478
lllet; roots of the equatio n f(x) • x3 _ 4x _
9 coned to three decimal pl;ices by using bisection
1.3125 1.34375 [6)
1.32032 -0.04052 0.04478 0.00165 _ - - o d.
1.3125 1.3281 3
Here I f(m) I - I0.00165 I - 0.00165 < 10
-l
Y!t .
So, the req uired root i~ 1.32032 fie;e ;(.r) = x' - 4x - 9
@4. 2072 Set C Q.No. 1ij
' (2) = 2·' - 4 )( ., - 9 = - 9
root of the eq uati on xJ _ 2r _ 5 "' 0 in (2, 3)
correct 1o s·ince f(?
f(3) = 3, -
- 4 >< 3 - 9 = 6
A pply the m ethod_of bisecli on to fi nd the 161 2 and 3.
three places of d ecim al. , ) a nd f('.\) ha ve opposi te sign, one root lies between
Please re fer lo 2070 Set C Q.No. 19

I
' ~
~=3 -~= 2.2307
330 Asmlta's NEB Solution of BHlc Mathematica-II 11 " ,r,-f(x1) 26 Untt 17. 331

((1111- - - 4
b m f( a) f(b)
_!M = 2 2307 - -869'3
}3_928=1.8811
a
2.5 -9 6 -3.375- - jl ,. ,r2-f'(X2) .
3
25
2
3 2.75
2.625
-3.375
-3.375
6
0.79688
0.7%1\11
-1 .41211
7752
°·
.!{!.1 · 1 8811 - 9.6156 = 1.8004
,. t , - f'(x~) = ·
2.5 2.75
2.75 -1 .41211
2.6875 0.79688 -0.33911 ,, ~ 0.0354
2.625
2.6875
2.6875
2.75
2.71 875
-0.33911
2.71875
-0.33911
2.7001 3
0.79688
0.22092
0.22092
-0.06099 ,, 1 8004 - 8.7243 = l .7963
"' t-1- f'(x.i) = ·
.f1!u. 0.000]9
0.22092
2.70313 2.71875 -0.06099
2.71094
0.07947
0.07947
0.00921
"XS - f'(xs) = 1. 7963 - 8.68 = 1.7962
271094 -0.06099
2.70704 11
2.7<n13 -0.06099
2.70509 0.00921 -0.0258.'l ColJIParing the value~ of xs and X6, we find
2.70704 . th _
that the digits Ill
2.70313
2.70704 -0.02583
2.70607 0.00921 -0.00823 e Hence, the reqwred root is l .796· e first thr
2.70509 0.00921 0,00058 salJI · ee places of d~ are
2.70704 -0.00823
2.70656 ~ A Q.No. 17a OR!
2.70607 -0.00823 0.00058
2.70607 2706.56
the reqwred root 1s 2.706. - v,e NeWton-Raphson's method to a p ~ te'42Wi
}r.
Smee a and bare equal upto 3 places of the dea.mal, thu
......rlliffl trrorlessthano.lXXJOJ.
C. NEWTON RAPHS ON'S METHO D ~ , f4J
&l&i·i-J:fi•i'i!ill•i§ti t.et,x=Vz
rt, xl =2

-
K1. 2070 Suf>P· Q.No. 17 a OR! xi-2=0
of the equatio n xJ + x - 1 = 0 in the interval
0(
Use Newton - Raphso n method to find the solution [4) ~tf(x) = xl -2
(0, 1), aCCUI'ilte to within 10"'. f(x) = 3x2
.&cl.
\Ve have, Xn+l = Xn - f(xn)
Here, f(x) = x' + x - 1
f'(x) = 3.r2 + 1 Let us take an initial guess (xo) = 1.
f(O) = -1 . ~ P-2
f(1) = 1 Now, Xt = Xo - f'(xo) = 1 - 3.12 = 1.333333
root lies between O and 1. Let us take initial guess
Since f(O) and f(1) have opposite signs, a real If(x1) I = 1(1.333333l3 - 2 I = o.370369 > 0.00001
.XO = 1.
By Newton-Raphso n 's method, w e hav e
_ JM _ 1.333333 - (1.333333 )3 - 2 .
.(l . 3-1 )2 = 1.26.188g
XJ - X1 -f'(xi) - 3 3 333
~ lf(x2)l = I (1 .263889)3 - 2 J = 0.018955 > o.CXXXJ1
Xn+l = Xn - f (Xn)
(1 -263889)3 - 2
fu:l_ = 1 263889 -
~ 1 Xi = X2 -
· 3.(1.263889)2 -1. 25 99:33
.r, = .xo - f'(.ro) = 1 <i = 0.75 f'(x2)
ll(XJ) I = I (1 .259933)3 - 21 = 0.000059 > 0.00001
I f(x,) I = 0.17875 > l C>-'
x.i =xi - M - {1 .259933 )'-2
fl!tl. 0.171875
· f'(x 1) - 1.259933 3 (1. 259933)2 - 1.259921
X2 = X1 - f' (.ri) = 0.75 - 2_ 6875 = 0.68605
t(xi)I = I (1 .259921)3 - 21 =O.OOOOCXXJ1 < 0.00001
I f(x2) I = 0.00895 > l Q-1
_ ~- _ 0.00895 _ ~ce the r_e quired root is 1.259921
.r, - x, - f' (x 2) - 0.686(b - 2 _411 99 - 0.68234 ·~•Set B Q.No. 17a oRj
I f(x1) I = o.00003 < 10-4 Using Newt on-Rap hson's method, find the positive root of xJ - 2x - Sa Olying between 2 and 3
co 141
Hence, the required root is 0.68234. ~ t to three places of decimal .

-
~2. 2073 Set C Q.No. 17a OR!
1 and 2 lo three places of decimal by Newton Let,f(x)., xJ _ 2x _ 5
Find a root of the equatio n x - x - 4 = 0 between
3
[4)
Raphson 's method. Then f'(x) = 3x2 - 2
f(2) = 21 -2 " 3 - 5 = -1
3
G iven equation is x1 - x - 4 = 0 Since /( ) = 31 - 2 >< 3 - 5 = 16
Let f(x) = x' - x - 4 let (2) anct f(3) ha ve opposite signs, a root lies between 2 and 3.
Us take uu· .
f(l ) = l3 - 1 - 4 = -4 ByNewt tia l guess (xu) = 2
f(2) = 2 1 - 2 - 4 = 2
011
-Raphso n 's me thod we have
Since f(l ) f(2) c (-4)" 2 c - 8 < 0, a real root lies
between 1 and 2. l" l " X ~ ,
n- f'(x.,)
f'(x) = 3x 2 -1 .

Let the inj tia( guess xo = 1. . l1 " l·o - ~ '-11


By Newton Rap lu.on 's m ethod , we have f'(xu) = 2 -7! = 2.1666
- .&cl.
f' (xn) ti .. x, - ~ - 0.8371
Xn•I - Xn - f'(.x,) - 2.1 666- ~ "" 2.0973
fu!!l. ~ ti " lh ~ 0.03072
xi = xo - f'(xo) = 1 - 2 "' 3 r(x2) .. 2.097'.' -11.196 .. 20945
3U
Aamlta'a NEB Solution of Basic Mathematics-II
COMPllrATIONAL METHOD Unit 17 333

= ~ = 2.0945 ~
~ 2070 Set C Q.No. 19 ~
X4 X3 -
r(x3) - 11.1607 = 2.0945 Solve 2r2 _ 3r _ 1 m O u l
~ s ng Newton-Raphson method talcin .
Comparing the values of l ) and x., we find that the digits in the first thn.>e places of decimal are Uw ~ gr.= 1 wdh error less than ID-4. (6)
same. Hence, the required root to three places of decimal is 2.094. Here, f(r) = 2x2 _ 3x _ 1
iilhf ·i):fi·\11 f i Ihl~t1 f(l) = 2 x l2 _ 3 x 1 -1 = -2
• f(2) = 2 X 22 _ 3 X 2 _ 1 = l
Ks. 2069 (Set Al Q.No. 19 orl Smee f(l) and f( 2) h
f'(x) = 4r - 3 ave opposite signs, so there is a real root between 1 and 2
[6)
Using Newton-Raphso n's method, find the square root of 153 correct to 3 places of decimals.
l:iD!iB Let us lake initial guess ro = 1
By Newton Raphson's meth d
Let x be the square root of 153. Xn+1 = Xn _ &cl, o ' we have
Then. x 2 = 153 f'(xn)
x 2 -153=0
Let f(r) = r2 - 153 x, =
Xo -
JM 1-2\
r(xo) = l - ~ = 3
a=153
Take, initial guess (Xo) = 10 X2 =x1 fu!.l. 3 8
- r(x,) = -9= 2.1111
By Newllln-Raphson 's method, we have
1.5802
x., =x,-JM-2111 1 - 5M44
Xn+1 !)
=½( X o + r(x2) - · = 1.8209

X4 = X3 _ .!M _1 ·8209 - 0.1684 =1.7816


½( ! ) + =
5
½( 10 + \ ; ) = 12.65
· r(x,)- 42836
x, = Xo
xs = x4 _,!M _ 1 7816 0.0033
r(x4)- · -4-1264=1.7808

X2 = ½(x ,+-;,) = ½( 1265 + 1~~) = 1237243 Xh = XS _ ,!M _ 1 7808


f'(xs)- ·
0.000}
-41232=1.7808
Here, I f(l.7808)_1 = 12(1.7808)2 - 3(1.7808)-1 I = 0 ,v,nn,,,,,.,o
x, ½( ! ) r 2+ = ½( 1237243 + 12~;;43) = 1236931 H ence, the requued root is 1 7808 -vvvu7, = < 0.0001
@s. 2070 Set D Q.No. 19 ~ · ·
X• = ½( r , +;;)
= ½( +
1236931 12~ 31 ) = 12.36931
mI!!la
Using Newton-Raphson m th d f
e o • ind the positive root of x3 -18 = 0 in (2. 3) [6]
are
Comparing the values of r , and x., we find that the digits in the first three places of decimal
same. Hence, the required square roots of 153 to three places of decimal is 12.369. Here, f(x) = xJ - 18
f'(x) = 3x2

-
146. 2069 (Set Bl Q.No. 19 orl f(2) = 2' -18 = -10
Using Newton luphson's method find the positive root of the equation f(x) = x3 - 2x - 5 = 0 lying f(3) = 33 - 18 = 9
[6)
between 2 and 3 correct to 3 places of decimals.
Since f(2) and f(3) have opposite signs, a real root lies between 2 d 3
Let 11\lbal guess x 0 = 2. an ·
Let,f(r) = x3 - 2r - 5 By Newton-Raphson 's method, we have
Then f'(x) = 3x 2 - 2 - fucl.
f(2) = 23 -2" 3 - 5 = -1 Xn+l - Xn- C(xn)
f(3) = 3 3 - 2 X 3 - 5 = 16
- . ~ i:!Ql
Since f(2) and £(3) have opposite signs, a root lies between 2 and 3. x, - .t o - C(xo) = 2 - 12 = 2.833-33
Let us take initial guess (xa) = 2 _ _ K!.!.l. 4.74529
By N e wton-Raphson's me thod, we have x2 - ,11 - C(xi) = 2.8333 - 24.08328= 2.63629
Xn+l = Xn -
.&d
f' (xn) _ _ .fu& 0.32228
3 f'(xi) = 2.63629 - 20_85007 = 2.62083
X - .l ~ -

XI = Xo - ~ - 1:!L
f' (Xo) - 2 - 6 - 21666 .1'4 = . .fu& 0.00182
.t , - f'(x,) = 2.62083 - 20 _60625 = 2.62074
IM o.8371 _
X2

X3
= XI - f'(X 1) = 2.1666 - 12 0824 - 2.09'13

= X2 - .!M
f' (x2) = 2.0973 -
~11.196 -
- 2.()(H5
rl±Comparing tl1e values of X) and X4, we find the digits in the first three places of decimal are equal, so

8 2 7
required roo t is 2.62 .
· 0 1 Set C Q.No. 19 OR!
.fu}l _ (--0.0005 ) ~ 2 0945 : ind a root of the equation x3 - x - 4 • O between 1 and 2 to thtte places of decimal by Newton-
X• = x, - f' (r,) -
2.0945 - 11.1607 . son m e thod. · (6]
~
Comparing the valu es o f r, and x., we fui d tl1at the d igits in U1e fi rst three places o f d ecimal a rc lill'
same . H ence, the required root lo three p laces of decimal 1s 2.094 . Given Cl(U a tio n is x' - x - 4 = O
~t f(x) = x' - x - 4
(l ) '" l l _ 1 _ 4 ., -4
334 Asmita's NEB Solution of Basic Mathematics-II

f(2) = 2' - 2 - 4 = 2 COMPUTATIONAL METHOD Unit 17 335


Since f(l) f(2) = (-4) x 2 = -S < 0, a real root lies between 1 and 2. Now, f(U) "' cos(O)- 01 = 1, & f('l) = _ 0.OOJ1 .
0 & 1. 523· Smee f(0) & f(l) are of opposite sign, a root lies between
f(x) =3.r2-1
Let the initial guess xo =1. Let us lake an initial guess xo • 0.5
By Newton Raphson's method, we have Prom Newton Raphson method, we have,
~ x1 = xo _ .&!!J, = 0 cos(0.5) - (0.5)1
Xn•l = Xn - f(Xn) f'(xo) ·5 - -sin(0.5) _ 3(0.5)2 = 1.657284686
~ !:1),_ .!M
:n = xo - f(xo) = 1 - 2 - 3 X
2
= Xi- f'(xi) = 1.226971721 M
X1 = X2 - f(xz) = 1.039895734
. - ~ lQ_ x- ~
X2 - X1 - f(.r i) = 3 - 26 - 2.2307 4
- x., - f'(x,) = 1.001681167
X; = X.. - ~ = 0.9999831544
x, = X2 - ~ ~-
f(X~ = 2.2307 -13.928 - 1.8811 Xs = Xs - J&
f'(><s) = 0.99999o429J.
X7=X,,-~=0.9991J754604
flcl_ 0.7752 _
- M
x, = .n - f(X)) -1.8811 - 9 _6156 -1 .8004 XH - X7 - f'(X7) = 0,999<J961572
x. -
- x. - M
f'(x.) = 0.9999700202
~- 0.0354_
X5 = X; - f(x,) - 1.8004 - 8.7243 -1 .7963 - M
Xio - X•i - f'(x,) = 0.9999736755
~ _ 0.00019 _
= xs - f(rs) - 1.7963 - 8 _68 -1.7962 Required root= 0.99997
Note: The angle is in radian measure.
Comparing the values of l.'S and l'.6, we find that the digits in the first three places of decimal are
same. Hence, the required root is 1.796. lss. 2074 Supp Q.No. 19 o~
jso. 2071 Seto Q.No. 19 oij Using Newton-Raphson method find .
correct to 3 places of decimals. • a root of the equation xJ - 2x - 5 = 0 lying between 2 and 3
Derive the formula for Newton-Raphson method Using Newton Raphson method, find a Please refer to 2069 Set B Q.N. 19 OR [6]
positive root of x3 + 3r - S = 0 lying between 1 and 2 correct to three places of decimals. [6]
lss. 2075 Sat A Q.No. 19 o~
!iB.!!iB Using Newton-Raphson method find t f .
Newton-Raphson Formula three places of decimals. ' a roo O th e equation r " + lr - S = 0 between 1 and 2 to
Let xo be the initial guess for the function y = f(r). Then for a point (x, y) sufficiently close to it, the
... Please refer to Model Set n, Q.No. 19 OR 161
function can be approximated b y its tangent line
@7. 2075 Sat B Q.No.~1!
" - f(.r,,)
x-x,, = f(ro) Fhor f(x) = xl - 4,perform 3 iterations of Newton-Raphson's method with starting point x.o = 2. Find
This tangent line crosses the x-axis when y = 0. Let us denote this new value of x by x,. Then, mmJmm
t e errors and percentage errors of Xo. x,, x, and XJ.
[6]
.!£cl.
x, = Xo - f(xo) Here,
Continuing in this way, we have f(x) = x3 -4
- .!{cl f'(x) = 3x2
Xn+l - Xn - f'{xn) and xo= 2
Second Part By Newton Raphson method, we have
Please refer to Model Set U, Q.No. 19 or -
Xn+l - Xn - f'(xn)
~
@1 . 2072 Set E Q.'NO. ~9 OA _ ~ (23 -4)
Find a root of the equation 2x1 - 3x - 1 = 0, x e (1, 2) using Newton Raphson method with error x, -xo-f'(xo) =2- 3 x 22 =1 .66667
less than to-<. [6]
Please refer to 2070 Set C Q .No. 19 or
@2. 2072 Supp Q.No. 19 OR!
Error (E1) = I IX1 ;/o = 1 1 · ~ ; 21 = 0.1999

Using Newton Raphson's method, find the root of the equation f(x)-= x 3 - x - 4 = 0 in (1, 2) correct Percentage error= E1 x 100 = 0.1999 x 100 = 19.99%
to 3 places of decimals. [6) IM n .6666?)' - 4
X2 = X1 - f'(xi) = 1.66667- x (1 .66667)l = 1.59111
Please refer to 2071 Set C Q.No. 19 OR 3
Js3. 2073 Set D Q.No. 19 OR!
Using Newton -Raphson method, find a root of the equation f(x) = x3 - x -4 = 0 in (1,2) correct to 3
Error (Ei) = I II
X2 - X1
- ;;- = 1.59111-1.66667
l.59ll1 I = 0.0475
places of decirn,1 ls. (6) Perce ntage Error = Ei x 100 = 0.0475 x 100 = -1.75%
Please refer to 2CY71 Set C Q .No. 19 OR
_ futl_ (1 .59111 )·1 - 4 _
JS4. 2073 Supp a.No. 19 otl x, - xi - f'(xz) = 1.59111 - 3 x (f59111 )2 - 1.58741
Newton Raphson's Method to find a positive root of cosx .. xl,
mll!mlE
161
E
rror (E.i) =
I I
~
x., - X2
=
11 .58741 - l.59lll
- i.5874l
I = 0.0023

Let f(x) = cosx - x3 = 0 Percentage Error = 0.002'1 x 100


r(x) = -sinx - 3x 1. -= 0.23 %
336 Aamlta"a NEB SOlutlOn of Buie Mathemettc,-11

- e
@s. 2075 Set C Q.No. 11]
Use Newton-Raphson method (formul a) to find the solutions of f(x) • 1 - 12x + xJ correct Upto
four decimal places. (6)

We draw the graph of the function in order to make the initial guess.
UNIT1
9
Here,
f(x) = 1 - 12x + x'
f(x) = 3x 2 -1 2
COMPUTATIONAL
f '(x)= 6x
Now, !...
f(x) = O ~ 3x2- 12=0
~ x2 = 4
1 ~
-4 I -2 ° ~ 4 METHOD {CONTINUED)
~ x = ±2
Also, f(2) = -15and f(- 2) = 17 t·Md·i'M:-i§•@ii¢h•i•i~•Ml=i•=i•i•
Hence f has local maximum at - 2 and minimum at 2. fi3t·i·i!fi·i'lfiii·iW Given equations are:
Also a tx=O,y=l .
From the graph. it is clear that there are three solutions, one near to 0, another less than -2 and the !1. 2070 Supp. Q.No, 1~ 2x + 3y = 4 ... (i)
3x+ 2y = --4 ... {ii)
remaining is greater than 0. Test the con~is~ency of the followin s stem by
Solution near 0 the Gauss elinnnation method: g Y
X - y - 2z = -1, 2x + y + z = 2,. Jx + 2y + 9z = 4
Multiplying equation (i) by ~ and then
TakeXil=O
1
subtracting from (i:i),
~ -o -
- -- l
x, = xc - f(Xo) - - (- 12) . - 0 ·08333
12-
mmiml (2] 3x + 2y = -4
9
Given equations are:
~ 3x +
2y =6
X2 = x, - f(x,) x-y-2z=-l ... (i)
1 - 12 (0.08333) + (0.08333)3 2x + y + z = 2 ... (ii)
5
= 0.08333 - 3(0.08333)2 _ 12 = 0.08338 3x + 2y + 9z = 4 ... (ill) -2 Y =-10 ... (iii)
Multiplying equation (i) by 2, then subtracting
~ from equation (ii), we have
Now, w e have the following equations
Xl = X2 - f'(x2)
2x +3y = 4 ... (i)
2x +y +z =2
_ 1 - 12 (0.08338) + (0.08338)3 _ 5
- 0.08338 3(0.08338)2 - 12 - 0.08338 2-r ~ 2y - 4z = -2 -2 Y = - 10 .. . (iii)
+ + +
Hence the solution i.s 0.0833 upto four places of decimal. From equation (iii), we have y = 4
Solution greater than 2 . 3y . ~ Sz = 4 ... (iv) Using y = 4 in equ ation {i), we have
Take Xo =3
Agam, multiplymg equation (i) by 3 and then 2x +3x4=4
subtracting from equation (ill), we have or, x = --4
_ ~- l - 12 x3 +3l
x, - Xo - f'(Xo) - 3 - 3x32 - 12 3x + 2y + 9z =4
The required solution is x= -1, y = 4.
3x - 3y - 6z = - 3
= 353333 + EMil"i·i:ii·l■Jfill·iit1
~
X2 = x, - f'(x,) = 3.426797 Sy + 15z = 7 .. (v)
p, . 201.uuee a.No. 1711
Xl = 3.421669 Multiplying equation (iv) by ¾ and Using Gauss-elimination methfMt solve the
x.. = 3.421658 following system of eq-tion: (4)
subtracting from equation (v)
Required root correct to 4 place of decimal = 3.4216 x + 3y - 2z =5, 3x + Sy + 6z = 7, 2x + 4y + 3z = 8.
Sy + 15z =7
Sol ution less than -2 • Please refer to 2070 Set C Q.N. 17a
25 20
TakeXo = - 3 Sy +3z =3 i,l.2075 Set A. Q.No. 17!
~ Solve the following system of equations by
x, = Xo - f (Xo)
20 1 Ga\155-elimination method. (4)
1 -12 " (-3) + (- J)l z . (vi) X + Jy - z '" ~ . 3x + 2y- z=3, -6x- 4y- 2z =18
= (- 3 )- 3 X (-3) 2 - 12 3 3
... Please refer to Model Set LIQ.No. 17a
= -3.66666 Now, we have the following three equations
- 2z = -1 -~- 2069 {Set A) Q.No. Hai

-
X - y .. (i)
X2 = X1 - iM
f' (x,) 3y + Sz =4 ... (iv) Using Gauss elimination method, solve the
20 1 following system of equations: (4)
= -3.51 5032
x, = - 3.505081 3 z =J .. (v i) X - 2y + 3z = 2., 2x - 3y + Z z 1, 3x - y + 2z = 9

x.. = - 3.505039 Prom eq ua tion (v i), we can fimt a finite vnlue


Thus, the required root up lo 4 places o f deci mal in t.hi 6 case is - 3.5050. of z anct then fini te values of x and y. So, the Given equations are:
Hence, the required solu tions are 0.0833, 3.4216 and -3.5050 ~ m of equation is consistent. x-2y+3z = 2 .. (i)
; ~ Supp. Q.No. 16q
2.t - 3y + z" 1
Jx - y + 2z ~ 9
.. (ii)
.. (iii)
;;auss e limina tion m ethod, solve
Jy .. 4, 3x + 2y C -4 [2)
338 Asmlta's NEB SQlutlon of Basic Mathematics-II

Multiplying equation_ (i) __ by 2 and then Now, we have U,e following three cquotiuns COMPUTATIONAL METHOD (CONTINUED) Unit 18 339
subtracting from equation (u) x, - 2r2 + 3.1"1 a JO ... (i) .>
[ 2071 Stt C Q.No, 17j
2x -3y + z = 1 7x1 - Sx~ = -19 ... (iv) From equation (vi), we have z = 3.
2-r -4y + 6z =4 17 85 Using Gauss-ellmlnatlo n method, solve the Using z = 3 in equation (iv), we have y = -1
following system of equation.
7-"-' =7 ... (vi) Using y = -1 and z = 3 in equation (i), we have
... (iv) X + Jy - Z • -2, Jx + 2y - Z • J, -6x - 4y - 2z • 18 x=l
y -Sz = -3 . From equation (i), we have x., = 5
Again, multiplying equati_o_n (i) by 3 then Using _,._, = 5 in equation (iv), we have ._ Please refer lo Model Set II, Q.No.17a
(4) ;==,,,The required solution is x = 1, y = -1, z = 3 .
subtracting from equation (m)
3x -y + 2z = 9
7x2 -40=-19 110. 2071 Set D Q.No. 17 aj t!! 2072 Set E Q.No. 17aj

+9z = 6
x2=3
Using Gauss-eliminatio n method, solve the Using Gauss-elimination method, solve the
3.-r -6y
+
Using x2 = 3, x., =Sin equation (i), we have following system of equation: x - 2y + 3z • h z followtng system of equations.
... (v) X\ - 2 X 3 + 3 X 5 = 10
- Jy + z .. 1, 3x - y + 2z " 9. (4) h2 + 3::a'),. 7, 3%, - 2x2·+ 2xJ =1, 2x, + 3x2-3x3 =5.
Sy - 7z _
(iv) by 5, then x, =1" ._ Please refer to 2069 (Set A) Q.No. 17a [4)
Multiplying equation
subtracting from (v) The required solution is x1 =1, x2 =3, x~ =5. 111. 2072SetCQ.No .17a~
Sy -7z = 3 lf. 2070SetCQ.No. 17ij - Solve by Gauss elimination method:
The given equations are:
Sy :25z
:-15 Solve, using Gauss elimination method, the
[4)
x + 3y-h = 5, h +Sy+ 6z = 7, 2x+ 4y+ 3z= 8(4)
2x2 + 3x, = 7
3x, - 2x2+ 2x1 = 1
following equations. ... Please refer to 2070 Set C Q.No. 17 a 2x, + 3x2-3x3 = 5
J8z = 18 ...(vi) .
Now, we have the following three equations
x+~-h=~h +~+k=~2x +~+h=& /12. 2072 Set D Q.No. 17a ~ The coefficient of the first variable .r1 is zero in
X _ 2y + 3z = 2 · .. (!) mmm Use Gauss elimination method to solve: (4) the first equation So, interchanging the first
y - Sz = -3 · · · (iv) Given equations are: = 8, 2x + Sy + 2z = 3, x + 2y + 4z = 11.
4x - y + z two equations, we have
18z = 18 ·· · (vi) X + 3y- 2z = 5 ... (i) B!.lmlll 3x, - 2x2+ 2x, = 1 ... (i)
From equation (vi), we have z = 1 3x + Sy + 6z = 7 ... (ii) Given equations are: 2x2 + 3x1 = 7 ... (ii)
Using z = 1 in equation (iv), we have 2x + 4y + ~z = 8 ... (iii)
4x- y + z = 8 ... (i) ~+~-~=5 -~
y-5 X 1 = - 3 Multiplying equation (i) by 3 and then
subtracting from (ii), we have
2x + Sy + 2z = 3 ... (ii) j
Multiplying equation (i) by and the subtracting
y=2 . . (") X + 2y + 4z = 11 ... (iii)
Again, using y = 2 and z = 1 m equation I , 3x + Sy + 6z = 7 from equation (iii)
Multiplying equation (i) by 1/2 and then
we have 3x + 9y - 6z = 15 from equation (ii) 2x, + 3x2 - 3x, = 5
subtracting
x-2><2+3><1= 2 + 4 4 2
2x + Sy + 2z = 3 2x1 - 3 x2 +3x1 =3
x=3 - 4y + 12z = -8 ... (iv) 1
1
The required solution is x = 3, Y = 2, z = 1· Again, multiplying equation (i) by 2 and then 2T - 2y +
2z = 4 - +

~- 2069 (Set B) Q.No. m4 subtracting from equation (iii), we have


2x + 4y + 3z 8
- +
13
3X2 -3.T1=3
13 13

Using Gauss eliminatio~ method, solve the 11 3


following system of equations: 2t + 6y -4z = 10 2Y+2z =-1 .. . (iv) or, .t2 - x, =1 ... (iv)
+ 1
x 2x2 + 3x3 = 10 2.r, + 3x2 - 2x3 = 1, and Again, multiplying equation {ii) by and
-:J~ 2x2 + 4l'3 = 13. , [4) -2y + 7z = -2 ... (v) Again, multiplying equation (i) by¼ and then 2
Multiplying equation (iv) by 1/2 and then subtracting from equation (iv), we have
mmJiEZl subtracting from (v)
subtracting from equation (iii) .t2 -.tJ = 1
Given equations are: X + 2y + 4z = 11 3 7
-2y + 7z = -2 X2 +2.t"J =2
2x1 + 3.r3 = 10 ... (i)
XJ - 1 1
-2y + 6z = -4 2x -4Y +4z = 2
2x1 + 3x2 - 2x3 = 1 ... (ii)
+
-x, - 2x2 + 4x3 = 13 ... (iii)
z= 2 ... (vi) . - + 5 5
Multiplying equaticm _ (i) __by 2 and then
Now, we have the following three equations 9 15 -2.t3 = -2 ··· (v)
subtracting from equation (11), w1; have 4Y+4z =9 .. .(v)
Now, we have the following three equations
2x1 + 3x2 - 2x3 = 1 X + 3y - 2z = 5 . -('.)
-4y + 12z = -8 ... (1v) 9 3.t1 - 2.T2 + 2.t, = 1 ... (i)
2x1 - 4x2 + 6XJ = 20 Again, multiplying equation (iv) by and
z = 2 . . .. (v1) 22 .t2 - .tJ =1 ... (ii)
subtracting from (v) 5 5
7x - Bx, = -19
.. . (iv)
Again, ad~g equation (i) and equ a tion (iv)
we have
From equation (vi), we have z = 2. _
8
Using z = 2 in eq uation (iv), we have y have
Using y = 8 and z = 2 m equat10n (1),
:e 9
4y+
15
4 z=9
-2 --C.• = -2
From equation (v), x., = 1
... (v)
.
-x, -
2x2 + 4x, = 13 -2 9 27 9 Using x., = I in equation (iv), we have xi_= 2 .
x,2x 2 + 3x, - 10
x=~~
The required sol ution is x =-15, Y =8• z - _:_ 4Y + 44 z = -22 Again, using .n = 2 and x, = 1 111 equation (1),
we have
- 4x2 + 7X3 - 23 . (v)
4 ~- 2070 Set D Q.No. 17 a! by =1
.tt
Multiplying equation (iv) by 7 and adding Solve the following system of eq uation (41 nz =n
69 207
... (vi) The required solution is x1 = 1, xi= 2, X3 = 1-
Ga.usslan elimlnalfon method. 4 + 3z"' 8.
with equation (v)
x + 3y _ 2z "' 5, 3x + Sy + 6z "' 7, 2x + Y Now, w e hav e t.he following three equations /1•. 2072 Supp Q.No. 17aj . .
-4x2 + 7x1 = 23 =8 ... (i) Solve the following system of equations usmg
... Please re fe·r lo 2070 Set C Q .No. 17 a 4.x - y + z
32 -76 Gauss elimination method: _14)
- 7X' =7 11 '.'\
4x2 x + 3y - 2z • S; 3x + Sy + 6z "' 7; 2x + 4y + 3z - 8
2Y•2z =-1 ... (iv)
.. . (vi)
,. Please refer to 2070 Set C Q.No. 17 a
69 2(!7
22z =22 ...(vi)
340 A1mlt1'• NEB Solution of B11lc M1them1tlc1-II

!15. 2073 Set C Q.No. mij [4 )


Again, using x., '" 1 and
3x, + 1 + 1 = 5
x2 = 1 in (!), we get COMPUTATIONAL METHOD (CONTINUED) Unit 18 341

Solve by Gauss elimination method. x, ml Multiplying equation (i) by! and then Next part:
3Xt + Xl + X3 =5, Xt - 4X2 + X3 = -2, Xl + Xl - 3Xl • -1. Given equations Me
subtracting from (IJ), we get
IDIIII 116. 2073 Set D Q.No. 17ij 3x + 4y + 5z ., 26 3x - y + 22 - 2 ... (i)
Given equations are: x + Sy + 2z = 6 ... (ii)
3x1 + Xl + x_, = 5 · · ·(i) Using Gauss-elimination method, solve the ~
3x + Y + 6z ., 30
2x + 3y + z = 0 ... (iii)
following system of equations:
X1- 4Xl + X3 = -2 .. . (ii)
2x - 3y + 3z =27, 4x + y - 2z =0, -6x - 4y + 2z .. o. Multiplying equation (i) by ½and subtracting
X1 + Xl - 3XJ = -1 ... (iii)
(4) from equation (ii)
. 1
Multiplying equation (i) by 3and subtracting x+5y+2z 6

from(ii)
Given equations are
2x-~+~= V ···W
Again, multiplying equation
subtracting, from equation (iii)
(i) by i& 1 1
- 3Y + 3z -3
2

X1 - 4XJ + x_, -2 3x + Sy + 6z = 31 + +
4x+y-2z=O .. .(ii)
1 1 ~ -6x-4y+2z= O ... (iii) 9 16 5 20
Xl + 3X2 + 3X• 3 _3x .:'.: zY .:'.: 6z =_30 3Y + 3Z 3 ... (iv)
Multiplying equation (i) by 2 and subtracting
1
from equation (ii) Again, multiplying (i) by ~ and subtracting
13 2 11 4x + y - 2z = 0 2 y-0.z= 1 ... (v)
- 3X2 - 3 x_; = -3 ... (iv) 4x-6y+ . 6z=S4 tdding equation (iv) & equation (v)
from(iii),
2x + 3y + z
- + + 0
1 2Y-O,z = 1 2 2
... (iv) 4
Again. multiplying equation (i) by 3and 7y-8z = -54
1
2x
3Y + 3Z -3
subtracting from (iii), Again, multiplying equation (i) by 3 and -2y-z=-4 + +
3x_, -1 adding with equation (iii) 11 1
Xl + Xl 4
6x - 9y + 9z = 81 -z = -3 ... (vi) 3Y + 3Z . (v)
1 1 ~ 3 ..
X1 + 3Xl + 3X3 -6x - 4y + 2z = 0 Now, we have the following three equations
3
-13y + llz = 81 ... (v) 2x + 3y + 4z = 20 .. . (i) Multiplying equation (iv) by 11 and
13 1 16
and subtracting from (v),
2 10 8 Again, multiplying equation (iv) by 7 -2y-z= -4 ,- ·· (iv)
4
11 1
3X2 - 3X3= -3 ... (v)
adding with equation (v) -z= -3 ... (vi) 3Y + 3Z 3
2 104 -702 From equation (vi), we have z = 3 11 55 55
Again, multiplying equation (iv) by 13 and 13y - T2 = -7- t uttin g the value of z in (iv), we get 3Y + 4aZ 12
1
adding with equation (v). -13y + llz = 81 -2y-3=-4
-135 ... (vi) 13 ~
2 4 22 27
-3X2 + 39X 3 -39 --yz=-7- 1 -16z 4 ... (vi)
or, - 2 y=-4+3=-1
Now, we have the following system of Now, we have the following three equations
2 10 8 or, y = 2 ... (i)
3 X2 - 3XJ -3 equations 3x - y + z: -2
Again putting the value of y & z in (i) 16 5 20
2x -3y + 3z = 27 ... (i)
3Y+3 2 =3 ... (iv)
126 126 .. (iv) 2x+3><2+4> <3=20
- 39 X3 = - 39 ... (vi) 7y- 8z = -54
or, 2x = 20 - 18 = 2 13 13
27
--;;-z=-7
135
.. . (vi) or, x = 1 -162 = -4 ... (vi)
2 4 22 : . X = 1, y = 2, Z = 3 From (vi), we have z = 4
-r2+ ~ 3 = -39 From equation (vi), z = 5 Using z = 4 in (iv),
Using z = 5 in equation (iv), [if 2074 Set B Q.No. 17ij 16 5 20
2 10 8
r2 - ? 3 = -3 7y = -14 Solve by Gauss elimination method: (4) 3Y+3• 4 =3
.. y = -2 ' x+ 3y - 2z = 5, Jx + Sy + 6z .- 7, 2x + 4y + 3z • 8.
... (vi) or y=O
126 126 Again, using z = 5 and y = -2 in (i), we get, "' Please refer to 2070 Set C Q.No. 17a
-39X3=-39 Using z ~ 4 & y = 0 in (i), we get
2x - 3 >< (-2) + 3 >< 5 = 27
Now, we have the following system of x=3 11•i·iH•i11H•MM 3x-0+4=-2
or, 3x =-6
equations. Hence, the solution is x = 3, y = -2, z = 5. x=-2
"@]§_75 Set B Q.No. 1st
3x1 + X2 + X3 = 5 ... (i)
!17. 2074 Set A Q.No. 17ij . .'. X = -2, y = 0, Z = 4
13 2 11 What are two steps of Gauss elimination
-3X2+ 3X3=-3 .. . (iv) Solve the following system of equations by iethod? Find the approximate solution of the
Gauss elimination method, . = lt.
e~:lowJ ng system of equations by Gaussl
126 126 2x + 3y + 4z = 20, 3x + 4y + Sz = 26, 3x + Sy + 6z · ·
-39 X3 = -39 ...(v i) 141 lt+llllnatton method: 3x - y + z" - 2,
~2z=6,2x +3y+z.,O . [6]
From equation (vi), we have, x1'" 1 limlDllml
Using X3 =l in (iv)
Given cq ualions are
13 11 2 l'he two steps of Gauss Elimination method
-3x2=-3 - 3 2x + 3y + 4z = 20 ... (i) . are
3x + 4y + Sz "' 26 ... (Ii) l,p
.". X2 =1 3x + Sy + 6z ., 31 ... (iii)
U. 8orward Elimination of unknowns
a.i;kw<1ru Substitution.
342 Asmlta's NEB Solution of Basic Mathematics-II

--- COMPUTATIONAL. METHOD (CONTINUED) Unlt18 343


,§3. 2072 Supp Q.No. 169
B. GAUSS-SE IDEL METHOD Examine whether the following equations arc
mmm+Dl 2069 (Set Bl Q.No. 17a §a
e_o.

--
12)
flbt·i-J!fi.J'iiiihl?ti diagonally dominant:
8x,-2X2+3X :l"'-1 Given [ ! ; ] [; ] = [;] Solve the following equations using Guess-
Seidel method:
-3x,+9x2- x.,=2 (4)
120. 2069 (Set A) Q.No. 16g i.c x + 2y ., 4 & 2x + 3y • 7 3x, + X2 ,. 5, x, - 3x2 "' 5
Examine whether the following system of 2x1-X2-7X :l-=3 Solving, we get x = 2 & y = 1
equations are ill conditione d
(2)
mmm p] 1 1
2x1 + X2 = 25; 2.001x1 + X2 = 25.01 i.e. [ ; ] =
Here, Xi =3(S-:r2), X2=
3 (x, -5)
lB!IEl I8 I > I -21 + 13 I = 5 Let us make a small h . Initially, x, = 0, X2 = 0
The condition for the ill-condition ed system of 191 > 1--31 + 1-11 =4 matrix of the system o~ ;11;;
_m the coefficient 1" iteration:
equations is that the detemlinan t of coefficient 1-71 = 7 > 121 + 1-1 I = 3 1.0001
2 0001
q tion.s as follows:
1
matrix is small enough or approxin1ately
equal to zero.
Hence, the given system
dominant.
is diagonally
_
[
2.0001 3:cxxn ][ ] = [
; i] x, =1(5-0) --1.667,
3
2"" iteration:
X2 = 3(1 .667 -5) = -1.111

Here, detenninant of roe.fficient matrix is 1.e. 1.0001x_+ 2.0001y = 4


,§4.2073 Set C Q.No. 16q 2.0001 + 3.0001 y ,,: 7 x, =l - + 1
3(:> 1.111)=203 7, .t2=3(2037 -5)=--0.988
I 2~ ~ I
= 2 " 1 - 1 " 2001 = 2 - 2001
Define well-cm1:ditioned and ill-conditioned of a
system of equation. (2)
Solving we get x = 20003 _
Again, let us make a small& Y- 0.9997
3"'iteration:
1
mmm hand side vector of th change U1 the right :ri =l - +
3 (:, 0.988) =1.996, :r2 =3{1.996-5 ) = -1.001

. um; 1·[~'Ett-
= --0.001
which is small enough in magnitude . Hence,
Well-conditioned: If a small change in fue 4"iteration:
the given system of equations is ill- coefficient of the variable shows only a small :ri =l + 1
deviation in Ule solution. Capital then fue 3 (5 1.001) = 2.000, .t2 =3(2.000- 5) = -1 .000
conditioneci 1.e. X+ 2y = 4.0001 & 2x + 3y = 7_0001
system of equations is said to. be well- 5"'iteration:
h 2072 Set C Q.No. 16q conditione d. Solvmg x = 1.9999 & y = 1 _0001 1
Write the conditions for the system of equations Hence the system is well conditioned beca :ri=3(5+1) =2, r,=½(2-5)= -1
Ill-conditio ned: If a small change in the
au x + au y = bi, an :r + au y = b:z, to be ill coefficient of the variable shows large small changes in coefficient matrix or ri~ From 4th and 51h iterations, the value of x, and
conditione d [2] hand side vector gave the small _ gh r2 are equal.
deviation in the solution, then the system of change m llie
solution. , ri = 2. .r2 = -1 .
B!IB equations is said to be ill-conditioned.
Given equations are: 12s. 2073 Supp Q.No. 169 121. 2074 Set B Q.No. 16q fil.
2070 Set C Q.No. 17 aog
a11 x + a12y = bi ~!ve the following equation using Ga.u ss
Interpret geometrically that a system of Examine whether the system of equations
82,r+az,y = bi
If the two straight lines represented by above
equations in two variables is ill-conditioned. [2) .
x _+ 3y - 2z = 0, 2x - 3y +z = 1 and 4x - 3 + z = 3 15
diagonally dominant or not Y
a:i.~!;ethod 3x1 + X2 = 5; x, + 2x:i = s. 141
two equations are almost parallel, then slopes ~ mmmm . r21
Given equations can be Written as..
must nearly be equal Consider the system of equations 1
x-y=1 Given equations are: x + 3y _ 3z = 0 .t1 =3 (S-.t2), .t2=½(s -.t1)
. an 821
Le. - , _ - 2x-3y + z = 1
a12 a22 x-1.00ly = 0 Initially, .t1 = 0, r2 = O
:::) a11azi - a 1.2a21 "' 0 Solving, x =1001, y =1000 4x-3y + z = 3 1.. iteration:
The left hand expression is the determinan t of Now, changing the coefficient of y in second ~ere, the absolute value of coefficient of 1
.ti = 3 (5 - 0) = 1.667, .t2 = ½(5 - 1.667) = 1.667
the coefficient matrix(:~:: ) equation to 0.999 then
x-y=1
diagonal element in first equation= I11 = 1
All d th e sum of the absolute value of
21111iteration:
1
Thus, the condition for U1e ill conditioned X -0.999y = 0 coefficients of the non-diagonal elements in ri = 3 (5-1.667) = 1.111, .Q =½(5-1.111) = 1.945
system of equations is that the determinan t of firstequati on= 131 + l-31 =3+3=6
the coefficient matrix is small enough or Solving, x = -999, y =- 1000 3"' iteration:
Here, small change in coefficient results the Since 1 < 6, Uie system is not diagonally
approxima tely equal to zero.
dominant Xt = ½(5 -1.945) = 1.018, .T2 = ½(5 -1.018) = 1.991
very large ~ge in the solution. Such system
'22. 2072 Set D Q.No. 16g
Test whether the system of equations
is called ill conditione d system. '[Th7s Set~ a.No. 1&cj 4"iteration:
1 1
12x + 3y - Sz =1, x + Sy + 3z =28 and 3:r + 7y + Geometrically, the ill - conditioned system
5 ~ th e followings system of equation
[2)
Xi= 3(5 -1.991) = 1.003, .Q = 2 (5 -1.003) = 1.998
13z = 1 is diagonally consistent? (2) represents Ule straigh t Jines that are alrno l agonal!y dominant?
1~1+3 5" Iteration:
mmn parallel. Even a slight shift on any line c~n111 13.i: .. / ·2 - SxJ = 1, x1 + Sx2 + 3XJ = 28, 3x, + 7x2 + 1 1
Xi= 3(5 - 1.998) = 1.001, X2 = 2 (5 -1.001) = 2.000
3
Here, make a great move of the intersection point "- Pl
112 I > I3 I + 1-s 1 = s ill condi tioned system . . ease refer to 2072 Set D Q.No. 16c 6• iteration:
11s 1 > Il l+ 13 1 =4 1 1
~6. 2074 Set A Q.No. 16q Xi= J (5 - 2) = 1, .t2=2(5-1) =2
1131 > 13 1 + 171 = 10 4
Since the absolute val ue of the diagonal 2 ] [ yx ] '" [ 7 ] well· The values of xi and r2 in 5th and 6 iteration
th
Is the system [ 12 3 7a o
coefficient is greater than Ule sum of Ule

,~~a::;
(ZI SellteJ e following equations using Guess- are same.
absolu te va lues of Ule other coefficients in U1a t conditione d? Justi fy your answer.
~ lllett-iod• x, = 1,
row, so the system is diagonally dominant. J.f\'1 ·-♦ 7x2 = - 5 , (4)
re er to Model Set II, Q.No. 17a or

____,_.. . .,...,~
__
Asmlta's NEB Solutlon of Basic Mathematics-II Jnl lteratlon COMPUTATIONAL METHOD (CONTINUED) Unit 18 345
344
1
=½(2 + 3 x 3 - 3) ~ 1
1
ez, 2010 Set D Q.No. 1'7 a Oij z X "' is (] 9 + 'J.OQJ + 5 X 0,998) c 1,()00 Xi =5(2 "l.044+2><] .899-])=0.977
Solve the following system of equations by ']lh iteration 1 1
Gauss Seidel method Y• 20 (-18 -1.000- 0.998) .. -1.000 XJ = 3 (6 -1.044 + 0.977) = 1.978
3x + y - za= 2., 2x - Sy + z = 20, x - 3y - Sz .. 3 [4) x=½(2+3+1)=2 1 Iteration rv
z .. 8 (7 - 3 )( 1,000 + 4 )( 1,000) = 1.000
!i51!!IIIml y
1
=5 (2 )( 2 + 1 - 20) = -3 4U.iteratton
1
Given equations can be written as: X1 =4(7-0.977-1 .978)=1.011
1
1 1
x= (2-y+z)
3 ... (i) Z = ½(2 + 3 X 3 - 3) =1 x=25(19+1+5><1)'"1
Xl =5(2 X 1.01] + 2 X 1.978-1)=0.996
1
y =¼(2.1· + z-20) .. (ii)
From 6th and ']lh iterations, x, y and z have r 20 (-1s-1-1)=-t 1
equal values XJ =3(6-1 .01]+0.966)=1.995
1
So, X = '.l, y = -3, Z = 1 z = 8 (7 - 3 >< 1 + 4 >< 1) = 1
z=¾(x-3y-3) ... (iii) Iteration V

Initially, r = 0, Y = 0, z = 0
t\3. 2072 Set C Q.No. 17al From 3rd and 4th iterati0 1
Using Gauss Seidel method, solve the equations and z are equal, so x = ns, ~e values of x, y X1 "'4 (7 - 0.996 -1 .995) = 1.002
1" iteration: . 1•Y- -1,z=l
3x + 2y = -9, 2x - 3y = -6 [4) '3&. a'Q!!j 1
Put x = 0, y = o, z = 0 in equation (1), we get 2072 Supp Q.No. 17
X2 = 5 (2 >< J.002 + 2 X 1.995 - ] ) = 0.999
Bmiml Solve the following e •
X =½(2-0 + 0) = 0.667 From the given equations we have, Seidel method: quation using Gauss- 1
XJ =3(6-1.002+0.999)=1 .999
Again, put most ~ t values i.e. x = 0.667 -9-2y 2x + 6 3x1 + x.i= 5 (4)
x= 3 'y=-3- X1-3x.i=5 Iteration VI
and z = 0 in equation (n.), 1
lnitially, X = 0, y = 0 • Please refer to 2069 (Set B) Q.No. 17a Or x, =4 (7 - 0.999-1.999) = 1.001
y = ¼(2 X 0.667 + 0 - 20) = -3.733 Iteration I: ps. 2073 Set '·D' Q;Nc),,17liQ!!j 1
And, put x = 0.667 and y = -3.733 in equation (iii), -9 - 2 x 0 2 x (-3) + 6 Using Gauss-Seidel method 1 X2 =5 (2" 1.001 + 2 >< 1.999-1) = 1.()()0
x = - ---=-3,y= =0
3 3 system of equations: ' so ve the following
1
Z =½ (Q.667 + 3 X 3.733 - 3) = 1.1()8 Iteration II: 4xt + X2 + XJ = 7, 2x1 - Sx.i + 2x.i = 1 (4] XJ = 3 (6 - 1.001 + 1.000) = 2.000
-9-2><0 2x(-3)+6 !iE!!IB , Xt - X2 + 3XJ = 6.
2""iteration x = - ---=-3, y= , =0 Iteration VII
3 3
Given equation are 1
X = ½(2 + 3.733 + 1.108) = 2280 From 1•' and 2nd iterations, the values of x and 4X1 + X2 + )(J = 7 X1 = 4 (7 - 1 - 2) = 1
y are equal, so x = -3, y = 0
)' = 51 (2 X 2280 + 1.108 - 20) = -2866
, · 2072 Set D Q;No. ftaj
!34
2x1 - 5x2 + 2x.i = 1
Xt - X2 + 3X3 = 6
1
X2 =5(2><1+2><2-1 ) =1
Using Gauss Seidel method, solve: 1
Z =! (2280 + 3
8
X 2866- 3) = 0.985. €J ;.:n,:,f,z
= 7, x + 20y + z = -18, 25x + y - Sz = 19 [4]
From these equations, we have
) 1
XJ "'3(6-1+1)=2
3rd iteration _!(7
Rewriting the given equations so as to make · X1 -4 -X2-)(J , X2 "'5(2x1+2x.J-1) From 6th and ']\h iterations, the value of x,, x,
X =½ (2 + 2866 + 0.985) = 1.950 them diagonally dominant, we have 1 and XJ are equal, so x1 = 1, xi= 1, X:l = 2.
XJ = 3 (6 - X1 + X2)
1
25x+y-5z=19
x+ 20y + z= -18
,ISJ,j·J1-i·i'liilhVfi
Y = 5 (2 X 1.950 + 0.985 -20) = -3.023 Initially, X1 = 0, X2 = 0, x_, = 0
3x + 4y + 8z = 7 Iteration I p
,7. 2010 Supp. Q.No.1~
1 From these equations, we have, Given the system
Z =s (1.950 + 3 X 3.023 - 3) = 1.002 1
1 1 Xi = 4 (7 - 0 - 0) = 1.75 lr-6y + 2z = 23
4 th iteration x=i5{19-y+5z), y= 20 (-18-x-z)
-4x+y_-z=-8
1 .1 x-3y + 7z=17
1 Xl =5(2><1 .75+2>< .0-1)=0.5
X =
3 (2 + 3.023 + 1.002) = 2.008 z = 8 (7 - 3x - 4y) Make it diagonally dominant and solve by
1 Initially x = 0, y = 0, z = 0 1 Gauss-siedel method with eD"Or less than 0.005.
Y =s (2 X 2()08 + 1.002-20) = -2996
1•1 iteration
x., =3 (6-1.75 + 0.5) = 1.583
(6]
1 1 Iteration II
z =s (2.008 + 3" 2996 - 3) = 1.000 X =2S (19 - 0+5 X 0) = 0.76
1 Writing the given equation so that the system
5th iteration 1 4
Xi = (7 - 0.5 - 1.583) = 1.229 will be diagonally dominant
1 y =20 (-18 -0.76-0) = - 0.938 -4x + y- z =-8
X = (2 + 2.996 + 1) = 1.999
3
z = ½((7 - 3 x 0.76 - 4 x (-0.938)1 = 1 -059
X2
"'sl (2" 1.229 + 2" 1.583 _ 1) = o.925 3.t - 6y + 2z = 23
x-3y + 7z = 17
1
y= 5(2 )( 1.999 + 1 - 20) = -3.001 21w1 iteration
~3 =! From these equations, we have
3 (6 -1 .229 + 0.925) = 1.899
Z
1
=s {1 ,999 + 3 X 3,001 - 3) = 1.000 X o= i (19 + 0.938 + 5" 1,059) = ] .009 Iteration III
X1 l
X=
1
4(8 + y - z)
6th iteration 1
1 y "' ~ (-1 8 -1.009 - 1.059) .. -1.003 "':w - o. 925 - 1.899) = 1.044 y=
6(-23+3x+2z)
X = (2 + 3.001 + ] ) = 2.000
3
1 z., ½(7 - 3 x 1.009 + 4 " 1.ocn) .. 0.995
Y = S (2 X 2 + ] - 20) = -3
Asmlt•'• NEB Solution of Buie MathemeHcs-11
346 COMPUTATIONAL METHOD (CONTINUED) Unit 18 347

z=~(17-x + 3y)
[; ~:~;] [;]-[~] e.: 2uio~
(E5a. ;;:0:=.~7;~ - - - - -- -
p.=;Q~.N
71F;s~u:::p:= 1 l

Initially, r = 0, y = 0, z = 0
which is in the form of AX
1
s B
... (i)
Usc the Gauss-Sie del method to solve the
systems
y- 5 (3 - 2 X I - 2 X 3) = -1
I
1" iteration

x= ¼(8 + 0 - 0) = 2
⇒ X • A- B
For A-1
3 1.52
~1.::w:• 2x+5y+2z• 3, x+2y+4z • 11 . l6J
1
z = 4 (11-J + 2 x 1) =3

From 6'h and 7t• iterations, the va lues of x, y


'

y =¼(-23 + 3 X 2 + 2 X 0) = -2.&,3
IAI • I
= 3.06-3.04
2 1.02 =3 X

= 0.02;t 0
1.02-2>< 1.52
Fro;n the given equations, we have
X a 4 (8 + y - z), y m ½(3 - 2.r - 2z)
and z are equal, so
x = 1, y = -I and z = 3
So, A-• exists. 1 ~ - 2071 Supp. 0..No. 19 OR!
½
z = (17 - 2 - 3 X 2.8.~3) = 0.929

-
1.02 -1.52 ] z= 4 (tl -x-2y) Givrn the system
Adj . A = [ -2 0.335% + 0.667y = 0.168, 0.333x + 0.266y ~ 0.067
3
2-iteratio n Initially, x = 0, y = 0, z = 0
1 •• iteration: Determine whether the system is ill conditioned
¼(8 _ 2833 _ 0.929) = 1.06 1 1 [ 1.02

"
.r = 1 by cb.ngi.ng the coefficient 0.667 to 0.666. l6]
A-1 = m Adj. A= 0.02 -2
r = 4 (8 + 0 - 0) =2
_l( + 3 x 1.06 + 2 x 0.929) = -2.994
y-6 - 23

z =~(1 7 -1.06- 3 X 2994) = 0.994


X
From (i)
=A-1B
_...L[ 1.02 -1 .52] [1] 1
1
y = 5 (3 - 2 )( 2 - 2>< 0) = -0.2
Given equations are:
0.835.r + 0.667y = 0.168
0.333.r + 0.266y = 0.067
1
-0.02 -2 +3 l z = 4 (11 - 2 + 2 X 0.2) = 2.35 The given equations m matnx form can be
2•• iteration: Written as:
3-it,eralion _L [1.02-1.5 2]
r=¾(S-2 994-0.994) = UXJ3
= O.o2 -2+3
X
1
= 4 (8 - 0.2 - 2.35) = 1.363
0.835 0.667 ]
[ 0.333 0.266 y - O067
[·I] _
[ 0. )68]

y = ¼(-23 + 3 X 1.003 + 2 X 0.994) = - 3.0(ll r;1 = r-:1 1


y = 5 (3 -2 >< l .363 - 2 x 235) = --0.885
which is in the form of A,'( = B
=> X = A· ' B

z =½ (17 - 1.()()3 -3 X 3.001) = 0.999


r= -25,y = 50
Again, changing the coefficient 1.02 to 1.03,
1
z =4 (11 -1.363 + 2 )( 0.885) = 2852 IA I = I 0.835 0.667
0.333 0.266
I (i)

we have the following equations 3"' iteration:


4• iteration
3.r+ l.52y = 1 1 = 0.835 X 0.266 - 0.667 X 0.333 = --0.CXXXXJl ~ 0
So, A-1 exists.
r =¾(8- 3.CXll - 0.999) =1 2.r+ 1.03y = 1 X = 4 (8 - 0.885 - 2852) = 1.066
The given equations in matrix form can b,• 1 Adj.A =[ 0.266 --0.667 ]
)' =¾ (-23 + 3 X 1+2 X 0.999) = -3 written as: )' = 5 (3 - 2 X 1.066 - 2 X 2.852) = -{)_967 -0.333 0.835
1
z=~ (17-1-3 x3) =1 [ ~ ~:~; ] [;] = [~]
1
z =4(11 -1 .066 + 2 )( 0.967) = 2967
1
A· =mAdj . A
which is in the form of AX =B 4 th
iteration: [ 0 "66 - 0.667 ]
S-iteatio n 1
⇒ X=A-1 B ... (i) 1 ==o:cmxn --0333 0.835
.r=¼(8-3 -1) = 1 ForA-1 X =4 (8 - 0.967 - 2.967) = 1.017 From(i) •
2 X =A-' B
I
3 1.52 1
¼(-23 + 3 =- 3 2 1.03 =3x1.03- 1.25x
)' = X } +2 X 1) IAI= y = 5 (3 - 2 )( 1.017 - 2 )( 2.967) = --0.994
=__I_ _ [ 0.266 --0.667 ] [ 0.168]
= 3.09 -3.04 = 0.05 ;t 0 1 -0.000001 --0.333 0.835 0.067
z=~(l7- 1-3x3)= 1 So, A- 1 exists. Z =4 (1} - 1.0}7 + 2 X 0.994) =2993
1 [ 026o X 0.168 - 0.667 X 0.067 ]
Therefore, exact solution of given system is =~
1.03 -1.52 ] S• h iteration:
3 -0.333 X 0.168 + 0.835 >< 0.067
.r = 1, y = -3, z = 1 Adj. A = [ -2 1
The solution with error less than 0.005 is X = 4 (8 - 0.994 - 2.993) = 1.003
l 1 [ 1.03 -13.52 ]
obtained in w iteration which is A-' = w Adj . A= 0.05 -2 1
.r = 1.003, y = -3.001 , = 0.999 )' = 5 (3 - 2 X 1.003 - 2 X 2.993) = -0.998 .r = 1, y = -1
From(ii) 1 Again, chan.ging tht• oo.tfficient 0.667 to 0.666,
l3s. 2070 Supp. Q.No. 19 OR! X =A-1 8 Z =4 (11 - 1.()()3 + 2 X 0.998) = 2.998 we have the following t'\lUations
Given the system 0.835.Y + 0.666v = 0. I o8
1 [ 1.03 -13.52 ] [11] 6• h iteration:

-
3.r+LS2y "'l = 0 .05 -2 0.333x + 0.26oy = 0.067
2r+l.02y "'l 1
X= 4 (8 - 0 .998 - 2.998) = 1.001 The given ~uabons in matrix form c,m be
Determine whether the system is ill-cond itioned _l__ [1.03-1.52 ] written as:
by cb.ngi.ng the coefficien t 1.02 to 1.03. 16) = 0.05 - 2+3 1
y "' 5(3 - 2 )( 1.001 -2 X 2.998) = - ] .()(X} 0.8..15 0.666 ] [.r ] [ 0.168 ]
Given equations are:
3r+ 1.52y = 1
Gl [-io =
8
]
. . we sec •1
1
z "' 4(11 - 1.(JOl + 2 X l .lXXJ) = 3.CXX)
[ 0.3.:13 0.266
wluch is in th.- fo rm of AX = B
y = 0.067

:. .r g - 9.8, y "' 20 7'b ite ration: ⇒ X=A ·1 B .. . (ii)


2.r+1 .02y = 1
From abovt' two se ts of _s olu~~~~ th ere is •1
The given equations in matrix form can be large d 1ange in th e sol ution h , syste111 x .. l
written as: small change in the codficit"nl. So, t c 4 (8 - 1 - 3) = I
is ill conditione d .
350 Asmlta's NEB Solution of Basic Mathematica-II COMPUTATIONAL METHOD (CONTINUED) 351
Unit 11

-(; : ;
li1i1IIILII

!1},➔ o)~
1 4 7 : 0 1 OJ
Gauss Elimination Method
_ o -36 -72 : 0 -10 1 R.,++R,
[ O O -22 : 1 -3 0 Given equations are ti?i·i·i:fi·l1iiiit•itt1
X+ y+Z• 6 .. . (i) ff. 2075 Set B Q.No. 19 oRj
-[~: : . : fs -¼]R, ➔(~)R,
0 5 7
3x - 4y .. - 5 ... (ii)
Solve the following syst"m 0f

Lt :JR1➔R1-R2
= 13 ... (iii) equations by

-(t?X~~{f)-
-3x + 2y + 4z matrix inversion method;
0 0 -22 1 -3 0 Multiplying equation
from equation (ii)
(i) by 3 and subtractm
g Jo: o: 3x+Sz•14,
2x+y-3z• 3,
'. !

l ~
1 0 -1 : 0 -½ ½] 3x-4y = -5 7 Rl ➔ RJ-5Rz x+y+z•4 .
~I
- S 1 R, ➔ R, -4R., 3x + 3y + 3z = 18 ~ 5
o 7 1
I
012 : 018°36 7
[ - - -7y ~ 3z = --23 ... (iv)
00-22 : 1-3

1 0 -1 0
0

-½ ½]
-ls-¼ Ri ➔ -(i)R,
Again, multiplying equation (1) by 3 and
adding with (iii), _(:: i ;3
7
1~
-7
0 J RJ ➔
7
34 RJ
1
3x - 2y + 4z = 13 0
- o 1 O
_3x ~ 3y ~ 3z= _ 18 lo o 1 3 5 7 which is in the form of AX = B .. . (i)

-(: :: f(fJ ➔
[ 1 3
ool : -n 22 0 Sy + 7z = 31 ... (v) where,

f Jx=U)s=( r)
5
App{lying R, ➔Rt + ;R.,; R:i1-2Ri.] and adding with
1 0 0 : 198 -22 9
Multiplying (iv) by
equation (v)
7
17 -34 -34
R, R1 -tRJ
3 A=U O 1
1 1 1 Sy + 7z 31 1._ 1_ 7 R2 ➔ Rz- 7 Rz We augment the matrix A with identify
0 1 0 : ~ 1: -36 15 115 0 0 1
17 34 34
-Sy z matrix I,
-22 22 7 7

(t~-~1-_tJ
0 0 1 : 0
102 3 0 5 : 1 0 0)

-~
37 ... (vi)
7Z (A: I) = ( 2 1 -3 : 0 1 O
7
111 : 001
-22
1 198
5 91 ] Now, we have the following three equations A-'=
X + y + Z =6 ... (i) 111 : 001 )
1 1 1 - ( 21-3 : 010 R1HR,
-7y - 3z = -23 ... (iv) 17 34 34
A·' =
[
~

-22 22
1~ -36
° z=
7
34102
... (vi)
7
From (i) X = A-18
305 : 100
1 1 1 :0 0 1)
- 0 -1 --5 : O 1 -2 Rz ➔ Rz-2R,
From the equation (vi), we have z = 3 ( R, ➔ R, - 3R,

=(;v -L34; J( ~)
0 -3 2 : 1 O -3
Using z = 3 in (iv), we get
-7y - 3 X 3= - 23 1 1 1 :0 0 1)
or, -7y=-14
X ~015:
( 0-12 Ri ➔ (-l)R,
:. y=2 3i 0-32 : 10 -3
Using y = 2, z = 3 in equation (i), we get 1 0 -4 : 0 1 -1)
x+2+3=6 - o 1 s o -1 2 Ri ➔ Rl +3Rz
( 0 0 17 : 1 -3 3 R, ➔ R, - Ri
or, x = 1
:. X =1, y = 2, Z =3

t
1 0 -4 : 0 1 -1)
Matrix Inversion Method 0 1 5 : 0 -1 2 1
- ( 1 -3 3 Ri ➔ 17Ri
Writg ren)(eqr)ti:n(s ~fta)trix form: 001 : 171717
( -3 2 4 z 13
~- 201, Supp Q.No. 17a oRj which is in the form of AX =B .. • (i) 100 : 171717
4 5 -SJ
Solve the following system of eqwdiom using 0 1 0 · ~ ~ !2 R, ➔ R, + 4R,

! ~ l x=(;
-
inverse matrix method:
3X + y + Z = 15, X + y + Z = 3, y - Z "' - 1.
[4]
:~e(re,; ),B=( !s) (
· 17 17 17
1 -3 3
Ri ➔ Ri-SR,

Please refer to 2071 Set CQ.N. 17a OR


-3 2 4 Jthe matrixz A with identify
13 001 : 171717
Now, we augment 5
is. 2075 Set C Q.No. 17!

!
matrix 1, 17 17
Solve the following system of equation, US_ing
Gauss-elim ination or inverse matrix method. [41
x + y + z = 6, 3x - 4y • -5, 4z - 3x + 2y • 13
(A: I) •
(
1 1 1
3 -4 0
-3 2 4
: 1 0 0)
: 0 1 0
: 0 0 1
.. A·'=
( 4 -2 19
17 17
-3 3
f, J
1 1 1 : 1 0 0) R2 ➔ Ri - 3R1 X = l, Y = 2, Z =3 17 17 17
- ( 0 -7 -3 : -3 1 0 RJ ➔ Rl + 3R1
0 5 7 : 301
of &a•lc Mathefllatlc•-11

19
352
Asrnlta'• NEB 5o1.,cton

From (i) AX • B
UNIT
~ x=(A-':17 .i.
17 ~J
17 14

or, X =
-5
17 17
-2 .!2.
17 (
3
4 ) X • 3, y • 0, Z =1
NUM ERIC AL INTE GRA TION
1 -3 .l. A rRAPEZOID AL RULES

-
417x 14+
7
:1
17 17" 3 -17
2- ,. 4]
fi,U·i·I li•.Ulii■ IIH61
11 , 2089 (Set Bl Q.No. 18d
19 4
-17 x H-17 " 3+ -17 >< 4
-5 2
or, X --
3
Given I • Jr dx, n • 4. Estimate t11e value of J using Trapezoidal ra1e. [2)
~ ><14 - r,x3+ 17 " 4 0
~ aum
Here, y = f(x) = x3
a = 0, b = 4, n = 4
b-a 4-0
We have, h =-n-=- -= 1
4
The five points to be considered are Xo = o, x = 1, x = 2, .r = 3, .r.. = 4. The
1 2 3 values of the
function at these oints are tabulated below.
End oint Xo = O x1 = t x2 • 2 .r3 = 3 .r.. = 4
= f x = x3 0 1 8 'ZJ M
Using trapezoidal rule, we have
4 h
f x 3dx "' 2 [yo+ 2y1 + 2y2 + 2y3 + Y•1

I
0
1
= 2 (0 + 2 X } +2 X 8 + 2 X 27 + 64) = 68
F· 2070 Set C Q.No. 16 d
Using trapezoidal rule, evaluate: (f- .../Sin'i dx, n • 2. [2)
liBYlirml
Here, y = f(x) = ~
a = 0, b = n/2, n = 2

b-a ~-O 1t
We have, h = -n- = - - =
2 4
The three points to be considered are:

Xo = 0, X1 = ~ , X2 = ~
The values of the function at these oints are tabulated below:
End point .ro • 0

= f x = sin x 0.8409
Hence, by us ing trapezoidal rule, we have
~2 ~
. l?- h 4 ooAllll
f v sin x d x,. 2 (yo+ 2y 1 + y,J "' 2 (0 + 2" ,.,..., .., 1] .. 1 0531
7

0
354 Asmita's NEB Solution of Baslc Mathematics-l
NUMERICAL INTEGRATION Unit 19 355
. 2070 Set DQ.No. 16 C Hence, by using
2 trapezoidal rule, we have
Using trapezoidal rule, evaluate, (-4:)d, n =3. 121
sOLUTION
(2r-1) dr yo+2y +2y +2ys+ yl
0
Here, f() = 312 4r

We have, h=
a0,b=3,n=3
h- 01
-1+2x (-0.5)+2*1+2*35+7]
0.25 [-1-1+2+7+7 3.5
The four points to be considered at xo = 0, xn= 1, 2, x 3 . 20174 Supp Q.No. 16
The values of the function at these points are tabulated as followS: Using trapezoidal rule, evaluate dx
End point 22
(1+n=2 12
y=f)= 3a2-4x Please refer to 2071 Set C
QN. 16c
Using trapezoidal rule, we have 7. 2075 Set CQ.No. 16
(3x-4) dx *~vo+2y +2ya+ ys]=|10+2* (-1)+2x4+ 15]
Use the
Trapezoidal Rule to
approximate the
integraldx Find the error
for the
- - 2 8 15- 10.5 approximation.
sOLUTIONN
42071Set CQ.No. 16 Here,
a 1, b-2
Using the trapezoidal rule evaluate n =2

0
SOLUTIONN
Here, a 0 , b = 1, n=2.
f0)-1, f2)-
Using trapezoidal rule,
hb-a-0
n
T h r e e p o i n t s t o b e c o n s i d e r e d a r e xo = 0, 1 = 0.5, 2 = 1 . db-
The values of function at these pointsaretabulated below
End point o 0 =0.5 X2=1 R-
y f(x) = (1+ 1 = 1 + 2 1 0.8 0.5
0.75
Hence, by using Trapezoidal rule, we have
1 Actual value =

dx
f1+2dr fta)+ 2f(r)+ fts))
= In2 - Inl

120.8+0.5] 0.775 =0.693147


Error = Actual value - approximate value

5. 2071 Set DQ.No. 16 0.693147-0.75


- 0.056853
Using the trapezoidal rule, evaluate: (2x2-1)dx, n =4. 2
MARKS QUESTIONS
sOLUTION &2069 (Set A) Q.No. 17 1
First part
Here, a 0, b = =
2, n =4 Estinate the following integral using Trape-Zoidal rule. i n 4
hb -05
h=- n
0
41
Estim the error with respect to the actual value
The fivepoints beconsidered are xo
to =0.5, x2 1, xg =1.5, x= 2
0, X1

The values offunction at these points are tabulated below SOLUTION


End point o=0 =0.52-1 -1.5 2 Here, y f()-
y 22-1 -1 -0.5 1 3.5
a 0, b 1, n =4
356 Asmita's NEB Solution of Basic Mathematics-

1-0 NUMERICAL INTEGRATION Unit 19 357


We have,h- n
40.25
=0.75, x41. The five
be considered are xo 0,
=
=
0.25, 2-0.5, x3 points to be considered are xo 1, x 1.25, x2 =1.5, x
= =
=
1.75 and z4=2
ne five points to
Deto The values of the function at these points are respectively.
The values of the function at these points are tabulated 0.75 =1 End point: tabulated below;
End point 0 0 =0.25 -0.5 Xo-1 1-125|2-15 x-1.75 2 |
0 . 8 |0.66666 0.57143 0.5
0.64 0.4444
y-F() 1+1 Using trapezoidal rule, we have
0.3265 0.25

Using trapezoidal rule, we have 2


x dx vo+2y1+2ya+ 2ys *yl
0

= 1+2x 0.8+2x 0.66666+2


x 0.57143 +0.5] 1+2x 0.64+2x04444 +2x0.3265+0.25]
-(0.125) (5.57619) 0.67701
-1+1.28+0.8888 +0.653+0.25]-0.509
10. 2071 Supp. Q.No. 17b
Actual vaBue 7 -log (+ -log 2-0.6915
Compute an approximate value of
Error Actual value Approximate value (f (1+91 dr by the composite trapezoid rule with three
0.69315 -0.67701 = 0.01614
points. Then comparing it with the actual value of the
B 2070 Supp.Q.No.175 SOLUTION integral, find the erTOr. 4
h and h =
by the composite
Compute two approximate values for dr using First part: Please see 2071 Set
CQNo. 16c
4 Next part:
trapezoid rule.

SOLUTION Actual value


Here, y = f{) = r2

a 1 , b=2
1[lan--an'1 --0785a
Error Actual value -Approximate value
Case I: When h - - 0.7854-0.775- 0.0104

We have,
11. 2072 Set EQ.No. 17E
h-b n
Evaluate using composite trapezoidal rule, the integral sinx dr, n =
4.
on.
4
or, n = 2
SOLUTION
The three points to be considered arex- 1, -15, a =2 respectively. The values of the Here, a = 0,b= t, n 4
functionat these pointsare tabulated below.
End point o1 X11.5
0.4444
y-fo)- r- The five points to be considered are
xo- 0, - u -
t
respectiveby. The
Hence, by using trapezoidal rule, we have
2
values offunction atthese points are tabulated below
rdro 2y yl-u 2*044+029]-0.5847 End point Xo 0

y=sinr 0
Case I1: When h = 2
USing trapezoidal rule, we have
We b-a
have, h= b J sinx dx » lyo+ 2y1+2y2 +2ys * y«l
Or,

'n=4

2 2)- 19%
Asmita's NEB Solution of Basic Mathematics-
358

6 MARKS QUESTIONS NUMERICAL INTEGR. Unit 19 359


12 2075 Set AQ.No. 17b OR
Evaluate, using composite trapezoidal
rule the integral: ,n =
5.
41 15. 2072 Sot CQ.No, 19 0OR
State and prove
sOLUTION Please see ModelTrapezoidal
Set 1,
rule of numerical
approximation.
16. 2072 Set DQ.No. 19 Q.No.
Here, a - 0, b = 1, n =5
19 [61
We have, h -0.2
5
The six points to be considered are to=0, 1i = 0.2, x2 0.4, x1 = 0.6, x = 0.8 and xs = 1. The values af
of Approximate the value using trapezoidal rule
theintegrand ateach of the points are as follows:
for e dr; n -2
e6
X
0 0.2 04 0.6 0.8 sOLUTION 1
0.83333 0.71429 0.625 0.55556 0.5 Here, a
-1, b =1, n =2

Using Composite 1rapezoidal rule, we have We have, h


The three
points to be
The values of function considered are xo
-1,x =0, x2=1
End point
at these points are
tabulated below respectively.
ye -1 10
1 + 2 x 0.83333+ 2x 0.71429+ 2x 0.625+2x 0.55556+ 0.5]
Hence, by using 0.3679
02 1 trapezoidal rule, we have
6.95636
0.695636
(e dx a)+2{tr«) +{()]
-1
13. 2075 Set BO.No. 17b
Compute an approximate value of lo (1+ x1 dx by using the composite trapezoid rule
[0.3679+2x1+2.7183] =2.5431
-

with three points. Then comparing with the actual value of the integral, find the error.
17. 2073 Set C Q.No. 19
First part: Please refer to 2071 Set C QNo. 16c
1
Second part:Pleaserefer to 2071 Supp Q.No.17b Define Trapezoidal rule. Evaluate using
142075Set BO.No.17D OR Trapezoidal rule: for n=4. [61
Approximate i 2 dx using Simpson's rule with h = SOLUTION
First part: Please see Model Set L, Q.No. 19
SOLUTION Second part:
Here,
Here, y f )
a-0,b-2h- =1+7
We have, a 0, b =1, n=4
h- h We have, h 4 -025
or, n=h The five points to be considered are xo =0, x 0.25, xy =0.5, x
=
=
0.75, x=1.
Thevalues of the function at these points are
2
End point tabulated below
Now, the end points of four subintervals are 0, 0.5, 1, 1.5 and 2. 0 =0.25 =05 075 1
The value of the integrand at these points are tabulated below:
0051 152
y-s) 0.8 0.66666 0.57143 0.5
y f(x)2 1 14142 2 2.82842 4 Using trapezoidal rule, we have
Using Simpson's rule, we have
JTT dr -bo 2y+2y+ 2ys*yl
0
2rdx vo4y+ 2y 4y *yd
2 1 + 2 x 0.8+2 x066666+ 2 x 0.57143+0.5]
=[1+4x 14142+ 2* 2+4x 2.82842+ 4]
(0.125) (5.57619) 0.67701
(25.97008)- 4.3283
360 Asmita's NEB Solution of Baslc Mathematics-l
NUMERICAL INTEGRATION Unit19 361
18. 2073 Sot DO.No. 17
Usit the trapezoidal rule, compute 2f(2-1) dx with 4 intervals. Find the absolute r B. SIMPSON'S RULE
2 MARKS aUESTIONS
of approximation from its actual value. [4] . 2072 Set EQ.No. 16c
sOLUTION 0.2
Find the
First Part: Please refer to 2071 Set D QNo. 16c approximate value of JV1-2 dxn=2,
Second part using Simpson's rule.
21
0
2 SOLUTION
Actual value 22-1) dx Here, y =f{*) =V1-22
0 a 0, b=0.2, n 2
We have, h- 0=01
22-3.333 The three points to be considered at zo 0, =

Absolute error 13.3333-3.5|=0.1667 0.1, x2 0.2 respectively. The values of the functions at
these points are tabulated below:
19. 2074 Set A.No.19 End point
La=0.1 =0.2
Using trapezoidal rule, evaluate 14Tn-4
+x
Estimate the error of approximation from its
y-f)-1-22 0.9899 0.9592
actual value. 6 Using Simpson's rule,
Please referto2069 Set AQNo. 17b 1
20. 2074 SotBQ.No. 19 we have
(V1-2 dr vo+ 4y+ yl -
State Trapezoidalrule, hence evaltuate (i+forn-4 correctto 3 places of decimal. 15
4x0.9899+0.9592]
0

0.1973
sOLUTION 2. 2073 Set DQ.No. 16
1t part: Please refer to Model Set 1 QNo. 19
2nd part 0.2
Here, a=0, b=2, n=4
Using Simpson's rule, evaluate: J 1-2 d,n -2
.h -05 SOLUTION
So, the five points to be considered are xo 0, x =
0.5, x=1, x =1.5, x =2. The value of the Here, a = 0, b= 0.2, n =2

integrand atthese points aretabulated below: h h=-2 0: =0.1


End point F 0 x=0.5x=1|x=1.5 =2
1 0.94118 0.5 0.16495 0.05882
Value of y f)=1+K The three points to be taken are 0, 0.1 and 0.2.
The values of the integrand at these points are tabulated below.
have
By Trapezoidal rule, we
0.1 0.2
2
v o +2y1+2ya 2ys+yd v-1-2 0.9933 0.9726

Now, using Simpson's rule,


-[1+2x0.94118 +2x0.5+2x0.16495+0.05882] 0.2

9427108)
=
1.06777-=1.068 (upto 3 places of decimals)
(1+4x0.9933+0.9726)

0.1982

2076 SetB O.No. 16d


the value of he In x dx with n =3. 12
APply Simpson's rule to approximate
Basic Mathematics-ll
362 Asmita's NEB Solution of

NUMERICAL INTEGRATION Unit 19 363


SOLUTION & 2070 S0tDQ.No. 17C
Here
a 1 , b4,n=3
Using the Simpson'srule, evaluate n -4.
h - =1
1, X1 Xo + h =1+1 =2 x2 X1 + h =
2 + 1 =
3 and
SOLUTION
The 4 points to be considered are Xo
a= and
xs=b=4. Given, y ( ) -
integrandateach of these points are tabulated below.
The values of thexo1X1=2x3 x=4
Here, a - 0 ,b 1,n =4
Ly f)=evnx0 5.12170 2206621 75.68911 We have, h 0.25
3
The five
Since n-3 whichis odd, so we have to apply Simpson's rule. Using Simpson's rule, we have points to be considered are xo= 0, x
The values of the function at these points are
0.25, z2 0.5, 13= 0.75, I4 =1.
=

End point tabulated below:


e rx dxbe+3 xy+3ya+ yil Xo0X1=0.25 205 |=0.75
03x5.12170+3 x 2206621 + 75.68911) 0.8
1
0.6666 0.5714 0.5
58.969815
Using Simpson's rule, we have
4 MARKS aUESTIONSs

42069(Set B)Q.No.17b (idbot4y+2yzt 4ys+ yd -1408+2x0.666+4x 0.5714+05]


0
Evaluate the following integral using Simpson's rule:J sinx dx, n =6.

SOLUTION
0(8318)
0.6932
Here, y-f(r) =sinr Z2071Set CQ.No. 177E
a=0,b=T,n =6
Using Simpson's 1/3 rule, evaluate: 1+2 dr, h= 025
We have, h=*
0
The seven points to be considered are xo=0, x1 sOLUTION
Here, f(x)=v1+2
a- 0, b=1, h= 0.25
We have,
57t h a
X and x = t. The values of the function at these points are tabulated below:
n
End point o=0
O, 0.25
y=f)=sin x 0.5 0.8660 1 0.8660| 0.5 n=4
The five points to be considered
Using Simpson's rule, we have are xo= 0, x1 -0.25, 2 0.5, x
The values of the function at these
=
0.75, x-1.
are tabulated below:
points

sin xdryo+ 4y+ 2y+ 4ys+ 2y4+ 4yst ysl End point o0 =0.25 =0.5 =0.75 =1
1 .1.0607 1.2247 14577 1.7321
0 y f)=v1+2
[0+ 4 x0.5+2 x0.8660+ 4x1+2x 0.8660 +4 x 0.5+0] Using Simpson's rule, we have
1
1 (11464) 2.008
t SVi+2xdx vo 4y1* 2ya* 4y»* yd
5 2070Sat GO.No.17b 0
dx 141
Using Simpson's rule evaluate lo1+n=4. [1+4x 1.0607+2 x 1.2247+4x 1.4577+ 1.7321]
Please see Model Set I, QNo. 17b
1.2712
364
Asmita's NEB Solution of Basic Mathematics- UMERICALINTITEGRATION Unit 365

8. 2071 Set DQ.No. 17 14x0+2x 1+4x4+9]


Estimate the following integral using Simpson's 1/3
rule: sinr dr, n -6 x284. 4.6667
13. 2076 Sot AQ.No. 17E
Please refer to 2069 Set Q.No.17b Using Simpson's rule,
B

Please
evaluate:
refer to 2070 Set D QNo. 17b 1x,n-4
2072 Set CQ.No.17b 4
5
* dr with 4
14. 2076 Sot CQ.No. 17bE
calculate =

Using Simpson's rule,


n
4 2
1 Using Simpson'srule, evaluate ((42-4x+1)d», n4
SOLUTION 4
Here, y -f{7)
=
* sOLUTION
a1,b=5, n = 4 Here,

-1 a =0,b 2,n=4
We have. h n We have, length of each
interval (h)
Thefive points to beconsidered arexo 1, - 2 »=3, 1=4, 45. Thevalues of the Thus the end points of the 4 --4=05
n

function at these points are tabulated below. end points of the sub intervalssubintenals are 0, 0.5, 1, 1.5
are given below:
and 2 The values of the
integrand at each
End point 1 =2 =3 =4 x=5 X 0.5
y-f)=r 16 81 256 625 y-4x2-4x+1| 0
Using Simpson's rule, we have Now, using simpson's rule, we have

rdalvo+ 4y +2ya+ 4ys+yd (4x-4x+ 1)dx -vot 4y1 +2y2+ 4ys +yd
0.5
1+4x 0+2*1+ 4x4+9]
-[1+4x16+2x81+4 x256+625]-625.33 28= 4.6667
10.2072 Supp Q.No. 17 6 MARKS QUESTIONS
Evaluate the following integral using Simpson's rule: ; 16. 2072 Set DQ.No. 19 OR
Evaluate V1+ dr using Simpson's rule with n-4.
Please see Model Set II, Q.No.17b I61
81. 2073 Sugp Q.No.17 0
SOLUTION
Evaluate the following using Simpson's rule: n= 4 14 Given, f(x) =y1+
Please refer to Model Set II, QNo. 17b a =0, b= 1, n =4
=
12. 2074 Supp Q.No. 17b We have, h - -0.25
Using Simpson's rule, evaluate (2x-1Pdx, n = 4. 14 The 5 points to be considered are xo 0, x 0.25, x2 =0.5, x =0.75 and =1. The values of
-

thefunction at these points are tabulated below.


sOLUTION End point Xo0 x=0.25y=0.5 =0.75u=1
Here, a 0, b = 2, n = 4
y=f =I+r 1 1.0078 1.0607 1.1924 14142
We have, h =0.5 Using Simpson's rule, we have
The five points to be considered are xo 0, 1 = 0.5, x2 = 1, zs = 1.5 and x = 2. The values of u e

integrandateach ofthe pointsare as follows: SVT dr


0.5
Ly-fla)=2x-1 0
Using Simpson's rule, we have - +4x 1.0078 2 x 1.0607+4 x1.1924+1.4142

(2x-1P dx lyo+ 4y+2ya+ 4ys+y - 1.1114


366 Asmita's NEB Solution of Basic Mathematlcs-ll

16. 2073 Set cONo. 19 OR 2


of decimal.
Using Simpson's rule, evaluate for n4, correct to 3 places

sOLUTION Mer
Here, a = 0, b = 2 n = 4

h n 2-05 1.5 and x 2 .


t Hast

xa =1, x=
points tobe considered are xo=0, x
H a s t
The five
=
0.5, e h

The value of the integrand at these points are tabulated below.

o0 x=0.5 x=1 1.5 2


yf) Y1 0.9412 y0.5 ys =0.1649 =0.0588
R a t e n
aya
Using Simpson's rule, we have

Hax
4 x0.9412+2 x 0.5+4 x0.1649 +0.0588

=1.081

17.2074 5et aN 190 16]


hence evaluate ( for n =
4 corect to 4 places of decimal.
Define Simpson's rule,

SOLUTION
Simpson's Rule: If a function f is continuous on closed interval fa, b], then,

b- 2fx) +4fxs)+ 2f(xa) t +2f(%n-)+4ffx-) +


{(x»)]
x) dx o)4fx) + ...

b
each of the
where la, b] is divided into n equal subintervals [o x] k, xal x2 xa].. xn-1 Xn],

length n

Second Part
17b.
Please refer to2070 (Set D) Q. No.

www

You might also like